Bộ đề phát triển đề tham khảo kỳ thi tốt nghiệp THPT môn Toán năm 2022 mới nhất

666 7

Miễn phí

Tải về máy để xem đầy đủ hơn, bản xem trước là bản PDF

Tags: #toán 12#đề thi toán#thptqg toán

Mô tả chi tiết

Tài liệu do Tailieuvip.com sưu tầm gồm 308 trang, được biên soạn bởi thầy giáo Lê Quang Xe, tuyển tập bộ đề phát triển đề tham khảo kỳ thi tốt nghiệp THPT năm 2022 môn Toán của Bộ Giáo dục và Đào tạo.

Các đề được xây dựng dựa trên ma trận đề tham khảo kỳ thi tốt nghiệp THPT năm 2022 môn Toán với các câu hỏi và bài toán có nội dung tương tự và độ khó tương đương, có đáp án và lời giải chi tiết.

Nội dung

BI–N SO„N: L– QUANG XET i Li»u l÷u h nh nëi bë BË — THAM KHƒO N‹M 2022MÆN TONMÆN TONN‹M HÅC: 2021 - 2022N‹M HÅC: 2021 - 2022MÖC LÖC· sè 11· sè 2 16· sè 3 32· sè 4 50· sè 5 66· sè 6 82· sè 7 98· sè 8 114· sè 9 130· sè 10 146· sè 11 162· sè 12 177· sè 13 194· sè 14 210· sè 15 226· sè 16 242· sè 17 258· sè 18 277· sè 19 291i/ 305 i/ 305 pGV: L¶ Quang Xe  Ô0967.003.131MÖC LÖCNÌI N€O CÂ Þ CH, NÌI  C CON ×ÍNGiiii/ 305 ii/ 305 pGV: L¶ Quang Xe  Ô0967.003.131PHT TRIšN — THAM KHƒO 2022 NÌI N€O CÂ Þ CH, NÌI  C CON ×ÍNG1BË GIO DÖC & €O T„OTR×ÍNG THPT NGUY™N T‡T TH€NH GV: L– QUANG XE - 0967.003.131 — SÈ 1 PHT TRIšN — THAM KHƒO 2022N‹M HÅC 2021 - 2022Mæn:To¡nThíi gian l m b i: 90 phót — THAM KHƒO PTMH2022dC¥u 1. Mæun cõa sè phùc z= 3 �ib¬ng A 8. B p10. C 10. D 2p 2. ÊLíi gi£i.Ta câ z= 3 �i) j zj = p 10.Chån ¡p ¡n B dC¥u 2. Trong khæng gian Oxyz, m°t c¦u (S ) : ( x+ 1) 2+ ( y� 2) 2+ z2= 9 câ b¡n k½nh b¬ng A 3. B 81. C 9. D 6.ÊLíi gi£i.Ta câ R2= 9 n¶n b¡n k½nh m°t c¦u R= 3 .Chån ¡p ¡n A dC¥u 3. iºm n o d÷îi ¥y thuëc ç thà cõa h m sè y= x4+ x2� 2? A iºmP(� 1; �1) . B iºmN(� 1; �2) . C iºmM(� 1; 0) . D iºmQ(� 1; 1) . ÊLíi gi£i.Thay iºm M(� 1; 0) v o h m sè y= x4+ x2� 2(thäa m¢n).Chån ¡p ¡n C dC¥u 4. Thº t½ch Vcõa khèi c¦u b¡n k½nh r÷ñc t½nh theo cæng thùc n o d÷îi ¥y? A V= 1 3r3. B V= 2 r3. C V= 4 r3. D V= 4 3r3. ÊLíi gi£i.Thº t½ch khèi c¦u câ b¡n k½nh rl  V= 4 3r3.Chån ¡p ¡n D dC¥u 5. Tr¶n kho£ng (0; +1), hå nguy¶n h m cõa h m sè f(x ) = x3 2l  A Zf(x )d x= 3 2x1 2+ C. B Zf(x )d x= 5 2x2 5+ C. C Zf(x )d x= 2 5x5 2+ C. D Zf(x )d x= 2 3x1 2+ C. ÊLíi gi£i.Hå nguy¶n h m cõa h m sè f(x ) = x3 2l  Zf(x ) d x= Zx3 2dx = 2 5x5 2+ C.Chån ¡p ¡n C 1/ 305 1/ 305 pGV: L¶ Quang Xe  Ô0967.003.131— SÈ 1 NÌI N€O CÂ Þ CH, NÌI  C CON ×ÍNG2dC¥u 6. Cho h m sè y= f(x ) câ b£ng x²t d§u cõa ¤o h m nh÷ sau xf0( x ) �1�2 0 1 4 +1 �0 +0 �0 +0 �Sè iºm cüc trà cõa h m sè ¢ cho l A 3. B 2. C 4. D 5.ÊLíi gi£i.Düa v o b£ng x²t d§u ¤o h m nhªn th§y f0( x ) êi d§u qua c¡c gi¡ trà x= �2, x = 0 ,x = 1 ,x = 4 .Vªy h m sè câ 4iºm cüc trà.Chån ¡p ¡n C dC¥u 7. Tªp nghi»m cõa b§t ph÷ìng tr¼nh 2x> 6l  A (log26; +1). B (�1 ; 3). C (3; +1). D (�1 ; log26). ÊLíi gi£i.Ta câ 2x> 6, x > log26.Vªy tªp nghi»m cõa b§t ph÷ìng tr¼nh l  S= (log26; +1).Chån ¡p ¡n A dC¥u 8. Cho khèi châp câ di»n t½ch ¡y B= 7 v  chi·u cao h= 6 . Thº t½ch cõa khèi châp ¢cho b¬ng A 42. B 126. C 14. D 56.ÊLíi gi£i.Thº t½ch cõa khèi châp V= 1 3hB=1 36 7 = 14 .Chån ¡p ¡n C dC¥u 9. Tªp x¡c ành cõa h m sè y= xp 2l  A R. B Rnf 0g . C (0; +1). D (2; +1). ÊLíi gi£i.H m sè y= xp 2x¡c ành khi v  ch¿ khi x >0.Vªy D= (0; + 1).Chån ¡p ¡n C dC¥u 10. Nghi»m cõa ph÷ìng tr¼nh log2(x + 4) = 3 l  A x= 5 . B x= 4 . C x= 2 . D x= 12 . ÊLíi gi£i.Ta câ log2(x + 4) = 3 ,¨x + 4 >0x + 4 = 2 3, ¨x > �4x = 4 ,x= 4 .Vªy x= 4 l  nghi»m cõa ph÷ìng tr¼nh.Chån ¡p ¡n B 2/ 305 2/ 305 pGV: L¶ Quang Xe  Ô0967.003.131PHT TRIšN — THAM KHƒO 2022 NÌI N€O CÂ Þ CH, NÌI  C CON ×ÍNG3dC¥u 11. N¸u5Z2 f(x )d x= 3 v 5Z2 g(x )d x= �2 th¼ 5Z2 [f (x ) + g(x )]d xb¬ng A 5. B �5. C 1. D 3.ÊLíi gi£i.Ta câ 5Z2 [f (x ) + g(x )] d x= 5Z2 f(x ) d x+ 5Z2 g(x ) d x= 3 + ( �2) = 1 .Chån ¡p ¡n C dC¥u 12. Cho sè phùc z= 3 �2i, khi â 2z b¬ng A 6� 2i. B 6� 4i. C 3� 4i. D �6 + 4 i. ÊLíi gi£i.Ta câ 2z = 2 (3 �2i) = 6 �4i.Chån ¡p ¡n B dC¥u 13. Trong khæng gian Oxyz, m°t ph¯ng (P ) : 2 x� 3y + 4 z� 1 = 0 câ mët vectì ph¡ptuy¸n l  A #n 4 = (�1; 2; �3) . B #n 3 = (�3; 4; �1) . C #n 2 = (2;�3; 4) . D #n 1 = (2; 3; 4). ÊLíi gi£i.M°t ph¯ng (P ) : 2 x� 3y + 4 z� 1 = 0 câ mët vectì ph¡p tuy¸n l  #n 2 = (2;�3; 4) .Chån ¡p ¡n C dC¥u 14. Trong khæng gian Oxyz, cho hai vectì #u = (1; 3; �2) v  #v = (2; 1; �1) . Tåa ë cõavectì #u � #v l  A (3; 4;�3) . B (� 1; 2; �3) . C (� 1; 2; �1) . D (1;�2; 1) . ÊLíi gi£i.Ta câ #u � #v = ( �1; 2; �1) .Chån ¡p ¡n C dC¥u 15. Tr¶n m°t ph¯ng tåa ë, cho M(2; 3) l  iºm biºu di¹n cõa sè phùc z. Ph¦n thüc cõaz b¬ng A 2. B 3. C �3. D �2. ÊLíi gi£i.V¼ M(2; 3) l  iºm biºu di¹n cõa sè phùc zn¶n z= 2 + 3 i.Vªy ph¦n tü cõa sè phùc zl  2.Chån ¡p ¡n A dC¥u 16. Ti»m cªn ùng cõa ç thà h m sè y= 3x + 2 x� 2 l  ÷íng th¯ng câ ph÷ìng tr¼nh A x= 2 . B x= �1. C x= 3 . D x= �2. ÊLíi gi£i.3/ 305 3/ 305 pGV: L¶ Quang Xe  Ô0967.003.131— SÈ 1 NÌI N€O CÂ Þ CH, NÌI  C CON ×ÍNG4Ta câlimx ! 2 3x + 2 x� 2 =1 n¶n ç thà h m sè nhªn ÷íng th¯ng x= 2 l m ti»m cªn ùng.Chån ¡p ¡n A dC¥u 17. Vîi måi sè thüc ad÷ìng, log2a 2b¬ng A 12log2a. B log2a+ 1 . C log2a� 1. D log2a� 2. ÊLíi gi£i.Ta câ log2a 2= log2a� log22 = log2a� 1.Chån ¡p ¡n C dC¥u 18.H m sè n o d÷îi ¥y câ ç thà nh÷ ÷íng cong trong h¼nhb¶n? A y= x4� 2x 2� 1. B y= x+ 1 x� 1. C y= x3� 3x � 1. D y= x2+ x� 1. xyOÊLíi gi£i.Düa v o h¼nh d¡ng ç thà ta th§y r¬ng ÷íng cong ð h¼nh v³ l  ç thà cõa h m sè bªc ba, do â tachån ÷ñc h m sè y= x3� 3x � 1.Chån ¡p ¡n C dC¥u 19. Trong khæng gian Oxyz, ÷íng th¯ng d:8><>: x= 1 + 2 ty = 2 �2tz = �3� 3t i qua iºm n o d÷îi¥y? A iºmQ(2; 2; 3) . B iºmN(2; �2; �3) . C i·mM(1; 2; �3) . D iºmP(1; 2; 3) . ÊLíi gi£i.D¹ th§y r¬ng ÷íng th¯ng dluæn i qua iºm M(1; 2; �3) .Chån ¡p ¡n C dC¥u 20. Vîinl  sè nguy¶n d÷ìng, cæng thùc n o d÷îi ¥y óng? A Pn =n!. B Pn =n� 1. C Pn = (n� 1)! . D Pn =n. ÊLíi gi£i.Sè ho¡n và cõa nph¦n tû l  Pn =n!.Chån ¡p ¡n A 4/ 305 4/ 305 pGV: L¶ Quang Xe  Ô0967.003.131PHT TRIšN — THAM KHƒO 2022 NÌI N€O CÂ Þ CH, NÌI  C CON ×ÍNG5dC¥u 21. Cho khèi l«ng trö câ di»n t½ch ¡y Bv  chi·u cao h. Thº t½ch Vcõa khèi l«ng trö¢ cho ÷ñc t½nh theo cæng thùc n o d÷îi ¥y? A V= 1 3Bh. B V= 4 3Bh. C V= 6 Bh. D V=Bh . ÊLíi gi£i.Thº t½ch Vcõa khèi l«ng trö câ di»n t½ch ¡y Bv  chi·u cao hl  V=Bh .Chån ¡p ¡n D dC¥u 22. Tr¶n kho£ng (0; +1), ¤o h m cõa h m sè y= log2xl  A y0= 1 xln 2 . B y0= ln 2 x. C y0= 1 x. D y0= 1 2x . ÊLíi gi£i.¤o h m cõa h m sè y= log2xtr¶n kho£ng (0; +1)l  y0= 1 xln 2 .Chån ¡p ¡n A dC¥u 23. Cho h m sè y= f(x ) câ b£ng bi¸n thi¶n nh÷ sau xy0 y �1�2 02 +1 �0 +0 �0 ++1 +1 �1 �1 11�1 �1 +1 +1 H m sè ¢ cho çng bi¸n tr¶n kho£ng n o d÷îi ¥y?A (0; +1). B (�1 ;� 2) . C (0; 2). D (� 2; 0) . ÊLíi gi£i.H m sè ¢ cho çng bi¸n tr¶n kho£ng (� 2; 0) .Chån ¡p ¡n D dC¥u 24. Cho h¼nh trö câ b¡n k½nh ¡y rv  ë d i ÷íng sinhl. Di»n t½ch xung quanh Sxq cõah¼nh trö ¢ cho ÷ñc t½nh theo cæng thùc n o d÷îi ¥y? A Sxq = 4rl. B Sxq = 2rl. C Sxq = 3rl. D Sxq =rl . ÊLíi gi£i.Di»n t½ch xung quanh cõa h¼nh trö l  Sxq = 2rl.Chån ¡p ¡n B dC¥u 25. N¸u5Z2 f(x )d x= 2 th¼ 5Z2 3f (x )d xb¬ng A 6. B 3. C 1_8 . D 2.ÊLíi gi£i.Ta câ 5Z2 3f (x )d x= 3 2 = 6 .5/ 305 5/ 305 pGV: L¶ Quang Xe  Ô0967.003.131— SÈ 1 NÌI N€O CÂ Þ CH, NÌI  C CON ×ÍNG6Chån ¡p ¡n A dC¥u 26. Cho c§p sè cëng (un)vîi u1 = 7v  cæng sai d= 4 . Gi¡ trà cõa u2 b¬ng A 11. B 3. C 74. D 28.ÊLíi gi£i.Ta câ u2 =u1 +d= 7 + 4 = 11 .Chån ¡p ¡n A dC¥u 27. Cho h m sè f(x ) = 1 + sin x. Kh¯ng ành n o d÷îi ¥y óng? A Zf(x )d x= x� cos x+ C. B Zf(x )d x= x+ sin x+ C. C Zf(x )d x= x+ cos x+ C. D Zf(x )d x= cos x+ C. ÊLíi gi£i.Ta câ Zf(x )d x= x� cos x+ C.Chån ¡p ¡n A dC¥u 28.Cho h m sè y= ax 4+ bx 2+ c(a; b; c 2R)câ ç thà l  ÷íng congtrong h¼nh b¶n. Gi¡ trà cüc ¤i cõa h m sè ¢ cho b¬ng A 0. B �1. C �3. D 2.xyO�2 2�1 �3 ÊLíi gi£i.Tø b£ng bi¸n thi¶n ta câ gi¡ trà cüc ¤i cõa h m sè ¢ cho b¬ng �1.Chån ¡p ¡n B dC¥u 29. Tr¶n o¤n [1; 5], h m sè y= x+ 4 x¤t gi¡ trà nhä nh§t t¤i iºm A x= 5 . B x= 2 . C x= 1 . D x= 4 . ÊLíi gi£i.Ta câ y0= 1 �4 x2 = x2� 4 x2 = 0,–x = 2 (nhªn)x = �2 (lo¤i).Ì f(1) = 1 + 4 1= 5.Ì f(2) = 2 + 4 2= 4.Ì f(5) = 5 + 4 5=29 5.6/ 305 6/ 305 pGV: L¶ Quang Xe  Ô0967.003.131PHT TRIšN — THAM KHƒO 2022 NÌI N€O CÂ Þ CH, NÌI  C CON ×ÍNG7Vªy gi¡ trà nhä nh§t cõa h m sè l 4t¤i iºm x= 2 .Chån ¡p ¡n B dC¥u 30. H m sè n o d÷îi ¥y nghàch bi¸n tr¶n R? A y= �x3� x. B y= �x4� x2. C y= �x3+ x. D y= x+ 2 x� 1. ÊLíi gi£i.Ta th§y h m sè y= �x3� xcâÌ Tªp x¡c ành D=R.Ì y0= �3x 2� 1< 0; 8x 2 R.Vªy h m sè y= �x3� xnghàch bi¸n tr¶n R.Chån ¡p ¡n A dC¥u 31. Vîi måia; bthäa m¢n log2a� 3 log2b= 2 , kh¯ng ành n o d÷îi ¥y óng? A a= 4 b3. B a= 3 b+ 4 . C a= 3 b+ 2 . D a= 4 b3 . ÊLíi gi£i.Ta câ log2a� 3 log2b= 2 ,log2a b3 = 2,a b3 = 22, a= 4 b3.Chån ¡p ¡n A dC¥u 32.Cho h¼nh hëp ABC D:A0B 0C 0D 0câ t§t c£ c¡c c¤nh b¬ng nhau (thamkh£o h¼nh b¶n). Gâc giúa hai ÷íng th¯ng A0C 0v  BD b¬ng A 90. B 30. C 45. D 60. A B CDA0 B0 C0 D0 ÊLíi gi£i.A 0C 0? BD n¶n gâc giúa A0C 0v  BD b¬ng 90.Chån ¡p ¡n A dC¥u 33. N¸u3Z1 f(x )d x= 2 th¼ 3Z1 [f (x ) + 2 x]d x b¬ng A 20. B 10. C 18. D 12.ÊLíi gi£i.Ta câ 3Z1 [f (x ) + 2 x]d x = 3Z1 f(x )d x+ 3Z1 2x dx = 2 + x2 31 = 2 + (32� 12) = 10 .Chån ¡p ¡n B 7/ 305 7/ 305 pGV: L¶ Quang Xe  Ô0967.003.131— SÈ 1 NÌI N€O CÂ Þ CH, NÌI  C CON ×ÍNG8dC¥u 34. Trong khæng gian Oxyz, cho iºm M(2; �5; 3) v  ÷íng th¯ng d:x 2=y+ 2 4=z� 3 �1 .M°t ph¯ng i qua Mv  vuæng gâc vîi dcâ ph÷ìng tr¼nh l  A 2x � 5y + 3 z� 38 = 0 . B 2x + 4 y� z+ 19 = 0 . C 2x + 4 y� z� 19 = 0 . D 2x + 4 y� z+ 11 = 0 . ÊLíi gi£i.V²c-tì ch¿ ph÷ìng cõa dl  #a = (2; 4; �1) .Ph÷ìng tr¼nh m°t ph¯ng i qua M(2; �5; 3) nhªn #a l m vec-tì ph¡p tuy¸n l 2( x� 2) + 4( y+ 5) �(z � 3) = 0 ,2x + 4 y� z+ 19 = 0 :Chån ¡p ¡n B dC¥u 35. Cho sè phùc zthäa m¢n iz = 5 + 2 i. Ph¦n £o cõa zb¬ng A 5. B 2. C �5. D �2. ÊLíi gi£i.Ta câ z= 5 + 2i i= 2�5i.Suy ra z= 2 + 5 i, do â ph¦n £o cõa zl  5.Chån ¡p ¡n A dC¥u 36.Cho h¼nh l«ng trö ùng ABC:A0B 0C 0câ ¡y ABCl  tam gi¡c vuængc¥n t¤i Bv  AB = 4 (tham kh£o h¼nh b¶n). Kho£ng c¡ch tø C¸nm°t ph¯ng (ABB 0A 0) b¬ng A 2p 2. B 2. C p2. D 4.AB CA0 B0 C0 ÊLíi gi£i.Ta câ ¨C B ?ABC B ?BB 0)C B ?(ABB 0A 0) .Suy ra d(C; (ABB 0A 0)) = C B.M  4ABC vuæng c¥n t¤i Bn¶n C B=AB = 4.Vªy d(C; (ABB 0A 0)) = C B= 4.Chån ¡p ¡n D dC¥u 37. Tø mët hëp chùa 16 qu£ c¦u gçm 7 qu£ m u ä v  9 qu£ m u xanh, l§y ng¨u nhi¶nçng thíi hai qu£. X¡c su§t º l§y ÷ñc hai qu£ câ m u kh¡c nhau b¬ng A 740. B 2140. C 310. D 215. ÊLíi gi£i.8/ 305 8/ 305 pGV: L¶ Quang Xe  Ô0967.003.131PHT TRIšN — THAM KHƒO 2022 NÌI N€O CÂ Þ CH, NÌI  C CON ×ÍNG9GåiAl  bi¸n cè chån ÷ñc hai qu£ m u kh¡c nhauChån 2qu£ tø 16qu£ n¶n khæng gian m¨u jnj= C 212Ì Chån 1qu£ ä tø 7qu£ ä câ C17 c¡ch.Ì Chån 1qu£ xanh tø 9qu£ xanh câ C19 c¡ch.Vªy sè c¡ch chån l  C17 C 19 = 63.X¡c su§t bi¸n cè Al  P= 63 C212 =21 40.Chån ¡p ¡n B dC¥u 38. Trong khæng gian Oxyz, cho ba iºm A(2; �2; 3) ; B(1; 3; 4) v C(3; �1; 5) . ÷íngth¯ng i qua Av  song song vîi BCcâ ph÷ìng tr¼nh l : A x� 2 2=y+ 4 �2 =z� 1 3. B x+ 2 2=y� 2 �4 =z+ 3 1. C x� 2 4=y+ 2 2=z� 3 9. D x� 2 2=y+ 2 �4 =z� 3 1. ÊLíi gi£i.# BC = (2; �4; 1) . ÷íng th¯ng i qua Asong song vîi BCn¶n nhªn # BC l m mët v²ctì ch¿ ph÷ìng.Ph÷ìng tr¼nh ÷íng th¯ng l  x� 2 2=y+ 2 �4 =z� 3 1.Chån ¡p ¡n D dC¥u 39. Câ bao nhi¶u sè nguy¶n xthäa m¢n (4x� 5:2 x+2+ 64) p 2� log(4 x)  0? A 22. B 25. C 23. D 24.ÊLíi gi£i.i·u ki»n x¡c ành: ¨4x > 02 � log(4 x)  0, ¨x > 0log 10(4x)  2, ¨x > 04 x  100 ,¨x > 0x  25 ,0< x 25.V¼ p 2� log(4 x)  0n¶n b§t ph÷ìng tr¼nh · b i ¢ cho t÷ìng ÷ìng vîi4 x� 52x+2+ 64 0, 4x� 20 2x+ 64 0, –2x 42 x 16 ,–x  2x  4So l¤i vîi i·u ki»n x¡c ành, ta câ tªp nghi»m cõa b§t ph÷ìng tr¼nh ¢ cho l  S= (0; 2] [[4; 25] .Vªy câ 22sè nguy¶n xthäa m¢n y¶u c¦u b i to¡n.Chån ¡p ¡n A dC¥u 40. Cho h m sè y= f(x ) câ b£ng bi¸n thi¶n nh÷ sau xy0 y �1�1 2 +1 +0 �0 +�1�1 11�5 �5 +1 +1 Sè nghi»m thüc ph¥n bi»t cõa ph÷ìng tr¼nhf0( f (x )) = 0 l  A 3. B 4. C 5. D 6.ÊLíi gi£i.9/ 305 9/ 305 pGV: L¶ Quang Xe  Ô0967.003.131— SÈ 1 NÌI N€O CÂ Þ CH, NÌI  C CON ×ÍNG10Ta câf0( f (x )) = 0 ,–f (x ) = �1f (x ) = 2Vîi f(x ) = �1, ¥y l  ph÷ìng tr¼nh ho nh ë giao iºm cõa ç thà h m sè y= f(x ) v  ÷íng th¯ngy = �1. Tø b£ng bi¸n thi¶n ta th§y ÷íng th¯ng y= �1 c­t ç thà h m sè y= f(x ) t¤i ba iºmph¥n bi»t, suy ra ph÷ìng tr¼nh f(x ) = �1 câ 3nghi»m thüc ph¥n bi»t.Vîi f(x ) = 2 , ¥y l  ph÷ìng tr¼nh ho nh ë giao iºm cõa ç thà h m sè y= f(x ) v  ÷íng th¯ngy = 2 . Tø b£ng bi¸n thi¶n ta th§y ÷íng th¯ng y= 2 c­t ç thà h m sè y= f(x ) t¤i mët iºm duynh§t, suy ra ph÷ìng tr¼nh f(x ) = 2 câ1nghi»m thüc (nghi»m n y kh¡c 3nghi»m cõa ph÷ìng tr¼nhf (x ) = 1 ).Vªy ph÷ìng tr¼nh f0( f (x )) = 0 câ4nghi»m thüc ph¥n bi»t.Chån ¡p ¡n B dC¥u 41. Cho h m sè y= f(x ) câ ¤o h m l  f0( x ) = 12 x2+ 2 ;8 x 2 R v  f(1) = 3 . Bi¸tF(x )l  nguy¶n h m cõa f(x ) thäa m¢n F(0) = 2 , khi â F(1) b¬ng A �3. B 1. C 2. D 7.ÊLíi gi£i.Ta câ f(x ) = Zf0( x ) d x= 4 x3+ 2 x+ C1. V¼f(1) = 3 n¶nC1=�3.Khi â f(x ) = 4 x3+ 2 x� 3.Ta câ F(x ) = Zf(x ) d x= x4+ x2� 3x + C2. V¼F(0) = 2 n¶nC2= 2.Khi â F(x ) = x4+ x2� 3x + 2 .Vªy F(1) = 1 .Chån ¡p ¡n B dC¥u 42. Cho khèi châp ·u S:ABC DcâAC = 4a, hai m°t ph¯ng (S AB )v  (S C D )vuænggâc vîi nhau. Thº t½ch cõa khèi châp ¢ cho b¬ng A 16p 23a3. B 8p 23a3. C 16a3. D 163a3. ÊLíi gi£i.Ta câ Sl  iºm chung cõa hai m°t ph¯ng (S AB )v  (S C D ).M°t kh¡c ABC D n¶n giao tuy¸n cõa (S AB )v  (S C D )l  ÷íng th¯ng dqua iºm Sv  song song vîi AB,C D .Gåi Ol  t¥m h¼nh vuæng suy ra S O?(ABC D ).Gåi Il  trung iºm AB,J l  trung iºm C D. Khi âÌ S I ?AB )S I ?d.Ì S J ?C D )S J ?d.Suy ra gâc giúa (S AB )v  (S C D )l  ÔI S J = 90 B AC DO SIJTa câAD=AC p2= 2 p 2a .V¼ 4I S J vuæng t¤i Sn¶n S O=1 2I J=1 2AD=p 2a .Thº t½ch S:ABC D l V= 1 3S O SABC D =1 3p 2a 8a 2= 8p 23a3.Chån ¡p ¡n B 10/305 10/305 pGV: L¶ Quang Xe  Ô0967.003.131PHT TRIšN — THAM KHƒO 2022 NÌI N€O CÂ Þ CH, NÌI  C CON ×ÍNG11dC¥u 43. Tr¶n tªp hñp c¡c sè phùc, x²t ph÷ìng tr¼nh z2� 2mz + 8m�12 = 0 (m l  tham sèthüc). Câ bao nhi¶u gi¡ trà nguy¶n cõa mº ph÷ìng tr¼nh â câ hai nghi»m ph¥n bi»t z1; z2thäam¢n jz1j= jz2j? A 5. B 6. C 3. D 4.ÊLíi gi£i.Ta câ 0= m2� 8m + 12 .Ì N¸u 0> 0th¼ ph÷ìng tr¼nh câ hai nghi¶m thüc. Khi â, jz1j= jz2j ,z1 =�z2 ,z1 +z2 =0 , m= 0 (thäa m¢n).Ì N¸u 0< 0, th¼ ph÷ìng tr¼nh câ hai nghi»m phùc. Khi â, l  hai sè phùc li¶n hñp n¶n ta luæncâ jz1j= jz2jhay m2� 8m + 12 <0, 2< m < 6luæn thäa m¢n.Vªy câ 4gi¡ trà nguy¶n cõa tham sè thäa m¢n.Chån ¡p ¡n D dC¥u 44. GåiSl  tªp hñp t§t c£ c¡c sè phùc zsao cho sè phùc w= 1 jz j � zcâ ph¦n thüc b¬ng1 8. X²t c¡c sè phùcz1; z22Sthäa m¢n jz1 �z2j= 2 , gi¡ trà lîn nh§t cõa P=jz1 �5ij2� j z2 �5ij2b¬ng A 16. B 20. C 10. D 32.ÊLíi gi£i.Gåi z= x+ yi (x; y 2R), i·u ki»n jz j � z6= 0 ( ); z1 =x1 +y1i;z 2 =x2 +y2i.Ta câ w= 1 €p x2+ y2� xŠ� yi = €p x2+ y2� xŠ+ yi €p x2+ y2� xŠ2+ y2.Theo ·, ta câ p x2+ y2� x 2 (x2+ y2) � 2x p x2+ y2 = 1 8, 8€p x2+ y2� xŠ= 2 x2+ 2 y2� 2x p x2+ y2, 4€p x2+ y2� xŠ= p x2+ y2 €p x2+ y2� xŠ, €p x2+ y2� xŠ €p x2+ y2� 4Š= 0, "p x2+ y2= 4p x2+ y2� x= 0 : xyABy1 y2 OTr÷íng hñp 1:p x2+ y2� x= 0 ,¨x  0y = 0 (khæng thäa m¢n i·u ki»n).Tr÷íng hñp 2: p x2+ y2= 4 ,x2+ y2= 16) x21 +y21 = 16v x22 +y22 = 16.Ta câ jz1 �z2j= 2 ,(x1 �x2)2+ ( y1 �y2)2= 4 ,(y1 �y2)2= 4 �(x1 �x2)2.Khi â P=x21 + (y1 �5)2� x22 �(y2 �5)2= �10 (y1 �y2) 10 jy1 �y2j= 10 È 4� (x1 �x2)2 20.D§u  = x£y ra khi v  ch¿ khi x1 =x2 v jy1 �y2j= 2 .Vªy max P= 20 .Chån ¡p ¡n A 11/305 11/305 pGV: L¶ Quang Xe  Ô0967.003.131— SÈ 1 NÌI N€O CÂ Þ CH, NÌI  C CON ×ÍNG12dC¥u 45. Cho h m sè f(x ) = 3 x4+ ax 3+ bx 2+ cx +d(a; b; c; d 2R)câ ba iºm cüc trà l  �2;� 1v  1. Gåi y= g(x ) l  h m sè bªc hai câ ç thà i qua ba iºm cüc trà cõa ç thà h m sè y= f(x ).Di»n t½ch h¼nh ph¯ng giîi h¤n bði hai ÷íng y= f(x ) v  y= g(x ) b¬ng A 50081. B 365. C 2932405. D 2948405. ÊLíi gi£i.Ta câ f0( x ) = 12 x3+ 3 ax2+ 2 bx+c. (1)M°t kh¡c, v¼ y= f(x ) l  h m sè bªc bèn v  câ ba iºm cüc trà �2, � 1, 1 n¶n suy raf 0( x ) = 12( x+ 3)( x+ 1)( x� 1) = 12( x3+ 2 x2� x� 2) = 12 x3+ 24 x2� 12x� 24: (2)Tø (1) v (2), ta câ h» ph÷ìng tr¼nh 8><>: 3a = 242 b = �12c = �24 ,8><>: a= 8b = �6c = �24 :Suy ra f(x ) = 3 x4+ 8 x3� 6x 2� 24x+ d.Ì C¡ch 1:Ta câ f(x ) = f0( x )�1 4x+ 1 6‹� 7x 2� 16x+ d+ 4 .Khi â ç thà i qua ba iºm cüc trà cõa f(x ) l  g(x ) = �7x 2� 16x+ d+ 4 .Do â ta câS= Z1� 2 jf (x ) � g(x )j dx = 1Z� 2 3 x 4+ 8 x3+ x2� 8x � 4 dx = 2948 405:Ì C¡ch 2:X²t ph÷ìng tr¼nh ho nh ë giao iºm cõa f(x ), g(x ) l  f(x ) = g(x ) , f(x ) � g(x ) = 0 .Nhªn x²t r¬ng f(x ) � g(x ) l  h m sè bªc bèn v  theo gi£ thi¸t, ph÷ìng tr¼nh tr¶n câ 3nghi»m� 2, � 1, 1. Khi âf (x ) � g(x ) = 3( x2� 1)( x+ 2)( mx+n)= �3 x 3+ 6 x2� 3x � 6(mx +n)= 3 mx4+ 3 nx3+ 6 mx3+ 6 nx2� 3mx 2� 3nx �6mx �6n= 3 mx4+ 3( n+ 2 m)x 3+ 3(2 n� m)x 2� 3(n+ 2 m)x � 6n:V¼ f(x ) l  h m sè bªc bèn v  g(x ) l  h m sè bªc hai, n¶n ta câ thº çng nh§t h» sè bªc 4 v bªc 3 cõa f(x ) v  f(x ) � g(x ). Suy ra m= 1 v n= 2 3.Khi â f(x ) � g(x ) = ( x+ 2)( x2� 1)(3 x+ 2) .Do âS= 1Z� 2 jf (x ) � g(x )j dx = 1Z� 2 ( x + 2)( x2� 1)(3 x+ 2) dx = 2948 405:Chån ¡p ¡n D dC¥u 46. Trong khæng gian Oxyz, cho iºm A(� 4; �3; 3) v  m°t ph¯ng (P ) : x+ y+ z= 0 .÷íng th¯ng i qua A, c­t tröc Ozv  song song vîi (P )câ ph÷ìng tr¼nh l  A x� 4 4=y� 3 3=z� 3 �7 . B x+ 4 4=y+ 3 3=z� 3 1.12/305 12/305 pGV: L¶ Quang Xe  Ô0967.003.131PHT TRIšN — THAM KHƒO 2022 NÌI N€O CÂ Þ CH, NÌI  C CON ×ÍNG13C x+ 4 �4 =y+ 3 3=z� 3 1. D x+ 8 4=y+ 6 3=z� 10 �7 . ÊLíi gi£i.Gåi dl  ÷íng th¯ng thäa · b i. °t M(0; 0; m) = d\ Oz .- M°t ph¯ng (P ) câ VTPT l  #n = (1; 1; 1) , ÷íng th¯ng dcâVTCP l  #u = # AM = (4; 3; m�3).- V¼ d (P )) #u ? #n , #u #n = 0 ,4 + 3 + m�3 = 0 ,m =�4.- d câ VTCP l  #u = (4; 3; �7) n¶n lo¤i ÷ñc c¡c ph÷ìng ¡nx + 4 4=y+ 3 3=z� 3 1v x+ 4 �4 =y+ 3 3=z� 3 1.- ÷íng th¯ng dqua A(� 4; �3; 3) v  câ VTCP #u = (4; 3; �7)n¶n dcâ PTCT l : x+ 4 4=y+ 3 3=z� 3 �7 . P dA M#n - V¼di qua iºm N(� 8; �6; 10) n¶nx+ 8 4=y+ 6 3=z� 10 �7 l  ph÷ìng tr¼nh cõad.Chån ¡p ¡n D dC¥u 47. Cho khèi nân ¿nh Scâ b¡n k½nh ¡y b¬ng 2p 3a . Gåi Av  Bl  hai iºm thuëc÷íng trán ¡y sao cho AB= 4a. Bi¸t kho£ng c¡ch tø t¥m cõa ¡y ¸n m°t ph¯ng (S AB )b¬ng2 a , thº t½ch cõa khèi nân ¢ cho b¬ng A 8p 23a3. B 4p 6a 3. C 16p 33a3. D 8p 2a 3. ÊLíi gi£i.Gåi Ol  t¥m ÷íng trán ¡y v  Ml  trung iºm cõa AB.Ta câ S O?(OAB )v  OM ?AB . Düng OH?S M t¤iH.Khi â kho£ng c¡ch tø O¸n m°t ph¯ng (S AB )l  OH = 2a.Ta t½nh ÷ñc OM2= OA 2� AM 2= 12 a2� 4a 2= 8 a2.Tam gi¡c S OMvuæng t¤i Ocâ OH l  ÷íng cao n¶n1 OH2= 1 OS2+ 1 OM2, 1 OS2= 1 OH2� 1 OM2= 1 4a 2 � 1 8a 2 = 1 8a 2:Suy ra OS= 2p 2a . O SAB MHThº t½ch cõa khèi nân ¢ cho l V= 1 3 €2p 3a Š22p 2a = 8 p 2a 3.Chån ¡p ¡n A dC¥u 48. Câ bao nhi¶u sè nguy¶n asao cho ùng vîi méi a, tçn t¤i ½t nh§t bèn sè nguy¶nb 2 (� 12; 12) thäa m¢n 4a2+ b 3b� a+ 65 ? A 4. B 6. C 5. D 7.ÊLíi gi£i.4 a2+ b 3b� a+ 65 ,3b 3a + 654a24b, �3 4‹b+ 65 3a�1 4‹b� 4a23a 0: (1)H m sè f(b ) = �3 4‹b+ 65 3a�1 4‹b� 4a23a.Ta câ f0( b ) = �3 4‹bln 3 4+ 653a�1 4‹bln 1 4<0, 8b.13/305 13/305 pGV: L¶ Quang Xe  Ô0967.003.131— SÈ 1 NÌI N€O CÂ Þ CH, NÌI  C CON ×ÍNG14B£ng bi¸n thi¶nxf0( b ) f(b ) �1 a+1 �0 �+1 +1 �4a23a �4a23a y= 0 Ta ÷ñc tªp nghi»mS= ( �1 ; ].S chùa ½t nh§t 4sè nguy¶n tè b2 (� 12; 12) , f�11;�10; �9; �8g  (�1 ; ], f(� 8) 0, �4 3‹8+ 65 3a48� 4a23a 0, a2 f� 3;�2; : : : ; 3g(TABLE �5! 5).Chån ¡p ¡n D dC¥u 49. Trong khæng gian Oxyz, cho m°t c¦u (S ) : ( x� 4)2+ ( y+ 3) 2+ ( z+ 6) 2= 50 v ÷íng th¯ng d:x 2=y+ 2 4=z� 3 �1 . Câ bao nhi¶u iºmMthuëc tröc ho nh, vîi ho nh ë l  sènguy¶n, m  tø Mk´ ÷ñc ¸n (S ) hai ti¸p tuy¸n còng vuæng gâc vîi d? A 29. B 33. C 55. D 28.ÊLíi gi£i.M°t c¦u (S ) câ t¥m I(4; �3; �6) ; R = 5 p 2.Ta câ M2Ox )M(a ; 0; 0) .Gåi (P )l  m°t ph¯ng chùa hai ti¸p tuy¸n tø M¸n (S ). Khi â (P )i qua M(a ; 0; 0) , vuæng gâc vîi÷íng th¯ng d, ph÷ìng tr¼nh m°t ph¯ng (P )l 2( x� a) + 4 y� z= 0 ,2x + 4 y� z� 2a = 0 :Ta câ Ml  iºm n¬m ngo i m°t c¦u, suy raÌ I M > R ,(a � 4)2+ 9 + 36 >50 , (a � 4)2> 5 (1)Ì d (I ;(P )) < R ,j8 � 12 + 6 �2a j p21<5p 2, j 2� 2a j < 5p 42(2)Tø (1) v  (2), suy ra ¨(a � 4)2> 5j 2 � 2a j < 5p 42,8<: a2� 8a + 11 >0a 2� 2a + 1 <350 3,8><>: –a  7a  1� 15 a 17 ,–� 15 a 17  a 17:V¼ a2 Z, suy ra câ 28iºm Mtho£ m¢n.Chån ¡p ¡n D dC¥u 50. Cho h m sè y= f(x ) câ ¤o h m l  f0( x ) = x2+ 10 x;8x 2 R. Câ bao nhi¶u gi¡ trànguy¶n cõa tham sè mº h m sè y= f(x 4� 8x 2+ m)câ óng 9 iºm cüc trà? A 16. B 9. C 15. D 10.ÊLíi gi£i.14/305 14/305 pGV: L¶ Quang Xe  Ô0967.003.131MÖC LÖCNÌI N€O CÂ Þ CH, NÌI  C CON ×ÍNG15Ta câf0( x ) = 0 ,–x = 0x = �10 :y 0= �4 x 3� 16xf 0�x 4� 8x 2+ m= 0 ,–4x 3� 16x= 0f 0�x 4� 8x 2+ m= 0, 26666664 x= 0x = 2x = �2x 4� 8x 2+ m = 0x 4� 8x 2+ m =�10, 26666664 x= 0x = 2x = �2m =�x4+ 8 x2(1)m + 10 = �x4+ 8 x2(2)º h m sè y= f(x 4� 8x 2+ m)câ 9iºm cüc trà th¼ f0(x 4� 8x 2+ m) = 0 ph£i câ 6nghi»m ph¥nbi»t.Suy ra ph÷ìng tr¼nh (1)ph£i câ 2nghi»m v  ph÷ìng tr¼nh (2)ph£i câ 4nghi»m.Ta câ: ¨� m 0� 16 <�m �10 <0, ¨m 0� 10 < m < 6, �10< m 0.Do m2Z n¶n m2 f� 9;�8; : : : ;� 1; 0 g.Vªy câ 10gi¡ trà nguy¶n mthäa m¢n · b i.Chån ¡p ¡n D H˜T15/305 15/305 pGV: L¶ Quang Xe  Ô0967.003.131— SÈ 2 NÌI N€O CÂ Þ CH, NÌI  C CON ×ÍNG16BË GIO DÖC & €O T„OTR×ÍNG THPT NGUY™N T‡T TH€NH GV: L– QUANG XE - 0967.003.131 — SÈ 2 PHT TRIšN — THAM KHƒO 2022N‹M HÅC 2021 - 2022Mæn:To¡nThíi gian l m b i: 90 phót — THAM KHƒO PTMH2022dC¥u 1. T¼m ph¦n £o cõa sè phùc z= 2 �3i. A 2. B 3. C �3. D �3i. ÊLíi gi£i.Ph n £o cõa sè phùc z= 2 �3i l  �3.Chån ¡p ¡n C dC¥u 2. Trong khæng gian Oxyz, cho m°t c¦u (S ) : ( x+ 1) 2+ y2+ ( z� 3)2= 4 . T¼m t¥m Iv b¡n k½nh rcõa m°t c¦u (S ). A I(1; 0; �3) ,r = 4 . B I(� 1; 0; 3) ,r = 2 . C I(� 1; 0; 3) ,r = 4 . D I(1; 0; �3) ,r = 2 . ÊLíi gi£i.M°t c¦u (S ) câ t¥m l  iºm I(� 1; 0; 3) v  b¡n k½nh r= 2 .Chån ¡p ¡n B dC¥u 3. T¥m èi xùng cõa ç thà h m sè y= x� 2 2x � 1l  A I�� 1 2;1 2‹. B I�1 2;1 2‹. C I�1 2;� 1‹. D I�� 1 2; 2 ‹. ÊLíi gi£i.ç thà h m sè y= ax+b cx+dvîiad�bc 6= 0 nhªn iºm I�� d c;a c‹l m t¥m èi xùng. Do â I�1 2;1 2‹l  t¥m èi xùng cõa ç thà h m sè ¢ cho.Chån ¡p ¡n B dC¥u 4. Tªp hñp t¥m c¡c m°t c¦u i qua ba iºm ph¥n bi»t khæng th¯ng h ng l  A mët m°t ph¯ng. B mët ÷íng th¯ng. C mët m°t trö. D mët m°t c¦u.ÊLíi gi£i.Gåi Il  t¥m m°t c¦u i qua ba iºm ph¥n bi»t khæng th¯ng h ng A,B ,C . Suy ra I A=I B =I C ,do â In¬m tr¶n tröc cõa ÷íng trán ngo¤i ti¸p tam gi¡c ABC.Chån ¡p ¡n B dC¥u 5. T¼m nguy¶n h m cõa h m sè f(x ) = sin 2 x. A Zsin 2 xdx = �cos 2x 2+C. B Zsin 2 xdx = �cos 2 x+ C. C Zsin 2 xdx = cos 2x 2+C. D Zsin 2 xdx = 2 cos 2 x+ C. ÊLíi gi£i.16/305 16/305 pGV: L¶ Quang Xe  Ô0967.003.131PHT TRIšN — THAM KHƒO 2022 NÌI N€O CÂ Þ CH, NÌI  C CON ×ÍNG17Ta câZsin 2 xdx = 1 2Zsin 2 xd(2 x) = �cos 2x 2+C.Chån ¡p ¡n A dC¥u 6. Cho h m sè y= f(x ) câ b£ng bi¸n thi¶n nh÷ sau xy0 y �10 3 +1 +0 �0 +�1�1 22�2 �2 +1 +1 T¼m gi¡ trà cüc ¤i v  gi¡ trà cüc tiºu cõa h m sè ¢ cho.A yC = 2v yCT =�2. B yC =�2 v  yCT = 2. C yC = 3v yCT = 0. D yC = 0v yCT = 3. ÊLíi gi£i.Theo b£ng bi¸n thi¶n ta câ yC = 2v yCT =�2.Chån ¡p ¡n A dC¥u 7. Bi¸t r¬ng S l  tªp nghi»m cõa b§t ph÷ìng tr¼nh log(�x2+ 100 x� 2400) <2câ d¤ngS = ( a;b) n f x0g. Gi¡ trà a+ b� x0 b¬ng A 50. B 150. C 30. D 100. ÊLíi gi£i.i·u ki»n �x2+ 100 x� 2400 >0, 40< x < 60:Ta câlog(�x2+ 100 x� 2400) <2, � x2+ 100 x� 2400 <102, (x � 50) 2> 0, x6= 50 :Vªy S= (40; 60) n f50g. Do â a+ b� x0 = 50.Chån ¡p ¡n A dC¥u 8. Cho khèi l«ng trö câ thº t½ch V, di»n t½ch ¡y l  Bv  chi·u cao h. Kh¯ng ành n osau ¥y óng? A V= 1 3Bh. B V=p Bh. C V=Bh . D V= 3 Bh. ÊLíi gi£i.Khèi l«ng trö câ thº t½ch V, di»n t½ch ¡y l  Bv  chi·u cao hth¼ V=Bh .Chån ¡p ¡n C dC¥u 9. T¼m tªp x¡c ành Dcõa h m sè y= ( x2� 5x + 6) �2019. A D= ( �1 ; 2)[(3; + 1). B D= ( �1 ; 2][[3; + 1). C D= (2; 3) . D D=Rn f 2; 3 g. ÊLíi gi£i.H m sè y= ( x2� 5x + 6) �2019x¡c ành khi v  ch¿ khix2� 5x + 6 6= 0 ,¨x 6= 2x 6= 3 :17/305 17/305 pGV: L¶ Quang Xe  Ô0967.003.131— SÈ 2 NÌI N€O CÂ Þ CH, NÌI  C CON ×ÍNG18Vªy tªp x¡c ành cõa h m sèy= ( x2� 5x + 6) �2019l D =Rn f 2; 3 g.Chån ¡p ¡n D dC¥u 10. Ph÷ìng tr¼nh log3(x + 2) = 3 câ nghi»m l  A 5. B 25. C 7. D �3. ÊLíi gi£i.Ta câ log3(x + 2) = 3 ,¨x > �2x + 2 = 27 ,x= 25 .Chån ¡p ¡n B dC¥u 11. Cho3Z1 f(x ) d x= 3 v 3Z1 g(x ) d x= 4 , khi â 3Z1 [4f(x ) � g(x )] d xb¬ng A 16. B 8. C 11. D 19. ÊLíi gi£i.Ta câ 3Z1 [4f(x ) � g(x )] d x= 4 3Z1 f(x ) d x� 3Z1 g(x ) d x= 4 3 � 4 = 8 .Chån ¡p ¡n B dC¥u 12. T¼m sè phùc zthäa m¢n z+ 2 �3i = 3 �2i. A z= 1 + i. B z= 1 �i. C z= 5 �5i. D z= 1 �5i. ÊLíi gi£i.z + 2 �3i = 3 �2i , z= 1 + i.Chån ¡p ¡n A dC¥u 13. Trong khæng gian Oxyzcho hai iºm A(2; 0; �1) ,B (1; 1; 0) v ( ) l  m°t ph¯ng trungtrüc cõa o¤n th¯ng AB. V²c-tì n o sau ¥y l  mët v²c-tì ph¡p tuy¸n cõa ( )? A #n (1; �1; �1) . B #n (1; 1; �1) . C #n (1; �1; 1) . D #n (1; 1; 1) . ÊLíi gi£i.Do ( ) l  m°t ph¯ng trung trüc cõa o¤n th¯ng ABn¶n( ) nhªn # AB (� 1; 1; 1) l m v²c-tì ph¡p tuy¸n.Suy ra #n (1; �1; �1) = �# AB công l  v²c-tì ph¡p tuy¸n cõa ( ).Chån ¡p ¡n A dC¥u 14. Trong khæng gian vîi h» tröc to¤ ë Oxyz;cho hai v²c-tì #u = (2; 3; �1) ;#v =(5; �4; m): T¼m mº #u ? #v : A m= 0 . B m= 4 . C m= 2 . D m=�2 . ÊLíi gi£i.Ta câ #u ? #v , #u #v = 0 ,10�12 �m = 0 ,m=�2:Chån ¡p ¡n D dC¥u 15. Cho sè phùc z= 2 + 5 i. iºm biºu di¹n sè phùc ztrong m°t ph¯ng Oxyl  A (2;�5) . B (5; 2). C (2; 5). D (� 2; 5) .18/305 18/305 pGV: L¶ Quang Xe  Ô0967.003.131PHT TRIšN — THAM KHƒO 2022 NÌI N€O CÂ Þ CH, NÌI  C CON ×ÍNG19ÊLíi gi£i.Sè phùc z= 2 + 5 i÷ñc biºu di¹n bði iºm M(2; 5) trong m°t ph¯ng Oxy.Chån ¡p ¡n C dC¥u 16. Cho h m sè y= f(x ) x¡c ành tr¶n Rv  limx ! +1 f(x ) = a; limx! x0 f(x ) = b. Ti»m cªnngang cõa ç thà h m sè ¢ cho l  ÷íng th¯ng A x= b. B y= b. C x= a. D y= a. ÊLíi gi£i.Ta câ limx ! +1 f(x ) = a) y= al  ti»m cªn ngang cõa ç thà h m sè.Chån ¡p ¡n D dC¥u 17. Cho0< a 6= 1 v b; c > 0thäa m¢n logab= 3 ,logac= �2. T½nh loga(a 3b 2p c). A �18 . B 7. C 10. D 8. ÊLíi gi£i.Ta câloga�a 3b 2p c= log aa3+ log ab2+ log ap c= 3 + 2 logab+ 1 2logac= 3 + 2 3 + 1 2(� 2) = 8 :Chån ¡p ¡n D dC¥u 18.Cho h m sè y= ax 3+ bx 2+ cx +dcâ ç thà nh÷ h¼nh v³ b¶n.M»nh · n o d÷îi ¥y l  óng? A a >0, b < 0, c < 0, d > 0. B a <0, b > 0, c > 0, d < 0. C a <0, b < 0, c > 0, d < 0. D a <0, b > 0, c < 0, d < 0. xyOÊLíi gi£i.+ Tø h¼nh d¡ng ç thà ta suy ra a <0.+ ç thà c­t tröc tung t¤i iºm câ tung ë ¥m n¶n d <0.+ H m sè câ hai iºm cüc trà tr¡i d§u, suy ra ac < 0) c > 0.+ V¼ ho nh ë iºm uèn ¥m n¶n ab > 0) b < 0.Chån ¡p ¡n C dC¥u 19. Trong khæng gian Oxyz, ÷íng th¯ng d: 8><>: x= ty = 1 �tz = 2 + t. ÷íng th¯ngdi qua iºmn o sau ¥y? A K(1; �1; 1) . B H(1; 2; 0) . C E(1; 1; 2) . D F(0; 1; 2) .19/305 19/305 pGV: L¶ Quang Xe  Ô0967.003.131— SÈ 2 NÌI N€O CÂ Þ CH, NÌI  C CON ×ÍNG20ÊLíi gi£i.÷íng th¯ng di qua iºm F(0; 1; 2) .Chån ¡p ¡n D dC¥u 20. Mët a gi¡c lçi câ 15c¤nh th¼ câ bao nhi¶u ÷íng ch²o? A 90. B 210. C 195. D 105. ÊLíi gi£i.Chån 2¿nh tø 15¿nh cõa a gi¡c ¢ cho, ta ÷ñc mët ÷íng ch²o ho°c mët c¤nh cõa a gi¡c.Do â, sè ÷íng ch²o cõa a gi¡c ¢ cho l  C215 �15 = 90 .Chån ¡p ¡n A dC¥u 21. Thº t½ch khèi hëp chú nhªt câ chi·u d i ba k½ch th÷îc l  2cm, 3cm, 4cm l  A 24cm3. B 9cm 3. C 18cm3. D 30cm3. ÊLíi gi£i.Thº t½ch khèi hëp chú nhªt l  V= 2 3 4 = 24 cm3. A BCD0 C0 A0 DB0 Chån ¡p ¡n A dC¥u 22. Cho h m sè f(x ) = log3(2x+ 1) . Gi¡ trà cõa f0(0) b¬ng A 2ln 3. B 0. C 2 ln 3. D 2. ÊLíi gi£i.Ta câ f0( x ) = (2x+ 1) 0 (2x+ 1) ln 3 =2 (2x+ 1) ln 3 )f0(0) = 2 ln 3.Chån ¡p ¡n A dC¥u 23. Cho h m sè y= f(x ) câ b£ng bi¸n thi¶n nh÷ h¼nh v³ d÷îi ¥y xy0 y �1�2 2 +1 +0 �0 +�1�1 3300 +1 +1 H m sè çng bi¸n tr¶n kho£ng n o d÷îi ¥y?A (2; +1). B (� 2; 2) . C (�1 ; 3). D (0; +1).20/305 20/305 pGV: L¶ Quang Xe  Ô0967.003.131PHT TRIšN — THAM KHƒO 2022 NÌI N€O CÂ Þ CH, NÌI  C CON ×ÍNG21ÊLíi gi£i.Düa v o b£ng bi¸n thi¶n ta câ h m sè çng bi¸n tr¶n kho£ng (2; +1).Chån ¡p ¡n A dC¥u 24. Mët khèi trö câ ë d i ÷íng sinh b¬ng 10, bi¸t thº t½ch cõa khèi trö b¬ng 90. T½nhdi»n t½ch xung quanh cõa khèi trö. A 60. B 78. C 81. D 90. ÊLíi gi£i.Gåi R,l l¦n l÷ñt l  b¡n k½nh v  ë d i ÷íng sinh cõa khèi trö. Chi·u cao cõa khèi trö công b¬ng ëd i ÷íng sinh cõa khèi trö.Thº t½ch Vcõa khèi trö l  V=R 2l. Suy ra R= É Vl=É 90 10= 3.Di»n t½ch xung quanh cõa khèi trö l  Sxq = 2Rl = 23 10 = 60 .Chån ¡p ¡n A dC¥u 25. Cho h m sè f(x ) li¶n töc tr¶n Rthäa m¢n 8Z1 f(x ) d x = 9 ,12Z4 f(x ) d x = 3 v 8Z4 f(x ) d x= 5 . T½nh 12Z1 f(x ) d x. A I= 17 . B I= 1 . C I= 11 . D I= 7 . ÊLíi gi£i.Ta câ 4Z1 f(x ) d x= 8Z1 f(x ) d x� 8Z4 f(x ) d x= 9 �5 = 4 :Suy ra 12Z1 f(x ) d x= 4Z1 f(x ) d x+ 12Z4 f(x ) d x= 4 + 3 = 7 :Chån ¡p ¡n D dC¥u 26. Cho c§p sè cëng (un)câ u1 =�5 v  cæng sai d= 3 . Sè 100l  sè h¤ng thù bao nhi¶ucõa c§p sè cëng ¢ cho? A 20. B 36. C 35. D 15. ÊLíi gi£i.Ta câ cæng thùc sè h¤ng cõa c§p sè cëng un =u1 + (n� 1)d.100 l  sè h¤ng thù nkhi v  ch¿ khi 100 =�5 + ( n� 1) 3 , 3n = 108 ,n= 36 .Vªy 100l  sè h¤ng thù 36.Chån ¡p ¡n B dC¥u 27. Hå nguy¶n h m cõa h m sè f(x ) = sin 2 xl  A F(x ) = �1 2cos 2x+ C. B F(x ) = cos 2 x+ C.21/305 21/305 pGV: L¶ Quang Xe  Ô0967.003.131— SÈ 2 NÌI N€O CÂ Þ CH, NÌI  C CON ×ÍNG22C F(x ) = 1 2cos 2x+ C. D F(x ) = �cos 2 x+ C. ÊLíi gi£i.Ta câ Zsin 2 xdx = �cos 2x 2+C:Chån ¡p ¡n A dC¥u 28. Cho h m sè y= f(x ) li¶n töc tr¶n Rv  câ b£ng bi¸n thi¶n nh÷ sau. xy0 y �11 2 +1 +0 �0 +�1�1 22�1 �1 +1 +1 Trong c¡c m»nh · sau, m»nh · n o óng?A H m sè khæng câ cüc trà. B H m sè ¤t cüc tiºu t¤ix= 1 . C H m sè ¤t cüc ¤i t¤ix= 2 . D H m sè ¤t cüc tiºu t¤ix= 2 . ÊLíi gi£i.Tø b£ng bi¸n thi¶n ta câ: H m sè ¤t cüc tiºu t¤i x= 2 .Chån ¡p ¡n D dC¥u 29. Gi¡ trà nhä nh§t cõa h m sè f(x ) = x3+ 3 xtr¶n o¤n [� 1; 2] b¬ng A 4. B �4. C 14. D �2. ÊLíi gi£i.Ta th§y f0( x ) = 3 x2+ 3 >0; 8x 2 [� 1; 2] )min[ � 1;2] f(x ) = f(� 1) = �4.Chån ¡p ¡n B dC¥u 30. Cho h m sè y= x3� 3x 2. M»nh · n o d÷îi ¥y óng? A H m sè nghàch bi¸n tr¶n kho£ng(0; 2). B H m sè nghàch bi¸n tr¶n kho£ng(2; +1). C H m sè çng bi¸n tr¶n kho£ng(0; 2). D H m sè çng bi¸n tr¶n kho£ng(�1 ; 0). ÊLíi gi£i.Ta câ y0= 3 x2� 6x , suy ra y0< 0, x2 (0; 2) v y0> 0, x2 (�1 ; 0)[(2; + 1).Vªy h m sè nghàch bi¸n tr¶n kho£ng (0; 2)v  çng bi¸n tr¶n méi kho£ng (�1 ; 0);(2; + 1).Chån ¡p ¡n C dC¥u 31. X²t t§t c£ c¡c sè thüc d÷ìng av  bthäa m¢n log3a= log27€a2p bŠ. M»nh · n od÷îi ¥y óng? A a= b2. B a3= b. C a= b. D a2= b. ÊLíi gi£i.Ta câlog3a= log27€a2p bŠ, log3a= 1 3log3€a2p bŠ22/305 22/305 pGV: L¶ Quang Xe  Ô0967.003.131PHT TRIšN — THAM KHƒO 2022 NÌI N€O CÂ Þ CH, NÌI  C CON ×ÍNG23,3 log3a= log3€a2p bŠ, log3a3= log 3€a2p bŠ, a3= a2p b, a= p b, a2= b:Chån ¡p ¡n D dC¥u 32. Cho h¼nh châp S:ABC Dcâ ¡y l  h¼nh thang vuæng t¤i Av  Dv  AB = 2a; AD =DC =a, S A ?AB ,S A ?AD ,S A =2a p 33. T½nh cos gâc giúa hai ÷íng th¯ngS Dv BC . A p4228. B p37. C p314. D p4214. ÊLíi gi£i.Gåi Kl  trung iºm cõa AB)tù gi¡c AK C Dl  h¼nh vuæng c¤nh a) AC =DK =ap 2.L¤i câ DK BC l  h¼nh b¼nh h nh suy ra DKBCSuy ra Ù�( S D ;BC ) = Ú�( S D ;DK ) = ÕS DK .Ta câ S D=S K =ap 213.cos ÕS DK =S D2+ DK 2� S K 2 2S D DK =p 4214. SKAD BCChån ¡p ¡n D dC¥u 33. T½ch ph¥n I= 2Z0 (x + 2) 3dx b¬ng A I= 56 . B I= 60 . C I= 240 . D I= 120 . ÊLíi gi£i.Ta câ I= 2Z0 (x + 2) 3dx = (x + 2) 4 4 20 =44� 24 4= 60.Chån ¡p ¡n B dC¥u 34. Trong khæng gian vîi h» tåa ë Oxyz;cho2iºm A(0; 1; 2) ; B(0;�1; 2) :Vi¸t ph÷ìngtr¼nh m°t ph¯ng trung trüc cõa o¤n AB:A z� 2 = 0 . B x� z+ 2 = 0 . C x= 0 . D y= 0 . ÊLíi gi£i.Gåi Ml  trung iºm cõa AB)M(0; 0; 2) :Suy ra (P ) l  m°t ph¯ng trung trüc cõa ABi qua Mv  nhªn # AB = (0; �2; 0) l m v²c-tì ph¡ptuy¸n.) (P ) : y= 0 :Chån ¡p ¡n D 23/305 23/305 pGV: L¶ Quang Xe  Ô0967.003.131— SÈ 2 NÌI N€O CÂ Þ CH, NÌI  C CON ×ÍNG24dC¥u 35.Gåi z1; z2l¦n l÷ñt câ iºm biºu di¹n l Mv Ntr¶n m°t ph¯ng phùc ð h¼nhb¶n. Khi â, ph¦n £o cõa sè phùc z1 z2 l  A 1417. B �1 4. C �5 17. D 12. xyO 3M21N�4 ÊLíi gi£i.Tø h¼nh v³ ta câ z1 = 3 + 2i; z2= 1�4i. Vªy z1 z2 =3 + 2i 1� 4i =� 5 17+14 17i.Chån ¡p ¡n A dC¥u 36. Cho h¼nh châp S:ABC Dcâ ¡yABC D l  h¼nh vuæng c¤nh a, c¤nh b¶n S Avuænggâc vîi m°t ph¯ng ¡y v  S A=a. Kho£ng c¡ch tø ÷íng th¯ng AB¸n m°t ph¯ng (S C D )b¬ng A ap 63. B ap 32. C a. D ap 22. ÊLíi gi£i.Gåi Hl  h¼nh chi¸u vuæng gâc Al¶n S D.Ta câ ¨C D ?S A �v¼ S A ?(ABC D )C D ?AD �v¼ ABC D l  h¼nh vuæng ) C D ?(S AD ).Do â ¨AH ?C D �v¼ C D ?(S AD )AH ?S D )AH ?(S C D ). SHA DB CVªyd (AB; (S C D )) = d ( A;(S C D )) =AH= S AAD pS A2+ AD 2= ap 22.Chån ¡p ¡n D dC¥u 37. Gieo hai çng xu A v  B mët c¡ch ëc lªp vîi nhau. çng xu A ch¸ t¤o c¥n èi,çng xu B ch¸ t¤o khæng c¥n èi n¶n x¡c su§t xu§t hi»n m°t s§p g§p 3 l¦n x¡c su§t xu§t hi»nm°t ngûa. X¡c su§t º gieo hai çng xu hai l¦n th¼ c£ hai çng xu ·u ngûa l  A 116. B 164. C 132. D 14. ÊLíi gi£i.X¡c su§t gieo hai çng xu mët l¦n ·u xu§t hi»n m°t ngûa l  1 21 4=1 8.Do â, x¡c su§t khi gieo hai çng xu hai l¦n ·u xu§t hi»n m°t ngûa l  1 81 8=1 64.Chån ¡p ¡n B dC¥u 38. Trong khæng gian Oxyz, cho iºm M(1; �2; 4) v (P ) : �2x + y� z+ 5 = 0 . ÷íngth¯ng i qua Mv  vuæng gâc vîi m°t ph¯ng (P )câ ph÷ìng tr¼nh ch½nh t­c l 24/305 24/305 pGV: L¶ Quang Xe  Ô0967.003.131PHT TRIšN — THAM KHƒO 2022 NÌI N€O CÂ Þ CH, NÌI  C CON ×ÍNG25A x� 2 1=y+ 1 �2 =z� 1 4. B x+ 1 �2 =y� 2 1=z+ 4 �1 . C x� 1 2=y+ 2 �1 =z� 4 1. D x� 1 �2 =y+ 2 �1 =z� 4 1. ÊLíi gi£i.V²c-tì ph¡p tuy¸n cõa m°t ph¯ng (P )l  #n = ( �2; 1; �1) .V¼ ÷íng th¯ng c¦n t¼m vuæng gâc vîi (P )n¶n v²c-tì ch¿ ph÷ìng cõa nâ l  #u = (2; �1; 1) .Ph÷ìng tr¼nh ch½nh t­c cõa ÷íng th¯ng c¦n t¼m l  x� 1 2=y+ 2 �1 =z� 4 1.Chån ¡p ¡n C dC¥u 39. B§t ph÷ìng tr¼nh 2x+2+ 8 2�x� 33 <0câ bao nhi¶u nghi»m nguy¶n? A Væ sè. B 6. C 7. D 4. ÊLíi gi£i.Ta câ2x+2+ 8 :2 �x� 33 <0 , 422x� 33 2x+ 8 <0, 1 4<2x< 8, � 2< x < 3:Suy ra b§t ph÷ìng tr¼nh câ 4nghi»m nguy¶n S= f� 1;0 ;1 ;2 g .Chån ¡p ¡n D dC¥u 40. Câ bao nhi¶u gi¡ trà nguy¶n cõa tham sè mº ph÷ìng tr¼nh cos 2x= mp 1 + tanxcos 2x câ nghi»m thuëc o¤n h0;  3i? A 3. B 4. C 1. D 2. ÊLíi gi£i.°t t= tan x) t2 [0; p 3].Khi â cos2x = 1 1 +t2 ;sin 2x = t2 1 +t2 v cos 2 x= 1� t2 1 +t2 .Vîi x2 h0;  3i, ta câ cos 2x= mp 1 + tanxcos 2x , m= cos 2x p1 + tanxcos 2x =1� t2 p1 +t.°t g(t) = 1� t2 p1 +t= (1�t) p 1 +t) g0( t) = �3t � 1 2p 1 +t<0;8 t 2 [0; p 3]Ta câ b£ng bi¸n thi¶n tg0( t) g(t) 0 p3�11p1 +p 3(1�p 3)p1 +p 3(1�p 3)Vªyp 1 +p 3(1�p 3)m 1, suy ra câ óng 3gi¡ trà nguy¶n cõa mthäa m¢n y¶u c¦u b i to¡n.Chån ¡p ¡n A 25/305 25/305 pGV: L¶ Quang Xe  Ô0967.003.131— SÈ 2 NÌI N€O CÂ Þ CH, NÌI  C CON ×ÍNG26dC¥u 41. GåiF(x ) l  mët nguy¶n h m cõa h m sè f(x ) = 4 x3� 3x + 2 thäa m¢n F(� 1) = �3 2.Khi â ph÷ìng tr¼nh F(x ) = 2 x+ 1 câ sè nghi»m thüc l  A 0. B 1. C 2. D 3. ÊLíi gi£i.Ta câ F(x ) = Zf(x )d x= Z�4 x 3� 3x + 2 dx = x4� 3 2x2+ 2 x+ C.Do F(� 1) = �3 2n¶n suy ra(� 1) 4� 3 2(� 1) 2+ 2 (� 1) + C=�3 2)C= 1 .Tø â F(x ) = x4� 3 2x2+ 2 x+ 1 . Bði vªyF (x ) = 2 x+ 1 ,x4� 3 2x2+ 2 x+ 1 = 2 x+ 1, x4� 3 2x2= 0, 24 x= 0x = p 62:Do â ph÷ìng tr¼nh F(x ) = 2 x+ 1 câ3nghi»m ph¥n bi»t.Chån ¡p ¡n D dC¥u 42. Cho h¼nh châp S:ABCcâAB = 4a, BC = 5a, C A = 3a; c¡c m°t ph¯ng (S AB ),( S B C ), (S C A )còng t¤o vîi m°t ph¯ng ¡y (ABC )mët gâc b¬ng 60v  h¼nh chi¸u vuæng gâccõa Sl¶n m°t ph¯ng ¡y l  mët iºm thuëc mi·n trong cõa tam gi¡c ABC. T½nh kho£ng c¡ch tøA ¸n mp (S B C ). A 2a p 35. B 3a . C 5a 2. D 6a p 35. ÊLíi gi£i.Gåi Hl  h¼nh chi¸u vuæng gâc cõa cõa Sl¶n mp (ABC ). Trong( ABC )gåi D,E ,F l¦n l÷ñt l  h¼nh chi¸u vuæng gâc cõa Hl¶nc¤nh BC,C A ,AB t÷ìng ùng.Theo · b i ta câ ÕS DH =ÕS E H =ÕS F H = 60) 4 S H D =4 S H E =4S H F )H D =H E =H F m Hð mi·n trong4 ABC n¶nHl  t¥m ÷íng trán nëi ti¸p 4ABC .Câ BC 2= 25 a2= 16 a2+9 a2= AB 2+ AC 2) 4 ABC vuæng t¤i A. SB HDAF CES4 ABC =1 2ABAC = 6a2v  nûa chu vi p= 6 a, do â r= S p=ahay H D =a) S H =H D tan 60 = ap 3) VS:ABC =1 36a 2a p 3 = 2a3p 3.4 S B C câS D ?BC (v¼BC ?S H; BC ?H D ) n¶n S4 S BC =1 2BCS D =1 25 a p a2+ 3 a2= 5 a2.L¤i câ VS:ABC =1 3S4 S BC d ( A; (S B C ))) d (A; (S B C )) =32a 3p 35a 2 =6a p 35.26/305 26/305 pGV: L¶ Quang Xe  Ô0967.003.131PHT TRIšN — THAM KHƒO 2022 NÌI N€O CÂ Þ CH, NÌI  C CON ×ÍNG27Vªy kho£ng c¡ch tøA¸n mp (S B C )b¬ng 6a p 35.Chån ¡p ¡n D dC¥u 43. Bi¸t r¬ng ph÷ìng tr¼nh z2+ bz +c= 0 ( b; c2R)câ mët nghi»m phùc l  z1 = 1 + 2i.Khi â A b+ c= 2 . B b+ c= 3 . C b+ c= 0 . D b+ c= 7 . ÊLíi gi£i.Ph÷ìng tr¼nh bªc hai h» sè thüc câ nghi»m phùc th¼ hai nghi»m â l  li¶n hñp cõa nhau.Ph÷ìng tr¼nh ¢ cho câ nghi»m cán l¤i l  z2 = 1�2i.Khi â, theo Vi-et ta câ z1 +z2 =�b) b= �2.V  z1 z2 =c) c= 5 . Suy ra b+ c= 3 .Chån ¡p ¡n B dC¥u 44. GåiMl  gi¡ trà lîn nh§t cõa 2 m�i�1 , vîiml  sè thüc. M»nh · n o d÷îi ¥yóng? A M2�12 5;5 2‹. B M2�5 2;7 2‹. C M2�0; 12 5‹. D M2�14 5;16 5‹. ÊLíi gi£i.Ta câ 2 m�i�1 = 2� m +i m�i = j2 � m +ij jm �ij =p (2�m)2+ 1 pm2+ 1 =Ê m2� 4m + 5 m2+ 1 :X²t h m sè f(m ) = Ê m2� 4m + 5 m2+ 1 )f0( m ) = 1 2Ê m2+ 1 m2� 4m + 5 4m 2� 8m �4 (m 2+ 1) 2) f0( m ) = 0 )4m 2� 8m �4 = 0 ,–m = 1 + p 2m = 1 �p 2:B£ng bi¸n thi¶n mf0( m ) f(m ) �11� p 2 1 +p 2 +1 +0 �0 +11 p2 + 1p2 + 1p2� 1 p2� 1 11VªyM= maxx2 R f(m ) = f(1 �p 2) =p 2 + 1)M 2�12 5;5 2‹.Chån ¡p ¡n A dC¥u 45.27/305 27/305 pGV: L¶ Quang Xe  Ô0967.003.131— SÈ 2 NÌI N€O CÂ Þ CH, NÌI  C CON ×ÍNG28Cho h m sè a thùc bªc bèny= f(x ) câ ç thà (C ), bi¸t r¬ng( C )i qua iºm A(� 1; 0) . Ti¸p tuy¸n t¤i Acõa ç thà (C )c­t( C )t¤i hai iºm câ ho nh ë l¦n l÷ñt l  0v  2. Bi¸t di»n t½ch h¼nhph¯ng giîi h¤n bði , ç thà (C )v  hai ÷íng th¯ng x= 0 ;x = 2câ di»n t½ch b¬ng 56 5. Di»n t½ch h¼nh ph¯ng giîi h¤n bði, ç thà( C )v  hai ÷íng th¯ng x= �1; x = 0 b¬ng A 25. B 120. C 110. D 15. xyO�1 1 213A BÊLíi gi£i.H m sè y= ax 4+ bx2+ c. TX: D=R.Ta câ y0= 4 ax3+ 2 bx.Ph÷ìng tr¼nh ti¸p tuy¸n cõa ç thà (C )t¤i A(� 1; 0) câ d¤ng y= ( �4a � 2b)( x + 1) .Do ti¸p tuy¸n t¤i Acõa ç thà (C )c­t (C )t¤i hai iºm câ ho nh ë l¦n l÷ñt l  0v  2n¶n ph÷ìngtr¼nh ax4+ bx2+ c= ( �4a � 2b)( x + 1) nhªn ba nghi»m l  x= �1; x = 0 ;x = 2 .Suy ra ¨c= �a� bb = �3a ,¨c= 2 ab = �3a .Vªy (C ) : y= ax 4� 3ax 2+ 2 a= a(x 4� 3x 2+ 2) v  : y= 2 a(x + 1) .Di»n t½ch h¼nh ph¯ng giîi h¤n bði , ç thà (C )v  hai ÷íng th¯ng x= 0 ;x = 2 b¬ng 56 5n¶n2Z0 j2 a (x + 1) �a(x 4� 3x 2+ 2) jd x = 56 5, 2Z0 (2a(x + 1) �a(x 4� 3x 2+ 2))d x= 56 5, a2Z0 (� x4+ 3 x2+ 2 x)d x= 56 5, a�� x5 5+x3+ x2‹ 20 =56 5, a28 5=56 5,a= 2 :Di»n t½ch h¼nh ph¯ng giîi h¤n bði , ç thà (C )v  hai ÷íng th¯ng x= �1; x = 0 l S = 0Z� 1 ja (x 4� 3x 2+ 2) �2a (x + 1) jd x = 0Z� 1 (2x4� 6x 2� 4x ) d x= 2 �x5 5�x3� x2‹ 0� 1 =2 5:C¡ch kh¡c:Ph÷ìng tr¼nh ÷íng th¯ng  :y= x+ 1 .Do v  (C )c­t nhau t¤i c¡c iºm câ ho nh ë �1; 0; 2 n¶n ta câ ph÷ìng tr¼nh a(x + 1) 2x ( x � 2) = 0 .Theo b i ta câ ph÷ìng tr¼nh a2Z0 j( x + 1) 2x (x � 2)jd x = 56 5)a= 2 .Tø â ta ÷ñc S= 0Z� 1 [2(x+ 1) 2x (x � 2)] d x= 2 5.Chån ¡p ¡n A 28/305 28/305 pGV: L¶ Quang Xe  Ô0967.003.131PHT TRIšN — THAM KHƒO 2022 NÌI N€O CÂ Þ CH, NÌI  C CON ×ÍNG29dC¥u 46. Trong khæng gian Oxyz, cho iºm A(1; 2; 3) v  ÷íng th¯ng d: x� 3 2=y� 1 1=z + 7 �2 . ÷íng th¯ng i quaA, vuæng gâc vîi dv  c­t tröc Oxcâ ph÷ìng tr¼nh l  A 8><>: x= �1 + 2 ty = 2 tz = 3 t . B 8><>: x= 1 + ty = 2 + 2 tz = 3 + 2 t. C 8><>: x= �1 + 2 ty = �2tz = t . D 8><>: x= 1 + ty = 2 + 2 tz = 3 + 3 t. ÊLíi gi£i.Gåi l  ÷íng th¯ng c¦n t¼m v  B=  \Ox )B(b ; 0; 0) v # BA = (1 �b; 2; 3) .Do ?d,  qua An¶n # BA #u d = 0,2(1 �b) + 2 �6 = 0 ,b= �1.Tø â qua B(� 1; 0; 0) , câ mët v²c-tì ch¿ ph÷ìng l  # BA = (2; 2; 3) n¶n :8><>: x= �1 + 2 ty = 2 tz = 3 t:Chån ¡p ¡n A dC¥u 47. Cho h¼nh vuæng ABC Dc¤nh1, iºm Ml  trung iºm cõa C D. Cho h¼nh vuæng(t½nh c£ iºm trong cõa nâ) quay quanh tröc l  ÷íng th¯ng AMta ÷ñc mët khèi trán xoay.T½nh thº t½ch khèi trán xoay â. A 7p 10 15. B 7p 5 30. C 7p 2 30. D 7p 2 15. ÊLíi gi£i. C0 KD A BCHB0 MThº t½chVcõa khèi trán xoay l  V=V1 +V2 �V3. Trong âV1 l  thº t½ch cõa khèi nân vîi ÷íngsinh AB, b¡n k½nh ¡y BH,V2 l  thº t½ch cõa khèi nân cöt vîi ÷íng sinhBC, b¡n k½nh ¡y lînBH , b¡n k½nh ¡y nhä C K,V3 l  thº t½ch cõa khèi nân vîi ÷íng sinhM C, b¡n k½nh ¡y C K.Ta câ AH=1 p5;BH =2 p5;C K =1 p5;M K =1 2p 5.Khi âV 1 = 1 3AH H B 2= 4p 5 75;V2 = 1 3(BH 2+ C K 2+ BH C K )H K =14p 5 75;V3 = 1 3M K C K 2= p 5 150:Vªy V= 7p 5 30:Chån ¡p ¡n B dC¥u 48. Sè gi¡ trà nguy¶n cõa tham sè mº b§t ph÷ìng tr¼nh ln (x2+ 2 x+ m)> 2 ln(2 x� 1)chùa óng 2sè nguy¶n l  A 4. B 8. C 3. D 9.29/305 29/305 pGV: L¶ Quang Xe  Ô0967.003.131— SÈ 2 NÌI N€O CÂ Þ CH, NÌI  C CON ×ÍNG30ÊLíi gi£i.Ta câln�x 2+ 2 x+ m> 2 ln(2 x� 1), 8<: ln�x 2+ 2 x+ m> ln(2 x� 1)2x > 1 2, 8<: x2+ 2 x+ m > (2x� 1)2x > 1 2, 8<: 3x 2� 6x + 1 < m (1)x > 1 2:X²t h m sè y= 3 x2� 6x + 1 < m vîix > 1 2câ b£ng bi¸n thi¶n xy 12 1 2 3 +1 �5 4�2 101 +1 º b§t ph÷ìng tr¼nh(1)câ nghi»m th¼ m >�2, khi â (1)luæn câ mët nghi»m nguy¶n x= 1 .Do â, º (1)câ óng 2nghi»m nguy¶n th¼ 2nghi»m nguy¶n â l  x= 1 v x= 2 . Khi â 2 m 10.Vªy câ 9gi¡ trà nguy¶n d÷ìng cõa tham sè mº b§t ph÷ìng tr¼nh ¢ cho chùa óng 2sè nguy¶n.Chån ¡p ¡n D dC¥u 49.Cho cèc n÷îc nh÷ h¼nh v³. Ph¦n tr¶n l  h¼nh nân ¿nh S, ¡y l h¼nh trán t¥m Ob¡n b½nh R= p 5dm, chi·u cao h= S O =p 7dm. Trong cèc n÷îc ¢ chùa mët l÷ñng n÷îc câ chi·u cao a= 2 dmso vîi ¿nh S. Ng÷íi ta bä v o cèc n÷îc mët vi¶n bi h¼nh c¦u th¼n÷îc d¥ng l¶n vøa phõ k½n qu£ c¦u. H¢y t½nh g¦n óng b¡n k½nhcõa vi¶n bi. A 0;9 dm. B 1;0 dm. C 1;1 dm. D 0;8 dm. SM OIÊLíi gi£i.30/305 30/305 pGV: L¶ Quang Xe  Ô0967.003.131MÖC LÖCNÌI N€O CÂ Þ CH, NÌI  C CON ×ÍNG31°trl  b¡n k½nh vi¶n bi, V1 l  thº t½ch n÷îc ban ¦u;V2 l  thºt½ch cõa vi¶n bi, Vl  têng thº t½ch cõa l÷ñng n÷îc v  vi¶n bi. °t = ÕOS M , ta câ tan = p 5p7,sin = p 5p12,cos = p 7p12. Suy raS D =I D +S I = r sin +r= r(1 + sin ) sin ;DX =S I tan = r(1 + sin ) cos :Do â SM OIXC DV=  3DX 2S D =r3(1 + sin )3 3 sin cos 2 :Gåi r1 l  b¡n k½nh ÷íng trán ¡y cõa l÷ñng n÷îc ban ¦u, ta câr1 =atan = 2 tan n¶nV 1 =  3r21 a = 8 tan 2 3:Ta câ ph÷ìng tr¼nh V=V1 +V2) r3(1 + sin )3 3 sin cos 2 =8 tan 2 3+4r 3 3:Gi£i ph÷ìng tr¼nh, ta ÷ñc r= 3Ê 20065 + 54p 150;9 .Chån ¡p ¡n A dC¥u 50. Cho h m sè f(x ) = ax4+ bx2+ cvîi a > 0, c > 2018 v a+ b+ c < 2018 . Sè iºmcüc trà cõa h m sè y= jf (x ) � 2018 jl  A 1. B 3. C 5. D 7. ÊLíi gi£i.Tø gi£ thi¸t a >0, c > 2018 v a+ b+ c < 2018 suy ra b <0n¶n ab < 0.Vªy h m sè f(x ) = ax4+ bx 2+ ccâ 3 cüc trà trong â A(0; c), B ‚�É �b 2a ;�  4a Œ,C ‚É �b 2a ;�  4a Œ.Khi â h m sè y= f(x ) � 2018 câ ba iºm cüc trà. (1)A 1(0;c� 2018) ; B1‚�É �b 2a ;�  4a �2018 Œ; C 1‚É �b 2a ;�  4a �2018 Œ:Ta câ c� 2018 >0n¶n A1n¬m b¶n tr¶n tröc ho nh.Ta câ f(1) = a+ b+ c� 2018 <0n¶n B1; C1n¬m b¶n d÷îi tröc ho nh.Vªy ç thà h m sè y= f(x ) � 2018 c­tOx t¤i4iºm ph¥n bi»t. (2)Tø (1) v (2) suy ra h m sè y= jf (x ) � 2018 jcâ 7iºm cüc trà.Chån ¡p ¡n D H˜T31/305 31/305 pGV: L¶ Quang Xe  Ô0967.003.131— SÈ 3 NÌI N€O CÂ Þ CH, NÌI  C CON ×ÍNG32BË GIO DÖC & €O T„OTR×ÍNG THPT NGUY™N T‡T TH€NH GV: L– QUANG XE - 0967.003.131 — SÈ 3 PHT TRIšN — THAM KHƒO 2022N‹M HÅC 2021 - 2022Mæn:To¡nThíi gian l m b i: 90 phót — THAM KHƒO PTMH2022dC¥u 1. C¡c sè thüc x; ytho£ m¢n x+ iy = 3 �4i vîi il  ìn và £o l  A ¨x = 3y = �4. B ¨x = �4y = 3 . C ¨x = 3y = 4 . D ¨x = 4y = 3 . ÊLíi gi£i.x + iy = 3 �4i , ¨x = 3y = �4:Chån ¡p ¡n A dC¥u 2. Trong khæng gian Oxyz, t¥m cõa m°t c¦u (S ) : x2+ y2+ z2� 4x + 2 y+ 6 z� 2 = 0 câtåa ë l  A A(� 4; 2; 6) . B C(4; �2; �6) . C B(� 2; 1; 3) . D D(2; �1; �3) . ÊLíi gi£i.T¥m cõa m°t c¦u (S ) : x2+ y2+ z2� 4x + 2 y+ 6 z� 2 = 0 câ tåa ë l  (2;�1; �3) .Chån ¡p ¡n D dC¥u 3. ç thà h m sè y= x3+ 3 x2� 2nhªn : A Tröc tung l m tröc èi xùng. B Gèc tåa ë O l m t¥m èi xùng.C iºmI(� 1; 0) l m t¥m èi xùng. D ÷íng th¯ngx= 1 l m tröc èi xùng. ÊLíi gi£i.Ta câ: y0= 3 x2+ 6 x= 0y 00= 6 x+ 6 = 0 ,x= �1) y= 0 .H m sè y= x3+ 3 x2� 2l  h m a thùc bªc ba n¶n nhªn iºm I(� 1; 0) l m t¥m èi xùng.Chån ¡p ¡n C dC¥u 4. Khèi c¦u b¡n k½nh R= 2 acâ thº t½ch l  A 8a 3 3. B 16a 2. C 32a 3 3. D 6a 3. ÊLíi gi£i.V = 4 3R3= 4 3(2 a)3= 32a 3 3.Chån ¡p ¡n C dC¥u 5. Hå nguy¶n h m cõa h m sè f(x ) = x2l  A x3 2+C. B x3+ C. C x3 3+C. D 3x 3+ C.32/305 32/305 pGV: L¶ Quang Xe  Ô0967.003.131PHT TRIšN — THAM KHƒO 2022 NÌI N€O CÂ Þ CH, NÌI  C CON ×ÍNG33ÊLíi gi£i.Ta câ Zf(x ) d x= Zx2dx = x3 3+C.Chån ¡p ¡n C dC¥u 6. Cho ç thà h m sè y= f(x ) nh÷ h¼nh v³ xyO 1�1 �5 2Sè iºm cüc trà cõa h m sè l A 2. B 3. C 4. D 5. ÊLíi gi£i.Nh¼n h¼nh v³ ta th§y ç thà h m sè câ 3cüc trà.Chån ¡p ¡n B dC¥u 7. Tªp nghi»m cõa b§t ph÷ìng tr¼nh 32x � 1> 27 l  A �1 2; +1‹. B (2; +1). C (3; +1). D �1 3; +1‹. ÊLíi gi£i.Ta câ 32x�1> 27 , 2x � 1> 3, x > 2.Vªy tªp nghi»m cõa b§t ph÷ìng tr¼nh ¢ cho l  (2; +1).Chån ¡p ¡n B dC¥u 8. Cho h¼nh l«ng trö ùng câ di»n t½ch ¡y l  3a 2, ë d i c¤nh b¶n b¬ng 2a . Thº t½ch cõakhèi l«ng trö b¬ng A 6a 3. B a3. C 3a 3. D 2a 3. ÊLíi gi£i.H¼nh l«ng trö ùng câ ë d i ÷íng cao b¬ng c¤nh b¶n n¶n h= 2 a, di»n t½ch ¡y l  B= 3 a2. Ta câthº t½ch khèi l«ng trö b¬ng V=Bh = 6a3.33/305 33/305 pGV: L¶ Quang Xe  Ô0967.003.131— SÈ 3 NÌI N€O CÂ Þ CH, NÌI  C CON ×ÍNG34Chån ¡p ¡n A dC¥u 9. H m sèy= x+ ( x� 1)ecâ tªp x¡c ành l  A Rn f 1g . B (1; +1). C Rn f 0; 1 g. D Rn f 0g . ÊLíi gi£i.H m sè x¡c ành khi ¨x > 0x � 1> 0,x > 1.Chån ¡p ¡n B dC¥u 10. T¼m nghi»m cõa ph÷ìng tr¼nh 3x� 1= 27 : A x= 9 . B x= 3 . C x= 4 . D x= 10 . ÊLíi gi£i.Ta câ 3x� 1= 27 ,3x� 1= 3 3, x� 1 = 3 ,x= 4Chån ¡p ¡n C dC¥u 11. T½ch ph¥n I=  3Z 4dx sin2x b¬ng A cot 3�cot  4. B cot 3+ cot 4. C �cot  3+ cot 4. D �cot  3�cot  4. ÊLíi gi£i.I =  3Z 4dx sin2x =�cot x  3 4=�cot  3�cot  4= �cot  3+ cot 4:Chån ¡p ¡n C dC¥u 12. Thu gån sè phùc z= i+ (2 �4i) � (3 �2i) v· d¤ng z= a+ bi,(a; b 2R). T½nhS = a� b. A S= 2 . B S= 0 . C S= �1. D S= �2. ÊLíi gi£i.Ta câ z= i+ (2 �4i) � (3 �2i) = �1� i) ¨a = �1b = �1 )S= a� b= �1� (� 1) = 0 .Chån ¡p ¡n B dC¥u 13. Trong khæng gian Oxyz, m°t ph¯ng (P ) : 2 x+ y+ 3 z� 1 = 0 câ mët v²c-tì ph¡ptuy¸n l  A #n 4= (1; 3; 2). B #n 1= (3; 1; 2). C #n 3= (2; 1; 3). D #n 2= (�1; 3; 2) . ÊLíi gi£i.M°t ph¯ng (P ) : 2 x+ y+ 3 z� 1 = 0 câ mët v²c-tì ph¡p tuy¸n l  #n 3 = (2; 1; 3).Chån ¡p ¡n C 34/305 34/305 pGV: L¶ Quang Xe  Ô0967.003.131PHT TRIšN — THAM KHƒO 2022 NÌI N€O CÂ Þ CH, NÌI  C CON ×ÍNG35dC¥u 14. Trong khæng gian vîi h» tröc tåa ë Oxyzcho iºm A(1; 2; 3) . H¼nh chi¸u vuæng gâccõa iºm Atr¶n m°t ph¯ng (Oxy )l  iºm A Q(0; 2; 0) . B M(0; 0; 3) . C P(1; 0; 0) . D .ÊLíi gi£i.H¼nh chi¸u vuæng gâc cõa iºm A(1; 2; 3) tr¶n m°t ph¯ng (Oxy )l  iºm N(1; 2; 0) .Chån ¡p ¡n D dC¥u 15.iºm Mtrong h¼nh v³ b¶n l  biºu di¹n sè phùc �2 �1 1 2 3 xy�1 0123 MA z= 3 + 2 i. B z= 3 �2i. C z= 2 �3i. D z= 2 + 3 i. ÊLíi gi£i.Tröc thüc (tröc Ox) ch¿ sè 2, tröc £o (tröc Oy) ch¿ sè 3. Vªy ¡p ¡n l  z= 2 + 3 i.Chån ¡p ¡n D dC¥u 16. ç thà h m sè y= 2x � 3 x+ 1 câ c¡c ÷íng ti»m cªn ùng v  ngang l¦n l÷ñt l  A x= 2 ,y = 1 . B x= �3, y = �1. C x= �1, y = �1. D x= �1, y = 2 . ÊLíi gi£i.Ta câ limx !1 2x � 3 x+ 1 = 2v  limx ! (� 1)  2x � 3 x+ 1 =1 n¶n ç thà h m sè ¢ cho nhªn ÷íng th¯ng x= �1l m ti»m cªn ùng v  nhªn ÷íng th¯ng y= 2 l m ti»n cªn ngang.Chån ¡p ¡n D dC¥u 17. Vîiav  bl  c¡c sè thüc d÷ìng, a6= 1 . Biºu thùc loga(a 2b ) b¬ng A 2� logab. B 2 + logab. C 1 + 2 logab. D 2 logab. ÊLíi gi£i.Ta câ loga(a 2b ) = logaa2+ log ab= 2 + logab.Chån ¡p ¡n C dC¥u 18.35/305 35/305 pGV: L¶ Quang Xe  Ô0967.003.131— SÈ 3 NÌI N€O CÂ Þ CH, NÌI  C CON ×ÍNG36÷íng cong ð h¼nh b¶n l  ç thà cõa mët trong bèn h m sè n o d÷îi¥y? A y= �x4+ 3 x2+ 1 . B y= �x3+ 3 x2. C y= �x3+ 3 x. D y= �x3� 3x 2. xyO 11ÊLíi gi£i.ç thà h m bªc ba vîi h» sè a <0.ç thà h m sè i qua iºm (3; 0)n¶n l  ç thà h m sè y= �x3+ 3 x2.Chån ¡p ¡n B dC¥u 19. Trong khæng gian Oxyz, ÷íng th¯ng d: x+ 3 1=y� 2 �1 =z� 1 2i qua iºm n od÷îi ¥y? A M(3; 2; 1) . B M(� 3; 2; 1) . C M(3; �2; �1) . D M(1; �1; 2) . ÊLíi gi£i.L¦n l÷ñt thay tåa ë c¡c iºm Mv o ph÷ìng tr¼nh cõa dta th§y iºm M(� 3; 2; 1) thäa m¢n ph÷ìngtr¼nh cõa d.Chån ¡p ¡n B dC¥u 20. Mët lîp håc câ 40håc sinh gçm 15nam v  25nú. Gi¡o vi¶n c¦n chån 3håc sinhtham gia lao ëng. Häi câ bao nhi¶u c¡ch chån kh¡c nhau? A 9880. B 59280. C 2300. D 455. ÊLíi gi£i.Méi c¡ch chån ra 3håc sinh º tham gia lao ëng tø 40håc sinh l  mët tê hñp chªp 3cõa 40ph¦ntû.Vªy câ C340 = 9880c¡ch.Chån ¡p ¡n A dC¥u 21.Cho l«ng trö ùng ABC:A0B 0C 0câ AA 0= a, tam gi¡c ABC·u v câ c¤nh b¬ng a. Thº t½ch cõa khèi l«ng trö ¢ cho l  A a3p 34. B a3p 312. C a3 2. D a3. BB0 CC0 A0 AÊLíi gi£i.Thº t½ch VABC:A 0B 0C 0= SABC A 0A = a2p 34a = a3p 34.Chån ¡p ¡n A 36/305 36/305 pGV: L¶ Quang Xe  Ô0967.003.131PHT TRIšN — THAM KHƒO 2022 NÌI N€O CÂ Þ CH, NÌI  C CON ×ÍNG37dC¥u 22. ¤o h m cõa h m sè y= 2020 xl  A y0= x2020 x� 1. B y0= 2020 xlog 2020 . C y0= 2020 xln 2020 . D y0= 2020x ln 2020. ÊLíi gi£i.Vîi y= 2020 x, ta câ y0= 2020 xln 2020 .Chån ¡p ¡n C dC¥u 23. Cho h m sè y= f(x ) x¡c ành v  li¶n töc tr¶n kho£ng (�1 ; +1), câ b£ng bi¸nthi¶n nh÷ h¼nh sau xy0 y �1�1 1 +1 +0 �0 +�1�1 22�1 �1 +1 +1 M»nh · n o sau ¥y óng?A H m sè çng bi¸n tr¶n kho£ng(�1 ;� 3) . B H m sè nghàch bi¸n tr¶n kho£ng(1; +1). C H m sè çng bi¸n tr¶n kho£ng(� 1; + 1). D H m sè nghàch bi¸n tr¶n kho£ng(�1 ; 1). ÊLíi gi£i.Ì M»nh ·: H m sè çng bi¸n tr¶n kho£ng (�1 ;� 3)  óng.Ì M»nh ·: H m sè nghàch bi¸n tr¶n kho£ng (1; +1) sai.Ì M»nh ·: H m sè çng bi¸n tr¶n kho£ng (� 1; + 1) sai.Ì M»nh ·: H m sè nghàch bi¸n tr¶n kho£ng (�1 ; 1) sai.Chån ¡p ¡n A dC¥u 24. Cho h¼nh vuæng ABC Dc¤nh1. Khi quay h¼nh vuæng ABC Dquanh c¤nh ABta÷ñc mët h¼nh trö, häi h¼nh trö n y câ di»n t½ch to n ph¦n b¬ng bao nhi¶u? A 3 . B 2 . C 2 + 2 . D 4 . ÊLíi gi£i.Khi quay h¼nh vuæng ABC DquanhABta ÷ñc h¼nh trö câ b¡n k½nhr = AD = 1, chi·u cao h= AB = 1.Do â, Stp = 2rh + 2r2= 2 + 2 = 4 . ADCBChån ¡p ¡n D dC¥u 25. Cho h m sè f(x ) li¶n töc tr¶n o¤n [0; 3]. N¸u 3Z0 f(x )d x= 2 th¼ t½ch ph¥n 3Z0 [x �37/305 37/305 pGV: L¶ Quang Xe  Ô0967.003.131— SÈ 3 NÌI N€O CÂ Þ CH, NÌI  C CON ×ÍNG383f (x )]d xcâ gi¡ trà b¬ng A �3. B 3. C 32. D �3 2. ÊLíi gi£i.Ta câ 3Z0 [x � 3f (x )]d x= 3Z0 xdx � 3 3Z0 f(x )d x= 1 2x2 30 �6 = 9 2�6 = �3 2.Chån ¡p ¡n D dC¥u 26. Cho c§p sè cëng (un)gçm c¡c sè h¤ng theo thù tü 2; a; 6; b . Khi â t½ch abb¬ng A 22. B 40. C 12. D 32. ÊLíi gi£i.Theo t½nh ch§t cõa c§p sè cëng: ¨2 + 6 = 2 aa + b= 12 )¨a = 4b = 8 )ab = 32 :Chån ¡p ¡n DdC¥u 27. Ph¡t biºu n o sau ¥y l  óng? A Zexdx = e �x+ C. B Zexdx = �ex+ C. C Zexdx = e x+ C. D Zexdx = �e�x+ C. ÊLíi gi£i.Ta câ Zexdx = e x+ C.Chån ¡p ¡n C dC¥u 28. Cho h m sè y= f(x ) câ b£ng bi¸n thi¶n nh÷ h¼nh v³. H m sè ¤t cüc ¤i t¤i iºm xf0( x ) f(x ) �1�3 1 4 +1 �0 +0 � �+1 +1 �2 �2 33�1 +1 �1�1A x= 3 . B x= �3. C x= 1 . D x= 4 . ÊLíi gi£i.Tø b£ng bi¸n thi¶n, nhªn th§y f0( x ) êi d§u tø +sang �t¤i x= 1 , do â h m sè ¤t cüc ¤i t¤iiºm x= 1 v yC = 3.Chån ¡p ¡n C dC¥u 29. H m sèy= x� m2 x+ 1 câ gi¡ trà nhä nh§t tr¶n[0; 1]b¬ng �1 khi38/305 38/305 pGV: L¶ Quang Xe  Ô0967.003.131PHT TRIšN — THAM KHƒO 2022 NÌI N€O CÂ Þ CH, NÌI  C CON ×ÍNG39A m=�1. B m=1. C m= 1 . D m= 0 . ÊLíi gi£i.Tªp x¡c ành Rn f� 1g .Ta câ y0= 1 +m2 (x + 1) 2>0;8 x 6= �1 suy ra h m sè çng bi¸n tr¶n [0; 1].L¤i câ y(0) = �m 2; y(1) = 1� m2 2. Do h m sè çng bi¸n tr¶n[0; 1]n¶ny(0) < y (1).Vªy gi¡ trà nhä nh§t cõa h m sè tr¶n [0; 1]l y(0) = �m 2= �1, m=1.Chån ¡p ¡n B dC¥u 30. H m sè n o sau ¥y l  h m sè çng bi¸n tr¶n R? A y= tan x+ cos x. B y= 3x � 4 2x + 1 . C y= p x2� x+ 4 . D y= x px2+ 1 . ÊLíi gi£i.Ì H m sè y= tan x+ cos xcâ tªp x¡c ành x6=  2+k ,k 2 Z do â khæng thº çng bi¸n tr¶n R.Ì H m sè y= 3x � 4 2x + 1 câ tªp x¡c ànhRn§� 1 2ªdo â khæng thº çng bi¸n tr¶n R.Ì H m sè y= p x2� x+ 4 câ tªp x¡c ành Rv  y0= 2x � 1 2p x2� x+ 4 .Suy ra y0= 0 ,x= 1 2v y0êi d§u khi xqua gi¡ trà 1 2.Suy ra h m sè khæng çng bi¸n tr¶n R.Ì H m sè y= x px2+ 1 câ tªp x¡c ànhRv  y0= 1 p(x 2+ 1) 3>0, 8x 2 R. Suy ra h m sè çngbi¸n tr¶n R.Chån ¡p ¡n D dC¥u 31. Chox, y l  hai sè d÷ìng kh¡c 1. M»nh · n o sau ¥y sai? A lnx+ ln y= ln( xy). B log25 xy= log 25 x+ log 25 y. C logxy= lnx lny. D logx y= logx� log y. ÊLíi gi£i.X²t ¯ng thùc log25 xy= log 25 x+ log 25 y. Chån x= y= 5 , ta câlog 25 xy= (log5xy)2= 4; log 25 x+ log 25 y= (log55) 2+ (log 55) 2= 2 :Vªy ¯ng thùc log25 xy= log 25 x+ log 25 yl  sai.Chån ¡p ¡n B dC¥u 32. Cho tù di»n ABC DcâAB =C D . Gåi I, J, E ,F l¦n l÷ñt l  trung iºm cõa AC,BC ,BD ,AD . Gâc giúa I Ev J F b¬ng A 30. B 45. C 90. D 60. ÊLíi gi£i.39/305 39/305 pGV: L¶ Quang Xe  Ô0967.003.131— SÈ 3 NÌI N€O CÂ Þ CH, NÌI  C CON ×ÍNG40Ta câI JAB v E F AB n¶nI JE F . T÷ìng tü J EI F n¶n tùgi¡c I J E F l  h¼nh b¼nh h nh.V¼ AB =C D n¶nI J=AB 2=C D 2=J E . Do â I J E Fl  h¼nh thoi,suy ra I E?J F . Hay (I E ; J F ) = 90. C DABJ I FEChån ¡p ¡n C dC¥u 33. Cho h m sè f(x ) thäa m¢n 2Z0 f(x )d x= �1 v  2Z1 f(x )d x= 4 . Gi¡ trà cõa 1Z0 f(x )d xb¬ng A 5. B �5. C �3. D 3. ÊLíi gi£i.Ta câ 2Z0 f(x ) d x= 1Z0 f(x ) d x+ 2Z1 f(x ) d x, � 1 =1Z0 f(x ) d x+ 4, 1Z0 f(x ) d x= �1� 4 = �5:Chån ¡p ¡n B dC¥u 34. Trong khæng gian vîi h» tåa ë Oxyz, cho hai iºm A(1; 4; 3) ,B (3; �6; 5) . Vi¸t ph÷ìngtr¼nh m°t ph¯ng trung trüc (P )cõa o¤n th¯ng AB. A x+ 5 y� z� 11 = 0 . B x+ 5 y� z+ 11 = 0 . C x� 5y + z+ 16 = 0 . D x� 5y + z� 11 = 0 . ÊLíi gi£i.Tåa ë trung iºm Mcõa o¤n th¯ng ABl M(2; �1; 4) .Ta câ # AB = (2; �10; 2) , n¶n suy ra m°t ph¯ng trung trüc (P ) cõa o¤n th¯ng ABcâ v²c-tì ph¡ptuy¸n #n = (1; �5; 1) .Ph÷ìng tr¼nh m°t ph¯ng (P )l 1 (x � 2) �5(y + 1) + 1 (y � 4) = 0 hay(P ) : x� 5y + z� 11 = 0 :Chån ¡p ¡n D dC¥u 35. Cho sè phùc z= (1 �2i) 2. T½nh mæ-un cõa sè phùc 1 z.40/305 40/305 pGV: L¶ Quang Xe  Ô0967.003.131PHT TRIšN — THAM KHƒO 2022 NÌI N€O CÂ Þ CH, NÌI  C CON ×ÍNG41A 15. B p5. C 125. D 1p5. ÊLíi gi£i.Ta câ z= (1 �2i) 2= 1 �4i + 4 i2= �3� 4i.) 1 z=1 �3� 4i =� 3 25+4 25i.Do â 1 z = Ê �� 3 25‹2+ �4 25‹2= 1 5.Chån ¡p ¡n A dC¥u 36. Cho h¼nh châp S:ABC D·u câAB= 2a; S O =avîi Ol  giao iºm cõa ACv BD . Kho£ng c¡ch tø iºm O¸n m°t ph¯ng (S C D )b¬ng A ap 32. B ap 2. C a2. D ap 22. ÊLíi gi£i.Gåi Il  trung iºm cõa C Dv Hl  trung iºm S I.V¼ S C =S D n¶n S I?C D m S O ?C D )C D ?(S OI ).Suy ra C D?OH . (1)L¤i câ OI=BC 2=an¶n 4S OI c¥n t¤i O) OH ?S I . (2)Tø (1) v  (2) suy ra OH?(S C D ).D¨n tîi d [O; (S C D )] =OH=S I 2=S O p 22=ap 22. A DOB SIHCChån ¡p ¡n D dC¥u 37. Mët · thi tr­c nghi»m gçm 50c¥u, méi c¥u câ 4ph÷ìng ¡n tr£ líi trong â ch¿ câ 1ph÷ìng ¡n óng, méi c¥u tr£ líi óng ÷ñc 0;2 iºm. B¤n An l m b i b¬ng c¡ch chån ng¨u nhi¶n1 trong 4 ph÷ìng ¡n ð méi c¥u. T½nh x¡c su§t º An ÷ñc 6iºm. A 1� 0;25 20:0 ;75 30. B 0;25 20:0 ;75 30. C 0;25 30:0 ;75 20. D 0;25 30:0 ;75 20:C 2050 . ÊLíi gi£i.º l m ÷ñc 6iºm th¼ An ph£i tr£ líi óng 30c¥u.X¡c su§t tr£ líi óng mët c¥u l  1 4= 0;25 .X¡c su§t º A¤t 6iºm l  0;25 300;75 20.Chån ¡p ¡n C dC¥u 38. Ph÷ìng tr¼nh ÷íng th¯ng song song vîi ÷íng th¯ng d: x� 1 1=y+ 2 1=z �1 v c­t hai ÷íng th¯ng d1: x+ 1 2=y+ 1 1=z� 2 �1 ;d2: x� 1 �1 =y� 2 1=z� 3 3l  A x� 1 1=y 1=z� 1 �1 . B x� 1 1=y� 2 1=z� 3 �1 . C x� 1 1=y �1 =z� 1 1. D x+ 1 �1 =y+ 1 �1 =z� 2 1. ÊLíi gi£i.41/305 41/305 pGV: L¶ Quang Xe  Ô0967.003.131— SÈ 3 NÌI N€O CÂ Þ CH, NÌI  C CON ×ÍNG42V²c-tì ch¿ ph÷ìng cõadl  #u = (1; 1; �1) .Gåi l  ÷íng th¯ng c¦n t¼m v  A=  \d1,B =  \d2. Suy ra: ¨A (� 1 + 2 a;� 1 + a; 2 �a)B (1 �b; 2 + b; 3 + 3 b) :Khi â: # AB = (�b� 2a + 2; b� a+ 3; 3 b+ a+ 1) .V¼ ÷íng th¯ng song song vîi ÷íng th¯ng dn¶n # AB còng ph÷ìng vîi #u .Suy ra: �b� 2a + 2 1=b� a+ 3 1=3b + a+ 1 �1 ,¨a = 1b = �1 ) ¨A(1; 0; 1)B (2; 1; 0) :Thay A(1; 0; 1) v o ÷íng th¯ng dta th§y A =2d.Vªy ph÷ìng tr¼nh ÷íng th¯ng  :x� 1 1=y 1=z� 1 �1 .Chån ¡p ¡n A dC¥u 39. T¼mmº b§t ph÷ìng tr¼nh log2x + 3 log x+ m 0nghi»m óng vîi måi xthuëctªp x¡c ành. A m 9 4. B m 9 4. C m <9 4. D m >�9 4. ÊLíi gi£i.i·u ki»n: x >0: °t t= log x; t2R:B§t ph÷ìng tr¼nh trð th nh t2+ 3 t+ m 0, m � t2� 3t (2) :B§t ph÷ìng tr¼nh ¢ cho câ nghi»m óng vîi måi xthuëc tªpx¡c ành ,(2) câ nghi»m óng vîi måi t2 R:X²t f(t) = �t2� 3t vîi t2 R:Ta câ: f(t) 0= �2t � 3;f0( t) = 0 ,t= �3 2:Tø b£ng bi¸n thi¶n suy ra m 9 4: tf0( t) f(t) �1�3 2 +1 +0 ��1�1 9494+1 +1 Chån ¡p ¡n A dC¥u 40. Câ bao nhi¶u sè nguy¶n d÷ìng mº hai ÷íng cong (C1) :y= 1 ln(x+ 2) +x+ 1 x� 2;( C2) :y= � 1 2x� 8+ 2x+ m c­t nhau t¤i bèn iºm ph¥n bi»t? A 3. B Væ sè. C 4. D 5. ÊLíi gi£i.X²t ph÷ìng tr¼nh1 ln(x+ 2) +x+ 1 x� 2=� 1 2x� 8+ 2x+ m , m=g(x ) = 1 ln(x+ 2) +x+ 1 x� 2+1 2x� 8�2x:i·u ki»n 8><>: ln(x+ 2) 6= 0x � 26= 02 x� 86= 0 ,8><>: �2< x 6= �1x 6= 2x 6= 3 .Vªy tªp x¡c ành l  D= ( �2; + 1)n f� 1; 2; 3 g.Ta câ g0( x ) = � 1 (x + 2) ln 2(x + 2) �3 (x � 2)2� 2xln 2 (2x� 8)2�2< 0; 8x 2 D.B£ng bi¸n thi¶n42/305 42/305 pGV: L¶ Quang Xe  Ô0967.003.131PHT TRIšN — THAM KHƒO 2022 NÌI N€O CÂ Þ CH, NÌI  C CON ×ÍNG43xg0( x ) g(x ) �2 �1 2 3 +1 � � � �511124511124�1 +1 �1 +1 �1 +1 �1�1Do âm <511 124. Vªym2 f 1; 2; 3; 4 g.Chån ¡p ¡n C dC¥u 41. H m sè n o sau ¥y l  nguy¶n h m cõa h m sè f(x ) = j2 x � 4j tr¶n kho£ng( �1 ; +1), ð â C; C0l  c¡c h¬ng sè tòy þ? A F(x ) = jx 2� 4x j+ C. B F(x ) = ¨x2� 4x + 2 C khix 2� x2+ 4 x+ 2 C� 8 khix < 2. C F(x ) = jx 2� 4x + Cj. D F(x ) = ¨x2� 4x + C khix 2� x2+ 4 x+ C0khi x < 2. ÊLíi gi£i.Ta câ f(x ) = j2 x � 4j = ¨2x � 4 khix 2� 2x + 4 khix < 2.X²t h m sè F(x ) = ¨x2� 4x + C khix 2� x2+ 4 x+ C0khi x < 2.Ì Vîi x > 2, ta câ F0( x ) = 2 x� 4 = f(x ).Ì Vîi x < 2, ta câ F0( x ) = �2x + 4 = f(x ).Ì X²t t¤i x= 2 , ta câ f(2) = 0 ,limx ! 2+ F(x ) � F(2) x� 2 = limx! 2+ x2� 4x + C � (C � 4) x� 2 = limx! 2+ (x � 2) = 0 ,limx ! 2� F(x ) � F(2) x� 2 = limx! 2� �x2+ 4 x+ C0� (C � 4) x� 2 .Do limx ! 2� (x � 2) = 0 n¶n i·u ki»n c¦n º F0(2) = f(2) = 0 l limx ! 2� (� x2+ 4 x+ C0� C+ 4) = 0, C0� C+ 8 = 0 ,C0= C� 8.Ng÷ñc l¤i, vîi C0= C� 8ta câ limx ! 2� F(x ) � F(2) x� 2 = limx! 2� �x2+ 4 x� 4 x� 2 = 0.Vªy n¸u chån h¬ng sè l  2C th¼ F(x ) = ¨x2� 4x + 2 C khix 2� x2+ 4 x+ 2 C� 8 khix < 2l  nguy¶n h m cõaf(x ) =j 2 x � 4j tr¶n (�1 ; +1).Chån ¡p ¡n B dC¥u 42. Cho h¼nh l«ng trö ùng ABCA 0B 0C 0câ ë d i c¤nh b¶n b¬ng 2a , ¡y l  tam gi¡cABC vuæng c¥n t¤i C;C A =C B =a. Gåi l  Mtrung iºm cõa c¤nh AA0. T½nh kho£ng c¡chgiúa hai ÷íng th¯ng ABv M C 0. A ap 33. B a3. C ap 32. D 2a 3. ÊLíi gi£i.43/305 43/305 pGV: L¶ Quang Xe  Ô0967.003.131— SÈ 3 NÌI N€O CÂ Þ CH, NÌI  C CON ×ÍNG44Ta câd(AB ;C M 0) = 6VCABM 0 ABC M 0 sin( AB;C M 0) .Gåi Nl  trung iºm cõa BB0suy ra ABM N , gâc giúa ABv M C 0b¬nggâc giúa M Nv M C 0. Ta câ M N=AB =ap 2, M C 0= ap 2, N C 0= ap 2.Suy ra sin(AB;C M 0) = sin 60 = p 32.V CABM 0= 1 3d(C; ( ABM ))S4 ABM =1 3ap 22p 22a2= a3 6.Vªy d(AB ;C M 0) = a3 ap 2a p 2p 32=ap 33. A0 C0 B0 A CBMNChån ¡p ¡n A dC¥u 43. Gåiz1; z2l  hai nghi»m phùc cõa ph÷ìng tr¼nhz2� 4z + 5 = 0 . Gi¡ trà cõa z21 +z22b¬ng A 6. B 16. C 26. D 8. ÊLíi gi£i.V¼ z1; z2l  nghi»m cõa ph÷ìng tr¼nhz2� 4z + 5 = 0 n¶n ta câ: ¨z1 +z2 = 4z 1 z2 = 5Khi â: z21 +z22 = (z1 +z2)2� 2z1z2 = 16�10 = 6 .Chån ¡p ¡n A dC¥u 44. Gåim0l  gi¡ trà nhä nh§t cõa 2� 1 m�i , vîiml  sè thüc. M»nh · n o d÷îi ¥yóng? A m20 2 �10 3;7 2‹. B m20 2 �0; 10 3‹. C m20 2 �7 2;9 2‹. D m20 2 �9 2;11 2‹. ÊLíi gi£i.Ta câ: 2� 1 m�i = 2(m �i) � 1 m�i = j2 m �1� 2ij jm �ij =p (2m �1)2+ 4 pm2+ 1 =Ê 4m 2� 4m + 5 m2+ 1 .X²t h m sè f(m ) = 4m 2� 4m + 5 m2+ 1 tr¶n tªp x¡c ànhD=R:Ta câ: f0( m ) = 4m 2� 2m �4 (m 2+ 1) 2;f0( m ) = 0 ,2664 m= 1 + p 174m = 1� p 174:Giîi h¤n: limm !1 f(m ) = 4 .B£ng bi¸n thi¶n mf0( m ) f(m ) �1 1� p 174 1 +p 174 +1 +0 �0 +44 153 + 9p 1734153 + 9p 1734153�9p 1734153�9p 1734 4444/305 44/305 pGV: L¶ Quang Xe  Ô0967.003.131PHT TRIšN — THAM KHƒO 2022 NÌI N€O CÂ Þ CH, NÌI  C CON ×ÍNG45Düa v o b£ng bi¸n thi¶n ta câminm 2R f(m ) = 153�9p 1734)m20 = 153�9p 17342�10 3;7 2‹.Chån ¡p ¡n A dC¥u 45.Cho h m sè f(x ) = mx4+ nx 3+ px 2+ qx +r, (m; n; p; q; r 2R). H msè y= f0( x ) câ ç thà nh÷ h¼nh v³ b¶n. Tªp nghi»m cõa ph÷ìng tr¼nhf (x ) = rcâ sè ph¦n tû l  A 2. B 1. C 3. D 4. xyO�1 12�2 5ÊLíi gi£i.Ta câ f0( x ) = 4 mx3+ 3 nx3+ 2 px+q (1).Tø ç thà h m sè f0( x ), ta câ ph÷ìng tr¼nh f0( x ) = 0 câ ba nghi»m ìnx = �1, x = �2 5v x= 1 2.Do â f0( x ) = m(x + 1) �x + 2 5‹ �x� 1 2‹, vîi m >0.S 1 = �2 5Z� 1 jm (x + 1) �x + 2 5‹ �x� 1 2‹jd x = 27m 625.S 2 = 0Z� 2 5jm (x + 1) �x + 2 5‹ �x� 1 2‹jd x = 27m 625. xyO�1 12�2 5S1 S2 S1 =S2 , �2 5Z� 1 f0( x ) d x= � 0Z� 2 5f0( x ) d x, f(� 1) = f(0) = r.B£ng bi¸n thi¶n xf0( x ) f(x ) �1�1 �2 5 0 12 +1 �0 +0 �0 ++1 +1 rr +1 +1 rSuy ra ph÷ìng tr¼nhf(x ) = rcâ t§t c£ 3nghi»m.Chån ¡p ¡n C dC¥u 46. Trong khæng gian vîi h» tåa ë Oxyz;gåi( ) l  m°t ph¯ng chùa ÷íng th¯ng : x� 2 1=y� 1 1=z 2v  vuæng gâc vîi m°t ph¯ng( ) : x+ y� 2z � 1 = 0 :Khi â giao tuy¸ncõa hai m°t ph¯ng ( ); ( ) câ ph÷ìng tr¼nh45/305 45/305 pGV: L¶ Quang Xe  Ô0967.003.131— SÈ 3 NÌI N€O CÂ Þ CH, NÌI  C CON ×ÍNG46A x1=y� 1 1=z� 1 �1 . B x� 2 1=y+ 1 �5 =z 2. C x+ 2 1=y� 1 �5 =z 2. D x1=y+ 1 1=z+ 1 1. ÊLíi gi£i.L§y M(2; 1; 0) 2 ) M 2( ):Ta câ câ v²c-tì ch¿ ph÷ìng l  #u = (1; 1; 2) v ( ) câ v²c-tì ph¡p tuy¸n l  #n = (1; 1; �2) :V¼ ( ) v  ( ) ? ( ) n¶n ( ) câ v²c-tì ph¡p tuy¸n l  [#u ;#n ] = ( �4; 4; 0)) ( ) : �x + y+ 1 = 0 :X²t h» cõa 2ph÷ìng tr¼nh m°t ph¯ng cõa ( ) v  ) ¨� x+ y+ 1 = 0x + y� 2z � 1 = 0 :Ta câ A(0; �1; �1) ; B (1; 0; 0) l  hai iºm thuëc giao tuy¸n dcõa ( ) v  ( ):Vªy di qua A(0; �1; �1) v  câ v²c-tì ch¿ ph÷ìng l  # AB = (1; 1; 1) )d:x 1=y+ 1 1=z+ 1 1:Chån ¡p ¡n D dC¥u 47. Cho h¼nh thang c¥n ABC DcâAD = 2AB = 2BC = 2C D = 2a. T½nh thº t½ch khèitrán xoay khi quay quanh h¼nh thang ABC Dquanh ÷íng th¯ng AB. A 7a 3 4. B 21a 3 4. C 21a 3 4. D 7a 3 8. ÊLíi gi£i. B CDOKA EGåiOl  giao iºm ABv C D . Khi â tam gi¡c OADl  tam gi¡c ·u.Gåi Kl  trung iºm cõa OB.Gåi El  trung iºm cõa ADkhi â tù gi¡c BC DEl  h¼nh thoi n¶n BE=1 2ADsuy ra tam gi¡cABD vuæng t¤i B.Gåi V1 l  thº t½ch khèi trán xoay ÷ñc t¤o ra khi quay tam gi¡cOADquanh ÷íng th¯ng OA.Chi·u cao cõa khèi trán xoay l  OB)OB =h= a.B¡n k½nh R= BD =p AD2� AB 2= ap 3.Khi â thº t½ch khèi nân ÷ñc sinh ra bði tam gi¡c OB Dl V OBD =1 3a(a p 3)2= a 3) V1 = 2a3:Gåi V2 l  thº t½ch khèi trán xoay ÷ñc t¤o ra khi quay tam gi¡cOB Cquanh ÷íng th¯ng OB.46/305 46/305 pGV: L¶ Quang Xe  Ô0967.003.131PHT TRIšN — THAM KHƒO 2022 NÌI N€O CÂ Þ CH, NÌI  C CON ×ÍNG47Thº t½ch khèi nân ÷ñc t¤o ra bði tam gi¡cOK Cl V OK C =1 3‚ap 32Œ2a 2=a3 8)V2 = 2VOK C =a3 4:Gåi Vl  thº t½ch cõa khèi trán xoay khi quay h¼nh thang ABC DquanhABV =V1 �V2 = 2a3� a3 4=7a 3 4:Chån ¡p ¡n A dC¥u 48. Cho b§t ph÷ìng tr¼nh log3a 11+log1 7€p x2+ 3 ax+ 10 + 4 Šlog3a (x 2+ 3 ax+ 12) 0.Gi¡ trà thüc cõa tham sè aº b§t ph÷ìng tr¼nh tr¶n câ nghi»m duy nh§t thuëc kho£ng n o sau¥y? A (� 1; 0) . B (1; 2). C (0; 1). D (2; +1). ÊLíi gi£i.°t m= 3 akhi â b§t ph÷ìng tr¼nh ¢ cho trð th nhlog m11 + log1 7€p x2+ mx + 10 + 4 Šlogm�x 2+ mx + 12  0: (1)i·u ki»n cõa b§t ph÷ìng tr¼nh l  m >0;m6= 1; x2+ mx + 10 0:Ta câ(1),1� log7(p x2+ mx + 10 + 4) log11(x 2+ mx + 12) log11m 0: (2)°t u= x2+ mx + 10 ; u0.Ì Vîi 0< m < 1.Ta câ(2) ,f(u ) = log7(p u+ 4) log11(u + 2) 1 = f(9) : (3)V¼ f(u ) l  h m t«ng tr¶n (0; +1)n¶n tø (3) ta câf (u )  f(9) ,u 9, x2+ mx + 1 0: (4)(4) væ sè nghi»m v¼  =m2� 4< 0vîi 8m 2(0; 1) . Suy ra 0< m < 1khæng thäa b i to¡n.Ì Vîi m > 1. Ta câ(2),f(u )  f(9) ,0 u 9, ¨x2+ mx + 10 0 (5)x 2+ mx + 1 0: (6)X²t (6), ta câ  =m2� 4.+ m2� 4< 0, 1< m < 2th¼ (6) væ nghi»m. Khæng thäa b i to¡n.+ m2� 4> 0, m > 2th¼ (6) câ nghi»m l  o¤n [x1;x2], lóc n y (5) nhªn hìn 1 sè cõa[ x1;x2]l m nghi»m. Khæng thäa b i to¡n.+ m2� 4 = 0 ,m= 2 th¼ (6) câ nghi»m duy nh§t x= �1 v  x= �1 thäa (5). Do â b§tph÷ìng tr¼nh câ nghi»m duy nh§t l  x= �1.Vªy m= 2 ,a= 2 3.Chån ¡p ¡n C 47/305 47/305 pGV: L¶ Quang Xe  Ô0967.003.131— SÈ 3 NÌI N€O CÂ Þ CH, NÌI  C CON ×ÍNG48dC¥u 49. Cho khèi c¦u t¥m Ob¡n k½nh 6cm. M°t ph¯ng (P )c¡ch Omët kho£ng xcm v  c­tkhèi c¦u theo ÷íng trán (C ). Mët khèi nân câ ¿nh thuëc m°t c¦u, ¡y l  h¼nh trán (C ). Bi¸tkhèi nân câ thº t½ch lîn nh§t, khi â gi¡ trà cõa xb¬ng bao nhi¶u? A 2cm. B 3cm. C 4cm. D 0cm. ÊLíi gi£i. M O0 O6cm r xP SGåi ¿nh cõa khèi nân l S, t¥m cõa h¼nh trán l  O0.Ta câ S; O; O 0th¯ng h ng, °t OO0= x(0 x 6).º khèi nân câ thº t½ch lîn nh§t th¼ On¬m giúa Sv  O0, khi â h= S O 0= S O +OO 0= x+ 6 .B¡n k½nh h¼nh trán (C )b¬ng r= p 62� x2= p 36�x2.Di»n t½ch h¼nh trán (C )b¬ng S= R 2= (36 �x2).Thº t½ch khèi nân V= 1 3S h=1 3(36 �x2)( x + 6) = 1 3(x + 6) 2(6 �x).X²t h m sè f(x ) = ( x+ 6) 2(6 �x). �p döng B§t ¯ng thùc Cauchy ta câf (x ) = ( x+ 6) 2(6 �x) = 4 x+ 6 2x+ 6 2(6 �x)  4…x+ 6 2+x+ 6 2+ 6�x 3�3, f(x )  4�12 3‹3= 4 4:D§u \ = " x£y ra khi x+ 6 2= 6�x, x= 2 .Vªy khèi nân câ thº t½ch lîn nh§t khi x= 2 .Chån ¡p ¡n A dC¥u 50. Cho h m sè f(x ) câ ¤o h m tr¶n Rl  f0( x ) = x(x � 1)( x� 4)2(x + 2) 3. Sè iºm cüctrà cõa h m sè f(x 2� 1) l  A 7. B 5. C 6. D 6. ÊLíi gi£i.48/305 48/305 pGV: L¶ Quang Xe  Ô0967.003.131MÖC LÖCNÌI N€O CÂ Þ CH, NÌI  C CON ×ÍNG49°ty= f(x 2� 1). Câ y0[f (x 2� 1)] 0= 2 xf (x 2� 1) = 2 x(x 2� 1)( x2� 2)( x2� 5)2(x 2+ 1) 3.y 0= 0 ,266664 x= 0x = 1x = p 2x = p 5:B£ng x²t d§u xy0 �1�p 5 �p 2 �1 0 1 p2 p5 +1 �0 �0 +0 �0 +0 �0 +0 +Düa v o b£ng x²t d§u h m sè câ5iºm cüc trà.Chån ¡p ¡n B H˜T49/305 49/305 pGV: L¶ Quang Xe  Ô0967.003.131— SÈ 4 NÌI N€O CÂ Þ CH, NÌI  C CON ×ÍNG50BË GIO DÖC & €O T„OTR×ÍNG THPT NGUY™N T‡T TH€NH GV: L– QUANG XE - 0967.003.131 — SÈ 4 PHT TRIšN — THAM KHƒO 2022N‹M HÅC 2021 - 2022Mæn:To¡nThíi gian l m b i: 90 phót — THAM KHƒO PTMH2022dC¥u 1. Cho sè phùc z= 1 �2i. Kh¯ng ành n o sau ¥y l  kh¯ng ành óng? A Ph¦n thüc cõa sè phùczl  1. B Ph¦n £o cõa sè phùczl  �2i. C Ph¦n £o cõa sè phùczl  2. D Sè phùczl  sè thu¦n £o. ÊLíi gi£i.Sè phùc z= 1 �2i câ ph¦n thüc l  1, ph¦n £o l  �2 v  khæng ph£i l  sè thu¦n £o.Chån ¡p ¡n A dC¥u 2. M°t c¦u S(I ;R )câ ph÷ìng tr¼nh (x � 1)2+ y2+ ( z+ 2) 2= 3 . T¥m v  b¡n k½nh cõam°t c¦u l  A I(� 1; 0; 2) ; R=p 3. B I(1; 0; �2) ; R =p 3. C I(1; 0; �2) ; R = 3 . D I(� 1; 0; 2) ; R= 3 . ÊLíi gi£i.Düa v o ph÷ìng tr¼nh m°t c¦u, ta câ I(1; 0; �2) v  R= p 3.Chån ¡p ¡n B dC¥u 3. iºm n o sau ¥y thuëc ç thà (C )cõa h m sè y= x2+ 3 x+ 3 x+ 1 ? A (3; 0). B (2; 1). C (0; 3). D (� 2; 1) . ÊLíi gi£i.Ta câ y(0) = 3 n¶n iºm câ to¤ ë (0; 3)thuëc ç thà (C )cõa h m sè â cho.Chån ¡p ¡n C dC¥u 4. Mët m°t c¦u ÷íng k½nh b¬ng 6cm. Khi â m°t c¦u câ di»n t½ch l  A 36cm 2. B 144cm 2. C 9 cm 2. D 12cm 2. ÊLíi gi£i.B¡n k½nh m°t c¦u l  3cm. Di»n t½ch m°t c¦u l  S= 4 32= 36 (cm 2).Chån ¡p ¡n A dC¥u 5. Hå t§t c£ c¡c nguy¶n h m cõa h m sè f(x ) = cos x+ 6 xl  A sinx+ 3 x2+ C. B �sin x+ 3 x2+ C. C sinx+ 6 x2+ C. D �sin x+ C. ÊLíi gi£i.Ta câ Zf(x ) d x= Z(cos x+ 6 x) d x= sin x+ 3 x2+ C.Chån ¡p ¡n A 50/305 50/305 pGV: L¶ Quang Xe  Ô0967.003.131PHT TRIšN — THAM KHƒO 2022 NÌI N€O CÂ Þ CH, NÌI  C CON ×ÍNG51dC¥u 6.Cho h m sè y= f(x ) li¶n töc tr¶n Rv  câ b£ngbi¸n thi¶n nh÷ h¼nh b¶n. Kh¯ng ành n o sau ¥yóng? A H m sè ¤t cüc tiºu t¤i iºmx= 3 . B H m sè câ gi¡ trà nhä nh§t tr¶nRb¬ng �1. C H m sè câ gi¡ trà cüc ¤i b¬ng1. D H m sè ch¿ câ mët iºm cüc trà. xy0 y �11 3 +1 +0 � +�1�1 22�1 �1 +1 +1 ÊLíi gi£i.Tø b£ng bi¸n thi¶n ta th§y h m sè ¤t cüc tiºu t¤i x= 3 .Chån ¡p ¡n A dC¥u 7. Tªp hñp nghi»m cõa b§t ph÷ìng tr¼nh 2x2< 26� xl  A (2; +1). B (�1 ;� 3) . C (� 3; 2) . D (� 2; 3) . ÊLíi gi£i.Ta câ 2x2< 26� x, x2< 6� x, x2+ x� 6< 0, � 3< x < 2:Chån ¡p ¡n C dC¥u 8. T½nh thº t½ch Vcõa khèi l«ng trö tam gi¡c ·u câ t§t c£ c¡c c¤nh b¬ng a. A a3 6. B a3p 34. C a3p 312. D a3p 32. ÊLíi gi£i.V =S¡y h = p 34a2a = a3p 34.Chån ¡p ¡n B dC¥u 9. T¼m tªp x¡c ành Dcõa h m sè f(x ) = ( x+ 1) . A D=R. B D= [ �1; + 1). C D= ( �1; + 1). D D= (0; + 1). ÊLíi gi£i.V¼ khæng nguy¶n, n¶n i·u ki»n x¡c ành x+ 1 >0, x > �1.Vªy tªp x¡c ành cõa h m sè l  D= ( �1; + 1).Chån ¡p ¡n C dC¥u 10. T¼m tªp nghi»m cõa ph÷ìng tr¼nh log3(2x2+ x+ 3) = 1 . A §0; �1 2ª. B f0g . C §� 1 2ª. D §0; 1 2ª. ÊLíi gi£i.Ta câ log3(2x2+ x+ 3) = 1 ,2x 2+ x+ 3 = 3 ,24 x= 0x = �1 2:Chån ¡p ¡n A 51/305 51/305 pGV: L¶ Quang Xe  Ô0967.003.131— SÈ 4 NÌI N€O CÂ Þ CH, NÌI  C CON ×ÍNG52dC¥u 11. N¸u1Z0 f(x ) d x= 3 v 5Z0 f(x ) d x= 2 th¼ 5Z1 f(x ) d xb¬ng A �3. B �1. C 1. D 5. ÊLíi gi£i.Ta câ 5Z0 f(x ) d x= 1Z0 f(x ) d x+ 5Z1 f(x ) d x) 5Z1 f(x ) d x= 2 �3 = �1.Chån ¡p ¡n B dC¥u 12. Cho hai sè phùc z= 10 + 3 iv  w= �4 + 5 i. T½nh jz + wj. A 100. B p14. C 10. D 10p 2. ÊLíi gi£i.Ta câ z+ w = 6 + 8 i) j z+ wj= p 62+ 8 2= 10 .Chån ¡p ¡n C dC¥u 13. Trong khæng gian vîi h» tröc tåa ë Oxyz, cho m°t ph¯ng (P ) câ ph÷ìng tr¼nh2 x � y� 1 = 0 . V²c-tì n o sau ¥y l  mët v²c-tì ph¡p tuy¸n cõa m°t ph¯ng (P )? A #n = (2; �1; �1) . B #n = (2; 0; �1) . C #n = (2; �1; 0) . D #n = ( �2; 1; 1) . ÊLíi gi£i.M°t ph¯ng (P )câ mët v²c-tì ph¡p tuy¸n #n = (2; �1; 0) .Chån ¡p ¡n C dC¥u 14. Trong khæng gian Oxyzcho hai iºm A(1; 4; 2); B(� 3; 0; �2) . Trung iºm cõa o¤nAB câ tåa ë l  A (3; 4; 4). B (2; 2; 0). C (1; 4; 4). D (� 1; 2; 0) . ÊLíi gi£i.Tåa ë trung iºm Icõa o¤n th¯ng AB÷ñc t½nh theo cæng thùc.I xA +xB 2;yA +yB 2;zA +zB 2) I�1 + ( �3) 2;4 + 0 2;2 + (�2) 2‹) I(� 1; 2; 0) .Chån ¡p ¡n D dC¥u 15. iºmMtrong h¼nh v³ b¶n d÷îi l  iºm biºu di¹n cõa sè phùc A z= �3 + 2 i. B z= 3 + 2 i. C z= �3� 2i. D z= 3 �2i. O xy�3 �2 MÊLíi gi£i.52/305 52/305 pGV: L¶ Quang Xe  Ô0967.003.131PHT TRIšN — THAM KHƒO 2022 NÌI N€O CÂ Þ CH, NÌI  C CON ×ÍNG53iºmMtrong h¼nh v³ l  iºm biºu di¹n cõa sè phùc z= �3� 2i.Chån ¡p ¡n C dC¥u 16. ç thà h m sè y= p 16�x2 xcâ bao nhi¶u ti»m cªn ngang? A 1. B 3. C 0. D 2. ÊLíi gi£i.Tªp x¡c ành cõa h m sè D= [ �4; 0) [(0; 4] n¶n ç thà h m sè khæng câ ti»m cªn ngang.Chån ¡p ¡n C dC¥u 17. Choa, b l  c¡c sè thüc thäa m¢n log2a= 3 ,log2b= 5 . T½nh gi¡ trà cõa biºu thùcP = log8(16a3b ). A P= 20 . B P= 46 3. C P= 4 . D P= 6 . ÊLíi gi£i.i·u ki»n x¡c ành l  a >0, b > 0.Ta câP= log8�16 a3b = log2(16a3b ) log28 =log216 + 3 log2a+ log2b log28 =4 + 33 + 5 3= 6:Chån ¡p ¡n D dC¥u 18.H m sè n o sau ¥y câ ç thà nh÷ h¼nh b¶n d÷îi? A y= 2 x4� 4x 2+ 1 . B y= �x4+ 2 x2+ 1 . C y= �2x 4+ 4 x2+ 1 . D y= x4� 2x 2+ 1 . xy�1 11ÊLíi gi£i.C¡c h m sè ¢ cho ·u l  h m tròng ph÷ìng d¤ng y= ax 4+ bx2+ c; (a 6= 0) .Theo ç thà ta câ a >0v  ç thà i qua c¡c iºm (� 1;0) ;(1 ;0) .Trong c¡c h m sè ¢ cho th¼ ç thà tr¶n l  cõa h m sè y= x4� 2x 2+ 1 .Chån ¡p ¡n D dC¥u 19. Cho ÷íng th¯ng  :8><>: x= 2 �ty = 3 + 2 tz = 1 + t(t 2 R), khi â i qua iºm Mcâ tåa ë l  A (2; 3; 0). B (2; 3; 1). C (1; 2; 1). D (1; 5; 3). ÊLíi gi£i.÷íng th¯ng  :8><>: x= 2 �ty = 3 + 2 tz = 1 + t(t 2 R) i qua iºm M(2; 3; 1) .Chån ¡p ¡n B 53/305 53/305 pGV: L¶ Quang Xe  Ô0967.003.131— SÈ 4 NÌI N€O CÂ Þ CH, NÌI  C CON ×ÍNG54dC¥u 20. Tø c¡c chú sè 1, 2, 3, 4, 5, 6 câ thº lªp ÷ñc bao nhi¶u sè tü nhi¶n câ 4chú sè kh¡cnhau? A 630. B 360. C 4096. D 72. ÊLíi gi£i.Chån 4sè tø 6 sè tü nhi¶n ¢ cho, sau â ho¡n và 4 sè ¢ chån. V¼ th¸ sè c¡ch chån mët sè tü nhi¶ncâ 4chú sè kh¡c nhau ch½nh l  ch¿nh hñp chªp 4cõa 6ph¦n tû: A46 = 360c¡ch chån.Chån ¡p ¡n B dC¥u 21. Cho khèi lªp ph÷ìng câ c¤nh b¬ng a. Thº t½ch cõa khèi lªp ph÷ìng ¢ cho b¬ng A a3. B 3a . C a2. D 3a 2. ÊLíi gi£i.Thº t½ch cõa khèi lªp ph÷ìng l  V=a3.Chån ¡p ¡n A dC¥u 22. T½nh ¤o h m cõa h m sè y= log2(x 2+ 1) A y0= 2x ln 2. B y0= 2x (x 2+ 1) ln 2 . C y0= 1 (x 2+ 1) ln 2 . D y0= 2x x2+ 1 . ÊLíi gi£i.¤o h m y0= (x 2+ 1) 0 (x 2+ 1) ln 2 =2x (x 2+ 1) ln 2 .Chån ¡p ¡n B dC¥u 23. Cho h m sè y= f(x ) câ b£ng bi¸n thi¶n nh÷ sau xf0( x ) f(x ) �1�p 2 0 p2 +1 �0 +0 �0 ++1 +1 �2 �2 22�2 �2 +1 +1 H m sèy= f(x ) çng bi¸n tr¶n kho£ng n o d÷îi ¥y? A (� 2; + 1). B (�1 ;� 2) . C (� 1; 0) . D (� 2; 2) . ÊLíi gi£i.Düa v o b£ng bi¸n thi¶n, h m sè y= f(x ) çng bi¸n tr¶n kho£ng (� p 2; 0)v (p 2; +1). Do â h msè y= f(x ) çng bi¸n tr¶n kho£ng (� 1; 0) .Chån ¡p ¡n C dC¥u 24. Gåil; h; R l¦n l÷ñt l  ë d i ÷íng sinh, chi·u cao v  b¡n k½nh ¡y cõa h¼nh trö (T).Di»n t½ch to n ph¦n Stp cõa h¼nh trö (T) l  A Stp =Rl + 2R2. B Stp =Rh +R 2. C Stp =Rl +R 2. D Stp = 2Rl + 2R2. ÊLíi gi£i.S tp =Sxq + 2S¡y = 2Rl + 2R2.54/305 54/305 pGV: L¶ Quang Xe  Ô0967.003.131PHT TRIšN — THAM KHƒO 2022 NÌI N€O CÂ Þ CH, NÌI  C CON ×ÍNG55Chån ¡p ¡n D dC¥u 25. T½ch ph¥n I=  3Z0 sinxdx b¬ng A p32. B �p 32. C 12. D �1 2. ÊLíi gi£i.Ta câ I=  3Z0 sinxdx = �cos x  30 =1 2.Chån ¡p ¡n C dC¥u 26. Mët c§p sè cëng gçm 5sè h¤ng. Hi»u sè h¤ng ¦u v  sè h¤ng cuèi b¬ng 20. T¼m cængsai dcõa c§p sè cëng ¢ cho. A d= �5. B d= 4 . C d= �4. D d= 5 . ÊLíi gi£i.Gåi n«m sè h¤ng cõa c§p sè cëng ¢ cho l¦n l÷ñt l : u1,u2,u3,u4,u5.Theo · b i ta câ: u1 �u5 = 20,u1 �(u1 + 4d) = 20 ,d= �5.Chån ¡p ¡n A dC¥u 27. Hå c¡c nguy¶n h m cõa h m sè f(x ) = e 2x+ sin xl  A 12e2x+ cos x+ C. B 2e2x+ cos x+ C. C 12e2x� cos x+ C. D 2e2x � 1� cos x+ C. ÊLíi gi£i.Ta câ Zf(x ) d x= Z�e2x+ sin xdx = 1 2e2x� cos x+ C.Chån ¡p ¡n C dC¥u 28. Cho h m sè y= f(x ) x¡c ành, li¶n töc tr¶n Rv  câ b£ng bi¸n thi¶n: xy0 y �10 1 +1 + �0 +�1�1 22�3 �3 +1 +1 Kh¯ng ành n o sau ¥y l  kh¯ng ành óng?A H m sè ¤t cüc ¤i t¤ix= 0 v  ¤t cüc tiºu t¤i x= 1 . B H m sè câ gi¡ trà cüc tiºu b¬ng2. C H m sè câ óng mët cüc trà.D H m sè câ gi¡ trà lîn nh§t b¬ng2v  gi¡ trà nhä nh§t b¬ng �3.55/305 55/305 pGV: L¶ Quang Xe  Ô0967.003.131— SÈ 4 NÌI N€O CÂ Þ CH, NÌI  C CON ×ÍNG56ÊLíi gi£i.Tø b£ng bi¸n thi¶n suy ra kh¯ng ành óng l : H m sè ¤t cüc ¤i t¤i x= 0 v  ¤t cüc tiºu t¤i x= 1 .Chån ¡p ¡n A dC¥u 29. T¼m gi¡ trà lîn nh§t cõa h m sè y= x+ e2xtr¶n o¤n [0; 1]. A 1. B e2. C 2e. D e2+ 1 . ÊLíi gi£i.Ta câ y0= 1 + 2 e2x> 08x 2 [0; 1] .Vªy maxx 2 [0;1] y= y(1) = e2+ 1 .Chån ¡p ¡n D dC¥u 30. Cho h m sè y= f(x ) câ ¤o h m f0( x ) = �x2� 4;8 x 2 R. M»nh · n o d÷îi ¥yóng? A H m sè çng bi¸n tr¶n kho£ng(�1 ;� 2) . B H m sè çng bi¸n tr¶n kho£ng(� 2; 2) . C H m sè çng bi¸n tr¶n kho£ng(�1 ; +1). D H m sè nghàch bi¸n tr¶n kho£ng(�1 ; +1). ÊLíi gi£i.Do f0( x ) = �x2� 4 = �(x 2+ 4) <0;8 x 2 R n¶n h m sè nghàch bi¸n tr¶n (�1 ; +1).Chån ¡p ¡n D dC¥u 31. X²t t§t c£ sè thüc d÷ìng av  bthäa m¢n log2a= log4(2ab ). M»nh · n o d÷îi ¥yóng? A b= a2. B a= 2 b. C b= 2 a. D a= b2. ÊLíi gi£i.Ta câlog2a= log4(2ab ) , log2a= 1 2log2(2ab ), 2 log2a= log2(2ab ), log2a2= log 22ab, a2= 2 ab, a= 2 b:Chån ¡p ¡n B dC¥u 32. Cho h¼nh châp S:ABC Dcâ ¡yABC D l  h¼nh vuæng c¤nh ap 3, c¤nh b¶n S Avuænggâc vîi m°t ph¯ng ¡y v  S A=a. T½nh gâc giúa hai ÷íng th¯ng BCv S D b¬ng A 30. B 60. C 45. D 90. ÊLíi gi£i.56/305 56/305 pGV: L¶ Quang Xe  Ô0967.003.131PHT TRIšN — THAM KHƒO 2022 NÌI N€O CÂ Þ CH, NÌI  C CON ×ÍNG57V¼ABC D l  h¼nh vuæng n¶n BCAD) Ù�( BC; S D ) =Ù�( AD; S D ) =ÕS DA .Ta câ tanÕS DA =S A S D=a ap 3=1 p3)ÕS DA = 30. SAB C DChån ¡p ¡n A dC¥u 33. ChoF(x ) l  mët nguy¶n h m cõa h m sè f(x ) = lnx x. T½nhI= F(e) �F(1) . A I= e . B I= 1 e. C I= 1 2. D I= 1 . ÊLíi gi£i.Ta câ I= Ze1 lnx xdx = Ze1 lnxd(ln x) = (lnx)2 2 e1 =1 2:Chån ¡p ¡n C dC¥u 34. Trong khæng gian vîi h» tåa ë Oxyz, ph÷ìng tr¼nh m°t ph¯ng i qua iºmM (2; �1; 3) v  song song vîi m°t ph¯ng (P ) : x+ y+ 2 z� 5 = 0 l  A x+ y+ 2 z� 13 = 0 . B x+ y+ 2 z+ 7 = 0 . C z+ y+ 2 z+ 14 = 0 . D z+ y+ 2 z� 7 = 0 . ÊLíi gi£i.Ph÷ìng tr¼nh m°t ph¯ng c¦n t¼m câ d¤ng x+ y+ 2 z+ c= 0 . Do m°t ph¯ng n y i qua Mn¶n2 � 1 + 2 3 + c= 0 ,c= �7:Vªy ph÷ìng tr¼nh m°t ph¯ng c¦n t¼m l  x+ y+ 2 z� 7 = 0 .Chån ¡p ¡n D dC¥u 35. T¼m mæ-un cõa sè phùc zthäa m¢n (2 + 3i) z + 12 i= 3 . A jz j = 3p 22113. B jz j = p 226. C jz j = p 106. D jz j = 153 13. ÊLíi gi£i.Ta câ z= 3� 12i 2 + 3i=�30 13�33 13i.Do â jz j = Ê �� 30 13‹2+ �� 33 13‹2= 3p 22113.Chån ¡p ¡n A dC¥u 36. Cho h¼nh châp tù gi¡c ·u S:AC BDcâ t§t c£ c¡c c¤nh b¬ng 1. Gåi Ol  h¼nh chi¸uvuæng gâc cõa Str¶n m°t ph¯ng (ABC D ). Kho£ng c¡ch tø O¸n m°t ph¯ng (S B C )b¬ng A 1p6. B p23. C 12. D 1p5.57/305 57/305 pGV: L¶ Quang Xe  Ô0967.003.131— SÈ 4 NÌI N€O CÂ Þ CH, NÌI  C CON ×ÍNG58ÊLíi gi£i.Gåi Ml  trung iºm cõa BC, h¤ OH?S M .D¹ chùng minh d(O; (S B C )) =OH.OB =p 22;S O 2= S B 2� OB 2= 1 2.Trong tam gi¡c S OMvuæng t¤i Ota câ1 OH2= 1 S O2+ 1 OM2= 5)OH =1 p5. SHAD MCO BChån ¡p ¡n D dC¥u 37. Câ hai hëp chùa c¡c qu£ c¦u. Hëp thù nh§t chùa 7qu£ c¦u m u ä v  5qu£ c¦um u xanh; hëp thù hai chùa 6qu£ c¦u m u ä v  4qu£ c¦u m u xanh. L§y ng¨u nhi¶n tø méihëp 1qu£ c¦u. X¡c su§t sao cho hai qu£ c¦u l§y ra còng m u ä b¬ng A 720. B 320. C 12. D 25. ÊLíi gi£i.X¡c su§t l§y ÷ñc qu£ c¦u ä tø hëp 1: P1 = 7 12.X¡c su§t l§y ÷ñc qu£ c¦u ä tø hëp 2: P2 = 6 10.X¡c su§t c¦n t¼m P = P1P2 = 7 20.Chån ¡p ¡n A dC¥u 38. Trong khæng gian Oxyz, ph÷ìng tr¼nh ÷íng th¯ng i qua iºm M(2; �1; 1) v  vuænggâc vîi m°t ph¯ng (P ) : 2 x� y+ 3 z+ 1 = 0 l  A x� 2 2=y+ 1 �1 =z� 1 3. B x+ 2 2=y� 1 �1 =z+ 3 1. C x� 2 2=y+ 1 �1 =z� 3 1. D x+ 2 2=y� 1 �1 =z+ 1 3. ÊLíi gi£i.M°t ph¯ng (P )câ v²c-tì ph¡p tuy¸n #n = (2; �1; 3) .V¼ dvuæng gâc vîi (P )n¶n dqua M(2; �1; 1) v  nhªn #n l m v²c-tì ch¿ ph÷ìng.Suy ra d: x� 2 2=y+ 1 �1 =z� 1 3.Chån ¡p ¡n A dC¥u 39. T¼m t§t c£ c¡c gi¡ trà cõa tham sè mº b§t ph÷ìng tr¼nh 4x� 1� m(2 x+ 1) >0nghi»m óng vîi måi x2 R. A m2(�1 ; 0]. B m2(0; + 1). C m2(0; 1) . D m2(�1 ; 0)[(1; + 1). ÊLíi gi£i.°t 2x= t; t > 0ta ÷ñc b§t ph÷ìng tr¼nh t2 4> m(t + 1) ,t2 t+ 1 >4m .58/305 58/305 pGV: L¶ Quang Xe  Ô0967.003.131PHT TRIšN — THAM KHƒO 2022 NÌI N€O CÂ Þ CH, NÌI  C CON ×ÍNG59X²t h m sèf(t) = t2 t+ 1 tr¶n kho£ng(0; +1), ta câ f0( t) = 1 � 1 (t + 1) 2.f 0( t) = 0 ,–t = 0 =2 (0; + 1)t = �2 =2 (0; + 1):Ta câ b£ng bi¸n thi¶n cõa h m sè f(t) tf0( t) f(t) 0 +1 +00 +1 +1 Vªy b§t ph÷ìng tr¼nh ¢ cho nghi»m óng vîi måix2 R khi v  ch¿ khi 0 4m , m0.Chån ¡p ¡n A dC¥u 40. Cho ç thà (C ) : y= x� 3 x+ 1 v  ÷íng th¯ngd: y = x+ 3 m. Bi¸t (C )c­t dt¤i haiiºm ph¥n bi»t A,B thäa m¢n ho nh ë trung iºm cõa o¤n ABb¬ng 6. Khi â gi¡ trà cõa mb¬ng A �4. B �2. C 1. D 3. ÊLíi gi£i.Ph÷ìng tr¼nh ho nh ë giao iºm cõa (C )vîi dl x � 3 x+ 1 =x+ 3 m, g(x ) = x2+ 3 mx + 3m+ 3 = 0 :( )º (C )c­t dt¤i hai iºm ph¥n bi»t th¼ ( ) câ hai nghi»m ph¥n bi»t kh¡c �1, ¨ >0g (� 1) 6= 0 ,¨9m 2� 12m �12 >01 � 3m + 3 m+ 3 6= 0 ,24 m <�2 3m > 2:Khi â, vîi A,B l  giao iºm, ho nh ë trung iºm Icõa AB l x I = xA +xB 2=�3m 2= 6,m=�4:(nhªn)Vªy m=�4 tho£ m¢n y¶u c¦u b i to¡n.Chån ¡p ¡n A dC¥u 41. Bi¸tF(x ) l  mët nguy¶n h m cõa f(x ) = 1� sin 3x sin2x v F 4= p 22. Câ bao nhi¶usè thüc x2 (0; 2018 ) º F(x ) = 1 . A 2018. B 1009. C 2017. D 2016. ÊLíi gi£i.Ta câ f(x ) = 1 sin2x �sin x, suy ra F(x ) = �cot x+ cos x+ C.Do F 4= p 22n¶nC= 1 , khi â F(x ) = �cot x+ cos x+ 1 .59/305 59/305 pGV: L¶ Quang Xe  Ô0967.003.131— SÈ 4 NÌI N€O CÂ Þ CH, NÌI  C CON ×ÍNG60VªyF(x ) = 1 ,cot x� cos x= 0 ,–cos x= 0sin x= 1 ,x=  2+k; k 2Z.Do x2 (0; 2018 ) ) 0<  2+k < 2018) 0< 1 2+k, tø â suy ra câ 2018sè thüc thäa m¢n y¶uc¦u b i to¡n.Chån ¡p ¡n A dC¥u 42.Cho h¼nh l«ng trö ·u ABC:A0B 0C 0. Bi¸t kho£ng c¡ch tø iºm C¸nm°t ph¯ng (ABC 0) b¬ng a, gâc giúa hai m°t ph¯ng (ABC 0) v  (BC C 0B 0)b¬ng vîi cos = 1 3(tham kh£o h¼nh v³ b¶n). Thº t½ch khèi l«ng tröABC:A 0B 0C 0b¬ng A 3a 3p 1510. B 3a 3p 1520. C 9a 3p 1510. D 9a 3p 1520. A0 C0 B0 A CBÊLíi gi£i.Gåi Ml  trung iºm cõa AB,G l  trång t¥m cõa tam gi¡c ABC.Ta câ ¨C C 0? ABC M ?AB )AB ?(C C 0M )) (C C 0M )? (ABC 0) .M  (C C 0M )\ (ABC 0) = C0M n¶n h¼nh chi¸u vuæng gâc Hcõa Cl¶n C0Mth¼ Hl  h¼nh chi¸u vuæng gâc cõa Cl¶n (ABC 0) .Suy ra d (C; (ABC 0)) = C H=a.Düng ÷íng th¯ng i qua Gv  song song vîi C H, c­t C0M t¤i N. A0 C0 B0 N HA CBM GTa câ¨GN ?(ABC 0)AG ?(BC C 0B 0) n¶n gâc giúa hai m°t ph¯ng(ABC 0) v  (BC C 0B 0) l  gâc ÕAGN = .Do GN =1 3C H=a 3,AG =GN cos )AB =AG p 3.Trong tam gi¡c C0C M vuæng t¤i C÷íng cao C Hcâ 1 C C02 = 1 C H2� 1 C M2= 5 9a 2) C C 0= 3a p 55v SABC =€ap 3Š2p 34=3a 2p 34.Vªy thº t½ch khèi l«ng trö b¬ng C C0 SABC =9a 3p 1520.Chån ¡p ¡n D dC¥u 43. Bi¸tz1; z2l  c¡c nghi»m phùc cõa ph÷ìng tr¼nhz2� 4z + 5 = 0 . Gi¡ trà biºu thùcz 1 z2 +z2 z1 l  A 35. B �4 5. C 165. D 65. ÊLíi gi£i.60/305 60/305 pGV: L¶ Quang Xe  Ô0967.003.131PHT TRIšN — THAM KHƒO 2022 NÌI N€O CÂ Þ CH, NÌI  C CON ×ÍNG61Theo ành lþ Vi-²t ta câz1 +z2 = 4; z1z2 = 5. Vªyz 1 z2 +z2 z1 =z21 +z22 z1 z2 =(z1 +z2)2� 2z1 z2 z1 z2 =42� 25 5=6 5:Chån ¡p ¡n D dC¥u 44. Cho sè phùc zthäa m¢n jz � 1j = jz � ij. T¼m gi¡ trà nhä nh§t cõa j2 z + 3 �2ij. A 32p 2. B 252. C p22. D 5p 22. ÊLíi gi£i.°t z= x+ yi vîi x, y 2 R. Ta câj z � 1j = jz � ij , j (x + yi)� 1j = j( x + yi)� ij, j (x � 1) + yij= jx + ( y� 1)ij, (x � 1)2+ y2= x2+ ( y� 1)2, y= x:Khi â z= x+ xi, vîi x2 R. Do âj 2 z + 3 �2ij = j2( x+ xi) + 3 �2ij= j(2 x+ 3) + (2 x� 2)ij= È (2x+ 3) 2+ (2 x� 2)2= È (2x+ 3) 2+ (2 x� 2)2= p 8x 2+ 4 x+ 13= Ê 8�x + 1 4‹2+ 25 25p 22:Do â, gi¡ trà nhä nh§t cõa j2 z + 3 �2ij l  b¬ng 5p 22khix= �1 4. Khi âz= �1 4�1 4i.Chån ¡p ¡n D dC¥u 45.Cho h m sè y= f(x ) câ ç thà h m sè y= f0( x ) c­ttröc Oxt¤i ba iºm câ ho nh ë a < b < cnh÷ h¼nhv³. X²t 4 m»nh · sau:(1): f(c ) < f (a ) < f (b ):(2): f(c ) > f (b ) > f (a ):(3): f(a ) > f (b ) > f (c ):(4): f(a ) > f (b ):Trong c¡c m»nh · tr¶n câ bao nhi¶u m»nh · óng? Oxyab cA 4. B 1. C 2. D 3.61/305 61/305 pGV: L¶ Quang Xe  Ô0967.003.131— SÈ 4 NÌI N€O CÂ Þ CH, NÌI  C CON ×ÍNG62ÊLíi gi£i.Tø ç thà h m sè y= f0( x ) ta câ b£ng bi¸n thi¶n nh÷ sau xy0 y �1 ab c+1 +0 �0 +0 �f(a ) f(a ) f(b ) f(b ) f(c ) f(c ) Tø â ta th§y m»nh ·(4)óng.Tø ç thà ta câ di»n t½ch h¼nh ph¯ng giîi h¤n c¡c ÷íng y= f0( x ); tröc Ox,x = a; x =bnhä hìndi»n t½ch h¼nh ph¯ng giîi h¤n bði c¡c ÷íng y= f0( x ), tröc Ox,x = b; x =c.Do â bZa (� f0( x )) d x < cZb f0( x ) d x, � f(x ) ba < f(x ) cb , �(f (b ) � f(a )) < f (c ) � f(b ) , f(a ) <f (c ): M  f(a ) > f (b ) ) f(a ) > f (b ) > f (c ); hay m»nh · (3)óng.Chån ¡p ¡n C dC¥u 46. Trong khæng gian Oxyz, cho hai iºm A(0; 0; �3) ,B (4; 0; 0) . ÷íng th¯ng i qua t¥m÷íng trán nëi ti¸p v  t¥m ÷íng trán ngo¤i ti¸p 4OAB câ ph÷ìng tr¼nh A 8><>: x= 1 �2ty = 0z = �1� t. B 8><>: x= 1 + 2 ty = 0z = �1� t. C 8><>: x= 1 �2ty = 0z = 1 + t. D 8><>: x= 1 + 2 ty = 1z = �1� t. ÊLíi gi£i.Tam gi¡c OABvuæng t¤i An¶n t¥m I÷íng trán ngo¤i ti¸p 4OAB l  trung iºm cõa AB. Suy raI �2; 0; �3 2‹.Gåi J(x ;y ;z ) l  t¥m ÷íng trán nëi ti¸p 4OAB , ta câAB # J O +OA # J B +OB # J A =#0, 5# J O + 3 # J B + 4 # J A =#0, 12# OJ = 4 # OA + 3# OB) J= (1; 0; �1) :Ta câ # I J =�� 1; 0; 1 2‹n¶n suy ra ÷íng th¯ng I Jcâ v²c-tì ch¿ ph÷ìng l  #u (2; 0; �1) .Ph÷ìng tr¼nh ÷íng th¯ng I Jl 8><>: x= 1 + 2 ty = 0z = �1� t.Chån ¡p ¡n B dC¥u 47.62/305 62/305 pGV: L¶ Quang Xe  Ô0967.003.131PHT TRIšN — THAM KHƒO 2022 NÌI N€O CÂ Þ CH, NÌI  C CON ×ÍNG63Trong m°t ph¯ng(P )cho tam gi¡c ·u v  h¼nh vuæng còng câ ë d i c¤nh b¬ng4 ÷ñc x¸p chçng l¶n nhau sao cho mët ¿nh cõa tam gi¡c tròng vîi t¥m cõah¼nh vuæng, tröc cõa tam gi¡c tròng vîi tröc cõa h¼nh vuæng (nh÷ h¼nh b¶n).Thº t½ch cõa vªt thº trán xoay t¤o th nh khi quay mæ h¼nh tr¶n quanh tröcAB b¬ng A 136+ 24 p 39. B 48+ 7 p 33. C 128+ 24 p 39. D 144+ 24 p 39. ABÊLíi gi£i. ABPQK HKhi xoay quanh tröcABta ÷ñc h¼nh gçm hai ph¦n:Ì Ph¦n h¼nh vuæng ph½a tr¶n trð th nh h¼nh trö câ b¡n k½nh ¡y b¬ng 2; chi·u cao b¬ng 4. Suyra thº t½ch khèi trö l  V1 =:22:4 = 16 .Ì Ph¦n d÷îi trð th nh h¼nh nân cöt. Ta câ AKl  ÷íng cao cõa tam gi¡c ·u c¤nh b¬ng 4n¶nAK = 2p 3.Gåi Hl  giao iºm cõa AKv P Q ,R 0l  b¡n k½nh ¡y nhä, Rl  b¡n k½nh cõa ¡y lîn, hl chi·u cao cõa h¼nh nân cöt.Ta câ h= H K =AK �AH = 2p 3� 2 = 2 €p 3� 1Šv  R= 2 .M  R0 R=AH AK=2 2p 3=1 p3)R0= R p3=2 p3.�p döng cæng thùc t½nh thº t½ch khèi nân cöt ta câV2 = 1 3h �R 2+ R02+ RR 0= 24p 3� 8 9:Vªy thº t½ch cõa khèi trán xoay l  V=V1 +V2 = 136+ 24 p 39.Chån ¡p ¡n A dC¥u 48. Cho0< x < y < 1: °t m= 1 y� x�ln y 1� y�ln x 1� x‹. M»nh · n o sau ¥yóng? A m >4. B m <1. C m= 4 . D m <2. ÊLíi gi£i.X²t h m sè f(t) = ln t 1� t�4t tr¶n (0; 1) )f0( t) = 1 t+1 1� t�4 = (2t� 1)2 t(1 �t) 0; 8t 2 (0; 1) .Suy ra h m sè f(t) çng bi¸n tr¶n (0; 1).63/305 63/305 pGV: L¶ Quang Xe  Ô0967.003.131— SÈ 4 NÌI N€O CÂ Þ CH, NÌI  C CON ×ÍNG64Do vªyf(y ) > f (x ) , ln y 1� y�4y > ln x 1� x�4x, 1 y� x�ln y 1� y�ln x 1� x‹> 4:Vªy m > 4:Chån ¡p ¡n A dC¥u 49. Cho m°t c¦u (S ) b¡n k½nh R= 5 cm . M°t ph¯ng (P ) c­t m°t c¦u (S ) theo giaotuy¸n l  ÷íng trán (C )câ chu vi b¬ng 8 cm. Bèn iºm A,B ,C ,D thay êi sao cho A,B ,Cthuëc ÷íng trán (C ), iºm Dthuëc (S ) (D khæng thuëc ÷íng trán (C )) v  tam gi¡c ABC·u.T½nh thº t½ch lîn nh§t cõa tù di»n ABC D. A 32p 3cm 3. B 60p 3cm 3. C 20p 3cm 3. D 96p 3cm 3. ÊLíi gi£i.Gåi Hl  h¼nh chi¸u vuæng gâc cõa iºm Dtr¶n m°tph¯ng (P ).Ta câ VABC D =VD:ABC =1 3DHSABC .Tam gi¡c ·u ABCcâ b¡n k½nh ÷íng trán ngo¤i ti¸pr = 8 2 = 4cm, n¶n câ c¤nh a= 4 p 3cm.) SABC =€4p 3Š2 p 34= 12p 3cm 2khæng êi.Do â thº t½ch tù di»n ABC Dlîn nh§t khi DHlînnh§t.Khi â DH=DO +OH =DO +p OA2� AH 2=5 + p 25�16 = 8 .( VD:ABC )max =1 38 12 p 3 = 32p 3cm 3. A BCDOHChån ¡p ¡n A dC¥u 50. Câ bao nhi¶u gi¡ trà nguy¶n cõa mthuëc (� 2020; 2020) º ç thà h m sè y= 1 3jx j3�mx 2+ ( m+ 6) jx j+ 2019 câ5iºm cüc trà? A 2018. B 2017. C 2016. D 2021. ÊLíi gi£i.X²t h m sè y= f(x ) = 1 3jx j3� mx 2+ ( m+ 6) jx j+ 2019 .TX: D=R.8 x 2 R ta câ f(� x) = 1 3j �xj3� m (� x)2+ ( m+ 6) j �xj+ 2019 = 1 3jx j3� mx 2+ ( m+ 6) jx j+ 2019 = f(x ).Suy ra h m sè y= 1 3jx j3� mx 2+ ( m+ 6) jx j+ 2019 l  h m sè ch®n.V¼ vªy ç thà cõa h m sè y= 1 3jx j3� mx 2+ ( m+ 6) jx j+ 2019 nhªn tröc tung l m tröc èi xùng.Do â ç thà h m sè y= 1 3jx j3� mx 2+ ( m+ 6) jx j+ 2019 câ5iºm cüc trà, ç thà h m sè g(x ) = 1 3x3� mx 2+ ( m+ 6) x+ 2019 câ óng 2iºm cüc trà n¬m v· b¶n ph£i tröc64/305 64/305 pGV: L¶ Quang Xe  Ô0967.003.131MÖC LÖCNÌI N€O CÂ Þ CH, NÌI  C CON ×ÍNG65tung, g0( x ) = x2� 2mx + (m+ 6) = 0 câ hai nghi»m d÷ìng ph¥n bi»t, 8><>: 0= m2� m �6> 0P =m + 6 >0S = 2 m > 0 ,8>>><>>>: –m < �2m > 3m > �6m > 0 ,m > 3.M  ¨m 2Zm 2(� 2020; 2020) )m2 f 4;5 ;6 ; : : : ; 2019g.Vªy sè gi¡ trà nguy¶n mc¦n t¼m l  2019�4 + 1 = 2016 .Chån ¡p ¡n C H˜T65/305 65/305 pGV: L¶ Quang Xe  Ô0967.003.131— SÈ 5 NÌI N€O CÂ Þ CH, NÌI  C CON ×ÍNG66BË GIO DÖC & €O T„OTR×ÍNG THPT NGUY™N T‡T TH€NH GV: L– QUANG XE - 0967.003.131 — SÈ 5 PHT TRIšN — THAM KHƒO 2022N‹M HÅC 2021 - 2022Mæn:To¡nThíi gian l m b i: 90 phót — THAM KHƒO PTMH2022dC¥u 1. Cho sè phùc z= 4 �3i câ iºm biºu di¹n tr¶n m°t ph¯ng tåa ë Oxyl M. T½nh ëd i OM . A 5. B 25. C p7. D 4. ÊLíi gi£i.Ta câ OM=jz j = p 42+ ( �3) 2= 5 :Chån ¡p ¡n A dC¥u 2. Trong khæng gian vîi h» tåa ë Oxyz, tåa ë t¥m Iv  b¡n k½nh Rcõa m°t c¦u câph÷ìng tr¼nh (x + 2) 2+ ( y� 3)2+ z2= 5 l  A I(2; 3; 0) ; R=p 5. B I(� 2; 3; 0) ; R=p 5. C I(2; 3; 1) ; R= 5 . D I(2; �2; 0) ; R= 5 . ÊLíi gi£i.Ta câ t¥m m°t c¦u l  I(� 2; 3; 0) , b¡n k½nh m°t c¦u l  R= p 5.Chån ¡p ¡n B dC¥u 3. Cho h m sè y= f(x ) câ limx ! +1 f(x ) = 0 v limx !�1 f(x ) = + 1. M»nh · n o sau ¥y l m»nh · óng? A ç thà h m sè n¬m ph½a tr¶n tröc ho nh.B ç thà h m sè câ mët ti»m cªn ùng l  ÷íng th¯ngy= 0 . C ç thà h m sè câ mët ti»m cªn ngang l  tröc ho nh.D ç thà h m sè khæng câ ti»m cªn ngang.ÊLíi gi£i.Tø limx ! +1 f(x ) = 0 suy ra ç thà h m sè câ mët ti»m cªn ngang l  tröc ho nh.Chån ¡p ¡n C dC¥u 4. M°t c¦u b¡n k½nh acâ di»n t½ch b¬ng A 43a2. B a2. C 4a 2. D 43a3. ÊLíi gi£i.Di»n t½ch m°t c¦u S= 4 r2= 4 a2.Chån ¡p ¡n C dC¥u 5. Hå nguy¶n h m cõa h m sè f(x ) = x3+ 2 l  A 4x 2+ 2 x+ C. B 14x4+ 2 x+ C. C x4+ 2 x+ C. D 3x 4+ 2 x+ C.66/305 66/305 pGV: L¶ Quang Xe  Ô0967.003.131PHT TRIšN — THAM KHƒO 2022 NÌI N€O CÂ Þ CH, NÌI  C CON ×ÍNG67ÊLíi gi£i.Ta câ Z�x 3+ 2 dx = 1 4x4+ 2 x+ C.Chån ¡p ¡n C dC¥u 6. Cho h m sè f(x ) câ ¤o h m li¶n töc tr¶n Rv  d§u cõa ¤o h m cho bði b£ng sau xf0( x ) �1�3 �2 �1 +1 +0 �0 +0 �H m sèf(x ) câ m§y iºm cüc trà? A 3. B 2. C 1. D 5. ÊLíi gi£i.Düa v o b£ng bi¸n thi¶n, ta th§y f0( x ) êi d§u 3l¦n n¶n h m sè câ 3cüc trà.Chån ¡p ¡n B dC¥u 7. T¼m sè nghi»m nguy¶n d÷ìng cõa b§t ph÷ìng tr¼nh log1 2(4x� 9) >log 1 2(x + 10) . A 6. B 4. C 5. D Væ sè.ÊLíi gi£i.i·u ki»n cõa b§t ph÷ìng tr¼nh l  x >9 4.Khi â b§t ph÷ìng tr¼nh ¢ cho th nh 4x � 9< x + 10 ,x < 19 3. (Doa= 1 2<1).So i·u ki»n ta ÷ñc 9 4< x <19 3. Doxnguy¶n d÷ìng n¶n x2 f 3;4 ;5 ;6 g .Chån ¡p ¡n B dC¥u 8. Cho l«ng trö ùng ABC:A0B 0C 0câ ¡y l  tam gi¡c vuæng t¤i B,AB = 2a, BC =a,AA 0= 2 ap 3. T½nh theo athº t½ch khèi l«ng trö ABC:A0B 0C 0. A a3p 33. B 2a 3p 33. C 4a 3p 3. D 2a 3p 3. ÊLíi gi£i.V ABC:A 0B 0C 0= AA 0 1 2BABC = 2a3p 3. ABA0 B0 CC0 Chån ¡p ¡n D dC¥u 9. Tªp x¡c ành cõa h m sè y= ( x� 1)1 3l  A (�1 ; 1). B R. C (1; +1). D Rn f 1g . ÊLíi gi£i.H m sè y= ( x� 1)1 3x¡c ành khi v  ch¿ khi x� 1> 0, x > 1.67/305 67/305 pGV: L¶ Quang Xe  Ô0967.003.131— SÈ 5 NÌI N€O CÂ Þ CH, NÌI  C CON ×ÍNG68Vªy tªp x¡c ành cõa h m sè l D= (1; + 1).Chån ¡p ¡n C dC¥u 10. T¼m tªp nghi»m Scõa ph÷ìng tr¼nh log3(2x+ 3) = 1 . A Sf3g . B S= f� 1g . C S= f0g . D S= f1g . ÊLíi gi£i.Ph÷ìng tr¼nh ¢ cho t÷ìng ÷ìng 2x + 3 = 3 hayx= 0 . Vªy S= f0g .Chån ¡p ¡n C dC¥u 11. T½nhI= 2Z� 1 6x 2dx . A I= 18 . B I= 22 . C I= 26 . D I= 14 . ÊLíi gi£i.Ta câ I= 2Z� 1 6x 2dx = �2 x 3 2� 1 = 2 2 3� (� 1) 3= 18 .Chån ¡p ¡n A dC¥u 12. Cho sè phùc z= 2 + 5 i. T¼m sè phùc w= iz + z. A w= �3� 3i. B w= 3 + 7 i. C w= �7� 7i. D w= 7 �3i. ÊLíi gi£i.Ta câ w= i(2 + 5 i) + (2 �5i) = �3� 3i.Chån ¡p ¡n A dC¥u 13. Trong khæng gian Oxyz, cho m°t ph¯ng (P ) : 2 x� 3y + 5 z� 12 = 0 . Khi â m°tph¯ng (P )câ mët v²c-tì ph¡p tuy¸n l  A #n = (2; 3; 5) . B #n = (2; �3; 5) . C #n = ( �2; �3; �5) . D #n = ( �2; 3; 5) . ÊLíi gi£i.M°t ph¯ng (P ) : 2 x� 3y + 5 z� 12 = 0 câ mët v²c-tì ph¡p tuy¸n l  #n = (2; �3; 5) .Chån ¡p ¡n B dC¥u 14. Trong khæng gian vîi h» tröc tåa ë Oxyz, h¼nh chi¸u vuæng gâc võa A(3; 2; �1) tr¶nm°t ph¯ng (Oxy )l  iºm A H(3; 2; 0) . B H(0; 0; 1) . C H(3; 2; �1) . D H(0; 2; 0) . ÊLíi gi£i.H¼nh chi¸u vuæng gâc cõa iºm A(3; 2; �1) tr¶n m°t ph¯ng (Oxy )câ tåa ë l  (3; 2; 0).Chån ¡p ¡n A dC¥u 15. iºm biºu di¹n cõa sè phùc z= 8 �itr¶n m°t ph¯ng tåa ë Oxyl  A M(8; �1) . B M(8; �i) . C M(8; i) . D M(� i; 8) . ÊLíi gi£i.68/305 68/305 pGV: L¶ Quang Xe  Ô0967.003.131PHT TRIšN — THAM KHƒO 2022 NÌI N€O CÂ Þ CH, NÌI  C CON ×ÍNG69iºm biºu di¹n cõa sè phùcz= 8 �itr¶n m°t ph¯ng tåa ë Oxyl  iºm M(8; �1) .Chån ¡p ¡n A dC¥u 16. Ph÷ìng tr¼nh ÷íng ti»m cªn ngang cõa ç thà h m sè y= 2 + 3 1� xl  A y= 3 . B y= �1. C x= 1 . D y= 2 . ÊLíi gi£i.V¼ limx !1 y= 2 n¶n ç thà h m sè ¢ cho câ duy nh§t mët ÷íng ti»m cªn ngang câ ph÷ìng tr¼nh l y = 2 .Chån ¡p ¡n D dC¥u 17. Choa >0, a 6= 1 v logap 2 = 3. T½nh gi¡ trà cõa biºu thùc T= log2a. A T= 3 2. B T= 1 6. C T= 2 3. D T= 1 9. ÊLíi gi£i.Ta câ logap 2 = 3,1 2loga2 = 3,log2a= 1 6.Vªy T= log2a= 1 6.Chån ¡p ¡n B dC¥u 18.Cho h m sè y= ax 4+ bx 2+ c, ( c6= 0 ) câ ç thà nh÷ h¼nh b¶n.Nhªn x²t n o d÷îi ¥y l  óng? A a <0;b < 0;c < 0. B a >0;b < 0;c < 0. C a <0;b > 0;c > 0. D a <0;b > 0;c < 0. xyOÊLíi gi£i.Do ç thà h m sè quay xuèng n¶n a <0.ç thà h m sè câ ba cüc trà n¶n ab < 0) b > 0.Do ç thà c­t tröc tung ð d÷îi tröc ho nh n¶n c <0.Vªy ta câ a <0;b > 0;c < 0.Chån ¡p ¡n D dC¥u 19. Trong khæng gian Oxyz, iºm n o d÷îi ¥y thuëc ÷íng th¯ngd : 8><>: x= �3� 2ty = 5 + tz = 3 t ? A P(� 3; �5; 0) . B Q(3; 5; 3) . C M(� 2; 1; 3) . D N(� 3; 5; 0) . ÊLíi gi£i.69/305 69/305 pGV: L¶ Quang Xe  Ô0967.003.131— SÈ 5 NÌI N€O CÂ Þ CH, NÌI  C CON ×ÍNG70Vîit= 0 thay v o h» ta ÷ñc 8><>: x= �3y = 5z = 0 . Vªy ÷íng th¯ngdi qua iºm N(� 3; 5; 0) .Chån ¡p ¡n D dC¥u 20. Sè c¡ch chån mët ban ch§p h nh gçm mët tr÷ðng ban, mët phâ ban v  mët thõ quÿ÷ñc chån tø 16th nh vi¶n l  A A316 . B 163. C 316. D C316 . ÊLíi gi£i.Sè c¡ch chån ra mët ban c¡n sü gçm tr÷ðng ban, mët phâ ban v  mët thõ quÿ tø 16håc sinh b¬ngsè c¡ch l§y ra 3håc sinh tø 16håc sinh v  x¸p v o 3và tr½ v  b¬ng A316 .Chån ¡p ¡n A dC¥u 21. Cho khèi hëp chú nhªt ABC D:A0B 0C 0D 0câ AB =a, AD =b, AA 0= c. Thº t½ch cõakhèi hëp chú nhªt ABC D:A0B 0C 0D 0b¬ng bao nhi¶u? A 13abc. B 3abc . C abc. D 12abc. ÊLíi gi£i.Thº t½ch khèi hëp chú nhªt l  V=abc .Chån ¡p ¡n C dC¥u 22. T½nh ¤o h m cõa h m sè y= 3 x+1. A y0= 3 x+1ln 3 . B y0= (1 + x) 3x. C y0= 3x+1 ln 3. D y0= 3x+1ln 3 1 +x. ÊLíi gi£i.Ta câ y0= 3 x+1ln 3 .Chån ¡p ¡n A dC¥u 23.Cho h m sè y= f(x ) câ ç thà nh÷ h¼nh v³ b¶n. H m sè ¢ cho çngbi¸n tr¶n kho£ng n o trong c¡c kho£ng d÷îi ¥y? A (� 1; 3) . B (�1 ;� 2) . C (�1 ; 3). D (� 2; 2) . xy�2 23�1 ÊLíi gi£i.Tø ç thà, suy ra y0> 0khi x2 (�1 ;� 2) .Chån ¡p ¡n B dC¥u 24. Cho mët khèi trö câ ÷íng k½nh cõa ¡y b¬ng vîi chi·u cao v  câ thº t½ch b¬ng 2 .T½nh chi·u cao hcõa khèi trö. A h= 2 . B h= 3p 24. C h= p 2. D h= 3p 4. ÊLíi gi£i.70/305 70/305 pGV: L¶ Quang Xe  Ô0967.003.131PHT TRIšN — THAM KHƒO 2022 NÌI N€O CÂ Þ CH, NÌI  C CON ×ÍNG71Thº t½ch cõa khèi tröV= 2 . Theo gi£ thi¸t, suy ra: �h 2‹2h = 2 , h3= 8 ,h= 2 .Chån ¡p ¡n A dC¥u 25. Cho h m sè f(x ) li¶n töc tr¶n kho£ng Kv  c¡c h¬ng sè a; b; c2K. M»nh · n od÷îi ¥y sai? A bZa kf (x ) d x= k bZa f(x ) d xvîi k2 R. B bZa f(x ) d x= cZa f(x ) d x+ bZc f(x ) d x. C bZa f(x ) d x= � bZa f(x ) d x. D bZa f(x ) d x6= bZa f(t) d t. ÊLíi gi£i.Ta câ bZa f(x ) d x= bZa f(t) d t.Chån ¡p ¡n D dC¥u 26. Cho c§p sè cëng (un)câ u1 = 11v  cæng sai d= 4 :H¢y t½nh u99 . A 401. B 403. C 402. D 404. ÊLíi gi£i.Ta câ :u99 =u1 + 98d= 11 + 98 4 = 403 .Chån ¡p ¡n B dC¥u 27. Trong c¡c kh¯ng ành sau, kh¯ng ành n o sai? A Zexd x = ex+1 x+ 1 +C. B Zxed x = xe+1 e + 1+C. C Zcos 2 xdx = 1 2sin 2x+ C. D Z1 xdx = ln jx j+ C. ÊLíi gi£i.Ta câÌ Zexd x = e x+ C.Ì Zxed x = xe+1 e + 1+C.Ì Zcos 2 xdx = 1 2sin 2x+ C.Ì Z1 xdx = ln jx j+ C.Chån ¡p ¡n A dC¥u 28. Cho h m sè y= f(x ) câ b£ng bi¸n thi¶n nh÷ sau:71/305 71/305 pGV: L¶ Quang Xe  Ô0967.003.131— SÈ 5 NÌI N€O CÂ Þ CH, NÌI  C CON ×ÍNG72xy0 y �1�1 3 +1 +0 �0 +�1�1 5511 +1 +1 Cüc tiºu cõa h m sè b¬ngA 1. B �1. C 3. D 5. ÊLíi gi£i.Düa v o b£ng bi¸n thi¶n, ta câ cüc tiºu cõa h m sè b¬ng 1.Chån ¡p ¡n A dC¥u 29. Têng gi¡ trà lîn nh§t Mv  gi¡ trà nhä nh§t mcõa h m sè f(x ) = ( x� 6)p x2+ 4 tr¶no¤n [0; 3]câ d¤ng a� bp cvîi al  sè nguy¶n v  b, c l  c¡c sè nguy¶n d÷ìng. T½nh S= a+ b+ c. A 4. B �2. C �22 . D 5. ÊLíi gi£i.Ta câ f0( x ) = p x2+ 4 + ( x� 6)  x px2+ 4 =x2+ 4 + ( x� 6) x px2+ 4 =2x 2� 6x + 4 px2+ 4 :X²t f0( x ) = 0 ,2x 2� 6x + 4 px2+ 4 = 0,2x 2� 6x + 4 = 0 ,–x = 1 (nhªn)x = 2 (nhªn) :Ta t½nh c¡c gi¡ trà sau: f(0) = �12 ,f(1) = �5p 5, f(2) = �8p 2, f(3) = �3p 13. So s¡nh c¡c gi¡ tràtr¶n ta ÷ñc: maxx 2 [0;3] =f(3) = �3p 13 =M;minx 2 [0;3] =f(0) = �12 = m:Theo gi£ thi¸t, ta câM+m =a� bp c, � 12�3p 13 =a� bp c) 8><>: a= �12b = 3c = 13 :Vªy S= a+ b+ c= �12 + 3 + 13 = 4 .Chån ¡p ¡n A dC¥u 30. Cho h m sè y= �x3 3+ 3x2� 5x + 1 . M»nh · n o d÷îi ¥y sai? A H m sè çng bi¸n tr¶n kho£ng(1; 5). B H m sè nghàch bi¸n tr¶n kho£ng(�1 ; 1). C H m sè nghàch bi¸n tr¶n kho£ng(1; 5). D H m sè nghàch bi¸n tr¶n kho£ng(5; +1). ÊLíi gi£i.y 0= �x2+ 6 x� 5; y0= 0 ,–x = 5x = 1 .B£ng bi¸n thi¶n:72/305 72/305 pGV: L¶ Quang Xe  Ô0967.003.131PHT TRIšN — THAM KHƒO 2022 NÌI N€O CÂ Þ CH, NÌI  C CON ×ÍNG73xy0 y �11 5 +1 �0 +0 �+1 +1 �4 3�4 3 283283�1�1H m sè çng bi¸n tr¶n kho£ng(1; 5).H m sè nghàch bi¸n tr¶n kho£ng (�1 ; 1)v (5; + 1).Chån ¡p ¡n C dC¥u 31. N¸u c¡c sè a, b thäa m¢n 2020a= bth¼ A a= 2020 1 b. B a= 1 2020b. C a= log2020b. D a= log1 2020b. ÊLíi gi£i.Ta câ 2020a= b, a= 1 2020b.Chån ¡p ¡n C dC¥u 32. Cho h¼nh châp S:ABC D, câS A=S B =S C =S D , ¡y l  tù gi¡c ABC Dthäa m¢nÒB = 90 , BC =a, AB =AD =ap 3. Sè o gâc ÕBAD l  A 30. B 60. C 90. D 120. ÊLíi gi£i.Gåi Hl  h¼nh chi¸u vuæng gâc cõa l¶n( S:ABC D ). V¼ S A=S B =S C =S D n¶nH A =H B =H C =H D .M  4ABC vuæng t¤i Bn¶n Hl  trung iºmAC .Tø gi£ thi¸t suy ra 4ABH =4ADH n¶nÕBAD = 2ÕBAC .Ta câ cosÕBAC =p 32)ÕBAC = 30)ÕBAD = 60. ABHCD SChån ¡p ¡n B dC¥u 33. Bi¸tF(x ) = x3l  mët nguy¶n h m cõa h m sè f(x ) tr¶n R. Gi¡ trà cõa 3Z1 [1 +f(x )] d xb¬ng73/305 73/305 pGV: L¶ Quang Xe  Ô0967.003.131— SÈ 5 NÌI N€O CÂ Þ CH, NÌI  C CON ×ÍNG74A 20. B 22. C 26. D 28. ÊLíi gi£i.Ta câ 3Z1 [1 +f(x )] d x= 2 + 3Z1 f(x ) d x= 2 + 3Z1 3x 2dx = 28 .Chån ¡p ¡n D dC¥u 34. Trong khæng gian vîi h» tåa ë Oxyz, cho ba iºm A(0; 6; 0) ,B(0; 0; �2) v C (� 3; 0; 0) . Ph÷ìng tr¼nh m°t ph¯ng (P )i qua ba iºm A,B ,C l  A �2x + y� 3z + 6 = 0 . B x6+y �2 +z �3 = 1. C 2x � y+ 3 z+ 6 = 0 . D x3+y �6 +z 2= 1. ÊLíi gi£i.Ph÷ìng tr¼nh m°t ph¯ng theo o¤n ch­n x a+y b+z c= 1:Ta câ (P ) : x �3 +y 6+z �2 = 1, � 2x + y� 3z = 6 ,2x � y+ 3 z+ 6 = 0 .Chån ¡p ¡n C dC¥u 35. Cho hai sè phùc z1 = 2 +i, z2 = 1�3i. T½nh T= j(1 + i) z1 + 2z2j. A T= 18 . B T= 3 p 2. C T= 0 . D T= 3 . ÊLíi gi£i.(1 + i) z1 + 2z2 = (1 +i)(2 + i) + 2(1 �3i) = 3 �3i ) j (1 + i) z1 + 2z2j= p 9 + 9 = 3p 2:Chån ¡p ¡n B dC¥u 36. Cho h¼nh châp S:ABC Dcâ ¡yABC D l  h¼nh vuæng c¤nh b¬ng a, c¤nh b¶n S Avuæng gâc vîi ¡y v  S A=ap 3. Kho£ng c¡ch tø D¸n m°t ph¯ng (S B C )b¬ng A 2a p 55. B ap 3. C a2. D ap 32. ÊLíi gi£i.Do AD (S B C )n¶n d(D; (S B C )) = d( A;(S B C )).K´ AH ?S B t¤iH, suy ra AHl  kho£ng c¡ch tø A¸n (S B C ).Ta câ 1 AH2= 1 S A2+ 1 AB2= 1 3a 2 + 1 a2 = 4 3a 2 )AH =ap 32.Vªy d(D; (S B C )) =ap 32. B C DASHChån ¡p ¡n D dC¥u 37. Mët hëp chùa 11qu£ c¦u trong â câ 5qu£ m u xanh v  6qu£ m u ä. L§y ng¨unhi¶n l¦n l÷ñt 2qu£ tø hëp â. T½nh x¡c su§t º 2l¦n ·u l§y ÷ñc qu£ c¦u m u xanh. A 955. B 211. C 411. D 511. ÊLíi gi£i.74/305 74/305 pGV: L¶ Quang Xe  Ô0967.003.131PHT TRIšN — THAM KHƒO 2022 NÌI N€O CÂ Þ CH, NÌI  C CON ×ÍNG75GåiAl  bi¸n cè 2l¦n ·u l§y ÷ñc qu£ c¦u m u xanh.Gåi A1l  bi¸n cè l¦n thù1l§y ÷ñc qu£ c¦u xanh.Gåi A2l  bi¸n cè l¦n thù2l§y ÷ñc qu£ c¦u xanh.Ta câ P(A1) = 5 11,P( A2) = 4 10. VªyP(A) = P( A1)P( A2) = 2 11.Chån ¡p ¡n B dC¥u 38. Trong khæng gian Oxyz, cho iºm M(2; 1; 0) v  ÷íng th¯ng d: x� 1 2=y+ 1 1=z �1.Vi¸t ph÷ìng tr¼nh ÷íng th¯ng i qua iºm Mv  song song vîi ÷íng th¯ng d. A x+ 2 2=y+ 1 1=z �1. B x� 2 4=y� 1 2=z �2. C x� 2 2=y� 1 1=z 1. D x� 2 4=y� 1 4=z 2. ÊLíi gi£i.÷íng th¯ng dcâ v²c-tì ch¿ ph÷ìng l  #u = (2; 1; �1) .÷íng th¯ng qua M(2; 1; 0) v  song song vîi ÷íng th¯ng dcông nhªn #u = (2; 1; �1) l m v²c-tì ch¿ph÷ìng cõa nâ.Vªy ph÷ìng tr¼nh ÷íng th¯ng c¦n t¼m l x� 2 2=y� 1 1=z �1 ,x� 2 4=y� 1 2=z �2:Chån ¡p ¡n B dC¥u 39. Cho d¢y sè (un)thäa m¢n log3u1 �2 log 2u1 + logu1 �2 = 0 v un+1 = 2un + 10(vîimåi n2 N). Gi¡ trà nhä nh§t cõa nº un >10100�10 b¬ng A 326. B 327. C 325. D 324. ÊLíi gi£i.i·u ki»n u1 >0.V¼ ¨u1 >0u n+1 = 2un + 10 )un >10 vîi måi n2 N) log u1 >1.Ta câ log3u1 �2 log 2u1 + logu1 �2 = 0 ,264 logu1 = 1 (lo¤i)log u1 = 1�p 3 (lo¤i )log u1 = 1 + p 3 (thäa m¢n ))u1 = 1010 p 3.Ta câ u2 = 210 10 p 3+ 10 = 10(2 110 p 3+ 2 0)u 3 = 210 (2 110 p 3+ 2 0) + 10 = 10(2 210 p 3+ 2 1): : :) un = 10(2 n� 110 p 3+ 2 n� 2)V¼ un >10100�10 ) 10(2 n� 110 p 3+ 2 n� 2) > 10100�10 ) 2n� 2> 1099� 1 210 p 3+ 1 )n > 324;2 .Chån ¡p ¡n C dC¥u 40. Câ bao nhi¶u gi¡ trà nguy¶n cõa tham sè mº ç thà cõa h m sè y= x3+ ( m+2) x2+ ( m2� m �3)x� m2c­t tröc ho nh t¤i ba iºm ph¥n bi»t? A 1. B 2. C 4. D 3.75/305 75/305 pGV: L¶ Quang Xe  Ô0967.003.131— SÈ 5 NÌI N€O CÂ Þ CH, NÌI  C CON ×ÍNG76ÊLíi gi£i.y = x3+ ( m+ 2) x2+ ( m2� m �3)x� m2= ( x� 1)[ x2+ ( m+ 3) x+ m2].Ph÷ìng tr¼nh ho nh ë giao iºm cõa ç thà h m sè yv  tröc ho nh l ( x � 1)[ x2+ ( m+ 3) x+ m2] = 0 ,–x = 1x 2+ ( m+ 3) x+ m2= 0 :Y¶u c¦u b i to¡n ,x2+ ( m+ 3) x+ m2= 0 câ2nghi»m ph¥n bi»t kh¡c 1., ¨(m + 3) 2� 4m 2> 0(1) 2+ ( m+ 3)(1) + m26= 0 ,¨� 3m 2+ 6 m+ 9 >0m 2+ m + 4 6= 0 , �1< m < 3.Vªy m2 f 0; 1; 2 g.Chån ¡p ¡n D dC¥u 41. Cho h m sè f(x ) x¡c ành tr¶n Rn f 0; 2 gv  thäa m¢n f0( x ) = 1 x2� 2x . Bi¸t r¬ngf (� 2) + f(4) = 0 v f�1 2‹+ f�3 2‹= 2018 . T½nhT= f(� 1) + f(1) + f(5) . A T= 1 2ln 5 + 1009. B T= 1 2ln9 5+ 1009. C T= 1 2ln9 5+ 2018. D T= 1 2ln9 5. ÊLíi gi£i.Ta câ f(x ) = Zf0( x ) d x= Z1 x2� 2x dx = 1 2ln x� 2 x +C.Suy ra f(x ) = 8>>>>><>>>>>: 1 2ln x� 2 x +C1 khix < 01 2ln x� 2 x +C2 khi0< x < 21 2ln x� 2 x +C3 khix > 2:Do f�1 2‹+ f�3 2‹= 2018 ,1 2�ln 3 + ln 1 3‹+ 2 C2= 2018,C2= 1009.L¤i câ f(� 2) + f(4) = 0 ,1 2�ln 2 + ln 1 2‹+ C1+C3= 0,C1+C3= 0.Do â T= f(� 1) + f(1) + f(5) = 1 2�ln 3 + ln 1 + ln 3 5‹+ C1+C2+C3= 1 2ln9 5+ 1009.Chån ¡p ¡n B dC¥u 42. Cho khèi châp tù gi¡c ·u S:ABC Dcâ thº t½ch b¬ng a3v  ¡y ABC D l  h¼nh vuængc¤nh a. T½nh cos vîi l  gâc giúa m°t b¶n vîi m°t ¡y. A cos = 1 p5. B cos = 1 p3. C cos = 1 p37. D cos = 1 p19. ÊLíi gi£i.76/305 76/305 pGV: L¶ Quang Xe  Ô0967.003.131PHT TRIšN — THAM KHƒO 2022 NÌI N€O CÂ Þ CH, NÌI  C CON ×ÍNG77GåiIl  trung iºm C D,O l  t¥m cõa h¼nh vuæng ABC D,suy ra S O?(ABC D ). Khi âS O = 3V SABC D =3a 3 a2 = 3a:M°t kh¡c, ta câ S O?C D v OI ?C D n¶n gâc giúa m°tb¶n (S C D )vîi m°t ¡y l  gâc ÔS I O .X²t tam gi¡c vuæng S OIcâS O = 3a, OI =a 2n¶n tanÔS I O = tan = S O OI= 6. Suy ra cos =É 11 + tan2 = É 11 + 62= 1 p37. AB OC I DSChån ¡p ¡n C dC¥u 43. Cho hai sè thüc b, c vîi c > 0. K½ hi»u A,B l  hai iºm cõa m°t ph¯ng phùc biºudi¹n hai nghi»m cõa ph÷ìng tr¼nh z2+ 2 bz+c= 0 . T¼m i·u ki»n cõa bv  csao cho tam gi¡cOAB l  tam gi¡c vuæng (vîi Ol  gèc tåa ë). A b= c. B b2= c. C 2b2= c. D b2= 2 c. ÊLíi gi£i.Theo b i ra ta gi£ sû A, B l  iºm biºu di¹n l¦n l÷ñt cõaz 1 =x+ yi,z2 =x� yi, suy ra Av  Bèi xùng nhau qua tröcho nh.�p döng ành lþ Vi-²t ta câz1 +z2 = 2x= �2b v  z1z2 =x2+ y2= c:º tam gi¡c OABvuæng khi v  ch¿ khi OM=M A =M B ,j x j= jy j , x2= y2= b2.Tø â suy ra 2b2= c. xyO ABMChån ¡p ¡n C dC¥u 44. Gi¡ trà lîn nh§t Mcõa i mi�1+m+ 1 m2+ 1 i thuëc kho£ng n o sau ¥y? A (0; 1). B �0; 3 5‹. C �4 5; 1 ‹. D (� 1; 0) . ÊLíi gi£i.Ta câ i mi�1+m+ 1 m2+ 1 i = i( � mi �1) m2+ 1 +m+ 1 m2+ 1 i = m m2+ 1 +m m2+ 1 i = m m2+ 1 p 2.Ta l¤i câ m2+ 1 2p m21 = 2 jm j ) m m2+ 1  1 2.Do â i mi�1+m+ 1 m2+ 1 i  p 22.Chån ¡p ¡n A 77/305 77/305 pGV: L¶ Quang Xe  Ô0967.003.131— SÈ 5 NÌI N€O CÂ Þ CH, NÌI  C CON ×ÍNG78dC¥u 45.Gåi Hl  h¼nh ph¯ng giîi h¤n bði ç thà h m sè y= �x2+ 4 xv  tröc ho nh. Hai ÷íng th¯ng y= m v  y= nchia (H )th nhba ph¦n câ di»n t½ch b¬ng nhau (tham kh£o h¼nh v³). Gi¡ trà cõabiºu thùc T= (4 �m)3+ (4 �n)3b¬ng A T= 320 9. B T= 75 2. C T= 512 15. D T= 405 . xyO y= n y= m ÊLíi gi£i.Gåi Sl  di»n t½ch h¼nh ph¯ng giîi h¤n bði ç thà h m sè y= �x2+ 4 xv  tröc Oxv  hai ÷íng th¯ng x= 0 ,x = 2 .Khi â S= 2Z0 (� x2+ 4 x) d x= 16 3.÷íng th¯ng y= m v  y= nchia Sth nh ba ph¦n b¬ng nhau câdi»n t½ch theo thù tü tø tr¶n xuèng l  S1;S2;S3.Gåi ho nh ë c¡c giao iºm cõa parabol vîi hai ÷íng th¯ng nh÷ h¼nhb¶n.Ta câS1 = 2 2Za (� x2+ 4 x� m) d x= 1 3S, �� x3 3+ 2x2� mx ‹ 2a =1 316 3, �16 3�2m ‹� �� a3 3+ 2a2� ma ‹= 16 9(1):M  x= al  nghi»m cõa ph÷ìng tr¼nh �x2+ 4 x= m n¶n ta câ� a2+ 4 a= m (2).Thay (2)v o (1)ta ÷ñc �2a 3 3+ 4a2� 8a + 32 9= 0,a 0;613277 .Suy ra m=�a2+ 4 a 2;077 .T÷ìng tü ta câS1 +S2 = 2 3S) 22Zb (� x2+ 4 x� n) d x= 2 32  2Z0 (� x2+ 4 x) d x, � 2 3b3+ 4 b2� 8b + 16 9= 0, b 0;252839 )n= �b2+ 4 b= 0 ;947428 :Khi â T= (4 �m)3+ (4 �n)3= 320 9. xyO y= n y= m b a24Chån ¡p ¡n A 78/305 78/305 pGV: L¶ Quang Xe  Ô0967.003.131PHT TRIšN — THAM KHƒO 2022 NÌI N€O CÂ Þ CH, NÌI  C CON ×ÍNG79dC¥u 46. Trong khæng gian vîi h» tåa ë Oxyzcho tam gi¡c ABCbi¸tA(2; 1; 0) ,B (3; 0; 2) ,C (4; 3; �4) . Vi¸t ph÷ìng tr¼nh ÷íng ph¥n gi¡c trong gâc A. A 8><>: x= 2y = 1 + tz = 0 : B 8><>: x= 2y = 1z = t: C 8><>: x= 2 + ty = 1z = 0 : D 8><>: x= 2 + ty = 1z = t: ÊLíi gi£i.Ta câ ¨AB =p 6AC = 2p 6. GåiD(x ;y ;z ) l  ch¥n ÷íng ph¥n gi¡c h¤ tø ¿nh Acõa tam gi¡c ABC.Câ ¨# BD = (x� 3;y;z � 2)# C D = (x� 4;y� 3;z+ 4) theo t½nh ch§t ÷íng ph¥n gi¡c, ta câ# BD =�AB AC# C D ,2# BD =�# C D ,8>><>>: x= 10 3y = 1z = 0 )D�10 3; 1; 0 ‹:Suy ra # AD =�4 3; 0; 0 ‹, suy ra v²c-tì ch¿ ph÷ìng cõa ÷íng ph¥n gi¡c trong gâc Al  #u = (1; 0; 0)khi â ph÷ìng tr¼nh ÷íng ph¥n gi¡c trong gâc Al  8><>: x= 2 + ty = 1z = 0 :Chån ¡p ¡n C dC¥u 47. Cho tù di»n ABC DcâAD ?DC; AC = 2a, tam gi¡c 4ABC vuæng c¥n t¤i B, gâcÕDAC = 60. Quay tù di»n quanh tröc AC÷ñc mët khèi trán xoay câ thº t½ch V. Chån ¡p ¡nóng. A V= a3 2. B V= 2a 3 3. C V= 9 + p 312a3. D V= �9 + 8 p 36a3. ÊLíi gi£i.Thº t½ch khèi trán xoay c¦n t¼m b¬ng têng thº t½ch hai h¼nh nân khi quay haitam gi¡c 4ADC v 4AB 0C quanh tröc ACtrø i têng thº t½ch khèi trán xoaykhi quay tam gi¡c 4AF C quanh tröc AC(Do tam giac 4AF C quay hai l¦n). A CD B0 FBD0 HKhèi trán xoay khi quay tam gi¡c4ADC quanh tröc AC.V 1 = 1 3r21 AC=1 3(AD sin 60 )2AC =1 3‚ap 32Œ22a = a3 2:Khèi trán xoay khi quay tam gi¡c AB0C quanh tröc ACl V 2 = 1 3r22 AC=1 3a2AC =1 3a 22a = 2a 3 3:79/305 79/305 pGV: L¶ Quang Xe  Ô0967.003.131— SÈ 5 NÌI N€O CÂ Þ CH, NÌI  C CON ×ÍNG80Ta câÕDAF = 60� 45= 15 ) AF =AD cos 15= a cos 15)F H =AF sin 45 = asin 45  cos 15=r3.Khèi trán xoay khi quay tam gi¡c 4AF C quanh tröc ACl V 3 = 1 3r23 AC=1 3 �asin 45  cos 15‹2AC =1 3 a21 22 +p 342a = 8� 4p 33a3:Thº t½ch khèi trán xoay thu ÷ñc l  V1 +V2 �V3 = a3 2+2a 3 3�4� 2p 33a3= �9 + 8 p 36a3.Chån ¡p ¡n D dC¥u 48. B§t ph÷ìng tr¼nh 5x + p 6x 2+ x3� x4log 2x >(x 2� x) log2x+ 5 + 5 p 6 +x� x2câtªp nghi»m l  S= ( a;b]. Khi â b� ab¬ng A 12. B 72. C 52. D 2. ÊLíi gi£i.i·u ki»n ¨x > 06 + x� x2 0,0< x 3.Vîi i·u ki»n 0< x 3ta câ5 x + p 6x 2+ x3� x4log2x >(x 2� x) log2x+ 5 + 5 p 6 +x� x2, €p 6 +x� x2� x+ 1 Š(x log2x� 5) >0, p 6 +x� x2< x �1 ( v¼ max(0;3] (x log2x� 5) <0), 8><>: x� 1> 06 + x� x2 06 + x� x2< (x � 1)2, 5 2< x3:Ta chùng minh max(0;3] (x log2x� 5) <0.X²t f(x ) = xlog2x� 5vîi x2 (0; 3] .Ta câ f0( x ) = log2x+ 1 ln 2.Ta câ f0( x ) = 0 ,x= 2 �1 ln 2.Ta câ limx ! 0+ (x log2x) = limx! 0+ log2x 1x= limx! 0+ 1 xln 2 �1 x2 = limx! 0+ �x ln 2= 0.Ta câ b£ng bi¸n thi¶n cõa h m sè f(x ) xf0( x ) f(x ) 02�1 ln 2 3�0 +�5 �5 f(2 �1 ln 2) f(2 �1 ln 2) f(3)  � 0;25 f(3)  � 0;2580/305 80/305 pGV: L¶ Quang Xe  Ô0967.003.131MÖC LÖCNÌI N€O CÂ Þ CH, NÌI  C CON ×ÍNG81Do vªy,max(0;3] f(x ) = f(3) <0.Chån ¡p ¡n A dC¥u 49.Th£ mët qu£ c¦u °c câ b¡n k½nh 3cm v o mët vªt h¼nh nân (câ¡y nân khæng k½n) (nh÷ h¼nh v³ b¶n). Cho bi¸t kho£ng c¡ch tøt¥m qu£ c¦u ¸n ¿nh nân l  5cm. T½nh thº t½ch (theo ìn vàcm 3) ph¦n khæng gian k½n giîi h¤n bði b· m°t qu£ c¦u v  b· m°ttrong cõa vªt h¼nh nân. A 12 5. B 14 5. C 16 5. D 18 5. ÊLíi gi£i.Theo gi£ thi¸t ta câ S I= 5 cm,I H= 3 cm.Gåi thº t½ch nân nhä l  V1, thº t½ch chäm c¦u l V2. Khi â thº t½chc¦n t¼m l  V1 �V2.X²t tam gi¡c vuæng H I ScâH S =p S I2� H I 2= 4 ,H S 2=S I S J )S J =16 5,J I = 5 �16 5=9 5)J K = 3�9 5=6 5.Ta câ V1 = 1 3H J2S J =1 3�12 5‹216 5=768 125.V 2 =J K 2�H I �J K 3‹= �6 5‹2�3 � 6 15‹= 468 125.Vªy thº t½ch c¦n t¼m l  V1 �V2 = 768 125� 468 125= 12 5. ISHK JChån ¡p ¡n A dC¥u 50. H m sèy= f(x ) = ( x� 1) (x � 2)    (x � 2018) câ bao nhi¶u iºm cüc ¤i? A 1009. B 2018. C 2017. D 1008.ÊLíi gi£i.Ta câ f(x ) = 0 ,26664 x= 1x = 2: : :x = 2018 :Vªy ph÷ìng tr¼nh f(x ) = 0 câ 2018 nghi»m ìn. Do â h m sè y= f(x ) câ 2017 iºm cüc trà.M°t kh¡c limx !�1 f(x ) = limx! +1 f(x ) = + 1n¶n h m sè y= f(x ) câ 1008 iºm cüc ¤i v  1009 iºmcüc tiºu.Chån ¡p ¡n D H˜T81/305 81/305 pGV: L¶ Quang Xe  Ô0967.003.131— SÈ 6 NÌI N€O CÂ Þ CH, NÌI  C CON ×ÍNG82BË GIO DÖC & €O T„OTR×ÍNG THPT NGUY™N T‡T TH€NH GV: L– QUANG XE - 0967.003.131 — SÈ 6 PHT TRIšN — THAM KHƒO 2022N‹M HÅC 2021 - 2022Mæn:To¡nThíi gian l m b i: 90 phót — THAM KHƒO PTMH2022dC¥u 1. Sè phùc z= 12 �3i câ ph¦n thüc v  ph¦n £o l¦n l÷ñt b¬ng A �12 v  3. B 12 v  3. C �3 v  12. D 12 v �3. ÊLíi gi£i.Sè phùc z= 12 �3i câ ph¦n thüc l  12v  ph¦n £o l  �3.Chån ¡p ¡n D dC¥u 2. Trong khæng gian Oxyz, cho m°t c¦u (S ) : x2+ ( y+ 2) 2+ ( z� 2)2= 8 . T¼m to¤ ët¥m Iv  b¡n k½nh Rcõa m°t c¦u (S ). A I(0; �2; 2); R= 64 . B I(0; 2; �2); R= 4 . C I(0; �2; 2); R= 2 p 2. D I(0; 2; �2); R= 2 p 2. ÊLíi gi£i.Ph÷ìng tr¼nh m°t c¦u l  (x � a)2+ ( y� b)2+ ( z� c)2= R2n¶n I(0; �2; 2); R= 2 p 2.Chån ¡p ¡n C dC¥u 3. Cho h m sè y= f(x ) câ limx ! +1 f(x ) = 0 v limx !�1 f(x ) = + 1. M»nh · n o sau ¥y l m»nh · óng? A ç thà h m sè n¬m ph½a tr¶n tröc ho nh.B ç thà h m sè câ mët ti»m cªn ùng l  ÷íng th¯ngy= 0 . C ç thà h m sè câ mët ti»m cªn ngang l  tröc ho nh.D ç thà h m sè khæng câ ti»m cªn ngang.ÊLíi gi£i.Tø limx ! +1 f(x ) = 0 suy ra ç thà h m sè câ mët ti»m cªn ngang l  tröc ho nh.Chån ¡p ¡n C dC¥u 4. T½nh thº t½ch khèi c¦u câ b¡n k½nh b¬ng 1. A 4 3. B 4 . C . D 3. ÊLíi gi£i.Thº t½ch khèi c¦u câ b¡n k½nh r= 1 l :V= 4 3r3= 4 313= 4 3.Chån ¡p ¡n A dC¥u 5. Hå nguy¶n h m cõa h m sè f(x ) = 5 xl  A 5x+1+C. B 5ln 5+C. C 5x+1 x+ 1 +C. D 5x ln 5+C.82/305 82/305 pGV: L¶ Quang Xe  Ô0967.003.131PHT TRIšN — THAM KHƒO 2022 NÌI N€O CÂ Þ CH, NÌI  C CON ×ÍNG83ÊLíi gi£i.�p döng cæng thùc Zaxdx = ax lna+C, ta câ Z5xdx = 5x ln 5+C:Chån ¡p ¡n D dC¥u 6. Cho h m sè y= f(x ) câ b£ng bi¸n thi¶n nh÷ sau. M»nh · n o d÷îi ¥y óng? xy0 y �10 1 +1 +0d +00 22�13 55A H m sè ¤t cüc ¤i t¤ix= 2 . B H m sè ¤t cüc tiºu t¤ix= 1 . C H m sè ¤t cüc ¤i t¤ix= 0 . D Gi¡ trà cüc tiºu cõa h m sèyC T = 3. ÊLíi gi£i.Chån ¡p ¡n C dC¥u 7. Tªp nghi»m cõa b§t ph÷ìng tr¼nh log 2x <log(x+ 6) l  A (6; +1). B (0; 6). C [0; 6). D (�1 ; 6). ÊLíi gi£i.i·u ki»n x¡c ành: x >0.B§t ph÷ìng tr¼nh ,2x < x + 6,x < 6. Vªy tªp nghi»m cõa b§t ph÷ìng tr¼nh l  (0; 6).Chån ¡p ¡n B dC¥u 8. Thº t½ch khèi l«ng trö khi bi¸t di»n t½ch ¡y Sv  chi·u cao hl  A S h. B 13S h. C 16S h. D 3S h . ÊLíi gi£i.Thº t½ch khèi l«ng trö khi bi¸t di»n t½ch ¡y Sv  chi·u cao hl  V=S h .Chån ¡p ¡n A dC¥u 9. Tªp x¡c ành cõa h m sè y= ( �x2+ 3 x+ 4) 1 3+ p 2� xl  A (� 1; 2) . B (� 1; 2] . C (�1 ; 2]. D [� 1; 2] . ÊLíi gi£i.H m sè ¢ cho x¡c ành khi v  ch¿ khi ¨� x2+ 3 x+ 4 >02 � x 0 ,¨� 1< x < 4x  2 ,x2 (� 1; 2] .Chån ¡p ¡n B dC¥u 10. Tªp nghi»m cõa ph÷ìng tr¼nh log2(x � 1)2= 2 l  A f3g . B f�1; 3 g. C f�3; 1 g. D f1g . ÊLíi gi£i.i·u ki»n: (x � 1)2> 0, x6= 1 .83/305 83/305 pGV: L¶ Quang Xe  Ô0967.003.131— SÈ 6 NÌI N€O CÂ Þ CH, NÌI  C CON ×ÍNG84Ta câlog2(x � 1)2= 2 ,(x � 1)2= 2 2, j x� 1j = 2 ,–x = �1x = 3 :Chån ¡p ¡n B dC¥u 11. Cho hai h m sè f(x ), g(x ) li¶n töc tr¶n [a ;b] v  a < c < b . M»nh · n o d÷îi ¥ysai ? A bZa [f (x ) + g(x )] d x= bZa f(x ) d x+ bZa g(x ) d x. B bZa kf(x ) d x= k bZa f(x ) d xvîi kl  h¬ng sè. C bZa f(x ) g(x ) dx = bZa f(x ) d x bZa g(x ) d x. D bZa f(x ) d x= cZa f(x ) d x+ bZc f(x ) d x. ÊLíi gi£i.Theo t½nh ch§t cõa t½ch ph¥n x¡c ành ta câ bZa [f (x ) + g(x )] d x= bZa f(x ) d x+ bZa g(x ) d xbZa kf(x ) d x= k bZa f(x ) d xvîi k l  h¬ng sè.bZa f(x ) d x= cZa f(x ) d x+ bZc f(x ) d x.Ta khæng câ t½nh ch§t bZa f(x ) g(x ) dx = bZa f(x ) d x bZa g(x ) d x.Chån ¡p ¡n C dC¥u 12. Cho hai sè phùc z1 = 1�2i; z2=�2 + i. Khi â z1 z2 b¬ng A �4 + 5 i. B �5i. C 4� 5i. D 5i. ÊLíi gi£i.z 1 z2 = (1�2i)( � 2 + i) = 5 i.Chån ¡p ¡n D 84/305 84/305 pGV: L¶ Quang Xe  Ô0967.003.131PHT TRIšN — THAM KHƒO 2022 NÌI N€O CÂ Þ CH, NÌI  C CON ×ÍNG85dC¥u 13. Cho m°t ph¯ng ( ) câ ph÷ìng tr¼nh 2x + 4 y� 3z + 1 = 0 , mët v²c-tì ph¡p tuy¸ncõa m°t ph¯ng ( ) l  A #n = (2; �4; �3) . B #n = ( �3; 4; 2) . C #n = (2; 4; 3) . D #n = (2; 4; �3) . ÊLíi gi£i.M°t ph¯ng ( ) câ mët v²c-tì ph¡p tuy¸n l  #n = (2; 4; �3) .Chån ¡p ¡n D dC¥u 14. Trong khæng gian vîi h» tåa ë Oxyz, t½nh tåa ë trång t¥m Gcõa tam gi¡c ABCvîi A(1; �1; 0) ,B (2; 0; �2) ,C (0; �2; �4) l  A G(1; �1; �2) . B G(1; �1; 2) . C G(� 1; �1; �2) . D G(� 1; 1; 2) . ÊLíi gi£i.Tåa ë iºm Gl 8>>>>><>>>>>: xG = xA +xB +xC 3=1 + 2 + 0 3= 1y G = yA +yB +yC 3=�1 + 0 �2 3=�1z G = zA +zB +zC 3=0� 2� 4 3=�2 )G(1; �1; �2) .Chån ¡p ¡n A dC¥u 15. Trong h¼nh v³ b¶n, iºm Pbiºn di¹n sè phùc z1, iºmQbiºu di¹n sè phùc z2. T¼msè phùc z= z1 +z2? A 1 + 3i. B �3 + i. C �1 + 2 i. D 2 +i. xyO�1 21 2PQÊLíi gi£i.Nh¼n v o h¼nh v³ tr¶n ta th§y z1 =�1 + 2 i, z2 = 2 +i. Khi â z1 +z2 = 1 + 3i.Chån ¡p ¡n A dC¥u 16. ç thà cõa h m sè n o d÷îi ¥y câ ÷íng ti»m cªn ùng? A y= x� 1 x. B y= e x. C y= p x2+ x� 2. D y= x2� x� 2 x+ 1 . ÊLíi gi£i.limx ! 0� x� 1 x= +1 ) x= 0 l  ti»m cªn ùng cõa ç thà h m sè y= x� 1 x.y = e xch¿ câ ti»m cªn ngang, khæng câ ti»m cªn ùng.y = p x2+ x� 2khæng câ ti»m cªn n o.Cán vîi y= x2� x� 2 x+ 1 th¼limx !� 1x2� x� 2 x+ 1 = limx!� 1(x + 1)( x� 2) x+ 1 = limx!� 1(x � 2) = �36= 1 .Chån ¡p ¡n A 85/305 85/305 pGV: L¶ Quang Xe  Ô0967.003.131— SÈ 6 NÌI N€O CÂ Þ CH, NÌI  C CON ×ÍNG86dC¥u 17. Vîia, b, c l  c¡c sè d÷ìng v  a6= 1 , m»nh · n o sau ¥y sai? A loga(b c) = logab+ logac. B loga(b c) = logablogac. C logabc= clogab. D loga�b c‹= log ab� logac. ÊLíi gi£i.Ta câ: loga(b c) = logab+ logac.log abc= clogab.log a�b c‹= log ab� logac.log a(b c) = logab+ logac6= logablogac.Chån ¡p ¡n B dC¥u 18.ç thà h¼nh b¶n l  cõa ç thà cõa h m sè n o trong c¡c h m sèsau? A y= x3� 3x 2+ 1 . B y= �x3 3+x2+ 1 . C y= 2 x3� 6x 2+ 1 . D y= �x3� 3x 2+ 1 . xyO�1 1 2 3�3 �2 �1 12ÊLíi gi£i.Theo h¼nh v³ th¼ ta nhªn th§y ¥y câ thº l  ç thà h m sè y= ax 3+ bx 2+ cx +dvîi a > 0v  i quaiºm A(2; �3) . Trong c¡c ¡p ¡n th¼ h m sè y= x3� 3x 2+ 1 câ dú ki»n phò hñp nh§t.Chån ¡p ¡n A dC¥u 19. Trong khæng gian Oxyz, ÷íng th¯ng  :8><>: x= 2 �ty = 1z = �2 + 3 tkhængi qua iºm n o sau¥y? A P(4; 1; �4) . B Q(3; 1; �5) . C M(2; 1; �2) . D N(0; 1; 4) . ÊLíi gi£i.Th¸ tåa ë tøng iºm v o ph÷ìng tr¼nh ÷íng th¯ng , ta th§y tåa ë iºm Pthäa m¢n.Chån ¡p ¡n A dC¥u 20. Tø c¡c chú sè 1; 5; 6; 7 câ thº lªp ÷ñc bao nhi¶u sè tü nhi¶n câ bèn chú sè kh¡cnhau? A 14. B 24. C 36. D 20. ÊLíi gi£i.Méi sè tü nhi¶n lªp ÷ñc tø bèn chú sè 1; 5; 6; 7 l  mët ho¡n và cõa 4ph¦n tû.Sè ho¡n và l  P4 = 4! = 24.Chån ¡p ¡n B 86/305 86/305 pGV: L¶ Quang Xe  Ô0967.003.131PHT TRIšN — THAM KHƒO 2022 NÌI N€O CÂ Þ CH, NÌI  C CON ×ÍNG87dC¥u 21.Cho h¼nh châp S:ABC Dcâ ¡yABC D l  h¼nh chú nhªt, AB=a,BC = 2a, S A = 2a, S A vuæng gâc vîi m°t ph¯ng (ABC D )(tham kh£oh¼nh v³). Thº t½ch Vcõa khèi châp S:ABC Db¬ng A V= 4a 3 3. B V= 6a 3 3. C V= 4 a3. D V= 8a 3 3. SAB C DÊLíi gi£i.Khèi châp S:ABC Dcâ ¡y l  h¼nh chú nhªt ABC Dv  chi·u cao S An¶n câ thº t½ch l V S:ABC D =1 3SABC D S A =1 3AB BC S A =1 3a 2a 2a = 4a 3 3:Chån ¡p ¡n A dC¥u 22. Trong c¡c h m sè d÷îi ¥y, h m sè n o nghàch bi¸n tr¶n tªp sè thüc R? A y=  3x. B y= log  4(2x2+ 1) . C y= �2 e‹x. D y= log2 3x. ÊLíi gi£i.H m sè y= �2 e‹xcâ tªp x¡c ành D=R v  câ cì sè 0< 2 e<1n¶n h m sè nghàch bi¸n tr¶n tªp sèthüc R.Chån ¡p ¡n C dC¥u 23. Cho h m sè y= f(x ) câ b£ng x²t d§u cõa ¤o h m nh÷ sau xy0 �1�2 0 2 +1 +0 � �0 +M»nh · n o d÷îi ¥y óng?A H m sè nghàch bi¸n tr¶n kho£ng(0; 2). B H m sè çng bi¸n tr¶n kho£ng(� 2; 0) . C H m sè nghàch bi¸n tr¶n kho£ng(�1 ;� 2) . D H m sè çng bi¸n tr¶n kho£ng(�1 ; 0). ÊLíi gi£i.Düa v o b£ng x²t d§u ta câ y0< 0; 8x 2 (0; 2) n¶n h m sè nghàch bi¸n tr¶n (0; 2).Chån ¡p ¡n A dC¥u 24. Cho tam gi¡c ABCvuæng t¤i Acâ AB =a; AC = 2a. T½nh ë d i ÷íng sinh lcõah¼nh nân nhªn ÷ñc khi quay tam gi¡c ABCquanh tröc AB. A l= ap 2. B l= 2 a. C =ap 3. D l= ap 5. ÊLíi gi£i.Ta câ l= BC =p AB2+ AC 2= p a2+ 4 a2= ap 5.87/305 87/305 pGV: L¶ Quang Xe  Ô0967.003.131— SÈ 6 NÌI N€O CÂ Þ CH, NÌI  C CON ×ÍNG88Chån ¡p ¡n D dC¥u 25. Bi¸tbZa f(x ) d x= 10 v bZa g(x ) d x= 5 . T½nh t½ch ph¥n I= bZa [3f(x )� 5g (x )] d x. A I= 5 . B I= �5. C I= 15 . D I= 10 . ÊLíi gi£i.Ta câ I= bZa [3f(x ) � 5g (x )] d x= 3 bZa f(x ) d x� 5 bZa g(x ) d x= 3 10 �55 = 5 .Chån ¡p ¡n A dC¥u 26. Cho c§p sè cëng (un)câ sè h¤ng ¦u u1 = 2, cæng sai d= 3 . Sè h¤ng thù 5cõa (un)b¬ng A 14. B 10. C 162. D 30. ÊLíi gi£i.Sè h¤ng têng qu¡t cõa c§p sè cëng câ sè h¤ng ¦u u1 v  cæng sai b¬ngdl  un =u1 + (n� 1)d.Vªy u5 =u1 + 4d= 2 + 4 3 = 14 .Chån ¡p ¡n A dC¥u 27. Hå nguy¶n h m cõa h m sè f(x ) = e �x� 1l  A ex+ x+ C. B �e�x� x+ C. C �ex� x+ C. D e�x� x+ C. ÊLíi gi£i.Ta câ Zf(x ) d x= Z�e�x� 1dx = �e�x� x+ C.Chån ¡p ¡n B dC¥u 28. Cho h m sè y= f(x ) li¶n töc tr¶n Rv  câ b£ng bi¸n thi¶n nh÷ sau xy0 y �11 3 +1 +0 � jj +�1�1 22�1 �1 +1 +1 Kh¯ng ành n o sau ¥y óng?A H m sè ¤t cüc tiºu t¤i iºmx= 3 . B H m sè câ gi¡ trà nhä nh§t tr¶nRb¬ng �1. C H m sè câ gi¡ trà cüc ¤i b¬ng1. D H m sè ch¿ câ mët iºm cüc trà.ÊLíi gi£i.Tø b£ng bi¸n thi¶n ta th§y h m sè ¤t cüc tiºu t¤i x= 3 .Chån ¡p ¡n A 88/305 88/305 pGV: L¶ Quang Xe  Ô0967.003.131PHT TRIšN — THAM KHƒO 2022 NÌI N€O CÂ Þ CH, NÌI  C CON ×ÍNG89dC¥u 29.Cho h m sè y= f(x ) li¶n töc tr¶n o¤n [� 1; 5] v  câ çthà nh÷ h¼nh v³ b¶n. Gåi Mv ml¦n l÷ñt l  gi¡ trà lînnh§t v  gi¡ trà nhä nh§t cõa h m sè ¢ cho tr¶n o¤n[ � 1; 5] . Gi¡ trà cõa M�m b¬ng A 6. B 3. C 5. D 1. O xy�1 1 2 3 4 5�2 �1 123ÊLíi gi£i.Düa v o ç thà, gi¡ trà lîn nh§t v  gi¡ trà nhä nh§t cõa h m sè f(x ) tr¶n [� 1; 5] l¦n l÷ñt l  M= 3 v m =�2. Do â M�m = 5 .Chån ¡p ¡n C dC¥u 30. H m sèf(x ) = x3 3�x2 2�6x + 3 4A çng bi¸n tr¶n kho£ng(� 2; + 1). B Nghàch bi¸n tr¶n kho£ng(�1 ;� 2) . C Nghàch bi¸n tr¶n kho£ng(� 2; 3) . D çng bi¸n tr¶n kho£ng(� 2; 3) . ÊLíi gi£i.Tªp x¡c ành: D=R.Ta câ f0( x ) = x2� x� 6.f 0( x ) = 0 ,–x = �2x = 3 :B£ng bi¸n thi¶n xf0( x ) �1�2 3 +1 +0 �0 +Vªy h m sè nghàch bi¸n tr¶n(� 2; 3) ; çng bi¸n tr¶n (�1 ;� 2) ;(3; + 1).Chån ¡p ¡n C dC¥u 31. Cholog35 =a;log57 =b, khi â log45175b¬ng A a+ b 2 +a. B a(2 + b) 2 +a. C a(a + b) 2 +a. D 2a + b 2 +a. ÊLíi gi£i.Ta câ log45175 = log5175 log545 =log5�7 52 log5�5 32= log57 + 2 2 log53 + 1 =2 +b 21 a+ 1 =a(2 + b) 2 +a.Chån ¡p ¡n B dC¥u 32. Tù di»n ·u ABC Dc¤nha, M l  trung iºm cõa c¤nh C D. Cæ-sin cõa gâc giúa AMv  BD l 89/305 89/305 pGV: L¶ Quang Xe  Ô0967.003.131— SÈ 6 NÌI N€O CÂ Þ CH, NÌI  C CON ×ÍNG90A p36. B p23. C p33. D p26. ÊLíi gi£i.Gåi Nl  trung iºm cõa BC. Do M N BD n¶n gâc giúa AMv BD b¬ng gâc giúa AMv M N . Suy ra gâc c¦n t¼m l  gâc ÖAM N .Ta câcosÖAM N =M A2+ M N 2� AN 2 2M A M N= ‚ap 32Œ2+ a 22� ‚ap 32Œ2 2ap 32a 2= p 36: DM BC NAChån ¡p ¡n A dC¥u 33. Cho h m sè f(x ) x¡c ành tr¶n Rthäa m¢n f0( x ) = ( x+ 1) 2, 8x 2 R v  f(0) = 1 .Gi¡ trà cõa biºu thùc f(� 1) + f(1) b¬ng A 4. B 103. C 2. D 20.ÊLíi gi£i.Ta câ: f(x ) = Z(x + 1) 2dx = (x + 1) 3 3+C.f (0) = 1 ,1 3+C = 1 ,C= 2 3Vªy f(x ) = (x + 1) 3 3+2 3n¶nf(� 1) + f(1) = 10 3Chån ¡p ¡n A dC¥u 34. Trong khæng gian Oxyz, cho hai iºm A(1; 2; 2) v B(3; 0; 2) . M°t ph¯ng trung trüccõa o¤n th¯ng ABcâ ph÷ìng tr¼nh l  A x+ y� z� 1 = 0 . B x+ y� 3 = 0 . C x� y� z+ 1 = 0 . D x� y� 1 = 0 . ÊLíi gi£i.Ta câ m°t ph¯ng trung trüc cõa o¤n ABqua trung iºm I(2; 1; 2) cõaAB v  nhªn # AB = (2; �2; 0)l m v²c-tì ph¡p tuy¸n n¶n câ d¤ng 2x � 2y � 2 = 0 hayx� y� 1 = 0 .Chån ¡p ¡n D dC¥u 35. T¼m sè phùc li¶n hñp cõa sè phùc z= �2 + i 1� 2i‹2+ i2019. A z= �1. B z= �1� i. C z= �1 + i. D z= i. ÊLíi gi£i.Ta câ z= �2 + i 1� 2i‹2+ i2019=i2+ i2019=�1� i.Suy ra z= �1 + i.Chån ¡p ¡n C 90/305 90/305 pGV: L¶ Quang Xe  Ô0967.003.131PHT TRIšN — THAM KHƒO 2022 NÌI N€O CÂ Þ CH, NÌI  C CON ×ÍNG91dC¥u 36. Cho h¼nh châp S:ABCcâS A ,S B ,S C æi mët vuæng gâc v  S A=a, S B =ap 2,S C =ap 3. T½nh kho£ng c¡ch tø S¸n (ABC ). A 11a 6. B ap 666. C 6a 11. D ap 6611. ÊLíi gi£i.Gåi Hl  h¼nh chi¸u cõa Sl¶n (ABC ).V¼ S A ,S B ,S C æi mët vuæng gâc n¶n 1 S H2= 1 S A2+ 1 S B2+ 1 S C2.Tø ¥y ta gi£i ÷ñc d(S; (ABC )) =S H=ap 6611. AB QS CHChån ¡p ¡n D dC¥u 37. Tø tªp hñp c¡c sè tü nhi¶n câ 5chú sè m  c¡c chú sè ·u kh¡c 0, l§y ng¨u nhi¶n 1sè. T½nh x¡c su§t º l§y ÷ñc sè câ m°t óng 3chú sè kh¡c nhau. A 140059049. B 140019683. C 14006561. D 1402187. ÊLíi gi£i.Sè c¡c sè tü nhi¶n gçm 5chú sè m  c¡c chú sè ·u kh¡c 0câ: 95sè.Sè ph¦n tû cõa khæng gian m¨u: j j= 9 5.Bi¸n cè A: l§y ÷ñc sè câ m°t óng 3chú sè kh¡c nhau.+ Chån ra 3chú sè tø 9chú sè 1;2 ;3 ; : : : ; 9l : C39 .+ Gi£ sû 3sè ÷ñc chån l  a; b; c. V¼ sè c¦n t¼m câ 5chú sè m  ch¿ câ m°t óng 3chú sè kh¡c nhaun¶n ta câ c¡c tr÷íng hñp sau:Tr÷íng hñp 1: a xu§t hi»n 3l¦n; bv  cxu§t hi»n 1l¦n: C35 2! = 20 sè.T÷ìng tü khi bv  cxu§t hi»n 3l¦n th¼ méi tr÷íng hñp â công th nh lªp ÷ñc 20sè.Tr÷íng hñp 2: a xu§t hi»n 2l¦n; bxu§t hi»n 2l¦n v  cxu§t hi»n 1l¦n: C25 C 23 1 = 30 sè.Tr÷íng hñp 3: a xu§t hi»n 2l¦n; bxu§t hi»n 1l¦n v  cxu§t hi»n 2l¦n: C25 C 23 1 = 30 sè.Tr÷íng hñp 4: a xu§t hi»n 1l¦n; bv  cméi sè xu§t hi»n 2l¦n: C25 C 23 1 = 30 sè.Do â: jA j= (20 3 + 30 3) C 39 = 12600.Vªy PA = 12600 95 =1400 6561.Chån ¡p ¡n C dC¥u 38. Trong khæng gian vîi h» tåa ë Oxyz, ph÷ìng tr¼nh ch½nh t­c cõa ÷íng th¯ng diqua iºm M(1; �2; 5) v  vuæng gâc vîi m°t ph¯ng ( ) : 4 x� 3y + 2 z+ 5 = 0 l  A x� 1 4=y+ 2 3=z� 5 2. B x� 1 4=y+ 2 �3 =z� 5 2. C x� 1 �4 =y+ 2 �3 =z� 5 �2 . D x� 1 �4 =y+ 2 �3 =z� 5 2. ÊLíi gi£i.M°t ph¯ng ( ) câ mët VTPT l  #n = (4; �3; 2) .Do d? ( ), suy ra ÷íng th¯ng dnhªn #n l m v²c-tì ch¿ ph÷ìng.Do â ph÷ìng tr¼nh ÷íng th¯ng dl  x� 1 4=y+ 2 �3 =z� 5 2.Chån ¡p ¡n B 91/305 91/305 pGV: L¶ Quang Xe  Ô0967.003.131— SÈ 6 NÌI N€O CÂ Þ CH, NÌI  C CON ×ÍNG92dC¥u 39. T¼m t§t c£ c¡c gi¡ trà thüc cõa tham sè mº b§t ph÷ìng tr¼nh p 2x+ 1+ p 3� 2x mnghi»m óng vîi måi x2 (�1 ; log23). A m4. B m <4. C m2p 2. D m <2p 2. ÊLíi gi£i.°t t= 2 x; t > 0, b§t ph÷ìng tr¼nh ¢ cho trð th nh p t+ 1 + p 3� t m. (1)º b§t ph÷ìng tr¼nh ¢ cho nghi»m óng vîi måi x2 (�1 ; log23)th¼ b§t ph÷ìng tr¼nh (1) nghi»móng vîi måi t2 (0; 3) .X²t h m sè f(t) = p t+ 1 + p 3� t; t 2(0; 3) , khi â (1) óng vîi måi t2 (0; 3) khi v  ch¿ khim max(0;3) f(t) . (2)Ta câ f0( t) = 1 2p t+ 1 �1 2p 3� t; f0( t) = 0 ,t= 1 . B£ng bi¸n thi¶n tf0( t) f(t) 0 1 3+0 �2p 22p 2Düa v o b£ng bi¸n thi¶n ta suy ramax(0;3) f(t) = 2 p 2. Tø (2) ta câ m2p 2.Chån ¡p ¡n C dC¥u 40. Têng t§t c£ c¡c gi¡ trà cõa tham sè mº ÷íng th¯ng d:y = �x+ m c­t ç thà h msè y= x� 2 x� 1(C )t¤i hai iºm ph¥n bi»t A,B sao cho OA+ 2019 OB = 4040 . A �5. B 2. C 4. D �7. ÊLíi gi£i.Ta câ ph÷ìng tr¼nh ho nh ë giao iºm cõa hai ÷íng l �x + m = x� 2 x� 1, ¨x 6= 1x 2� mx +m �2 = 0 :i·u ki»n º hai ÷íng câ hai giao iºm l  ¨1� m +m �26= 0m 2� 4m + 8 >0 ,m2R.Gåi xA l  ho nh ë cõaAth¼ câ x2A �mxA+m �2 = 0 v A= ( xA ;� xA +m).Gåi xB l  ho nh ë cõaBth¼ câ x2B �mxB+m �2 = 0 v B= ( xB ;� xB +m).Th§y OA2= x2A + (�xA +m)2= 2( x2A �mxA+m �2) + m2� 2m + 4 = m2� 2m + 4 .OB 2= x2B + (�xB +m)2= 2( x2B �mxB+m �2) + m2� 2m + 4 = m2� 2m + 4 .Suy ra OA=OB , do â OA+ 2019 OB = 4040 ,2020 OA = 4040 ,OA = 2.Vîi OA = 2 ,m2� 2m + 4 = 4 ,m2� 2m = 0 ,–m = 0m = 2 :Vªy tø ph÷ìng tr¼nh tr¶n th§y têng c¡c gi¡ trà cõa tham sè ml 2.Chån ¡p ¡n B 92/305 92/305 pGV: L¶ Quang Xe  Ô0967.003.131PHT TRIšN — THAM KHƒO 2022 NÌI N€O CÂ Þ CH, NÌI  C CON ×ÍNG93dC¥u 41. ChoF(x ) l  mët nguy¶n h m cõa f(x ) = 2x + 1 x4+ 2 x3+ x2 tr¶n kho£ng(0; +1)thäam¢n F(1) = 1 2. Gi¡ trà cõa biºu thùcS= F(1) + F(2) + F(3) +   +F(2019) l  A 20192020. B 2019:2021 2020. C 20181 2020. D �2019 2020. ÊLíi gi£i.Ta câ F(x ) = Z2x + 1 x4+ 2 x3+ x2 dx = Z2x + 1 x2(x + 1) 2dx = Z�1 x2 � 1 (x + 1) 2‹dx .Suy ra F(x ) = �1 x+1 x+ 1 +cm  F(1) = 1 2n¶nc= 1 .Hay F(x ) = �1 x+1 x+ 1 + 1.Ta câS= F(1) + F(2) + F(3) +   +F(2019)= �� 1 1+1 2+ 1 ‹+ �� 1 2+1 3+ 1 ‹+ �� 1 3+1 4+ 1 ‹+    +�� 1 2019+1 2020+ 1‹= �1 + 1 2020+ 20191 = 2018 + 1 2020= 20181 2020:Chån ¡p ¡n C dC¥u 42. Cho h¼nh châp tù gi¡c ·u S:ABC Dcâ c¤nh ¡y b¬ng av  chi·u cao b¬ng h. Gåi Ol  t¥m cõa ¡y ABC D. T½nh kho£ng c¡ch tø O¸n (S AB ). A ahp2a 2+ 4 h2. B ahpa2+ 4 h2. C ahpa2+ h2. D ah2p a2+ h2. ÊLíi gi£i.Ta câ S O?(ABC D ).K´ OK ?AB; (K 2AB ), OH ?S K; (H 2S K ).V¼ ¨OH ?S K (S AB )OH ?AB (S AB ); (AB ?(S OK )) OH ?(S AB ).Suy ra: kho£ng c¡ch tø O¸n (S AB )l :d (O; (S AB ))=OH .X²t S OK vuæng t¤i O, ta câ:1 OH2= 1 OK2+ 1 S O2) OH = OKS O pOK2+ S O 2= ah 2É a2 4+h2 =ah pa2+ 4 h2.Vªy kho£ng c¡ch c¦n t¼m l d(O; (S AB ))=OH = ah pa2+ 4 h2: HB C DSAK OChån ¡p ¡n B 93/305 93/305 pGV: L¶ Quang Xe  Ô0967.003.131— SÈ 6 NÌI N€O CÂ Þ CH, NÌI  C CON ×ÍNG94dC¥u 43. Bi¸t sè phùc zthäa m¢n jz � 3� 4ij = p 5v  biºu thùc T= jz + 2 j2� j z� ij2¤tgi¡ trà lîn nh§t. T½nh jz j. A jz j = p 33. B jz j = 50 . C jz j = p 10. D jz j = 5 p 2. ÊLíi gi£i.°t z= x+ yi (x; y 2R), theo gi£ thi¸t jz � 3� 4ij = p 5, (x � 3)2+ ( y� 4)2= 5 ( C).Ngo i ra T= jz + 2 j2� j z� ij2, 4x + 2 y+ 3 �T= 0 () ¤t gi¡ trà lîn nh§t.Rã r ng (C )v  () câ iºm chung do â j23 + Tj 2p 5p 5, 13T 33.V¼ T¤t gi¡ trà lîn nh§t n¶n T= 33 suy ra 4x + 2 y� 30 = 0 ,y= 15 �2x thay v o (C )ta ÷ñc5 x 2� 50x+ 125 = 0 ,x= 5 )y= 5 . Vªy jz j = 5 p 2.Chån ¡p ¡n D dC¥u 44. X²t c¡c sè phùc zthäa m¢n jiz �3j = jz � 2� ij. T¼m ph¦n thüc cõa sè phùc zsaocho jz j nhä nh§t. A 15. B �2 5. C �1 5. D 25. ÊLíi gi£i.Gåi z= x+ yi, vîi x; y2R.j iz �3j = jz � 2� ij , j xi�y� 3j = j( x � 2) + ( y� 1)ij, x2+ ( y+ 3) 2= ( x� 2)2+ ( y� 1)2, x+ 2 y= �1, x= �2y � 1:Khi â, jz j = p x2+ y2= È (� 1� 2y )2+ y2= Ê 5�y + 2 5‹2+ 1 51 p5.Suy ra, jz jmin = 1 p5khiy= �2 5)x= �1 5.Chån ¡p ¡n C dC¥u 45.Cho hai h m sè f(x ) = ax3+ bx 2+ cx +3 4v g(x ) = dx2+ ex �3 4( a; b; c; d; e 2R). Bi¸t r¬ng ç thà cõa h m sè y= f(x ) v  y= g(x )c­t nhau t¤i ba iºm câ ho nh ë l¦n l÷ñt l  �2; 1; 3 (tham kh£oh¼nh v³). H¼nh ph¯ng giîi h¤n bði hai ç thà ¢ cho câ di»n t½chb¬ng A 25348. B 12524. C 12548. D 25324. x�2 1 3yOÊLíi gi£i.Ph÷ìng tr¼nh ho nh ë giao iºm ax3+ bx 2+ cx +3 4=dx 2+ ex �3 4,ax3+ ( b� d)x 2+ ( c� e)x + 3 2= 0.°t h(x ) = ax3+ ( b� d)x 2+ ( c� e)x + 3 2.Düa v o ç thà ta câ h(x ) = 0 câ ba nghi»m l  x= �2; x = 1 ;x = 3 .94/305 94/305 pGV: L¶ Quang Xe  Ô0967.003.131PHT TRIšN — THAM KHƒO 2022 NÌI N€O CÂ Þ CH, NÌI  C CON ×ÍNG95Khi â ta câ h»8>>>>><>>>>>: �8a + 4( b� d) � 2(c� e) = �3 2a + ( b� d) + ( c� e) = �3 227 a+ 9( b� d) + 3( c� e) = �3 2,8>>>>><>>>>>: a= 1 4b � d= �1 2c � e= �5 4:Khi â di»n t½ch h¼nh ph¯ng c¦n t½nh l S= 3Z� 2 jf (x ) � g(x )j d x = 1Z� 2 1 4x3� 1 2x2� 5 4x+ 3 2 dx + 3Z1 1 4x3� 1 2x2� 5 4x+ 3 2 dx= 63 16+4 3=253 48:Chån ¡p ¡n A dC¥u 46. Cho ÷íng th¯ng (d ) câ ph÷ìng tr¼nh 4x + 3 y� 5 = 0 v  ÷íng th¯ng ()câ ph÷ìngtr¼nh x+ 2 y� 5 = 0 . Ph÷ìng tr¼nh ÷íng th¯ng (d 0) l  £nh cõa ÷íng th¯ng (d ) qua ph²p èixùng tröc ()l  A x� 3 = 0 . B x+ y� 1 = 0 . C 3x + 2 y� 5 = 0 . D y� 3 = 0 . ÊLíi gi£i.Ta câ ¨4x + 3 y� 5 = 0x + 2 y� 5 = 0 ,¨x = �1y = 3 , suy ra(d ) c­t () t¤iI(� 1; 3) . ÷íng th¯ng (d 0) l  £nh cõa÷íng th¯ng (d ) qua ph²p èi xùng tröc ()n¶nI2 (d 0) .Trong 4 ÷íng th¯ng ¢ cho ch¿ câ ÷íng th¯ng câ ph÷ìng tr¼nh y� 3 = 0 quaI, do â (d 0) : y� 3 = 0 .Chån ¡p ¡n D dC¥u 47. Cho h¼nh nân câ chi·u cao b¬ng 6. Mët m°t ph¯ng i qua ¿nh h¼nh nân v  c­t h¼nhnân theo mët thi¸t di»n l  tam gi¡c vuæng c¥n câ c¤nh huy·n b¬ng 10p 2. T½nh thº t½ch cõa khèinân ÷ñc giîi h¤n bði h¼nh nân ¢ cho b¬ng A 32p 5 3. B 32. C 32p 3 . D 128. ÊLíi gi£i.Gi£ sû thi¸t di»n l  tam gi¡c vuæng c¥n S ABcâ c¤nh b¬ng lnh÷ h¼nhv³ ) lp 2 = 10p 2) l= 10 .Ta câ: r= OB =p S B2� S O 2= p l2� h2= 8 .) Thº t½ch khèi nân: V= 1 3r2h = 1 3826 = 128 . A BSO HMChån ¡p ¡n D 95/305 95/305 pGV: L¶ Quang Xe  Ô0967.003.131— SÈ 6 NÌI N€O CÂ Þ CH, NÌI  C CON ×ÍNG96dC¥u 48. Chox, y l  c¡c sè thüc d÷ìng thäa m¢n logx+ log y log( x2+ y). T¼m gi¡ trà nhänh§t cõa P= 2 x+ y. A 3 + 2p 6. B 4 + 2p 3. C 8. D 5 + 3p 2. ÊLíi gi£i.i·u ki»n x >0, y > 0. Ta câ logx+ log y log( x2+ y) , xy x2+ y ( ).Tø ( ) ) y(x � 1) x2> 0) x > 1) P� 2x < P �2. M  y= P� 2x > 0. Suy ra P >2.Ta câ ( ) , x(P � 2x )  x2+ P � 2x , 3x 2� (2 + P)x + P  0 ( ). B§t ph÷ìng tr¼nh ( )l  b§tph÷ìng tr¼nh bªc hai ©n x. º ( )câ nghi»m xth¼ 0, P2� 8P + 4 0, –P  4 + 2 p 3P  4� 2p 3(lo¤i v¼ P >2):Vîi P= 4 + 2 p 3th¼ ( ), x= 3 + p 33(thäa m¢nx >0). Khi â y= 6 + 4 p 33(thäa m¢ny>0).Vªy minP= 4 + 2 p 3.Chån ¡p ¡n B dC¥u 49. Trong khæng gian cho mët h¼nh c¦u (S ) t¥m Ocâ b¡n k½nh Rv  mët iºm Schotr÷îc sao cho S O= 2R. Tø Sk´ c¡c ti¸p tuy¸n vîi m°t c¦u vîi ti¸p iºm thuëc ÷íng tron (C1).Tr¶n m°t ph¯ng (P )chùa ÷íng trán (C1)ta l§y iºm Ethay êi n¬m ngo i m°t c¦u (S ). Gåi Nl  h¼nh nân câ ¿nh l  Ev  ¡y l  ÷íng trán (C2)gçm c¡c ti¸p iºm cõa ti¸p tuy¸n k´ tø E¸nm°t c¦u (S ). Bi¸t r¬ng hai ÷íng trán (C1)v  (C2)luæn câ còng b¡n k½nh. T½nh theo Rb¡n k½nhR 0cõa ÷íng trán cè ành m  Edi ëng tr¶n â. A R0= Rp 154. B R0= Rp 152. C R0= 3R 2. D R0= Rp 172. ÊLíi gi£i. DSOCRr1 Gåi b¡n k½nh cõa(C1), (C2)l¦n l÷ñt l  r1,r2. GåiCl  t¥m cõa (C1)v  Dl  mët iºm tr¶n (C1).Ta câ 4OS D vuæng t¤i Dn¶n C DOS =DO DS .Do â r1 =C D =Rp OS2� R2 OS=RÉ 1� R2 OS2.T÷ìng tü ta câ r2 =RÉ 1� R2 OE2. M r1 =r2 n¶nOE=OS = 2R.Suy ra Edi ëng tr¶n ÷íng trán giao tuy¸n cõa m°t c¦u t¥m Ob¡n k½nh 2R vîi m°t ph¯ng (P ).L¤i câ OC=OD2 OS=R 2)R0= p OE2� OC 2= É 4R 2� R2 4=Rp 152.Chån ¡p ¡n B 96/305 96/305 pGV: L¶ Quang Xe  Ô0967.003.131MÖC LÖCNÌI N€O CÂ Þ CH, NÌI  C CON ×ÍNG97dC¥u 50.Cho h m sè y= 1 4x4+ ax 3+ bx2+ cx +dcâ ç thà cõa h m y= f0( x )nh÷ h¼nh v³ b¶n. Sè iºm cüc trà cõa h m sè y= f(f 0( x )) l  A 11. B 7. C 9. D 5. xyO�1 2ÊLíi gi£i.Ta câ f0( x ) = x3+ 3 ax2+ 2 bx+cv  ç thà f0( x ) c­t tröc ho nh t¤i 3 iºm câ ho nh ë l¦n l÷ñt b¬ng� 1; 0; 2 , do â ta câ f0( x ) = ( x+ 1)( x� 0)( x� 2) = x3� x2� 2x .Do â y0= f00(x ) f 0( f 0( x )) = (3 x2� 2x � 2)( x3� x2� 2x + 1)( x3� x2� 2x )( x3� x2� 2x � 2).Ph÷ìng tr¼nh y0= 0 câ 9 nghi»m bëi l´ ph¥n bi»t.Vªy h m sè y= f(f 0( x )) câ 9 iºm cüc trà.Chån ¡p ¡n C H˜T97/305 97/305 pGV: L¶ Quang Xe  Ô0967.003.131— SÈ 7 NÌI N€O CÂ Þ CH, NÌI  C CON ×ÍNG98BË GIO DÖC & €O T„OTR×ÍNG THPT NGUY™N T‡T TH€NH GV: L– QUANG XE - 0967.003.131 — SÈ 7 PHT TRIšN — THAM KHƒO 2022N‹M HÅC 2021 - 2022Mæn:To¡nThíi gian l m b i: 90 phót — THAM KHƒO PTMH2022dC¥u 1. Cho sè phùc z= 3 �2i. Ph¦n £o cõa sè phùc zb¬ng A �2. B 3. C 2. D �2i. ÊLíi gi£i.Sè phùc câ d¤ng z= a+ bi(a; b 2R)th¼ câ ph¦n £o l  b.Vîi z= 3 �2i n¶n ph¦n £o l  �2.Chån ¡p ¡n A dC¥u 2. Trong khæng gian tåa ë Oxyz, cho m°t c¦u (S ) câ ph÷ìng tr¼nh x2+ y2+ z2� 2x +4 y + 4 z� 16 = 0 . Tåa ë t¥m Icõa m°t c¦u l  A I(� 2; 4; 4) . B I(1; �2; �2) . C I(� 1; 2; 2) . D I(2; �2; �4) . ÊLíi gi£i.M°t c¦u (S ) : x2+ y2+ z2� 2x + 4 y+ 4 z� 16 = 0 câ t¥m I(1; �2; �2) .Chån ¡p ¡n B dC¥u 3. H m sè n o sau ¥y câ tªp x¡c ành D=R? A y= p x� 1 x2+ 1 . B y= x� 1 2x � 1. C y= x3� 2x 2+ 1 . D y= p x3+ 1 . ÊLíi gi£i.H m sè y= x3� 2x 2+ 1 câ tªp x¡c ành D=R.Chån ¡p ¡n C dC¥u 4. T½nh di»n t½ch m°t c¦u b¡n k½nh r= 1 . A S= . B S= 4 . C S= 4 2. D S= 4 3. ÊLíi gi£i.�p döng cæng thùc t½nh di»n t½ch m°t c¦u b¡n k½nh r, ta câ S= 4 r2= 4 .Chån ¡p ¡n B dC¥u 5. Hå nguy¶n h m cõa h m sè y= cos 3 xl  A sin 3x 3+C. B �sin 3x 3+C. C sin 3x+ C. D �sin 3 x+ C. ÊLíi gi£i.�p döng cæng thùc Zcos( ax+b) d x= sin(ax+b) a+C ta câ Zcos 3 xdx = sin 3x 3+C:Chån ¡p ¡n A 98/305 98/305 pGV: L¶ Quang Xe  Ô0967.003.131PHT TRIšN — THAM KHƒO 2022 NÌI N€O CÂ Þ CH, NÌI  C CON ×ÍNG99dC¥u 6. Cho h m sè y= f(x ) câ b£ng x²t d§u ¤o h m f0( x ) nh÷ sau xf0( x ) �1�1 0 1 2 +1 �0 +0 �0 +0 +Sè iºm cüc trà cõa h m sè ¢ cho l A 4. B 2. C 3. D 1. ÊLíi gi£i.¤o h m cõa h m sè êi d§u khi qua c¡c iºm x= �1, x = 0 ,x = 1 n¶n h m sè ¢ cho câ 3iºmcüc trà.Chån ¡p ¡n C dC¥u 7. Tªp nghi»m cõa b§t ph÷ìng tr¼nh log2(2x+ 1) 1l  A ��1 ;1 2˜. B �� 1 2; +1‹. C �� 1 2;1 2˜. D ��1 ;1 2‹. ÊLíi gi£i.i·u ki»n x¡c ành: 2x + 1 >0, x > �1 2.B§t ph÷ìng tr¼nh ¢ cho t÷ìng ÷ìng vîi 2x + 1 2, x 1 2.Do â tªp nghi»m cõa b§t ph÷ìng tr¼nh l  �� 1 2;1 2˜.Chån ¡p ¡n C dC¥u 8. Mët khèi l¥p ph÷ìng câ thº t½ch b¬ng 3p 3a 3, th¼ c¤nh cõa khèi lªp ph÷ìng â b¬ng A ap 3. B 3a . C 3p 3a . D ap 33. ÊLíi gi£i.Gåi ë d i c¤nh cõa h¼nh lªp ph÷ìng l  x(vd)Khi â: V=x3= 3 p 3a 3) x= ap 3Chån ¡p ¡n A dC¥u 9. Tªp x¡c ành cõa h m sè y= ( x2� 3x + 2) l  A (1; 2). B (�1 ; 1)[(2; + 1). C (�1 ; 1][[2; + 1). D Rn f 1; 2 g. ÊLíi gi£i.i·u ki»n x2� 3x + 2 >0, x < 1ho°c x >2.Vªy tªp x¡c ành l  D= ( �1 ; 1)[(2; + 1).Chån ¡p ¡n B dC¥u 10. T¼m t§t c£ c¡c gi¡ trà cõa tham sè mº ph÷ìng tr¼nh 6x= 2020 �m câ nghi»m. A m2(�1 ; 2020) . B m2(�1 ; +1). C m2(2020; + 1). D m2(�1 ; 2020] . ÊLíi gi£i.Ph÷ìng tr¼nh ¢ cho câ nghi»m khi v  ch¿ khi 2020�m > 0hay m < 2020.Chån ¡p ¡n A 99/305 99/305 pGV: L¶ Quang Xe  Ô0967.003.131— SÈ 7 NÌI N€O CÂ Þ CH, NÌI  C CON ×ÍNG100dC¥u 11. Cho h m sè y= f(x ) li¶n töc tr¶n Rthäa m¢n 3Z1 f(x ) d x= 8 ,3Z2 f(x ) d x= 5 . Gi¡trà cõa t½ch ph¥n 2Z1 f(x ) d xb¬ng A �3. B 40. C 3. D 13. ÊLíi gi£i.3Z1 f(x ) d x= 2Z1 f(x ) d x+ 3Z2 f(x ) d x, 2Z1 f(x ) d x= 3Z1 f(x ) d x� 3Z2 f(x ) d x= 8 �5 = 3 .Chån ¡p ¡n C dC¥u 12. Cho hai sè phùc z1 = 4�3i v  z2 = 7 + 3i. T¼m sè phùc z= z1 �z2. A z= 11 . B z= 3 + 6 i. C z= �1� 10i. D z= �3� 6i. ÊLíi gi£i.z = z1 �z2 = (4�3i) � (7 + 3 i) = (4 �7) + ( �3i � 3i) = �3� 6i.Chån ¡p ¡n D dC¥u 13. Trong khæng gian Oxyz, cho m°t ph¯ng (P ) : 1 2x� 2y + z+ 5 = 0 . V²c-tì n o d÷îi¥y l  v²c-tì ph¡p tuy¸n cõa m°t ph¯ng (P )? A #n = (1; �2; 1) . B #n = (1; �4; 2) . C #n = (2; �2; 1) . D #n = ( �2; 1; 5) . ÊLíi gi£i.Mët v²c-tì ph¡p tuy¸n cõa m°t ph¯ng (P )l  #n = (1; �4; 2) .Chån ¡p ¡n B dC¥u 14. Trong khæng gian vîi h» tåa ë (Oxyz ), cho # OA = 3#k � #i . Tåa ë cõa iºm Al  A A(3; 0; �1) . B A(� 1; 0; 3) . C A(� 1; 3; 0) . D A(3; �1; 0) . ÊLíi gi£i.Ta câ # OA = 3#k � #i ) A(� 1; 0; 3) .Chån ¡p ¡n B dC¥u 15.iºm Mtrong h¼nh b¶n l  iºm biºu di¹n cõa sè phùc z.M»nh · n o sau ¥y óng? A Sè phùczcâ ph¦n thüc l  3v  ph¦n £o l  �4. B Sè phùczcâ ph¦n thüc l  3v  ph¦n £o l  �4i. C Sè phùczcâ ph¦n thüc l  �4 v  ph¦n £o l  3. D Sè phùczcâ ph¦n thüc l  �4 v  ph¦n £o l  3i. O xy�4 M3ÊLíi gi£i.100/305 100/305 pGV: L¶ Quang Xe  Ô0967.003.131PHT TRIšN — THAM KHƒO 2022 NÌI N€O CÂ Þ CH, NÌI  C CON ×ÍNG101Sè phùczcâ ph¦n thüc l  3v  ph¦n £o l  �4.Chån ¡p ¡n A dC¥u 16. Ti»m cªn ùng cõa ç thà h m sè y= 2x + 1 x+ 1 l  A y= �1 2. B x= 1 . C x= �1. D y= 2 . ÊLíi gi£i.Ta câ limx !� 1 2x + 1 x+ 1 =1 n¶n ç thà h m sè câ ti»m cªn ùng l  x= �1.Chån ¡p ¡n C dC¥u 17. T½nh gi¡ trà cõa bi¸u thùc K= logap ap avîi 0< a 6= 1 . A K=4 3. B K=3 2. C K=3 4. D K=�3 4. ÊLíi gi£i.Ta câ K= loga€a 1 2a 1 4Š= log aa 3 4= 3 4.Chån ¡p ¡n C dC¥u 18.ç thà cõa h m sè n o d÷îi ¥y câ d¤ng nh÷ ÷íng cong h¼nh b¶n? A y= x3� 3x 2. B y= �x4+ 2 x2� 2. C y= �x4+ 2 x2+ 2 . D y= �x4� 2. O xyÊLíi gi£i.D¹ th§y ç thà h m sè ¢ cho l  ç thà h m sè bªc bèn tròng ph÷ìng câ h» sè a <0(lo¤i A).M°t kh¡c ç thà h m sè c­t tröc tung t¤i iºm câ tung ë ¥m n¶n d <0(lo¤i C).ç thà h m sè câ 3 iºm cüc trà n¶n ab <0.Suy ra h m sè c¦n t¼m l  y= �x4+ 2 x2� 2.Chån ¡p ¡n B dC¥u 19. Trong khæng gian Oxyz, iºm n o d÷îi ¥y thuëc ÷íng th¯ng (d ) : x+ 1 1=y+ 1 2=z � 1 3? A Q(1; �2; �1) . B N(� 1; 3; 2) . C P(0; 1; 4) . D M(1; 2; 1) . ÊLíi gi£i.L¦n l÷ñt thay tåa ë 4iºm Q,N ,P v  Qv o ph÷ìng tr¼nh ÷íng th¯ng (d ).Ta th§y P(0; 1; 4) l  thäa m¢n, tùc l  0 + 1 1=1 + 1 2=4� 1 3.101/305 101/305 pGV: L¶ Quang Xe  Ô0967.003.131— SÈ 7 NÌI N€O CÂ Þ CH, NÌI  C CON ×ÍNG102Vªy ÷íng th¯ng(d ) : x+ 1 1=y+ 1 2=z� 1 3i qua iºmP(0; 1; 4) .Chån ¡p ¡n C dC¥u 20. Trong m°t ph¯ng cho 10iºm ph¥n bi»t sao cho khæng câ ba iºm n o th¯ng h ng.Häi câ bao nhi¶u tam gi¡c ÷ñc t¤o th nh m  ¿nh tròng vîi c¡c iºm ¢ cho? A C310 . B A710 . C 103. D C310 . ÊLíi gi£i.Sè c¡c tam gi¡c t¤o th nh l  C310 .Chån ¡p ¡n A dC¥u 21. Cho khèi châp tù gi¡c ·u câ thº t½ch b¬ng 16cm3v  c¤nh ¡y b¬ng 4cm, chi·u caocõa khèi châp â b¬ng A 3p 2cm. B 4cm. C 3cm. D 2p 3cm. ÊLíi gi£i.¡y l  h¼nh vuæng c¤nh b¬ng 4cm câ di»n t½ch b¬ngS = 4 2= 16 cm2.V = 1 3Sh ) h= 3V S= 3cm. 4cm SAB OC DChån ¡p ¡n C dC¥u 22. ¤o h m cõa h m sè y= e 2x +1l  A y0= 4 e2x. B y0= 2 e2x +1. C y0= e2x +1 ln 2. D y0= 2e2x +1 ln 10. ÊLíi gi£i.Ta câ y0= 2 e2x +1.Chån ¡p ¡n B dC¥u 23. Cho h m sè f(x ) câ b£ng bi¸n thi¶n nh÷ sau xf0( x ) f(x ) �1�1 1 +1 �0 +0 �+1 +1 �2 �2 22�1�1M»nh · n o d÷îi ¥y óng?A H m sèy= f(x ) çng bi¸n tr¶n kho£ng (� 2; 2) . B H m sèy= f(x ) nghàch bi¸n tr¶n kho£ng (� 1; + 1).102/305 102/305 pGV: L¶ Quang Xe  Ô0967.003.131PHT TRIšN — THAM KHƒO 2022 NÌI N€O CÂ Þ CH, NÌI  C CON ×ÍNG103C H m sèy= f(x ) nghàch bi¸n tr¶n kho£ng (�1 ; 1). D H m sèy= f(x ) çng bi¸n tr¶n kho£ng (� 1; 1) . ÊLíi gi£i.Düa v o b£ng bi¸n thi¶n, ta th§y f0( x ) > 08x 2 (� 1; 1) , do â h m sè y= f(x ) çng bi¸n tr¶n( � 1; 1) .Chån ¡p ¡n D dC¥u 24. H¼nh trö câ b¡n k½nh ¡y b¬ng av  chi·u cao b¬ng ap 3. Khi â di»n t½ch to n ph¦ncõa h¼nh trö b¬ng A 2a 2(p 3� 1). B a2(1 + p 3). C a2p 3. D 2a 2(1 + p 3). ÊLíi gi£i.Ta câ Stp = 2S¡y +Sxq = 2a2+ 2 aa p 3 = 2a2(1 + p 3):Chån ¡p ¡n D dC¥u 25. Cho t½ch ph¥n I= 2Z0 f(x ) d x= 2 . T½nh t½ch ph¥n J= 2Z0 [3f(x ) � 2] d x. A J= 6 . B J= 2 . C J= 8 . D J= 4 . ÊLíi gi£i.Ta câ J= 2Z0 [3f(x ) � 2] d x= 3 2Z0 f(x ) d x� 2 2Z0 dx = 3 2 � (2x) 20 = 6�4 = 2 .Chån ¡p ¡n B dC¥u 26. Cho c§p sè cëng (un), bi¸t u2 = 3v u4 = 7. Gi¡ trà cõa u2019 b¬ng A 4040. B 4400. C 4038. D 4037. ÊLíi gi£i.Ta câ u4 =u2 + 2d, d= 2 . Khi â u2019 =u2 + 2017d= 3 + 2017 2 = 4037 .Chån ¡p ¡n D dC¥u 27. H m sèf(x ) = e 3xcâ nguy¶n h m l  h m sè n o sau ¥y? A y= 3e 3x+ C. B y= (3e) x+ C. C y= e 3x+ C. D y= 1 3e3x+ C. ÊLíi gi£i.Ta câ Ze3xdx = 1 3e3x+ C.Chån ¡p ¡n D dC¥u 28.103/305 103/305 pGV: L¶ Quang Xe  Ô0967.003.131— SÈ 7 NÌI N€O CÂ Þ CH, NÌI  C CON ×ÍNG104Düa v o b£ng bi¸n thi¶n cõa h m sèy = f(x ) nh÷ h¼nh v³ b¶n, h¢y chånkh¯ng ành óng. xy0 y �1�2 3 +1 + �0 +�1�1 +1 +1 A H m sèf(x ) khæng x¡c ành t¤i x= 3 . B H m sèf(x ) câ 1cüc trà. C H m sèf(x ) câ 2cüc trà. D H m sèf(x ) khæng câ cüc trà. ÊLíi gi£i.Düa v o b£ng bi¸n thi¶n ta th§y y= f(x ) câ mët cüc trà.Chån ¡p ¡n B dC¥u 29. Cho h m sè y= f(x ) câ ¤o h m tr¶n o¤n [a ;b]. Ta x²t c¡c kh¯ng ành sau(i)N¸u h m sè f(x ) ¤t cüc ¤i t¤i iºm x0 2(a ;b) th¼ f(x0)l  gi¡ trà lîn nh§t cõa f(x ) tr¶n[ a ;b].(ii)N¸u h m sè f(x ) ¤t cüc ¤i t¤i iºm x0 2(a ;b) th¼ f(x0)l  gi¡ trà nhä nh§t cõa f(x ) tr¶n[ a ;b].(iii)N¸u h m sè f(x ) ¤t cüc ¤i t¤i iºm x0 v  ¤t cüc tiºu t¤i iºmx1 (x0; x12(a ;b)) th¼ taluæn câ f(x0)> f (x1).Sè kh¯ng ành óng l ? A 1. B 2. C 0. D 3. ÊLíi gi£i.Düa v o h¼nh v³ minh håa d÷îi ¥y. Ta câ c¡c kh¯ng ành ¢ cho ·u sai. xyOa x0 x1 bf(x0) f(x1) f(a ) f(b ) Chån ¡p ¡n C dC¥u 30. H m sè n o d÷îi ¥y çng bi¸n tr¶n kho£ng (�1 ; +1)? A y= 3 x3+ 3 x� 2. B y= 2 x3� 5x + 1 . C y= x4+ 3 x2. D y= x� 2 x+ 1 . ÊLíi gi£i.Ì X²t y= 3 x3+ 3 x� 2câ y0= 9 x2+ 2 >0;8 x 2 R n¶n chån y= 3 x3+ 3 x� 2.104/305 104/305 pGV: L¶ Quang Xe  Ô0967.003.131PHT TRIšN — THAM KHƒO 2022 NÌI N€O CÂ Þ CH, NÌI  C CON ×ÍNG105ÌX²t y= 2 x3� 5x + 1 cây0= 6 x2� 5; y 0= 0 l  ph÷ìng tr¼nh bªc 2câ nghi»m n¶n khæng thºçng bi¸n tr¶n (�1 ; +1).Ì X²t y= x4+ 3 x2câ y0= 4 x3+ 6 x;y 0= 0 câ nghi»m x= 0 n¶ny0s³ êi d§u khi qua x= 0 n¶nkhæng thº çng bi¸n tr¶n (�1 ; +1).Ì X²t y= x� 2 x+ 1 câ tªp x¡c ành l D=Rn f� 1g n¶n khæng thº çng bi¸n tr¶n (�1 ; +1).Chån ¡p ¡n A dC¥u 31. °ta= log126, b = log127. H¢y biºu di¹n log27theo av  b. A ba+ 1 . B b1� a. C ab� 1. D ab+ 1 . ÊLíi gi£i.log 27 = log127 log122=log127 log1212 6=log127 1� log126=b 1� a.Chån ¡p ¡n B dC¥u 32.Cho h¼nh lªp ph÷ìng ABC D:A0B 0C 0D 0(tham kh£o h¼nh v³ b¶n).Gâc giúa hai ÷íng th¯ng ACv A0D b¬ng A 45. B 30. C 60. D 90. A0 B B0 C C0 ADD0 ÊLíi gi£i.Ta câ: ACA0C 0) (AC; A 0D ) = ( A0C 0; A 0D ).M°t kh¡c: A0C 0= A0D =DC 0= ap 2n¶n suy ra 4A0DC 0·u.Do â (A 0C 0; A 0D ) = 60 . A0 B B0 C C0 ADD0 60 Chån ¡p ¡n C dC¥u 33. Cho h m sè y= f(x ) li¶n töc tr¶n [1; 9]thäa m¢n 9Z1 f(x ) d x= 7 v 5Z4 f(x ) d x= 3 .T½nh gi¡ trà biºu thùc P= 4Z1 f(x ) d x+ 9Z5 f(x ) d x. A 4. B 3. C 10. D 2.105/305 105/305 pGV: L¶ Quang Xe  Ô0967.003.131— SÈ 7 NÌI N€O CÂ Þ CH, NÌI  C CON ×ÍNG106ÊLíi gi£i.P = 4Z1 f(x ) d x+ 9Z5 f(x ) d x= 9Z1 f(x ) d x� 5Z4 f(x ) d x= 7 �3 = 4 .Chån ¡p ¡n A dC¥u 34. Trong khæng gian vîi h» tåa ë Oxyz, cho iºm M(� 2; 4; 2) . Vi¸t ph÷ìng tr¼nhm°t ph¯ng (P ) i qua 3iºm M1;M2;M3l¦n l÷ñt l  h¼nh chi¸u cõaMtr¶n c¡c tröc tåa ëOx; Oy; Oz . A (P ) : x �2 +y 4+z 2= 0. B (P ) : x 2+y �4 +z �2 = 1. C (P ) : x �1 +y 2+z 1= 1. D (P ) : x �2 +y 4+z 2= 1. ÊLíi gi£i.Ta câ M1(� 2; 0; 0) ,M2(0; 4; 0),M3(0; 0; 2))(P ) : x �2 +y 4+z 2= 1.Chån ¡p ¡n D dC¥u 35. Cho sè phùc z. ¯ng thùc n o sau ¥y sai? A jz j = j zj. B z z= jz j2. C z� zil  sè thu¦n £o. D z+ zl  sè thüc. ÊLíi gi£i.Vîi z= x+ yi (x , y 2 R) th¼ z= x� yi. Khi âÌ jz j = p x2+ y2= j zj.Ì z z= ( x+ yi)(x � yi) = x2+ y2= jz j2.Ì z� zi=x+ yi �(x � yi) i= 2yl  sè thüc.Ì z+ z= x+ yi +x� yi = 2 xl  sè thüc.Chån ¡p ¡n C dC¥u 36.Cho h¼nh châp ·u S:ABC Dcâ c¤nh ¡y b¬ng av  c¤nh b¶n t¤ovîi ¡y mët gâc 30. T½nh kho£ng c¡ch tø ¿nh S¸n m°t ph¯ng( ABC D ). A ap 66. B ap 33. C ap 62. D ap 32. B AC DSÊLíi gi£i.106/305 106/305 pGV: L¶ Quang Xe  Ô0967.003.131PHT TRIšN — THAM KHƒO 2022 NÌI N€O CÂ Þ CH, NÌI  C CON ×ÍNG107GåiO=AC \BD .S:ABC D l  h¼nh châp ·u suy ra S O?(ABC D ). Suy ra OCl  h¼nhchi¸u cõa S Cl¶n m°t ph¯ng (ABC D ). Gâc giúa S Cv (ABC D )l ÕS C O = 30.Kho£ng c¡ch tø S¸n m°t ph¯ng (ABC D ).d ( S; ( ABC D )) =S O=OC tan ÕS C O =a p21 p3=ap 66: B AC DOSChån ¡p ¡n A dC¥u 37. Mët tói chùa 6bi xanh, 4bi ä. L§y ng¨u nhi¶n 2bi. T½nh x¡c su§t º ÷ñc c£ haibi ·u m u ä. A 512. B 215. C 745. D 815. ÊLíi gi£i.Gåi Al  bi¸n cè l§y ÷ñc c£ hai vi¶n bi m u ä ta câ n(A ) = C 14 C 13 = 12v  khæng gian m¨un () = C 210 = 90:Vªy x¡c su§t c¦n t¼m l  P(A ) = 12 90=2 15:Chån ¡p ¡n B dC¥u 38. Trong khæng gian Oxyz, ÷íng th¯ng i qua iºm A(0; 2; 5) çng thíi vuæng gâcvîi hai ÷íng th¯ng d1: x� 1 �1 =y� 4 1=z+ 2 �2 v d2: 8><>: x= ty = �2� 2tz = 3 câ ph÷ìng tr¼nh l  A  :8><>: x= �ty = 2 �tz = 5 + 2 t: B  :8><>: x= �ty = 2 + 2 tz = 5 : C  :8><>: x= �4ty = 2 �2tz = 5 + t: D  :8><>: x= 4y = �2 + 2 tz = 1 + 5 t: ÊLíi gi£i.d 1 câ mët VTCP l  #u 1= (�1; 1; �2) .d 2 câ mët VTCP l  #u 2= (1;�2; 0) .Ta câ [#u 1; #u 2] = (�4; �2; 1) .V¼ ÷íng th¯ng vuæng gâc vîi hai ÷íng th¯ng d1 v d2 n¶n ta chån mët VTCP #u  = [ #u 1; #u 2] =( � 4; �2; 1) .Vªy  :8><>: x= �4ty = 2 �2tz = 5 + t:Chån ¡p ¡n C dC¥u 39. Tªp t§t c£ c¡c gi¡ trà thüc cõa xthäa m¢n b§t ph÷ìng tr¼nh 29x� 36x 6x� 4x 2l ( �1 ;a ][ (b ; c]. T½nh (a + b+ c)! . A 0. B 1. C 2. D 6. ÊLíi gi£i.107/305 107/305 pGV: L¶ Quang Xe  Ô0967.003.131— SÈ 7 NÌI N€O CÂ Þ CH, NÌI  C CON ×ÍNG108Ta câ b§t ph÷ìng tr¼nh t÷ìng ÷ìng vîi29x� 56x+ 2 4x 6x� 4x 0. °t t= �3 2‹xta câ( t � 2)(2 t� 1) t� 1 0, 24 t 1 21 < t 2:Suy ra 2664 �3 2‹x 1 21 < �3 2‹x 2, 24 x � log3 220 < x log3 22:Do â, a= �log3 22, b = 0 ,c = log3 22.Vªy (a + b+ c)! = 0! = 1 .Chån ¡p ¡n B dC¥u 40. T¼m t§t c£ c¡c gi¡ trà cõa tham sè mº ç thà h m sè y= x3� 3x 2+ mx �m c­t÷íng th¯ng y= �2 t¤i ba iºm A,B ,C sao cho AB=BC . A m= 0 . B mtòy þ. C m63. D m>3. ÊLíi gi£i.Ì X²t ph÷ìng tr¼nh ho nh ë giao iºm x3� 3x 2+ mx �m + 2 = 0 ,(x � 1)( x2� 2x + m �2) = 0 .Ì Tr÷íng hñp 1 : n¸u ph÷ìng tr¼nh x2� 2x + m �2 = 0 , câ0= 0 ,m= 3 câ nghi»mx 1 =x2 = 1khi â A B C, thäa m¢n i·u ki»n b i to¡n AB=BC = 0.Ì Tr÷íng hñp 2 : n¸u ph÷ìng tr¼nh x2� 2x + m �2 = 0 câ hai nghi»m ph¥n bi»t kh¡c 1, t÷ìng÷ìng vîi m <3, khi â AB=BC vîiB(1; �2) l  t¥m èi xùng cõa ç thà.Vªy gi¡ trà tham sè mc¦n t¼m l  m3.Chó þ : ç thà h m sè bªc ba nhªn iºm uèn l m t¥m èi xùng.Chån ¡p ¡n C dC¥u 41. X²t h m sè f(x ) = x2+ ax + ln jbx + 1 j+ cvîi a; b; c 2R. Bi¸t f0( x ) = 4x 2+ 4 x+ 3 2x + 1v  f(0) = 1 . T½nh gi¡ trà S= c(2 a� b)2. A 23. B 1. C 4. D 0. ÊLíi gi£i.f (x ) = Zf0( x ) d x= Z4x 2+ 4 x+ 3 2x + 1 dx = Z�2x + 1 + 2 2x + 1 ‹dx = x2+ x+ ln j2 x + 1 j+ C.Suy ra a= 1 ,b = 2 .L¤i câ: f(0) = 1 )C= 1 hayc= 1 . Vªy S= c(2 a� b)2= 0 .Chån ¡p ¡n D dC¥u 42. T½nh thº t½ch khèi châp tam gi¡c ·u S:ABCbi¸t chi·u cao h¼nh châp b¬ng hv ÕS B A = . A V= h3p 33 tan2 � 1. B V= h3p 21� 3 tan 2 . C V= h3p 31� 3 tan 2 . D V= h3p 33 tan2 + 1 . ÊLíi gi£i.108/305 108/305 pGV: L¶ Quang Xe  Ô0967.003.131PHT TRIšN — THAM KHƒO 2022 NÌI N€O CÂ Þ CH, NÌI  C CON ×ÍNG109°tAB= 2x. Gåi Ml  trung iºm ABv Ol  trångt¥m tam gi¡c ABC)S O ?(ABC )v  S M ?AB .X²t tam gi¡c S M Bcâtan = S M M B)S M =xtan .V¼ 4ABC ·u v  Ml  trung iºm ABn¶nC M =2x p 32=xp 3) OM =x p3.X²t tam gi¡c S OMcâS M 2= S O 2+ OM 2) x2tan 2 = h2+ x2 3) x= hp 3p3 tan2 � 1.Di»n t½ch tam gi¡c ABCl 1 22x x p 3 =3p 3h 2 3 tan2 � 1.Thº t½ch h¼nh châp S:ABCl V= p 3h 3 3 tan2 � 1. SMBCAOChån ¡p ¡n A dC¥u 43. Gåiz1 v z2 l  hai nghi»m phùc cõa ph÷ong tr¼nhz2� 2z + 10 = 0 . TinhA=j z 2i j+ jz 2i j A A= 20 . B A= 10 . C A= 30 . D A= 50 . ÊLíi gi£i.Ph÷ìng tr¼nh z2� 2z + 10 = 0(1) câ hai nghi»m phùc l  z= 1 + 3iv  z2 = 1�3i. Ta câ: A=j (1 �3i) 2j + j(1 + 3 i) 2j = j � 8� 6ij + j � 8 + 6 ij = 20 . Vªy A= 20 .Chån ¡p ¡n A dC¥u 44. X²t c¡c sè phùc zthäa m¢n j zj = jz + 2 ij, gi¡ trà nhä nh§t cõa jz � ij + jz � 4jb¬ng A 5. B 4. C 3p 3. D 6. ÊLíi gi£i.Gåi z= x+ yi,(x; y 2R)l  sè phùc thäa m¢n b i to¡n. Khi â,j zj = jz + 2 ij , È x2+ ( �y)2= È x2+ ( y+ 2) 2, 4y + 4 = 0 ,y= �1:Nh÷ th¸ jz � ij + jz � 4j = È x2+ ( y� 1)2+ È (x � 4)2+ y2= p x2+ 2 2+ È (4�x)2+ 1 2 È (x + 4 �x)2+ (2 + 1) 2= 5 :¯ng thùc x£y ra khi x 2=4� x 1,x= 8 3, tùc l z= 8 3�i.Vªy gi¡ trà nhä nh§t cõa jz � ij + jz � 4j b¬ng 5.Chån ¡p ¡n A dC¥u 45.109/305 109/305 pGV: L¶ Quang Xe  Ô0967.003.131— SÈ 7 NÌI N€O CÂ Þ CH, NÌI  C CON ×ÍNG110Cho ÷íng cong(C ) : y= 8 x� 27 x3v  ÷íng th¯ng y= m c­t( C )t¤i hai iºm ph¥n bi»t n¬m trong gâc ph¦n t÷ thù nh§t cõah» tröc tåa ë Oxyv  chia th nh 2mi·n ph¯ng (g¤ch såc v  k´caro) câ di»n t½ch b¬ng nhau (tham kh£o h¼nh v³ b¶n). M»nh· n o sau ¥y l  óng ? A 0< m < 1 2. B 12< m <1. C 1< m < 3 2. D 32< m <2. O xyy= 8 x� 27x3 y= m ÊLíi gi£i.X²t ph÷ìng tr¼nh ho nh ë giao iºm 8x � 27x3= m.Gi£ sû 8x � 27x3= m câ hai nghi»m x= a, x = bvîi 0< a < b .Suy ra 8a � 27a3= m v  8b � 27b3= m.Ta câaZ0 �m �8x + 27 x3dx = bZa �8 x � 27x3� mdx) � mb+ 4b2� 27 4b4= 0) m=�27 4b3+ 4 b) 8b � 27b3= �27 4b3+ 4 b) b= 4 9) m= 32 27:Chån ¡p ¡n C dC¥u 46. Trong khæng gian vîi h» tåa ë Oxyz, cho hai ÷íng th¯ng d1,d2 l¦n l÷ñt câ ph÷ìngtr¼nh l  x 1=y+ 1 2=z 1v x 1=y� 1 �2 =z� 1 3. ÷íng th¯ngdc­t c£ hai ÷íng th¯ng d1,d2 v song song vîi ÷íng th¯ng  :x� 4 1=y� 7 4=z� 3 �2 câ ph÷ìng tr¼nh l  A x+ 1 1=y� 1 4=z+ 4 �2 . B x� 1 1=y� 1 4=z� 4 �2 . C x� 1 1=y+ 1 4=z� 4 �2 . D x+ 1 1=y+ 1 4=z+ 4 �2 . ÊLíi gi£i.Gi£ sû dc­t d1; d2l¦n l÷ñt t¤iA,B . Hai ÷íng th¯ng d1,d2 câ ph÷ìng tr¼nh tham sè l¦n l÷ñt l d 1: 8><>: x= ty = �1 + 2 tz = t ;(t 2 R) d2: 8><>: x= my = 1 �2mz = 1 + 3 m;(m 2R):A 2d1 )A(t; � 1 + 2 t; t) v  B2d2 )B(m ; 1 �2m ; 1 + 3 m).110/305 110/305 pGV: L¶ Quang Xe  Ô0967.003.131PHT TRIšN — THAM KHƒO 2022 NÌI N€O CÂ Þ CH, NÌI  C CON ×ÍNG111Khi â# AB = (m�t; � 2m �2t + 2; 3 m�t+ 1) .÷íng th¯ng câ v²c-tì ch¿ ph÷ìng #u = (1; 4; �2) .V¼ d  n¶n m�t 1=�2m �2t + 2 4=3m �t+ 1 �2 ,¨m = 1t = 2 )B(1; �1; 4) .Vªy di qua Bv  câ v²c-tì ch¿ ph÷ìng #u n¶n câ ph÷ìng tr¼nh x� 1 1=y+ 1 4=z� 4 �2 .Chån ¡p ¡n C dC¥u 47. Cho m°t c¦u t¥m O, b¡n k½nh R. H¼nh trö (H )câ b¡n k½nh ¡y l  rnëi ti¸p m°tc¦u. Thº t½ch khèi trö ÷ñc t¤o n¶n bði (H )câ thº t½ch lîn nh§t khi rb¬ng A r= p 3R . B r= p 22R. C r= p 6R . D r= p 63R. ÊLíi gi£i.H¼nh trö nëi ti¸p trong m°t c¦u câ t¥m ¡y l  E, câ b¡n k½nh E A=r(0 <r < R ), ÷íng cao K E= 2E I.X²t tam gi¡c vuæng I E AcâI E =p I A2� E A 2= p R2� r2.Thº t½ch khèi trö l  V=r 2h = r 22I E = 2r2p R2� r2.X²t h m sè y= r2p R2� r2vîi ( 0< r < R ).Câ y0= 2 rp R2� r2+ r2 �2r 2p R2� r2 = 2rp R2� r2� r3 pR2� r2 =2 rR 2� 3r3 pR2� r2 . KIEAy0= 0 ,2rR 3� 3r3= 0 ,r(2 R2� 3r2) = 0 ,r= p 63R.B£ng bi¸n thi¶n ry0 y 0 p63R R+0 �Nh¼n b£ng bi¸n thi¶n ta th§yy y‚p 63RŒ) ymax =y‚p 63RŒ.D§u b¬ng x£y ra ,r= p 63R. Vªy thº t½ch h¼nh trö lîn nh§t ,ymax ,r= p 63R.Chån ¡p ¡n D dC¥u 48. Cho h m sè y= f(x ) li¶n töc tr¶n o¤n [� 1; 9] v  câ ç thà l  ÷íng cong trong h¼nhv³ d÷îi ¥y111/305 111/305 pGV: L¶ Quang Xe  Ô0967.003.131— SÈ 7 NÌI N€O CÂ Þ CH, NÌI  C CON ×ÍNG112xy�1 1 2 3 4 5 6 7 8 9�4 �3 �2 �1 12OCâ t§t c£ bao nhi¶u gi¡ trà nguy¶n cõa tham sèmº b§t ph÷ìng tr¼nh 163 f(x )� [f 2(x ) + 2 f(x )�8] 4f(x ) (m 2� 3m )6f(x )nghi»m óng vîi måi gi¡ trà 2[� 1; 9] ? A 32. B 31. C 5. D 6. ÊLíi gi£i.+) Tø ç thà h m sè suy ra �4 f(x )  2,8 x 2 [� 1; 9] , °t t= f(x ) ) t2 [� 4; 2] .+) Ta t¼m m º 163t� [t 2+ 2 t� 8]4t (m 2� 3m )6tóng 8t 2 [� 4; 2], 16 2t �[t 2+ 2 t� 8]�2 3‹t m2� 3m; 8t 2 [� 4; 2] .Ta câ 16 2t 4; 8t 2 [� 4; 2] , d§u b¬ng x£y ra khi t= 2 .M  t2+ 2 t� 8 0; 8t 2 [� 4; 2] )[t 2+ 2 t� 8]�2 3‹t 0; 8t 2 [� 4; 2] , d§u b¬ng x£y ra khi t= 2 .) 16 2t �[t 2+ 2 t� 8]�2 3‹t 4; 8t 2 [� 4; 2] .Vªy m2� 3m 4, � 1 m 4, m  mnguy¶n n¶n m2 f� 1; 0; 1; 2; 3; 4 g.Chån ¡p ¡n D dC¥u 49. Cho h¼nh nân ¿nh S, ¡y l  ÷íng trán (O ;r ). Mët m°t ph¯ng i qua ¿nh cõa h¼nhnân c­t ÷íng trán ¡y t¤i hai iºm Av  Bsao cho S A=AB =8r 5. T½nh theorkho£ng c¡ch tøO ¸n m°t ph¯ng (S AB )? A 2p 2r 5. B 3p 13r 20. C 3p 2r 20. D p13r 20. ÊLíi gi£i.112/305 112/305 pGV: L¶ Quang Xe  Ô0967.003.131MÖC LÖCNÌI N€O CÂ Þ CH, NÌI  C CON ×ÍNG113Ta câ4S AB ·u c¤nh 8r 5)S I =p 328 5r= 4p 35r:Gåi Il  trung iºm cõa AB)¨AB ?OIAB ?S I) AB ?(S I O )) (S I O )? (S AB ):Tø Odüng OH?S I )OH ?(S AB )) d(O ; ( S AB )) =OH:I O =p AO2� AI 2= Ê r2� �4r 5‹2= 3 5r:) OH =OIS O S I=3r 5p S I2� OI 2 4p 3r 5= 3r 5s ‚4p 3r 5Œ2� �3 5‹2 4p 3r 5=3p 1320r: HBAI SOChån ¡p ¡n B dC¥u 50. Cho hai h m sè f(x ) v  g(x ) thäa m¢n8>><>>: g(x ) = x+ 5 �p x+ 5f (1) <5p 3f (x ) = È g(x ) (10 f(x ) � 3) + f(x ) � 3g 2(x ) ;8 x  � 5:H m sè y= f(x ) câ bao nhi¶u iºm cüc trà? A 4. B 1. C 3. D 2. ÊLíi gi£i.Vîi i·u ki»n x2 [� 5; + 1), ta câ3 f 2(x ) = 10 g(x ) f (x ) � 3g (x ) + f(x ) � 3g 2(x ), 3f (x ) ( f(x ) � 3g (x )) � (f (x ) � 3g (x )) � g(x ) ( f(x ) � 3g (x )) = 0, [f (x ) � 3g (x )] [3 f(x ) � 1� g(x ))] = 0, 24 f(x ) = 3 g(x )f (x ) = 1 3+1 3g(x ):Tø gi£ thi¸t f(1) <5suy ra f(x ) = 1 3+1 3g(x ).Ta câ f0( x ) = 1 3g0( x ) = 1 3�1 6p x+ 5 =1 32p x+ 5 �1 2p x+ 5 .Ta câ f0( x ) = 0 ,p x+ 5 = 1 2,x= �19 4.M°t kh¡c f00(x ) = 1 121 p(x + 5) 3>0suy ra h m sè f(x ) ¤t cüc tiºu t¤i x= �19 4.Chån ¡p ¡n B H˜T113/305 113/305 pGV: L¶ Quang Xe  Ô0967.003.131— SÈ 8 NÌI N€O CÂ Þ CH, NÌI  C CON ×ÍNG114BË GIO DÖC & €O T„OTR×ÍNG THPT NGUY™N T‡T TH€NH GV: L– QUANG XE - 0967.003.131 — SÈ 8 PHT TRIšN — THAM KHƒO 2022N‹M HÅC 2021 - 2022Mæn:To¡nThíi gian l m b i: 90 phót — THAM KHƒO PTMH2022dC¥u 1. Cho sè phùc z= 6 �7i. T¼m sè phùc li¶n hñp cõa sè phùc z. A z= �6 + 7 i. B z= �6� 7i. C z= 6 + 7 i. D z= �i. ÊLíi gi£i.Ta câ z= 6 + 7 i.Chån ¡p ¡n C dC¥u 2. Trong khæng gian vîi h» tåa ë Oxyz, cho m°t c¦u (S ) : x2+ y2� 2x + 4 y� 6z � 11 = 0 .T¼m tåa ë t¥m Iv  b¡n k½nh Rcõa (S ). A I(1; �2; 3) ; R=p 3. B I(1; �2; 3) ; R= 5 . C I(� 1; 2; �3) ; R =p 3. D I(� 1; 2; �3) ; R = 5 . ÊLíi gi£i.Ta câ: a= 1 ; b=�2; c = 3 ; d=�11 . Vªy t¥m I(1; �2; 3) ; R=p a2+ b2+ c2� d= 5 .Chån ¡p ¡n B dC¥u 3. ç thà h m sè y= x3+ 3 x2� 2nhªn : A Tröc tung l m tröc èi xùng. B Gèc tåa ë O l m t¥m èi xùng.C iºmI(� 1; 0) l m t¥m èi xùng. D ÷íng th¯ngx= 1 l m tröc èi xùng. ÊLíi gi£i.Ta câ: y0= 3 x2+ 6 x= 0y 00= 6 x+ 6 = 0 ,x= �1) y= 0 .H m sè y= x3+ 3 x2� 2l  h m a thùc bªc ba n¶n nhªn iºm I(� 1; 0) l m t¥m èi xùng.Chån ¡p ¡n C dC¥u 4. Cho m°t c¦u câ di»n t½ch l  72cm2. B¡n k½nh Rcõa khèi c¦u l  A R= p 6 (cm ). B R= 6 ( cm). C R= 3 ( cm). D R= 3 p 2 (cm ). ÊLíi gi£i.Ta câ S= 4 R2= 72 ) R= É 72 4 =p 18 = 3p 2 (cm ).Chån ¡p ¡n D dC¥u 5. T¼m hå nguy¶n h m cõa h m sè y= x2� 3x+ 1 x. A x3 3�3x ln 3�lnjx j+ C; C 2R. B x3 3�3x ln 3+ lnjx j+ C; C 2R. C x3 3�3x+ 1 x2 +C; C 2R. D x3 3�3x ln 3�1 x2 +C; C 2R.114/305 114/305 pGV: L¶ Quang Xe  Ô0967.003.131PHT TRIšN — THAM KHƒO 2022 NÌI N€O CÂ Þ CH, NÌI  C CON ×ÍNG115ÊLíi gi£i.Ta câ Z�x2� 3x+ 1 x‹dx = x3 3�3x ln 3+ lnjx j+ C; C 2R:Chån ¡p ¡n B dC¥u 6. Cho h m sè y= f(x ) x¡c ành li¶n töc v  li¶n töc tr¶n Rv  câ b£ng bi¸n thi¶n nh÷sau xy0 y �1�2 0 +1 +0 �0 +�1�1 00�4 �4 +1 +1 Kh¯ng ành n o sau ¥y óng?A H m sè câ hai cüc trà. B H m sè câ gi¡ trà cüc tiºu b¬ng khæng.C H m sè câ gi¡ trà cüc ¤i b¬ng�4. D H m sè câ gi¡ trà cüc ¤i t¤ix= 0 . ÊLíi gi£i.Düa v o b£ng bi¸n thi¶n, ta th§y h m sè ¤t cüc trà t¤i x= �2 v  x= 0 .Chån ¡p ¡n A dC¥u 7. Tªp nghi»m Scõa b§t ph÷ìng tr¼nh log1 5(3x� 5) >log1 5(x + 1) l  A S= (2; + 1). B S= �5 3; 3 ‹. C S= ( �1 ; 3). D S= �3 5; 3 ‹. ÊLíi gi£i.i·u ki»n 3x � 5> 0, x > 5 3:Khi âlog 1 5(3x� 5) >log1 5(x + 1) ,3x � 5< x + 1,x < 3:K¸t hñp vîi i·u ki»n ta câ tªp nghi»m cõa b§t ph÷ìng tr¼nh l  S= �5 3; 3 ‹.Chån ¡p ¡n B dC¥u 8. Bi¸t thº t½ch mët khèi lªp ph÷ìng b¬ng 16p 2a 3, vªy c¤nh cõa khèi lªp ph÷ìng ¢ chob¬ng bao nhi¶u? A 8a p 2. B 2a p 2. C 4a p 2. D ap 2. ÊLíi gi£i.V¼ khèi lªp ph÷ìng câ thº t½ch b¬ng 16p 2a 3n¶n khèi lªp ph÷ìng câ c¤nh l  3p 16p 2a 3= 2 ap 2.Chån ¡p ¡n B dC¥u 9. T¼m tªp x¡c ành cõa h m sè y= ( x2� 3x + 2) �1 3. A (�1 ; 1)[(2; + 1). B Rn f 1; 2 g. C (1; 2). D R. ÊLíi gi£i.115/305 115/305 pGV: L¶ Quang Xe  Ô0967.003.131— SÈ 8 NÌI N€O CÂ Þ CH, NÌI  C CON ×ÍNG116V¼�1 3khæng nguy¶n n¶n:y= ( x2� 3x + 2) �1 3x¡c ành khi v  ch¿ khi x2� 3x + 2 >0, –x < 1x > 2:Chån ¡p ¡n A dC¥u 10. Sè nghi»m thüc cõa ph÷ìng tr¼nh log3x= �p 2l  A 3. B 2. C 1. D 0. ÊLíi gi£i.Ta câ log3x= �p 2, x= 3 �p 2.Chån ¡p ¡n C dC¥u 11. Cho h m sè f(x ) li¶n töc tr¶n o¤n [0; 3]v 2Z0 f(x ) d x= 1 ,3Z2 f(x ) d x= 4 . T½nhI = 3Z0 f(x ) d x. A I= 5 . B I= �3. C I= 3 . D I= 4 . ÊLíi gi£i.Ta câ I= 3Z0 f(x ) d x= 2Z0 f(x ) d x+ 3Z2 f(x ) d x= 1 + 4 = 5 .Chån ¡p ¡n A dC¥u 12.Cho sè phùc z= �1 + 2 i; w= 2�i. iºm n o trong h¼nh b¶n biºu di¹n sèphùc z+ w? A P. B N. C Q. D M. xyO 1�1 1�1 M QPNÊLíi gi£i.V¼ z+ w = �1 + 2 i+ 2 �i= 1 + in¶n iºm biºu di¹n l  iºm P(1; 1) .Chån ¡p ¡n A dC¥u 13. Trong khæng gian vîi h» tåa ë Oxyzcho m°t ph¯ng (P ) : 2 x� 4y + 6 z� 1 = 0 . M°tph¯ng (P )câ mët v²c-tì ph¡p tuy¸n l  A #n = (1; �2; 3) . B #n = (2; 4; 6) . C #n = (1; 2; 3) . D #n = ( �1; 2; 3) . ÊLíi gi£i.M°t ph¯ng (P )nhªn #v = (2; �4; 6) l m v²c-tì ph¡p tuy¸n. Do â v²c-tì #n = (1; �2; 3) = 1 2#v côngl  mët v²c-tì ph¡p tuy¸n cõa (P ).Chån ¡p ¡n A 116/305 116/305 pGV: L¶ Quang Xe  Ô0967.003.131PHT TRIšN — THAM KHƒO 2022 NÌI N€O CÂ Þ CH, NÌI  C CON ×ÍNG117dC¥u 14. Trong khæng gian vîi h» tröc tåa ë Oxyz, cho v²c-tì #a biºu di¹n cõa c¡c v²c-tì ìnvà l  #a = 2 #i � 3#j + #k . T¼m tåa ë cõa vec-tì #a l  A (1; 2;�3) . B (2;�3; 1) . C (2; 1;�3) . D 91;�3; 2) . ÊLíi gi£i.Theo ành ngh¾a th¼ #u = x#i + y#j + z#k , #u = ( x;y ;z ).Do â v²c-tì #a = (2; �3; 1) .Chån ¡p ¡n B dC¥u 15.iºm Mtrong h¼nh v³ b¶n l  iºm biºu di¹n cho sè phùc z= a+ bi. M»nh· n o sau ¥y l  óng? A a= �3 v  b= 2 . B a= 2 v b= �3. C a= �3 v  b= 2 i. D a= 2 v b= �3i. xyOM�3 2ÊLíi gi£i.Tø h¼nh v³ ta câ M(2; �3) . Suy ra z= 2 �3i.Vªy a= 2 v b= �3.Chån ¡p ¡n B dC¥u 16. Ti»m cªn ngang cõa ç thà h m sè y= 5 x� 1l  ÷íng th¯ng câ ph÷ìng tr¼nh A y= 0 . B y= 5 . C x= 1 . D x= 0 . ÊLíi gi£i.Ta câ limx ! +1 5 x� 1= 0)y= 0 l  ti»m cªn ngang cõa ç thà h m sè.Chån ¡p ¡n A dC¥u 17. Choa; b > 0, a 6= 1 thäa logab= 3 . T½nh P= loga2b3. A P= 18 . B P= 2 . C P= 9 2. D P= 1 2. ÊLíi gi£i.Ta câ P= 3 2logab= 3 23 = 9 2.Chån ¡p ¡n C dC¥u 18.Gi£ sû h m sè y= ax 4+ bx 2+ ccâ ç thà l  h¼nh b¶n. Kh¯ng ành n o sau¥y l  kh¯ng ành óng? A a >0; b < 0; c = 1 . B a >0; b > 0; c = 1 . C a <0; b > 0; c = 1 . D a >0; b > 0; c > 0. xyO 1�1 1ÊLíi gi£i.Düa v o ç thà ta câ Ì ç thà h÷îng l¶n n¶n a >0.117/305 117/305 pGV: L¶ Quang Xe  Ô0967.003.131— SÈ 8 NÌI N€O CÂ Þ CH, NÌI  C CON ×ÍNG118Ìç thà h m sè câ 3iºm cüc trà n¶n ab <0suy ra b <0.Ì Vîi x= 0 )y= c= 1 .Vªy ¡p ¡n óng l  a >0; b < 0; c = 1 .Chån ¡p ¡n A dC¥u 19. Trong khæng gian vîi h» tröc tåa ë Oxyz, cho ÷íng th¯ng d: 8><>: x= 1y = 2 + 3 tz = 5 �t (t 2 R).÷íng th¯ng di qua iºm n o d÷îi ¥y? A M1(1; 5; 4). B M2(� 1; �2; �5) . C M3(0; 3;�1) . D M4(1; 2;�5) . ÊLíi gi£i.Vîi t= 1 ta câ mët iºm thuëc dl  (1; 5; 4) .Chån ¡p ¡n A dC¥u 20. Tªp hñpMcâ30 ph¦n tû. sè c¡c tªp con gçm 5ph¦n tû cõa Ml  A 305. B A430 . C C530 . D 304. ÊLíi gi£i.Ta câ méi mët tªp con gçm 5ph¦n tû cõa Ml  mët tê hñp chªp 5cõa 30ph¦n tû.Vªy sè c¡c tªp nh÷ vªy l  C530 .Chån ¡p ¡n C dC¥u 21. Thº t½ch cõa khèi châp câ di»n t½ch ¡y l  12m2v  chi·u cao 5m l  A 20m3. B 30m3. C 60m3. D 10m3. ÊLíi gi£i.Ta câ V= 1 3S¡y h = 1 312 5 = 20 m3.Chån ¡p ¡n A dC¥u 22. T½nh ¤o h m cõa h m sè y= log3(2x+ 1) . A y0= 1 (2x+ 1) ln 3 . B y0= 1 2x + 1 . C y0= 2 (2x+ 1) ln 3 . D y0= (2 x+ 1) ln 3 . ÊLíi gi£i.Ta câ y0= (2x+ 1) 0 (2x+ 1) ln 3 =2 (2x+ 1) ln 3 .Chån ¡p ¡n C dC¥u 23. Cho h m sè y= f(x ) câ b£ng bi¸n thi¶n b¶n d÷îi xy0 y �1�2 3 +1 �0 +0 �+1 +1 11 44�1�1118/305 118/305 pGV: L¶ Quang Xe  Ô0967.003.131PHT TRIšN — THAM KHƒO 2022 NÌI N€O CÂ Þ CH, NÌI  C CON ×ÍNG119H m sè ¢ cho çng bi¸n tr¶n kho£ng n o ?A (� 2; + 1). B (� 2; 3) . C (3; +1). D (�1 ;� 2) . ÊLíi gi£i.Tø b£ng bi¸n thi¶n, suy ra y0> 0khi x2 (� 2; 3) .Chån ¡p ¡n B dC¥u 24. Cho h¼nh nân câ b¡n k½nh ¡y r= 3 v  ë d i ÷íng sinh l= 4 . T½nh di»n t½ch xungquanh Sxp cõa h¼nh nân ¢ cho. A Sxq = 8 p 3 . B Sxq = 12. C Sxq = 4 p 3 . D Sxq =p 39. ÊLíi gi£i.Di»n t½ch xung quanh Sxp cõa h¼nh nân l S xp = 1 3324 = 12 . AO SChån ¡p ¡n B dC¥u 25. Câ bao nhi¶u sè thüc aº 1Z0 �4 ax 3� 3a 2x 2+ 2 x+ 1 dx = 0 ? A 2. B 0. C 1. D 3. ÊLíi gi£i.Ta câ 1Z0 �4 ax 3� 3a 2x 2+ 2 x+ 1 dx = 0 ,(ax 4� a2x 3+ x2+ x) 10 = 0, a� a2+ 2 = 0 ,–a = �1a = 2 :Chån ¡p ¡n B dC¥u 26. C§p sè cëng (un)câ sè h¤ng ¦u u1 = 3, cæng sai d= �2 th¼ sè h¤ng thù 5l  A u5 = 8. B u5 = 1. C u5 =�5. D u5 =�7. ÊLíi gi£i.u 5 =u1 + 4d= 3 �42 = �5.Chån ¡p ¡n C dC¥u 27. Hå t§t c£ c¡c nguy¶n h m cõa h m sè f(x ) = 1 x�6x 2l  A lnjx j � 2x 3+ C. B �ln jx j � 2x 3+ C. C �1 x2 �12x+ C. D lnjx j � 6x 3+ C. ÊLíi gi£i.119/305 119/305 pGV: L¶ Quang Xe  Ô0967.003.131— SÈ 8 NÌI N€O CÂ Þ CH, NÌI  C CON ×ÍNG120Hå t§t c£ c¡c nguy¶n h m cõa h m sèf(x ) = 1 x�6x 2l Z f(x ) d x= Z�1 x�6x 2‹dx = ln jx j � 2x 3+ C:Chån ¡p ¡n A dC¥u 28. Cho h m sè f(x ) li¶n töc tr¶n Rv  câ b£ng x²t d§u ¤o h m nh÷ sau xf0( x ) �1�2 0 2 4 +1 + � jj +0 +0 �H m sè ¢ cho câ bao nhi¶u iºm cüc tiºuA 3. B 0. C 2. D 1. ÊLíi gi£i.Tø b£ng bi¸n thi¶n ta th§y h m sè câ 1iºm cüc tiºu.Chån ¡p ¡n D dC¥u 29. Câ bao nhi¶u gi¡ trà nguy¶n ¥m cõa tham sè mº h m sè y= 4x + m 2x + m + 3 çng bi¸n(0; 1) . A 1. B 5. C 4. D 3. ÊLíi gi£i.Tªp x¡c ành D=Rn§� m+ 3 2ª, khi â y0= 2m + 12 (2x+ m + 3) .º h m sè çng bi¸n tr¶n (0; 1)th¼8>>>><>>>>: y0> 0; 8x 2 (0; 1)264 �m+ 3 21� m+ 3 20 ,8><>: 2m + 12 >0–m  � 5m  � 3 ,8><>: m >�6–m  � 5m  � 3,m2(� 6; �5] [[� 3; + 1).Vªy c¡c gi¡ trà nguy¶n ¥m c¦n t¼m cõa ml f� 5;�3; �2; �1g .Chån ¡p ¡n C dC¥u 30. T¼m kho£ng çng bi¸n cõa h m sè y= x3+ 3 x2� 9x . A (�1 ; 1). B (�1 ;� 3) [(1; + 1). C (�1 ;� 3) v  (1; + 1). D (� 3; 1) . ÊLíi gi£i.Tªp x¡c ành D=R.Ta câ y0= 3 x2+ 6 x� 9 = 0 ,–x = �3x = 1 . Khi â b£ng bi¸n thi¶n cõa h m sè nh÷ sau120/305 120/305 pGV: L¶ Quang Xe  Ô0967.003.131PHT TRIšN — THAM KHƒO 2022 NÌI N€O CÂ Þ CH, NÌI  C CON ×ÍNG121xf0( x ) f(x ) �1�3 1 +1 +0 �0 +�1�1 2727�5 �5 +1 +1 Tø â suy ra h m sèy= x3+ 3 x2� 9x çng bi¸n tr¶n (�1 ;� 3) v  (1; + 1).Chån ¡p ¡n C dC¥u 31. Choal  sè thüc d÷ìng v  kh¡c 1. Gi¡ trà cõa loga2( 3p a) b¬ng A 6. B 32. C 16. D 23. ÊLíi gi£i.Vîi ad÷ìng v  kh¡c 1th¼ loga2( 3p a) = loga2 �a1 3‹= 1 6logaa= 1 6.Chån ¡p ¡n C dC¥u 32. Cho tù di»n ·u ABC Dc¤nha. T½nh cæ-sin gâc giúa hai ÷íng th¯ng ABv C I vîiI l  trung iºm cõa AD. A p36. B 12. C p34. D p32. ÊLíi gi£i.Gåi Ml  trung iºm BD. Khi â M IAB n¶n gâc giúa ABv C Il  gâc giúa M Iv C I l ÕC I M .Ta câ C I=C M =ap 32,M I =a 2.cos ÕC I M =I C2+ I M 2� C M 2 2I C I M =a2 42ap 32a 2=p 36. B DICAMChån ¡p ¡n A dC¥u 33. T½nh t½ch ph¥n I= 1Z0 (x + 1) 2dx . A I= 4 . B I= 3 . C I= 7 3. D I= 1 3. ÊLíi gi£i.Ta câ I= 1Z0 (x + 1) 2dx = 1Z0 (x + 1) 2d( x+ 1) = (x + 1) 3 3 10 =7 3.Chån ¡p ¡n C 121/305 121/305 pGV: L¶ Quang Xe  Ô0967.003.131— SÈ 8 NÌI N€O CÂ Þ CH, NÌI  C CON ×ÍNG122dC¥u 34. Trong khæng gian vîi h» tåa ë Oxyz, ph÷ìng tr¼nh n o ÷ñc cho d÷îi ¥y l  ph÷ìngtr¼nh m°t ph¯ng (Oyz )? A x= y+ z. B y� z= 0 . C y+ z= 0 . D x= 0 . ÊLíi gi£i.Trong khæng gian vîi h» tåa ë Oxyz, ph÷ìng tr¼nh cõa m°t ph¯ng (Oyz )l  x= 0 .Chån ¡p ¡n D dC¥u 35. Cho sè phùc zthäa m¢n (1 + 2i) z = 8 + i. Sè phùc li¶n hñp z cõa zl  A z = �2� 3i. B z = �2 + 3 i. C z = 2 + 3 i. D z = 2 �3i. ÊLíi gi£i.Ta câ (1 + 2i) z = 8 + i, z= 8 +i 1 + 2i= 2�3i ) z = 2 + 3 i.Chån ¡p ¡n C dC¥u 36. T½nh chi·u cao cõa h¼nh châp tù gi¡c ·u câ c¤nh ¡y b¬ng av  c¤nh b¶n b¬ngb . A p4b2+ 2 a2 2. B p4b2� 2a 2 2. C p4b2� a2 2. D p4b2+ a2 2. ÊLíi gi£i.Gi£ sû S:ABC D l  h¼nh châp ·u v  O=AC \BD . Khi â ABC Dl  h¼nh vuæng v  S O?(ABC D ).S C =b; OC =ap 22) S O =p S C2� OC 2= p 4b2� 2a 2 2.Vªy chi·u cao cõa khèi châp l  S O=p 4b2� 2a 2 2. AB C DSOChån ¡p ¡n B dC¥u 37. Mët b i kiºm tra câ 5c¥u theo 5mùc ë kh¡c nhau, x¡c su§t º b¤n An l m óngc¥u 1l  100 % v  gi£m d¦n ·u 10% khi sang méi c¥u ti¸p theo. T½nh x¡c su§t º b¤n An l móng h¸t c£ b i kiºm tra â. A 18125. B 1896250. C 189625. D 36125. ÊLíi gi£i.X¡c su§t º An l m óng c¥u 1l  1.X¡c su§t º An l m óng c¥u 2l  0;9 .X¡c su§t º An l m óng c¥u 3l  0;8 .X¡c su§t º An l m óng c¥u 4l  0;7 .X¡c su§t º An l m óng c¥u 5l  0;6 .Theo quy t­c nh¥n ta câ x¡c su§t º An l m óng h¸t 5c¥u l  10;9 0;8 0;7 0;6 = 189 625.Chån ¡p ¡n C 122/305 122/305 pGV: L¶ Quang Xe  Ô0967.003.131PHT TRIšN — THAM KHƒO 2022 NÌI N€O CÂ Þ CH, NÌI  C CON ×ÍNG123dC¥u 38. Trong khæng gian Oxyz, cho hai m°t ph¯ng c­t nhau (P ) : 2 x� y+ 3 z+ 1 = 0 v ( Q ) : x� y+ z+ 5 = 0 . ÷íng th¯ng dl  giao tuy¸n cõa (P )v  (Q )câ ph÷ìng tr¼nh l  A x� 4 2=y� 9 1=z� 1 �1 . B x� 4 2=y� 9 1=z 1. C x� 4 2=y� 9 1=z �1. D x� 4 2=y+ 9 1=z �1. ÊLíi gi£i.V²c-tì ph¡p tuy¸n cõa (P )v  (Q )l¦n l÷ñt l  #n = (2; �1; 3) v #n 0= (1; �1; 1) . Do â mët v²c-tì ch¿ph÷ìng cõa ÷íng th¯ng dl  #u = ”#n ; #n 0—= (2; 1; �1) .Cho z= 0 x²t h» ph÷ìng tr¼nh ¨2x � y+ 1 = 0x � y+ 5 = 0 ,¨x = 4y = 9 . Suy ra iºmM(4; 9; 0) 2d.Vªy ph÷ìng tr¼nh ch½nh t­c cõa ÷íng th¯ng dl  x� 4 2=y� 9 1=z �1.Chån ¡p ¡n C dC¥u 39. Sè c¡c nghi»m nguy¶n nhä hìn 2019cõa b§t ph÷ìng tr¼nh log2(16x) + 5 log x 42 0l  A 2015. B 2018. C 2017. D 2016. ÊLíi gi£i.i·u ki»n x >0, x 6= 4 .Ta câlog2(16x) + 5 log x 42 0, 4 + log2x+ 5 log2x 40, 4 + log2x+ 5 log2x� 20, log22 x+ 2 log2x� 3 log2x� 2 0, –� 3 log2x 1log 2x >2, 24 1 8x 2x > 4:Do xnguy¶n v  nhä hìn 2019n¶nx2 f 1; 2; 5; 6; : : :; 2018 g. Vªy câ 2016sè nghi»m nguy¶n.Chån ¡p ¡n D dC¥u 40. T¼m t§t c£ c¡c gi¡ trà cõa tham sè mº ç thà (C ) : y= ( x� 2)( x2� 2mx +m)c­ttröc ho nh t¤i ba iºm ph¥n bi»t câ ho nh ë d÷ìng. A m2(1; + 1). B m2(1; + 1)n §4 3ª. C m2(0; + 1). D m2(�1 ; 0)[�1; 4 3‹[ �4 3; +1‹. ÊLíi gi£i.Ph÷ìng tr¼nh ho nh ë giao iºm l  (x � 2)( x2� 2mx +m) = 0 ,–x = 2x 2� 2mx +m = 0 :(1)123/305 123/305 pGV: L¶ Quang Xe  Ô0967.003.131— SÈ 8 NÌI N€O CÂ Þ CH, NÌI  C CON ×ÍNG124ç thà(C ) : y= ( x� 2)( x2� 2mx +m)c­t tröc ho nh t¤i ba iºm ph¥n bi»t câ ho nh ë d÷ìng khiv  ch¿ khi ph÷ìng tr¼nh (2)câ hai nghi»m d÷ìng ph¥n bi»t kh¡c 2., 8>>><>>>: 0= m2� m > 0S = 2 m > 0P =m > 02 2� 4m +m 6= 0 ,8<: m >1m 6= 4 3:Chån ¡p ¡n B dC¥u 41. Cho h m sè f(x ) x¡c ành tr¶n Rn f� 1; 1gthäa m¢n f0( x ) = 1 x2� 1. Bi¸tf(3) +f (� 3) = 4 v f�1 3‹+ f�� 1 3‹= 2 . T½nh gi¡ trà cõa biºu thùc T= f(� 5) + f(0) + f(2) . A T= 5 �1 2ln 2. B T= 6 �1 2ln 2. C T= 5 + 1 2ln 2. D T= 6 + 1 2ln 2. ÊLíi gi£i.Ta câ Z1 x2� 1dx = 1 2Z�1 x� 1�1 x+ 1 ‹dx = 1 2ln x� 1 x+ 1 +C.Do h m sè f(x ) li¶n töc tr¶n c¡c kho£ng (�1 ;� 1) ;(� 1; 1) ;(1; + 1)n¶nf (x ) = 8>>>>>>><>>>>>>>: 1 2ln x� 1 x+ 1 +C1 khix2 (1; + 1);1 2ln x� 1 x+ 1 +C2 khix2 (�1 ;� 1) ;1 2ln x� 1 x+ 1 +C3 khix2 (� 1; 1) :Theo · b i ta câ f(� 3) = 1 2ln 2 +C2;f (3) = 1 2ln1 2+C1:M  f(3) + f(� 3) = 4 ,C1+C2= 4: (1)T÷ìng tü f�1 3‹+ f�� 1 3‹= 2 ,2C3= 2,C3= 1:Ta câ 8>>>><>>>>: f(� 5) = 1 2ln3 2+C2f (0) = 1f (2) = 1 2ln1 3+C1 )f(� 5) + f(0) + f(2) = 1 2ln1 2+ 1 +C1+C2.Tø (1) suy ra f(� 5) + f(0) + f(2) = �1 2ln 2 + 1 +C1+C2=�1 2ln 2 + 5.Chån ¡p ¡n A dC¥u 42. Cho h¼nh châp S:ABC Dcâ ¡y l  h¼nh thang c¥n vîi ¡y lîn AB= 2a; AD =BC =C D =am°t b¶n S ABl  tam gi¡c c¥n ¿nh Sv  n¬m trong m°t ph¯ng vuæng gâc vîi m°t ph¯ng124/305 124/305 pGV: L¶ Quang Xe  Ô0967.003.131PHT TRIšN — THAM KHƒO 2022 NÌI N€O CÂ Þ CH, NÌI  C CON ×ÍNG125ABC D. Bi¸t kho£ng c¡ch tø Atîi m°t ph¯ng (S B C )b¬ng 2a p 155, t½nh theoathº t½ch Vcõakhèi châp. A V= 3a 3p 34. B V= 3a 3 4. C V= 3a 3p 54. D V= 3a 3p 28. ÊLíi gi£i.Gåi O; I l  trung iºm cõa AB; B C;H l  h¼nh chi¸uvuæng gâc cõa Ol¶n S I.Tam gi¡c S ABc¥n t¤i S) S O ?AB .M  (S AB )? (ABC D )) S O ?(ABC D ).M  ABC D l  h¼nh thang c¥n vîi ¡yAB = 2a; AD =BC =C D =a:Suy ra 4OAD; 4OC D; 4OB C l  c¡c tam gi¡c ·u,c¤nh aS ABC D = 3SOBC = 3a2p 34=3a 2p 34. BAD CSOIHDoOl  trung iºm cõa ABn¶nd (A; ( S B C )) = 2 d ( O; ( S B C )).(1)4 OB C ·u,Il  trung iºm cõa BC)8<: OI?BCOI =ap 32:M  BC ?S O (doS O?(ABC D ))) BC ?(S OI )) BC ?OH . L¤i câ S I?OH )OH ?(S B C )) d (O; ( S B C )) =OH.(2)Tø (1),(2) suy ra d (A; ( S B C )) = 2 OH =2a p 155)OH =ap 155.4 S OI vuæng t¤i O,OH ?S I , ta câ1 S O2+ 1 OI2= 1 OH2, 1 S O2+ 1 34a2 =1 35a2 ,S O =ap 3Thº t½ch khèi châp S:ABC Dl V= 1 3S OSABC D =1 3a p 33a 2p 34=3a 3 4.Chån ¡p ¡n B dC¥u 43. Bi¸tz1 v z2 l  hai nghi»m cõa ph÷ìng tr¼nh2z 2+ p 3z + 3 = 0 :Khi â gi¡ trà cõaz 21 +z22 l  A 9. B 4. C 94. D �9 4. ÊLíi gi£i.Ta câ z21 +z22 = (z1 +z2)2� 2z1z2 = ‚� p 32Œ2� 23 2=�9 4:Chån ¡p ¡n D dC¥u 44. GåiMl  gi¡ trà lîn nh§t cõa 1 m�i�1 , vîiml  sè thüc. Gi¡ trà M2g¦n vîi sèn o nh§t trong c¡c sè d÷îi ¥y? A 2;62 . B 2;64 . C 1;62 . D 1;64 .125/305 125/305 pGV: L¶ Quang Xe  Ô0967.003.131— SÈ 8 NÌI N€O CÂ Þ CH, NÌI  C CON ×ÍNG126ÊLíi gi£i.Ta câ 1 m�i�1 = j1 � m +ij jm �ij =p (1�m)2+ 1 pm2+ 1 =Ê m2� 2m + 2 m2+ 1 =É 1 +1� 2m m2+ 1 .X²t f(m ) = 1� 2m m2+ 1 .f 0( m ) = 0 ,2m 2� 2m �2 (m 2+ 1) 2= 0,m= 1 + p 52ho°cm= 1� p 52.B£ng bi¸n thi¶n mf0( m ) f(m ) �1 1� p 52 1 +p 52 +1 +0 �0 +00 1 +p 521 +p 52 00Do âM=Ê 1 +1 +p 52)M2= 3 + p 522;62 .Chån ¡p ¡n A dC¥u 45. X¡c ànhmº ç thà h m sè (C ) : y= 5 x4� 8x 2+ m c­t tröc ho nh t¤i 4iºm ph¥nbi»t sao cho di»n t½ch h¼nh ph¯ng giîi h¤n bði (C )v  tröc ho nh câ ph¦n tr¶n v  ph¦n d÷îi b¬ngnhau. A 916. B 169. C 9. D 2516. ÊLíi gi£i.Ph÷ìng tr¼nh ho nh ë giao iºm cõa ç thà (C )v  tröc ho nh l  5x 4� 8x 2+ m = 0 .°t t= x2, t  0. Ta câ 5t2� 8t + m = 0 : (1)ç thà (C )c­t tröc ho nh t¤i bèn iºm ph¥n bi»t khi v  ch¿ khi ph÷ìng tr¼nh (1)câ hai nghi»m d÷ìngph¥n bi»t 8><>: 0> 0P > 0S > 0, 8>>><>>>: 16�5m > 0m 5>08 5>0 ,0< m < 16 5:Ta câ h m sè y= f(x ) = 5 x4� 8x 2+ m l  h m sè ch®n n¶n S1 +S2 =S3 )S2 = 1 2S3. Gåix 1 < x2< x3< x4l  bèn ho nh ë giao iºm cõa(Cm)vîi tröc ho nh ta câS 2 = 1 2S3 ) x4Zx 3 (� f(x )) d x= x3Z0 f(x ) d x:, x4Zx 3 f(x ) d x+ x3Z0 f(x ) d x= 0 ,x4Z0 f(x ) d x= 0 ,x4Z0 (5x4� 8x 2+ m) d x= 0, �x5� 8 3x3+ mx ‹ x40 = 0,x54 � 8 3x34 +mx4= 0,24 x4 = 0x 44 � 8 3x24 +m = 0 (2)126/305 126/305 pGV: L¶ Quang Xe  Ô0967.003.131PHT TRIšN — THAM KHƒO 2022 NÌI N€O CÂ Þ CH, NÌI  C CON ×ÍNG127Vîix4 = 0)m= 0 (lo¤i).X²t (2),(5x44 �8x 24 +m)� 4x 44 + 16 3x24 = 0,4x 44 � 16 3x24 = 0,x24 = 4 3)m= 16 9(nhªn).Chån ¡p ¡n B dC¥u 46. Trong khæng gian vîi h» tröc tåa ë Oxyz, cho m°t c¦u (S ) : x2+ y2+ z2� 2x + 4 y= 0v  m°t ph¯ng (P ) : 3 x� 2y + 5 z� 2019 = 0 . C¡c ti¸p di»n vîi m°t c¦u (S ) song song vîi m°tph¯ng (P )ti¸p xóc vîi (S ) t¤i hai iºm (A )v  B. Ph÷ìng tr¼nh ÷íng th¯ng ABl  A AB:8><>: x= �1 + 3 ty = 2 �2tz = 5 t . B AB:8><>: x= 4 + 3 ty = �4� 2tz = 5 + 5 t. C AB:8><>: x= �1 + ty = �2� 2tz = 0 . D AB:8><>: x= 3 + ty = �2� 2tz = 5 . ÊLíi gi£i.Ti¸p di»n cõa m°t c¦u (S ) luæn vuæng gâc vîi b¡n k½nh cõa m°t c¦u (S ) i qua ti¸p iºm, suy ra÷íng th¯ng ABvuæng gâc vîi (P )v  i qua t¥m Icõa m°t c¦u (S ). Hay nâi c¡ch kh¡c, ÷íng th¯ngAB câ v²c-tì ch¿ ph÷ìng #u AB =#n (P ) = (3;�2; 5) v  i qua iºm I(1; �2; 0) . Do â ABcâ ph÷ìngtr¼nh l AB:8><>: x= �1 + 3 ty = 2 �2tz = 5 t:Chån ¡p ¡n A dC¥u 47. Cho tù di»n ABC DcâAD ?(ABC ), ABC l  tam gi¡c vuæng t¤i B. Bi¸t BC=a,AB =ap 3, AD = 3a. Quay c¡c tam gi¡c ABCv ABD (bao gçm c£ iºm b¶n trong 2tam gi¡c)xung quanh ÷íng th¯ng ABta ÷ñc 2khèi trán xoay. Thº t½ch ph¦n chung cõa 2khèi trán xoayâ b¬ng A 5p 3a 3 16. B 3p 3a 3 16. C 8p 3a 3 3. D 4p 3a 3 16. ÊLíi gi£i. ADB C D1 D2 C1 C2 N MAB HTù di»nABC Dcâ¨DA ?(ABC )BC ?(ABD ), do â khi quay hai tam gi¡cABCv ABD quanh tröc ABta s³÷ñc hai h¼nh nân trán xoay, v  ph¦n chung cõa hai h¼nh nân n y l  ph¦n tæ m u x¡m ð h¼nh tr¶n.Khi â x²t m°t ph¯ng qua tröc ABcõa hai h¼nh nân (C1C2D2D1), gåi M=BD2\AC2;N =127/305 127/305 pGV: L¶ Quang Xe  Ô0967.003.131— SÈ 8 NÌI N€O CÂ Þ CH, NÌI  C CON ×ÍNG128BD1\AC1.Ta câ 4BM C2v4D2M A, n¶nBM D2M =BC2 AD2=1 3)H M AD2=1 4)H M BC2=3 4)H M =3a 4:Suy ra Ì Thº t½ch khèi nân (AN M )l  V1 = 1 3H M2AH =1 39a 2 16AH .Ì Thº t½ch khèi nân (BN M )l  V2 = 1 3H M2BH =1 39a 2 16BH .Vªy thº t½ch khèi trán xoay c¦n t¼m l V1 +V2 = 1 39a 2 16(AH +BH ) = 1 39a 2 16AH =3p 3a 3 16:Chån ¡p ¡n B dC¥u 48. T¼m tªp nghi»m cõa b§t ph÷ìng tr¼nh 3x> 5� 2x . A [1; +1). B (�1 ; 1]. C (1; +1). D ?. ÊLíi gi£i.Ta câ 3x> 5� 2x , 3x+ 2 x� 5> 0.X²t h m sè f(x ) = 3 x+ 2 x� 5, suy ra f0( x ) = 3 xln 3 + 2 >0;8 x 2 R. Do â h m sè çng bi¸n tr¶nR .Suy ra f(x ) > 0 = f(1) ,x> 1.Chån ¡p ¡n A dC¥u 49. Cho h¼nh c¦u t¥m Ob¡n k½nh R= 5 , ti¸p xóc vîi m°t ph¯ng (P ). Mët h¼nh nântrán xoay câ ¡y n¬m tr¶n (P ), câ chi·u cao h= 15 , câ b¡n k½nh ¡y b¬ng R. H¼nh c¦u v  h¼nhnân n¬m v· mët ph½a èi vîi m°t ph¯ng (P ). Ng÷íi ta c­t hai h¼nh â bði m°t ph¯ng (Q )songsong vîi (P )v  thu ÷ñc hai thi¸t di»n câ têng di»n t½ch l  S. Gåi xl  kho£ng c¡ch giúa (P )v ( Q ) (0 < x 5). Bi¸t r¬ng S¤t gi¡ trà lîn nh§t khi x= a bph¥n sè a btèi gi£n . T½nh gi¡ tràT = a+ b. SOP R 3R A T= 17 . B T= 19 . C T= 18 . D T= 23 .128/305 128/305 pGV: L¶ Quang Xe  Ô0967.003.131MÖC LÖCNÌI N€O CÂ Þ CH, NÌI  C CON ×ÍNG129ÊLíi gi£i.V¼ d (( P); (Q )) = x2 (0; 5] )d (O; (Q )) = R� x= 5 �xn¶n m°t c¦u (S ) s³c­t (Q )theo giao tuy¸n l  ÷íng trán (C1)câ b¡n k½nhR (C1) = È R2� d2(O; (Q )) = È 25�(5 �x)2:Di»n t½ch cõa h¼nh trán thi¸t di»n l  S(C1) =R 2( C1) =[25 �(5 �x)2].Gi£ sû h¼nh nân (N )câ t¥m ÷íng trán ¡y l  Hv  m°t ph¯ng (Q )c­t (N )theo giao tuy¸n l  ÷íng trán (C2)câ t¥m I. Theo ành l½ Thales ta câS I S H=R(C2) R(N ) ,h� x h=R(C2) 5,R(C2) = 15�x 3: SHIDi»n t½ch cõa h¼nh trán thi¸t di»n l S(C2) =R 2( C2) =�15 �x 3‹2.Têng di»n t½ch thi¸t di»n l S= S(C1) +S(C2) =25 �(5 �x)2+ �15 �x 3‹2= –� 8 9�x � 15 4‹2+ 75 2™ 75 2:¯ng thùc x£y ra khi x= 15 4.Chån ¡p ¡n B dC¥u 50. Cho h m sè f(x ) = x3+ ax 2+ bx �2thäa m¢n ¨a+ b > 13 + 2 a+ b < 0. Sè iºm cüc trà cõah m sè y= jf (jx j) j l  A 9. B 11. C 2. D 5. ÊLíi gi£i. xyO1 2�2 xyO 1 22D¹ th§yf(0) = �2 < 0, f(1) = a+ b� 1> 0, f(2) = 2(2 a+ b+ 3) <0v  limx ! +1 = +1n¶n ph÷ìngtr¼nh f(x ) = 0 câ ba nghi»m ph¥n bi»t l¦n l÷ñt thuëc c¡c kho£ng (0; 1),(1; 2) ,(2; + 1). Do â, ç thàh m sè f(x ) câ hai iºm cüc trà, mët iºm n¬m tr¶n tröc ho nh mët iºm n¬m d÷îi tröc ho nh (xemh¼nh v³). Tø â, h m sè y= jf (jx j) j câ t§t c£ 11iºm cüc trà.Chån ¡p ¡n B H˜T129/305 129/305 pGV: L¶ Quang Xe  Ô0967.003.131— SÈ 9 NÌI N€O CÂ Þ CH, NÌI  C CON ×ÍNG130BË GIO DÖC & €O T„OTR×ÍNG THPT NGUY™N T‡T TH€NH GV: L– QUANG XE - 0967.003.131 — SÈ 9 PHT TRIšN — THAM KHƒO 2022N‹M HÅC 2021 - 2022Mæn:To¡nThíi gian l m b i: 90 phót — THAM KHƒO PTMH2022dC¥u 1. Cho sè phùc z= a+ bi (a; b 2R) tòy þ. M»nh · n o sau ¥y óng? A z2= jz j2. B Sè phùc li¶n hñp cõazcâ mæ-un b¬ng mæ-un cõa iz. C iºmM(� a;b) l  iºm biºu di¹n cõa z. D Mæ-un cõazl  mët sè thüc d÷ìng. ÊLíi gi£i.Ta câ j zj = ja � bij= p a2+ b2v  jiz j= j� b+ aij= p a2+ b2. Vªy j zj = jiz j.Chån ¡p ¡n B dC¥u 2. Trong khæng gian Oxyz, cho m°t c¦u (S ) : ( x� 1)2+ ( y+ 2) 2+ ( z� 5)2= 9 . T¼m tåaë t¥m cõa m°t c¦u (S ). A (1;�2; �5) . B (1;�2; 5) . C (� 1; �2; 5) . D (1; 2; 5). ÊLíi gi£i.( S ) : ( x� 1)2+ ( y+ 2) 2+ ( z� 5)2= 9 th¼(S ) câ t¥m l  I(1; �2; 5) .Chån ¡p ¡n B dC¥u 3. T¥m èi xùng cõa ç thà h m sè y= x� 2 2x � 1l  A I�� 1 2;1 2‹. B I�1 2;1 2‹. C I�1 2;� 1‹. D I�� 1 2; 2 ‹. ÊLíi gi£i.ç thà h m sè y= ax+b cx+dvîiad�bc 6= 0 nhªn iºm I�� d c;a c‹l m t¥m èi xùng. Do â I�1 2;1 2‹l  t¥m èi xùng cõa ç thà h m sè ¢ cho.Chån ¡p ¡n B dC¥u 4. Mët m°t c¦u câ di»n t½ch l  16. T½nh b¡n k½nh Rcõa m°t c¦u. A R= 2 . B R= 2 . C R= 4 . D R= 4 . ÊLíi gi£i.Ta câ S= 4 R2n¶n 16= 4 R2suy ra R= 2 .Chån ¡p ¡n B dC¥u 5. Hå nguy¶n h m cõa h m sè f(x ) = sin x� 8x l  A cosx� 4x 2+ C. B �cos x� 4x 2+ C. C cosx+ 4 x2+ C. D �cos x+ C. ÊLíi gi£i.130/305 130/305 pGV: L¶ Quang Xe  Ô0967.003.131PHT TRIšN — THAM KHƒO 2022 NÌI N€O CÂ Þ CH, NÌI  C CON ×ÍNG131Ta câZ(sin x� 8x ) d x= �cos x� 4x 2+ C.Chån ¡p ¡n B dC¥u 6. Cho h m sè y= f(x ) câ b£ng bi¸n thi¶n nh÷ h¼nh v³ b¶n. M»nh · n o d÷îi ¥yóng? xy0 y �10 1 +1 �0 +0 �+1 +1 44 55�1�1A yC = 5. B minR y= 4 . C minR y= 5 . D yCT = 0. ÊLíi gi£i.Nh¼n v o b£ng bi¸n thi¶n ta th§y gi¡ trà cüc ¤i cõa h m sè l  5Chån ¡p ¡n A dC¥u 7. Câ bao nhi¶u sè nguy¶n d÷ìng xthäa m¢n b§t ph÷ìng tr¼nh �1 2‹2x> �1 2‹x+7? A 6. B 5. C 7. D Væ sè.ÊLíi gi£i.Vîi x2 Z; x > 0b§t ph÷ìng tr¼nh trð th nh 2x < x + 7,0< x < 7, x2 f 1; 2;   ; 6g.Vªy sè gi¡ trà xthäa · b i l  6.Chån ¡p ¡n A dC¥u 8. T½nh thº t½ch cõa khèi l«ng trö bi¸t di»n t½ch ¡y l  2a 2v  chi·u cao l  3a . A V= 2 3a3. B 3a 3. C 2a 3. D 6a 3. ÊLíi gi£i.Cæng thùc t½nh thº t½ch khèi l«ng trö l V=Sh = 2 a23a = 6 a3:Chån ¡p ¡n D dC¥u 9. T¼m tªp x¡c ành Dcõa h m sè y= ( x2� 3x )�2020. A D= ( �1 ; 0)[(3; + 1). B D= ( �1 ; 0][[3; + 1). C D= (0; 3) . D D=Rn f 0; 3 g. ÊLíi gi£i.H m sè y= ( x2� 3x )�2020x¡c ành khi v  ch¿ khix2� 3x 6= 0 ,¨x 6= 0x 6= 3 :Vªy tªp x¡c ành cõa h m sè y= ( x2� 3x )�2020l D =Rn f 0; 3 g.Chån ¡p ¡n D 131/305 131/305 pGV: L¶ Quang Xe  Ô0967.003.131— SÈ 9 NÌI N€O CÂ Þ CH, NÌI  C CON ×ÍNG132dC¥u 10. Gi£i ph÷ìng tr¼nh 2x= 3 . A x= 2 p 3. B x= log23. C x= log32. D x= 3 p 2. ÊLíi gi£i.Ta câ 2x= 3 ,x= log23.Chån ¡p ¡n B dC¥u 11. Cho h m sè f(x ) li¶n töc tr¶n kho£ng Kv  c¡c h¬ng sè a; b; c2K. M»nh · n od÷îi ¥y sai? A bZa kf (x ) d x= k bZa f(x ) d xvîi k2 R. B bZa f(x ) d x= cZa f(x ) d x+ bZc f(x ) d x. C bZa f(x ) d x= � bZa f(x ) d x. D bZa f(x ) d x6= bZa f(t) d t. ÊLíi gi£i.Ta câ bZa f(x ) d x= bZa f(t) d t.Chån ¡p ¡n D dC¥u 12. Cho sè phùc z= 3 �4i. Sè phùc w= z� 4 + 2 ib¬ng A w= �1� 2i. B w= 7 �6i. C �1 + 2 i. D �1� 6i. ÊLíi gi£i.Ta câ: w= 3 �4i � 4 + 2 i= �1� 2i.Chån ¡p ¡n A dC¥u 13. Trong khæng gian Oxyz, cho m°t ph¯ng (P ) : 1 2x� 2y + z+ 5 = 0 . Vectì n o d÷îi¥y l  v²c-tì ph¡p tuy¸n cõa m°t ph¯ng (P )? A #n 2= (1;�2; 1) . B #n 3= (1;�4; 2) . C #n 1= (2;�2; 1) . D #n 4= (�2; 1; 5) . ÊLíi gi£i.Tø ph÷ìng tr¼nh cõa (P )suy ra mët v²c-tì ph¡p tuy¸n cõa m°t ph¯ng (P )l  #n = �1 2;� 2; 1 ‹.M°t kh¡c #n 3 = (1;�4; 2) = 2 �1 2;� 2; 1 ‹= 2 #n n¶n #n 3 = (1;�4; 2) công l  mët v²c-tì ph¡p tuy¸ncõa m°t ph¯ng (P ).Chån ¡p ¡n B dC¥u 14. Trong khæng gian vîi h» tåa ë Oxyz, cho ba iºm A(1; 2; �1) , B(2; �1; 3) ,C (� 3; 5; 1) . T¼m tåa ë iºm Dsao cho tù gi¡c ABC Dl  h¼nh b¼nh h nh. A D(� 2; 8; �3) . B D(� 4; 8; �5) . C D(� 2; 2; 5) . D D(� 4; 8; �3) . ÊLíi gi£i.132/305 132/305 pGV: L¶ Quang Xe  Ô0967.003.131PHT TRIšN — THAM KHƒO 2022 NÌI N€O CÂ Þ CH, NÌI  C CON ×ÍNG133Ta câ# AB = (1; �3; 4) ,# DC = (�3� xD ; 5�yD ; 1�zD ).ABC D l  h¼nh b¼nh h nh khi v  ch¿ khi# AB =# DC ,8><>: 1 =�3� xD� 3 = 5 �yD4 = 1 �zD ,8><>: xD =�4y D = 8z D =�3:Vªy D(� 4; 8; �3) . BACDChån ¡p ¡n D dC¥u 15. Tr¶n m°t ph¯ng tåa ë, tªp hñp iºm biºu di¹n sè phùc zthäa m¢n i·u ki»nj z + 2 �5ij = 6 l  ÷íng trán câ t¥m Iv  b¡n k½nh Rl¦n l÷ñt l  A I(� 2; 5) v R= 36 . B I(� 2; 5) v R= 6 . C I(2; �5) v  R= 36 . D I(2; �5) v  R= 6 . ÊLíi gi£i.Gåi z= x+ iy (x; y 2R). Ta câ jz + 2 �5ij = 6 ,(x + 2) 2+ ( y� 5)2= 36 .Suy ra tªp hñp iºm biºu di¹n zl  ÷íng trán câ t¥m I(� 2; 5) v  b¡n k½nh R= 6 .Chån ¡p ¡n B dC¥u 16. ÷íng th¯ng n o d÷îi ¥y l  ti»m c¥n ngang cõa ç thà h m sè y= 3x � 1 2x + 1 ? A y= �1 2. B x= �1 2. C y= 3 2. D x= 3 2. ÊLíi gi£i.Ti»m cªn ngang cõa ç thà h m sè y= 3x � 1 2x + 1 l y= a c=3 2.Chån ¡p ¡n C dC¥u 17. Chologab= 3 ;logac= �2. Khi â loga(a 3b 2p c) b¬ng A 8. B 13. C 5. D 10. ÊLíi gi£i.Ta câ loga(a 3b 2p c) = logaa3+ log ab2+ log ap c= 3 + 2 logab+ 1 2logac= 3 + 2 3 + 1 2(� 2) = 8 .Chån ¡p ¡n A dC¥u 18. B£ng bi¸n thi¶n trong h¼nh v³ d÷îi ¥y l  b£ng bi¸n thi¶n cõa h m sè n o? xy0 y �1�1 0 1 +1 �0 +0 �0 ++1 +1 �4 �4 �3 �3 �4 �4 +1 +1 A y= �x4+ x2� 3. B y= x4+ 2 x2� 3. C y= �x4+ 2 x2� 3. D y= x4� 2x 2� 3. ÊLíi gi£i.133/305 133/305 pGV: L¶ Quang Xe  Ô0967.003.131— SÈ 9 NÌI N€O CÂ Þ CH, NÌI  C CON ×ÍNG134¥y l  b£ng bi¸n thi¶n cõa h m sèy= ax 4+ bx2+ cvîi a > 0v  b < 0.Chån ¡p ¡n D dC¥u 19. Trong khæng gian Oxyz, cho ÷íng th¯ng d: x� 1 2=y 1=z+ 1 2. iºm n o d÷îi ¥ythuëc d? A P(3; 1; 1) . B N(0; �1; �2) . C Q(3; 2; 2) . D M(2; 1; 0) . ÊLíi gi£i.L¦n l÷ñt th¸ tåa ë c¡c iºm v o ph÷ìng tr¼nh cõa dta th§y ch¿ câ P(3; 1; 1) thäa m¢n.Chån ¡p ¡n A dC¥u 20. Trong m°t ph¯ng cho 15iºm ph¥n bi»t trong â khæng câ 3iºm n o th¯ng h ng.Sè tam gi¡c câ c¡c ¿nh l  3trong sè 15iºm ¢ cho b¬ng bao nhi¶u? A A315 . B 15!. C C315 . D 153. ÊLíi gi£i.Sè tam gi¡c câ c¡c ¿nh l  3trong sè 15iºm ¢ cho l  C315 .Chån ¡p ¡n C dC¥u 21. Cho h¼nh châp S:ABC Dcâ ¡yABC D l  h¼nh thoi c¤nh a, ÕBAD = 60, gåi Il giao iºm cõa ACv BD . H¼nh chi¸u vuæng gâc cõa Str¶n m°t ph¯ng (ABC D )l  trung iºmH cõa o¤n BI. Gâc giúa S Cv ABC D b¬ng45. Thº t½ch khèi châp S:ABC Dl  A a3p 3948. B a3p 398. C a3p 3924. D a3p 3912. ÊLíi gi£i.Ta câ 4BAD l  tam gi¡c ·u n¶nBD =a, AI =C I =ap 32.C H =p I C2+ I H 2= s ‚ap 32Œ2+ a 42= ap 134.Û�S C; (ABC D ) =Ø�S C; H C = ÕS C H = 45.4 S H C vuæng c¥n t¤i Hn¶n suy ra S H=H C =ap 134.S ABC D =1 2BDAC =1 2aa p 3 =a2p 32.V S:ABC D =1 3S HSABC D =1 3ap 134a2p 32=a3p 3924. SD BCAI HChån ¡p ¡n C dC¥u 22. T½nh ¤o h m cõa h m sè y= 3 1� 2x. A y0= 3 1� 2xln 3 . B y0= (1 �2x )3 �2x. C y0= �2:3 1� 2xln 3 . D �2:3 1� 2x. ÊLíi gi£i.Ta câ y0= (1 �2x )03 1� 2xln 3 = �2:3 1� 2xln 3Chån ¡p ¡n C 134/305 134/305 pGV: L¶ Quang Xe  Ô0967.003.131PHT TRIšN — THAM KHƒO 2022 NÌI N€O CÂ Þ CH, NÌI  C CON ×ÍNG135dC¥u 23. Cho h m sè y= f(x ) câ b£ng bi¸n thi¶n nh÷ h¼nh v³. M»nh · n o d÷îi ¥y óng? xy0 y �1�1 1 +1 +0 �0 +�1�1 33�1 �1 +1 +1 A H m sè nghàch bi¸n tr¶n kho£ng(� 1; 3) . B H m sè çng bi¸n tr¶n kho£ng(� 1; + 1). C H m sè nghàch bi¸n tr¶n kho£ng(� 1; 1) . D H m sè çng bi¸n tr¶n kho£ng(�1 ; 1). ÊLíi gi£i.Düa v o b£ng bi¸n thi¶n, m»nh · óng l  h m sè nghàch bi¸n tr¶n kho£ng (� 1; 1) .Chån ¡p ¡n C dC¥u 24. Di»n t½ch xung quanh cõa h¼nh trö câ b¡n k½nh ¡y R= 2 v  ÷íng sinh l= 3b¬ng A 4 . B 6 . C 24. D 12. ÊLíi gi£i.Di»n t½ch xung quanh cõa h¼nh trö l  Sxq = 2R l = 2 2 3 = 12 :Chån ¡p ¡n D dC¥u 25. N¸u2Z1 f(x ) d x= 3 ;5Z2 f(x ) d x= �1 th¼ 5Z1 f(x ) d xb¬ng A 3. B 4. C 2. D �2. ÊLíi gi£i.Theo t½nh ch§t t½ch ph¥n 5Z1 f(x ) d x= 2Z1 f(x ) d x+ 5Z2 f(x ) d x= 3 + ( �1) = 2 :Chån ¡p ¡n C dC¥u 26. Cho c§p sè cëng (un)câ u1 = 2v  cæng sai d= �3. T½nh u4. A u4 =�7. B u4 =�1. C u4 =�10 . D u4 = 11. ÊLíi gi£i.Ta câ u4 =u1 + 3d= 2 + 3 (� 3) = �7.Chån ¡p ¡n A dC¥u 27. T½nhZ(x � sin 2 x) d x. A x2 2+ cos 2x+ C. B x2+ 1 2cos 2x+ C.135/305 135/305 pGV: L¶ Quang Xe  Ô0967.003.131— SÈ 9 NÌI N€O CÂ Þ CH, NÌI  C CON ×ÍNG136C x2 2+1 2cos 2x+ C. D x2 2+ sinx+ C. ÊLíi gi£i.Ta câ Z(x � sin 2 x) d x= x2 2+1 2cos 2x+ C.Chån ¡p ¡n C dC¥u 28. Cho h m sè y= f(x ) câ ç thà nh÷ h¼nh v³. H m sè y= jf (x )j câ bao nhi¶u iºmcüc ¤i? A 5. B 4. C 6. D 3. xyO�2 �1 1 2�1 1ÊLíi gi£i.Ta câ ç thà y= jf (x )j nh÷ h¼nh b¶n:Düa v o ç thà h m sè y= jf (x )j, ta th§y h m sè câ ba iºmcüc trà. xyO�2 �1 1 2�1 1Chån ¡p ¡n D 136/305 136/305 pGV: L¶ Quang Xe  Ô0967.003.131PHT TRIšN — THAM KHƒO 2022 NÌI N€O CÂ Þ CH, NÌI  C CON ×ÍNG137dC¥u 29. Gi¡ trà lîn nh§t cõa h m sè y= p 4x � x2l  A 4. B 2. C 0. D �2. ÊLíi gi£i.Ta câ p 4x � x2= p 4� (x � 2)2 2v  d§u b¬ng x£y ra khi x= 2 . Vªy GTLN cõa h m sè b¬ng 2.Chån ¡p ¡n B dC¥u 30. Kho£ng nghàch bi¸n cõa h m sè y= p 2x � x2l  A (�1 ; 1). B (1; 2). C (1; +1). D (0; 1). ÊLíi gi£i.Tªp x¡c ành cõa h m sè l  D= [0; 2] .y 0= 1� x p2x � x2;y0= 0 ,x= 1 .B£ng bi¸n thi¶n xy0 y 0 1 2+0 �00 1100Vªy h m sè nghàch bi¸n tr¶n kho£ng(1; 2).Chån ¡p ¡n B dC¥u 31. Choa >0, a 6= 1 . T¼m m»nh · óngtrong c¡c m»nh · sau A loga(xy ) = logaxlogay. B logaxn= nlogax(x > 0, n 6= 0 ). C loga1 =a. D logaxcâ ngh¾a vîi 8x . ÊLíi gi£i.Trong c¡c m»nh · ¢ cho, m»nh · óng l   logaxn= nlogax(x > 0, n 6= 0 ).Chån ¡p ¡n B dC¥u 32. Cho khèi l«ng trö ABC:A0B 0C 0câ ¡y ABCl  tam gi¡c ·u c¤nh a, h¼nh chi¸u vuænggâc cõa A0l¶n m°t ph¯ng (ABC )tròng vîi trung iºm c¤nh AC, ÷íng th¯ng A0B t¤o vîi m°tph¯ng (ABC )mët gâc 30. Gåi l  gâc giúa hai ÷íng th¯ng ABv C C 0. T½nh cos . A cos = p 24. B cos = p 2. C cos = p 52. D cos = p 23. ÊLíi gi£i.Ta câ A0H ?(ABC )n¶n ÖA 0BH = (A0B; (ABC )) = 30 . Suy raA 0H =BH tan 30 = a 2;A 0B = BH cos 30=a;AA 0= p AH2+ A0H 2= ap 22: CA B B0 C0 A0 H137/305 137/305 pGV: L¶ Quang Xe  Ô0967.003.131— SÈ 9 NÌI N€O CÂ Þ CH, NÌI  C CON ×ÍNG138Do âcos = cos( AB; AA 0) = A0A 2+ AB 2� A0B 2 2A 0A AB =p 24.Chån ¡p ¡n A dC¥u 33. Cho t½ch ph¥n eZ0 �3 x 2� 2x dx = me 3+ ne 2vîi m,n 2 Z. Khi â jm �nj b¬ng baonhi¶u? A 2. B 5. C 3. D 0. ÊLíi gi£i.Ta câ eZ0 �3 x 2� 2x dx = ( x3� x2) e0 =�e3� e2� �0 3� 02= e3� e2.Do â ¨m = 1n = �1 ) jm�nj= 2 .Chån ¡p ¡n A dC¥u 34. Trong khæng gian Oxyz cho hai iºm A(1; �3; 2) v B(3; 1; 4) . Khi â, m°t ph¯ngtrung trüc cõa o¤n th¯ng ABcâ ph÷ìng tr¼nh l  A x� 2y + z� 7 = 0 . B 2x � y+ 3 z� 4 = 0 . C 2x + 4 y+ 2 z� 3 = 0 . D x+ 2 y+ z� 3 = 0 . ÊLíi gi£i.Ta câ ¨M (2; �1; 3) l  trung iºm c£ o¤n th¯ng AB# AB = (2; 4; 2) :M°t ph¯ng trung trüc cõa o¤n th¯ng ABcâ ph÷ìng tr¼nh l  x+ 2 y+ z� 3 = 0 .Chån ¡p ¡n D dC¥u 35. iºm biºu di¹n cõa sè phùc z= 1 2� 3i tr¶n m°t ph¯ng tåa ëOxycâ tåa ë l  A (3;�3) . B �2 13;3 13‹. C (3;�2) . D (2;�3) . ÊLíi gi£i.z = 1 2� 3i =2 13+3 13i.Chån ¡p ¡n B dC¥u 36. Cho h¼nh l«ng trö tam gi¡c ·u ABC:A0B 0C 0câ t§t c£ c¡c c¤nh b¬ng a. Kho£ng c¡chd tø A¸n m°t ph¯ng (A 0BC )b¬ng A d= ap 34. B d= ap 217. C d= ap 64. D d= ap 22. ÊLíi gi£i.138/305 138/305 pGV: L¶ Quang Xe  Ô0967.003.131PHT TRIšN — THAM KHƒO 2022 NÌI N€O CÂ Þ CH, NÌI  C CON ×ÍNG139GåiMl  trung iºm cõa BC, ta c⨠AM ?BCAA 0? BC )BC ?(AA 0M ):Vªy (AA 0M )? (A 0BC )theo giao tuy¸n A0M .K´ AH ?A0M trong (AA 0M ), ta suy ra AH?(A 0BC ).Ta câ AM=ap 32, x²t tam gi¡cAA0M câ1 AH2= 1 AA02 + 1 AM2)AH =ap 217:Vªy kho£ng c¡ch tø A¸n (A 0BC )l  d= ap 217. BA CMA0 C0 H B0 Chån ¡p ¡n B dC¥u 37. Mët chi¸c m¡y câ hai ëng cì I v  II ho¤t ëng ëc lªp vîi nhau. X¡c su§t º ëngcì I v  ëng cì II ho¤t ëng tèt l¦n l÷ñt l  0;99 v 0;98 . X¡c su§t º câ ½t nh§t mët ëng cì ho¤tëng tèt l  A 0;9881 . B 0;9972 . C 0;9998 . D 0;9702 . ÊLíi gi£i.Gåi A= ½t nh§t mët ëng cì ho¤t ëng tèt A= khæng câ ëng cì ho¤t ëng tèt.Câ P� A= 0 ;01 0;02 ) P(A) = 1 �0;01 0;02 = 0 ;9998 .Chån ¡p ¡n C dC¥u 38. Trong khæng gian Oxyz, cho iºm A(2; 3; 4) v  hai m°t ph¯ng (P ) : 2 x� 3y � z+ 1 = 0 ,( Q ) : x+ 2 y� 3z + 10 = 0 . Ph÷ìng tr¼nh ÷íng th¯ng di qua Asong song vîi c£ (P )v  (Q )l  A x� 2 �11 =y� 3 �5 =z� 4 7. B x� 2 11=y� 3 �5 =z� 4 7. C x� 2 11=y� 3 5=z� 4 �7 . D x� 2 11=y� 3 5=z� 4 7. ÊLíi gi£i.Ta câ (P ) : 2 x� 3y � z+ 1 = 0 câ mët v²c-tì ph¡p tuy¸n l  #n (P ) = (2;�3; �1) .( Q ) : x+ 2 y� 3z + 10 = 0 câ mët v²c-tì ph¡p tuy¸n l  #n (Q ) = (1; 2;�3) .÷íng th¯ng dcâ mët v²c-tì ch¿ ph÷ìng l  #u d.Do ÷íng th¯ng dsong song vîi (P )v  (Q )n¶n )#u d= #n (P ); #n (Q )= (11; 5; 7) .M°t kh¡c ÷íng th¯ng di qua A(2; 3; 4) v  câ v²c-tì ch¿ ph÷ìng #u d = (11; 5; 7)n¶n ph÷ìng tr¼nhch½nh t­c cõa dl  x� 2 11=y� 3 5=z� 4 7.Chån ¡p ¡n D dC¥u 39. Câ bao nhi¶u gi¡ trà nguy¶n cõa tham sè m2[0; 10] º tªp nghi»m cõa b§t ph÷ìngtr¼nh È log22 x+ 3 log 1 2x2� 7< m (log4x2� 7) chùa kho£ng (256; +1)? A 7. B 10. C 8. D 9. ÊLíi gi£i.139/305 139/305 pGV: L¶ Quang Xe  Ô0967.003.131— SÈ 9 NÌI N€O CÂ Þ CH, NÌI  C CON ×ÍNG140X²t tr¶n(256; +1), khi â b§t ph÷ìng tr¼nh t÷ìng ÷ìngÈ log22 x� 6 log2x� 7< m (log2x� 7) (1) :°t t= log2xvîi x > 256)t= log2x >8.(1) ,p t2� 6t � 7< m (t � 7), È (t + 1)( t� 7) < m (t � 7), p t+ 1 < m p t� 7, É t+ 1 t� 7< m( )( do t� 7> 1> 0):B§t ph÷ìng tr¼nh ¢ cho câ tªp nghi»m chùa (256; +1)khi v  ch¿ khi (*) nghi»m óng vîi måi t >8.Ta câ 8t > 8th¼ t+ 1 t� 7= 1 +8 t� 7)1< t+ 1 t� 7<1 + 8 8� 7= 9)1< É t+ 1 t� 7<3:Tø â t¼m ÷ñc i·u ki»n cõa tham sè ml m 3. Vªy câ 8gi¡ trà nguy¶n c¦n t¼m l  3;4 ;5 ;6 ;7 ;8 ;9 ;10 .Chån ¡p ¡n C dC¥u 40. ç thà h m sè y= 4x + 4 x� 1 v y= x2� 1c­t nhau t¤i bao nhi¶u iºm? A 0. B 1. C 2. D 3. ÊLíi gi£i.Ph÷ìng tr¼nh ho nh ë giao iºm cõa ç thà h m sè y= 4x + 4 x� 1 v y= x2� 1l 4 x + 4 x� 1 =x2� 1, ¨4( x+ 1) = ( x� 1)( x2� 1)x 6= 1, ¨(x + 1) 2(x � 3) =x 6= 1 ,–x = �1x = 3 :Do â ç thà h m sè y= 4x + 4 x� 1 v y= x2� 1c­t nhau t¤i 2iºm.Chån ¡p ¡n C dC¥u 41. Bi¸tF(x ) l  mët nguy¶n h m cõa h m sè f(x ) = 1 x� 2, thäa m¢nF(3) = 1 v F (1) = 2 . Gi¡ trà cõa F(0) + F(4) b¬ng A 2 ln 2 + 3. B 2 ln 2 + 2. C 2 ln 2 + 4. D 2 ln 2. ÊLíi gi£i.Ta câ Zf(x ) d x= Z1 x� 2dx = ln jx � 2j + C:Suy ra F(x ) = ¨ln jx � 2j + C1 n¸ux > 2ln jx � 2j + C2 n¸ux < 2:Ì Tø F(3) = 1 )lnj3 � 2j + C1= 1,C1= 1.Ì Tø F(1) = 2 )lnj1 � 2j + C2= 2,C2= 2.140/305 140/305 pGV: L¶ Quang Xe  Ô0967.003.131PHT TRIšN — THAM KHƒO 2022 NÌI N€O CÂ Þ CH, NÌI  C CON ×ÍNG141VªyF(0) + F(4) = ln j0 � 2j + 2 + ln j4 � 2j + 1 = 2 ln 2 + 3 :Chån ¡p ¡n A dC¥u 42. Cho h¼nh châp S:ABCcâS A vuæng gâc vîi ¡y, m°t ph¯ng (S AB )vuæng gâc vîim°t ph¯ng (S B C ), gâc giúa hai m°t ph¯ng (S AC )v  (S B C )l  60, S B =ap 2, ÕBS C = 45. Thºt½ch khèi châp S:ABCtheo a l  A V= a3p 215. B V= 2 p 3a 3. C V= 2 p 2a 3. D V= 2a 3p 315. ÊLíi gi£i.Thº t½ch khèi châp V= 1 3S ASABC .K´ AH ?S B suy ra AH?(S B C ).Do BC ?S A v BC ?AH n¶nBC ?(S AB ), doâ tam gi¡c ABCvuæng t¤i B.K´ BI ?AC )BI ?S C v  k´ BK?S C )S C ?( BI K ).Do â gâc giúa hai m°t ph¯ng (S AC )v  (S B C )l ÕBK I = 60.Do ÕBS C = 45n¶n S B=BC =ap 2v  Kl  trungiºm cõa S Cn¶n BK =S B p 22=a.Trong tam gi¡c vuæng BI KcâBI =BK sin 60 = ap 32:Trong tam gi¡c vuæng ABCcâ1 BI2= 1 AB2+ 1 BC2)AB = BIBC pBC2� BI 2= ap 305: A CBSIH KSABC =1 2ABBC =a2p 155;S A =p S B2� AB 2= 2a p 55.Vªy V= 1 3S ASABC =2a 3p 315.Chån ¡p ¡n D dC¥u 43. K½ hi»uz1; z2; z3; z4l  bèn nghi»m phùc cõa ph÷ìng tr¼nh� z 2� 3z + 6  �z2+ 3 z+ 3 � z�9 + 2 z2+ z2= 0 :Gi¡ trà cõa biºu thùc jz1j+ jz2j+ jz3j+ jz4jb¬ng A 2p 3(1 +p 2). B 2. C 2p 2(1 +p 2). D 2p 3(1 +p 3). ÊLíi gi£i.Ta th§y �z 2� 3z + 6  �z2+ 3 z+ 3 � z�9 + 2 z2+ z2= 0, z2� �z2� 3z + 6 + �z 2+ 3 z+ 3 z+ �z 2� 3z + 6  �z2+ 3 z+ 3 = 0, –z = z2� 3z + 6z = z2+ 3 z+ 3141/305 141/305 pGV: L¶ Quang Xe  Ô0967.003.131— SÈ 9 NÌI N€O CÂ Þ CH, NÌI  C CON ×ÍNG142,–z2� 4z + 6 = 0z 2+ 2 z+ 3 = 0, –z = 2 p 2iz = �1 p 2i:Khi â, ta ÷ñc jz1j+ jz2j+ jz3j+ jz4j= 2  2 + p 2i + �1 + p 2i = 2 p 3€1 + p 2Š.Chån ¡p ¡n A dC¥u 44. Cho sè phùc zthäa m¢n jz j = 1 . T¼m gi¡ trà lîn nh§t cõa biºu thùc T= jz + 2 j+2 jz � 2j A maxT= 5 p 2. B maxT= 2 p 10. C maxT= 3 p 5. D maxT= 2 p 5. ÊLíi gi£i.Gi£ sû z= a+ bi (a; b 2R). Khi â, do jz j = 1 n¶na2+ b2= 1 .Ta câ: T= p (a + 2) 2+ b2+ 2 p (a � 2)2+ b2:Theo b§t ¯ng thùc Bunhiacopski ta câ:h È (a + 2) 2+ b2+ 2 È (a � 2)2+ b2i2 (12+2 2) (a + 2) 2+ b2+ ( a� 2)2+ b2= 5 2( a2+ b2) + 8 = 50 :Vªy max T= p 50 = 5p 2.Chån ¡p ¡n A dC¥u 45.Cho h m sè y= f(x ) câ ç thà tr¶n o¤n [� 3; 1] nh÷ h¼nh v³. Di»nt½ch c¡c ph¦n A,B ,C tr¶n h¼nh v³ câ di»n t½ch l¦n l÷ñt l  8, 3 5v 4 5. T½nh t½ch ph¥n 0Z� 2 (f (2 x+ 1) + 3) d x. A �41 5. B �42 5. C �21 5. D �82 5. O xy-3 1ÊLíi gi£i.Di»n t½ch c¡c ph¦n A,B ,C tr¶n h¼nh v³ câ di»n t½ch l¦n l÷ñt l  8, 3 5v 4 5n¶n1Z� 3 f(x )d x= �8 + 3 5�4 5=�41 5:°t t= 2 x+ 1 )dt = 2d x. Ta l¤i câ0Z� 2 (f (2 x+ 1) + 3) d x= 1Z� 3 (f (t) + 1) 2d t= 2 1Z� 3 f(t)d t+ 2 1Z� 3 dt = 2 �� 41 5‹+ 2 4 = �42 5:142/305 142/305 pGV: L¶ Quang Xe  Ô0967.003.131PHT TRIšN — THAM KHƒO 2022 NÌI N€O CÂ Þ CH, NÌI  C CON ×ÍNG143Chån ¡p ¡n B dC¥u 46. Trong khæng gian Oxyz, vi¸t ph÷ìng tr¼nh ÷íng th¯ng n¬m trong m°t ph¯ng( P ) : 2 x� y� z+ 4 = 0 v  vuæng gâc vîi ÷íng th¯ng d: x 1=y� 1 2=z+ 2 �3 . Bi¸ti qua iºmM (0; 1; 3) . A x1=y� 1 �1 =z� 3 1. B x1=y� 1 1=z� 3 1. C x1=y+ 1 �1 =z+ 3 1. D x1=y+ 1 1=z+ 3 1. ÊLíi gi£i.# n P = (2;�1; �1)# u d= (1; 2;�3) «) [#n P;#u d] = (5; 5; 5) = 5(1; 1; 1): (P )) ? #n P.M°t kh¡c ?dn¶n câ mët VTCP #u = (1; 1; 1) .Ta câ ¨Qua M(0; 1; 3)VTCP #u = (1; 1; 1) ) : x 1=y� 1 1=z� 3 1:Chån ¡p ¡n B dC¥u 47. Cho khèi l°ng trö tam gi¡c ·u ABC:A0B 0C 0. C¡c m°t ph¯ng (AB 0C )v  (A 0BC 0)chia l«ng trö th nh 4ph¦n. Thº t½ch ph¦n nhä nh§t trong 4ph¦n ÷ñc t¤o ra b¬ng bao nhi¶u bi¸tthº t½ch Vcõa l«ng trö b¬ng 1? A 124. B 112. C 18. D 136. ÊLíi gi£i.Ta câ AB0\ A0B =M ;BC 0\ B0C =N.Do ABB 0A 0; B C C 0B 0l  c¡c h¼nh chú nhªt n¶n M; Nl¦n l÷ñt l  trungiºm cõa A0B; C 0B .Gåi V1 =VB:B 0M N ; V2=VB:AC N M ; V3=VB0:A 0C 0N M 0; V4=VAA 0M C C 0NSuy ra 8>>>>><>>>>>: V2 =VB0:ABC �V1 = 1 3V�V1V 3 =VB:A 0B 0C 0� V1 = 1 3V�V1V 4 =V�(V1 +V2 +V3) = 1 3V+V1:Ta câ V B:B 0M N VB:B 0A 0C 0 = BM BA0 BN BC0= 1 4)VB:B 0M N =1 4VB:B 0A 0C 0= 1 12V= 1 12.Suy ra V2 =V3 = 1 4;V4 = 5 12.Vªy thº t½ch ph¦n nhä nh§t l  V1 = 1 12. A BCA'C' B'N MChån ¡p ¡n D dC¥u 48. Choal  sè thüc d÷ìng, a6= 1 . Bi¸t b§t ph÷ìng tr¼nh logax 3x � 3nghi»m óngvîi måi x >0. Sè athuëc tªp hñp n o sau ¥y? A (5; +1). B (2; 3). C (1; 2). D (3; 5]. ÊLíi gi£i.Nhªn th§y n¸u 0< a < 1th¼ b§t ph÷ìng tr¼nh logax 3x � 3khæng nghi»m óng vîi x= ado âa > 1.143/305 143/305 pGV: L¶ Quang Xe  Ô0967.003.131— SÈ 9 NÌI N€O CÂ Þ CH, NÌI  C CON ×ÍNG144Thayx= av o b§t ph÷ìng tr¼nh logax 3x � 3ta câ 1 3a � 3, a 4 3.Thay x= 1 av o b§t ph÷ìng tr¼nhloga1 a3 a�3suy ra a 3 2.Vªy 4 3a 3 2suy raa2 (1; 2) .Chån ¡p ¡n C dC¥u 49. Trong khæng gian vîi h» tåa ë Oxyz, cho ph÷ìng tr¼nh m°t c¦u( Sm ) :x2+ y2+ z2+ ( m+ 2) x+ 2 my �2mz �m �3 = 0 . Bi¸t vîi måi sè thüc mth¼ (Sm )luænchùa mët ÷íng trán cè ành. B¡n k½nh rcõa ÷íng trán â l  A r= 1 3. B r= 4p 23. C r= p 23. D r= p 3. ÊLíi gi£i.Gåi M(x ;y ;z ) l  iºm cè ành m  (Sm )luæn i qua. Suy rax 2+ y2+ z2+ ( m+ 2) x+ 2 my �2mz �m �3 = 0 ;8 m 2R., m(x + 2 y� 2z � 1) = �(x 2+ y2+ z2+ 2 x� 3) ;8 m 2R., ¨x + 2 y� 2z � 1 = 0x 2+ y2+ z2+ 2 x� 3 = 0 .Suy ra tªp hñp ÷íng trán cè ành ÷ñc t¤o ra bði giao cõa m°t ph¯ng (P ) : x+ 2 y� 2z � 1 = 0 v m°t c¦u (S ) : x2+ y2+ z2+ 2 x� 3 = 0 .M°t c¦u (S ) t¥m I(� 1; 0; 0) v  b¡n k½nh R= 2 v h= d(I ; (P )) = 2 3.Suy ra b¡n k½nh cõa ÷íng trán l  r= p R2� h2= Ê 22� �2 3‹2= 4p 23.Chån ¡p ¡n B dC¥u 50. Cho h m sè f(x ) = x3� 3x 2. Sè iºm cüc trà cõa h m sè f(f (x )) l  A 2. B 3. C 5. D 4. ÊLíi gi£i.Ta câ f[f (x )] = [ f(x )] 3� 3[f(x )] 2= [ f(x )] 2(f (x ) � 3), suy ra[ f (f (x ))] 0= 3 f(x )f 0( x ) [ f(x ) � 2] = 0 ,264 f(x ) = 0f 0( x ) = 0f (x ) � 2 = 0 :Ph÷ìng tr¼nh f(x ) = 0 ,x= 0; x= 3 .Ph÷ìng tr¼nh f0( x ) = 0 ,x= 0; x= 2 .X²t h m sè y= x3� 3x 2� 2câ b£ng bi¸n thi¶n nh÷ sau xy0 y �10 2 +1 +0 �0 +�1�1 �2 �2 �4 �4 +1 +1 3-2Tø b£ng bi¸n thi¶n, suy ra ph÷ìng tr¼nhf(x )� 2 = 0 ,x3� 3x 2� 2 = 0 câ duy nh§t nghi»m x0 >3.Lªp b£ng bi¸n thi¶n cõa h m sè f[f (x )] nh÷ sau144/305 144/305 pGV: L¶ Quang Xe  Ô0967.003.131MÖC LÖCNÌI N€O CÂ Þ CH, NÌI  C CON ×ÍNG145xf0( x ) f(x ) f(x ) � 2 [f (f (x ))] 0 f[f (x )] �10 1 2 3 x0 +1 +0 � �0 + + +�0 � � �0 + +� � � � �0 ++0 �0 �0 +0 �0 +�1�1 +1 +1 Düa v o b£ng bi¸n thi¶n, suy ra h m sèf[f (x )] câ 4iºm cüc trà.Chån ¡p ¡n D H˜T145/305 145/305 pGV: L¶ Quang Xe  Ô0967.003.131— SÈ 10 NÌI N€O CÂ Þ CH, NÌI  C CON ×ÍNG146BË GIO DÖC & €O T„OTR×ÍNG THPT NGUY™N T‡T TH€NH GV: L– QUANG XE - 0967.003.131 — SÈ 10 PHT TRIšN — THAM KHƒO 2022N‹M HÅC 2021 - 2022Mæn:To¡nThíi gian l m b i: 90 phót — THAM KHƒO PTMH2022dC¥u 1. Sè phùc z= 5 �7i câ sè phùc li¶n hñp l  A z= 5 + 7 i. B z= �5 + 7 i. C z= 7 �5i. D z= �5� 7i. ÊLíi gi£i.Sè phùc z= a+ bi câ sè phùc li¶n hñp l  z= a� bi.Chån ¡p ¡n A dC¥u 2. Trong khæng gian vîi h» tröc tåa ë Oxyz, m°t c¦u (S ) : ( x� 5) 2+ ( y� 1) 2+ ( z+ 2) 2= 3câ b¡n k½nh b¬ng A 3. B 9. C 2p 3. D p3. ÊLíi gi£i.B¡n k½nh cõa m°t c¦u: R= p 3.Chån ¡p ¡n D dC¥u 3. iºm n o sau ¥y thuëc ç thà (C )cõa h m sè y= x2+ 3 x+ 3 x+ 1 ? A (3; 0). B (2; 1). C (0; 3). D (� 2; 1) . ÊLíi gi£i.Ta câ y(0) = 3 n¶n iºm câ to¤ ë (0; 3)thuëc ç thà (C )cõa h m sè â cho.Chån ¡p ¡n C dC¥u 4. Di»n t½ch m°t c¦u b¡n k½nh ab¬ng A 4a 3. B 4a 2 3. C 4a 2. D 4a 3 3. ÊLíi gi£i.Di»n t½ch m°t c¦u b¡n k½nh b¬ng al  S= 4 R2= 4 a2.Chån ¡p ¡n C dC¥u 5. Cæng thùc nguy¶n h m n o sau ¥y l  sai? A Zdx x= lnx+ C. B Zx dx = x +1 + 1 +C. C Zaxdx = ax lna+C(< 6= �1) . D Z1 cos2x dx = tan x+ C. ÊLíi gi£i.Düa v o cæng thùc nguy¶n h m cì b£n. (óng l  Zdx x= lnjx j+ C).Chån ¡p ¡n A 146/305 146/305 pGV: L¶ Quang Xe  Ô0967.003.131PHT TRIšN — THAM KHƒO 2022 NÌI N€O CÂ Þ CH, NÌI  C CON ×ÍNG147dC¥u 6.Cho h m sè f(x ) câ b£ng bi¸nthi¶n nh÷ h¼nh v³. Gi¡ trà cüctiºu cõa h m sè b¬ng A 3. B 1. C 0. D �1. xf0( x ) f(x ) �1�1 0 1 +1 �0 +0 �0 ++1 +1 00 3300 +1 +1 ÊLíi gi£i.Düa v o b£ng bi¸n thi¶n, gi¡ trà cüc tiºu cõa h m sè b¬ng 0.Chån ¡p ¡n C dC¥u 7. T¼m tªp nghi»m cõa b§t ph÷ìng tr¼nh log3(x � 2) >2. A (�1 ; 11) . B (2; +1). C [11; +1). D (11; +1). ÊLíi gi£i.i·u ki»n: x� 2> 0, x > 2:V¼ 3> 1n¶n log3(x � 2) >2, x� 2> 32, x> 11.Vªy tªp nghi»m cõa b§t ph÷ìng tr¼nh l  [11; +1).Chån ¡p ¡n C dC¥u 8. Cho h¼nh châp S:ABC Dcâ ¡yABC D l  h¼nh chú nhªt, AB=a, AD = 2a, S Avuæng gâc vîi (ABC D ), S A =ap 3. Thº t½ch cõa khèi châp S:ABC Dl  A a3p 33. B 2a 3p 33. C 2a 3p 3. D a3p 3. ÊLíi gi£i.Ta câ: V= 1 3S ASABC D =1 3S AAB AD =2a 3p 33.Chån ¡p ¡n B dC¥u 9. Tªp x¡c ành cõa h m sè y= ( x� 1)1 2l  A (0; +1). B [1; +1). C (1; +1). D (�1 ; +1). ÊLíi gi£i.Biºu thùc (x � 1)1 2câ ngh¾a khi v  ch¿ khi x >1.Vªy tªp x¡c ành cõa h m sè ¢ cho l  (1; +1).Chån ¡p ¡n C dC¥u 10. Gi£i ph÷ìng tr¼nh log2(1�x) = 2 . A x= �4. B x= 3 . C x= �3. D x= 5 . ÊLíi gi£i.Ta câ log2(1�x) = 2 ,¨1� x > 01 � x= 2 2,x= �3.Chån ¡p ¡n C 147/305 147/305 pGV: L¶ Quang Xe  Ô0967.003.131— SÈ 10 NÌI N€O CÂ Þ CH, NÌI  C CON ×ÍNG148dC¥u 11. T½nh t½ch ph¥n I= 2Z1 x� 1 xdx . A I= 1 �ln 2 . B I= 7 4. C I= 1 + ln 2 . D I= 2 ln 2 . ÊLíi gi£i.Ta câ I= 2Z1 �1 � 1 x‹dx = ( x� lnjx j) 21 = 1�ln 2 .Chån ¡p ¡n A dC¥u 12. Cho sè phùc z= 1 �1 3i. T½nh sè phùc w= i z+ 3 z. A w= 8 3. B w= 8 3+i. C w= 10 3+i. D w= 10 3. ÊLíi gi£i.Ta câ w= i�1 + 1 3i‹+ 3 �1 � 1 3i‹= �3 � 1 3‹+ i(1 �1) = 8 3:Chån ¡p ¡n A dC¥u 13. Trong khæng gian Oxyz, cho m°t ph¯ng (P ) : x� 4y + 3 z� 2 = 0 . Mët v²c-tì ph¡ptuy¸n cõa m°t ph¯ng (P )l  A #n 2= (1; 4; 3). B #n 3= (�1; 4; �3) . C #n 4= (�4; 3; �2) . D #n 1= (0;�4; 3) . ÊLíi gi£i.( P )câ v²c-tì ph¡p tuy¸n l  #n = (1; �4; 3) n¶n#n 3 = (�1; 4; �3) = �#n công l  v²c-tì ph¡p tuy¸ncõa (P ).Chån ¡p ¡n B dC¥u 14. Trong khæng gian vîi h» tåa ë OxyzchoA(1; 2; 3) ,B (5; 2; 0) . Khi â A # AB = 5. B # AB = 2 p 3. C # AB = p 61. D # AB = 3. ÊLíi gi£i. # AB = p (5�1)2+ (2 �2)2+ (0 �3)2= 5 .Chån ¡p ¡n A dC¥u 15. iºm biºu di¹n cõa sè phùc z= 1 2� 3i l  A �2 13;3 13‹. B (4;�1) . C (2;�3) . D (3;�2) . ÊLíi gi£i.Ta câ z= 1 2� 3i =2 + 3i (2�3i)(2 + 3 i) =2 + 3i 13=2 13+3 13i. Suy ra iºm biºu di¹n cõa sè phùcz = 2 13+3 13il  �2 13;3 13‹.Chån ¡p ¡n A 148/305 148/305 pGV: L¶ Quang Xe  Ô0967.003.131PHT TRIšN — THAM KHƒO 2022 NÌI N€O CÂ Þ CH, NÌI  C CON ×ÍNG149dC¥u 16. Ti»m cªn ùng cõa ç thà h m sè y= 2� x x+ 3 l  A y= �3. B y= �1. C x= �3. D x= 2 . ÊLíi gi£i.H m sè y= 2� x x+ 3 x¡c ành tr¶nRn f� 3g .Ta câ limx !� 3� 2� x x+ 3 =�1 ,limx !� 3+ 2� x x+ 3 = +1.Do â x= �3 l  ti»m cªn ùng cõa ç thà h m sè.Chån ¡p ¡n C dC¥u 17. Choa >0, a 6= 1 ,b > 0v  logab= 2 . Gi¡ trà cõa logab(a 2) b¬ng A 16. B 12. C 23. D 1. ÊLíi gi£i.Ta câ logab= 2 ,b= a2. Thay v o P, ta ÷ñcP = logaa 2(a 2) = log a3(a 2) = 2 3.Chån ¡p ¡n C dC¥u 18. ç thà n o trong h¼nh d÷îi ¥y l  ç thà cõa h m sè y= x4+ 2 x2� 3? A O xy�2 1 2�3 1. B O xy�1 1�3 1.C O xy�2 1 2�3 1. D O xy�2 1 2�4 �3 1.ÊLíi gi£i.149/305 149/305 pGV: L¶ Quang Xe  Ô0967.003.131— SÈ 10 NÌI N€O CÂ Þ CH, NÌI  C CON ×ÍNG150Ta cây0= 4 x3+ 4 x, y0= 0 ,x= 0 .Tø b£ng bi¸n thi¶n suy ra h m sè câ 1cüc tiºu.Cho y= 0 ta câ x4+ 2 x2� 3 = 0 ,–x 2= 1x 2= �3 ,x= 1:Suy ra ç thà h m sè i qua iºm (� 1; 0) v (1; 0) . xf0( x ) f(x ) �10 +1 �0 ++1 +1 �3 �3 +1 +1 Chån ¡p ¡n B dC¥u 19. Trong khæng gian Oxyz, ÷íng th¯ng d: x� 1 2=y� 2 �1 =z� 3 2i qua iºm n od÷îi ¥y? A Q(2; �1; 2) . B M(� 1; �2; �3) . C P(1; 2; 3) . D N(� 2; 1; �2) . ÊLíi gi£i.Thay l¦n l÷ñt tåa ë c¡c iºm ¢ cho v o ph÷ìng tr¼nh cõa ÷íng th¯ng d, ta câÌ Vîi M(� 1; �2; �3) th¼ �1� 1 26= �2� 2 �1 , suy radkhæng i qua iºm M.Ì Vîi N(� 2; 1; �2) th¼ �2� 1 26= 1� 2 �1 , suy radkhæng i qua iºm N.Ì Vîi P(1; 2; 3) th¼1� 1 2=2� 2 �1 =3� 3 2= 0, suy ra di qua iºm P.Ì Vîi Q(2; �1; 2) th¼2� 1 26= �1� 2 �1 , suy radkhæng i qua iºm Q.Chån ¡p ¡n C dC¥u 20. Tø7chú sè 1, 2, 3, 4, 5, 6, 7 câ thº lªp ÷ñc bao nhi¶u sè tü nhi¶n câ 4chú sè æimët kh¡c nhau? A 74. B 76 5 4. C 7!. D 7!6! 5! 4! . ÊLíi gi£i.Sè c¡c sè tü nhi¶n thäa m¢n l  A47 = 76 5 4.Chån ¡p ¡n B dC¥u 21. Mët khèi châp câ thº t½ch b¬ng 15v  di»n t½ch ¡y b¬ng 9. Chi·u cao cõa khèi châpâ b¬ng A 53. B 5. C 95. D 6. ÊLíi gi£i.Thº t½ch khèi châp l  V= 1 3B h vîi Bl  di»n t½ch ¡y, hl  chi·u cao.Do â h= 3V B=315 9= 5.Chån ¡p ¡n B dC¥u 22. T½nh ¤o h m cõa h m sè y= 2017 x: A y0= 2017 xln 2017 . B y0= 2017 x.150/305 150/305 pGV: L¶ Quang Xe  Ô0967.003.131PHT TRIšN — THAM KHƒO 2022 NÌI N€O CÂ Þ CH, NÌI  C CON ×ÍNG151C y0= 2017x ln 2017. D y0= x2017 x� 1. ÊLíi gi£i.y 0= (2017 x)0= 2017 xln 2017 :Chån ¡p ¡n A dC¥u 23. Cho h m sè phò hñp b£ng bi¸n thi¶n sau. M»nh · n o d÷îi ¥y óng? xy0 y �1�1 0 1 +1 +0 � �0 +�1 �1 1111�1 +1 55 +1 +1 A H m sè çng bi¸n tr¶n kho£ng(�1 ;� 1) [(1; + 1)v  nghàch bi¸n tr¶n (� 1; 0) [(0; 1) . B H m sè çng bi¸n tr¶n hai kho£ng(�1 ;� 1) ;(11; + 1)v  nghàch bi¸n tr¶n kho£ng (� 1; 11) . C H m sè çng bi¸n tr¶n hai kho£ng(�1 ;� 1) ;(1; + 1)v  nghàch bi¸n tr¶n kho£ng (� 1; 1) . D H m sè çng bi¸n tr¶n hai kho£ng(�1 ;� 1) ;(1; + 1)v  nghàch bi¸n tr¶n hai kho£ng( � 1; 0); (0; 1) . ÊLíi gi£i.Qua b£ng bi¸n thi¶n ta th§y h m sè çng bi¸n tr¶n hai kho£ng (�1 ;� 1) ;(1; + 1)v  nghàch bi¸ntr¶n hai kho£ng (� 1; 0); (0; 1) .Chån ¡p ¡n D dC¥u 24. Mët h¼nh nân trán xoay câ ÷íng cao h, b¡n k½nh ¡y rv  ÷íng sinh l. Biºu thùcn o sau ¥y dòng º t½nh di»n t½ch xung quanh cõa h¼nh nân? A Sxq =r h . B Sxq = 2h . C Sxq = 2r h . D Sxq =r l. ÊLíi gi£i.Di»n t½ch xung quanh h¼nh nân câ b¡n k½nh ¡y rv  ÷íng sinh ll  Sxq =r l.Chån ¡p ¡n D dC¥u 25. N¸u4Z0 f(x ) d x= 5 v 4Z2 f(x ) d x= �1 th¼ 2Z0 f(x ) d xb¬ng A 6. B 4. C �4. D �6. ÊLíi gi£i.4Z0 f(x ) d x= 5 , 2Z0 f(x ) d x+ 4Z2 f(x ) d x= 5 , 2Z0 f(x ) d x= 5 �4Z2 f(x ) d x= 5 + 1 = 6 .Chån ¡p ¡n A dC¥u 26. T¼m sè h¤ng ¦u u1 v  cæng saidcõa c§p sè cëng câ u2 = 3v u3 = 4. A u1 = 1; d= 1 . B u1 = 2; d=�1. C u1 = 2; d= 1 . D u1 = 1; d=�1.151/305 151/305 pGV: L¶ Quang Xe  Ô0967.003.131— SÈ 10 NÌI N€O CÂ Þ CH, NÌI  C CON ×ÍNG152ÊLíi gi£i.Ta câ d= u3 �u2 = 1v u1 =u2 �d= 2 .Chån ¡p ¡n C dC¥u 27. Hå nguy¶n h m cõa h m sè f(x ) = 1 x+ 1 l  A logj1 + xj+ C. B ln(1 +x) + C. C �1 (1 +x)2 +C. D lnj1 + xj+ C. ÊLíi gi£i.Ta câ Z1 x+ 1 dx = Z1 x+ 1 d(x+ 1) = ln jx + 1 j+ C.Chån ¡p ¡n D dC¥u 28. Cho h m sè y= f(x ) li¶n töc tr¶n Rv  câ b£ng x²t d§u cõa ¤o h m nh÷ sau. H msè ¢ cho câ bao nhi¶u iºm cüc trà? xf0( x ) �1�1 0 2 4 +1 +0 � +0 �0 +A 2. B 1. C 4. D 3. ÊLíi gi£i.Düa v o b£ng x²t d§u f0( x ), h m sè f(x ) câ 4iºm x0 m  t¤i âf0( x ) êi d§u khi xqua iºm x0.Vªy h m sè ¢ cho câ 4iºm cüc trà.Chån ¡p ¡n C dC¥u 29. Gi¡ trà lîn nh§t cõa h m sè y= x3� 3x + 2 tr¶n o¤n [� 3; 3] b¬ng A 20. B 4. C 0. D �16 . ÊLíi gi£i.Ta câ y0= 3 x2� 3; y0= 0 ,x= 1.y (� 3) = �16 ;y(� 1) = 4 ;y(1) = 0 ;y(3) = 20 .H m sè li¶n töc v  câ ¤o h m tr¶n o¤n [� 3; 3] do â ymax = 20.Chån ¡p ¡n A dC¥u 30. H m sèy= p �x2+ 2 xnghàch bi¸n tr¶n kho£ng n o? A (0; 1). B (1; 2). C (�1 ; 1). D (1; +1). ÊLíi gi£i.Ì TX: D= [0; 2] .Ì y0= �x + 1 p�x2+ 2 x)y0= 0 , �x + 1 p�x2+ 2 x= 0,x= 1 .Ta câ b£ng x²t d§u cõa h m sè y= f0( x ) nh÷ b¶n d÷îi xf0( x ) 0 1 2+0 -152/305 152/305 pGV: L¶ Quang Xe  Ô0967.003.131PHT TRIšN — THAM KHƒO 2022 NÌI N€O CÂ Þ CH, NÌI  C CON ×ÍNG153Vªy h m sè nghàch bi¸n tr¶n(1; 2).Chån ¡p ¡n B dC¥u 31. Bi¸tlog3x= a, log3y= b. T½nh logp 3(3xy )theo a, b. A logp 3(3xy ) = 2(1 + a+ b). B logp 3(3xy ) = 1 +a+ b 2. C logp 3(3xy ) = 2 + a+ b. D logp 3(3xy ) = 1 + a+ b 2. ÊLíi gi£i.Ta câ logp 3(3xy ) = log p 33 + log p 3x+ log p 3y= 2 + 2 log3x+ 2 log3y= 2 + 2 a+ 2 b= 2(1 + a+ b).Chån ¡p ¡n A dC¥u 32. Cho h¼nh lªp ph÷ìng ABC D:A0B 0C 0D 0câ I ; J t÷ìng ùng l  trung iºm cõa BC; B B0.Gâc giúa hai ÷íng th¯ng AC; I Jb¬ng A 30. B 60. C 45. D 120. ÊLíi gi£i.Ta câ I JB0C n¶n suy ra (AC; I J ) = (AC; B 0C ).V¼ ABC D:A 0B 0C 0D 0l  h¼nh lªp ph÷ìng (°t AB=a) n¶n tacâ B0C =AC =AB 0= ap 2. Suy ra tam gi¡c AB0C ·u n¶n( AC; B 0C ) = 60 .Vªy Ø�( AC; I J ) =Ú�( B 0C; AC ) = 60 . A BICD0 C0 DB0 JA0 Chån ¡p ¡n B dC¥u 33. Cho2Z1 f(u ) d u= 4 v 4Z1 f(v ) d v= 18 . T½nh t½ch ph¥n I= 4Z2 •f (x ) � 3 x˜dx . A I= 22 �3 ln 2 . B I= 14 + 3 ln 2 . C I= 14 �3 ln 2 . D I= 22 + 3 ln 2 . ÊLíi gi£i.Ta câI= 4Z2 •f (x ) � 3 x˜dx = 4Z2 f(x ) d x� 4Z2 3 xdx= 4Z1 f(x ) d x� 2Z1 f(x ) d x� 3 ln xj42= 18 �4� 3 (ln 4 �ln 2) :Vªy I= 14 �3 ln 2 .Chån ¡p ¡n C 153/305 153/305 pGV: L¶ Quang Xe  Ô0967.003.131— SÈ 10 NÌI N€O CÂ Þ CH, NÌI  C CON ×ÍNG154dC¥u 34. Trong khæng gian vîi h» tåa ë Oxyz, cho hai iºm A(1; 1; 1) v B(1; 3; 5) . Vi¸tph÷ìng tr¼nh m°t ph¯ng trung trüc cõa o¤n AB. A y� 2z � 6 = 0 . B y� 3z + 4 = 0 . C y+ 2 z� 8 = 0 . D y� 2z + 2 = 0 . ÊLíi gi£i.Ta câ I(1; 2; 3) l  trung iºm cõa o¤n AB.M°t ph¯ng trung trüc cõa o¤n th¯ng ABi qua Iv  câ v²c-tì ph¡p tuy¸n # AB = (0; 2; 4) = 2(0; 1; 2) ,suy ra ph÷ìng tr¼nh m°t ph¯ng trung trüc c¦n t¼m l 0(x� 1) + 1( y� 2) + 2( z� 3) = 0 ,y+ 2 z� 8 = 0 .Chån ¡p ¡n C dC¥u 35. Cho sè phùc zthäa m¢n (1 + 3i) z � 5 = 7 i:M»nh · n o sau ¥y óng? A z= �13 5+4 5i. B z= 13 5�4 5i. C z= 13 5+4 5i. D z= �13 5�4 5i. ÊLíi gi£i.Ta câ z= 5 + 7i 1 + 3i=13 5�4 5i) z= 13 5+4 5iChån ¡p ¡n C dC¥u 36.Cho h¼nh châp S:ABC DcâS A ?(ABC D ), S A =av ABC D l  h¼nh vuæng câ c¤nh b¬ng a. T½nh kho£ng c¡ch dtø iºm A¸n m°t ph¯ng (S C D ). Kh¯ng ành n o sau ¥yóng? A d =ap 22. B d =ap 2. C d =ap 32. D d =a 2. B AC DSÊLíi gi£i.Gåi Hl  trung iºm o¤n S A, v¼ S A=AD =an¶n AH ?S D .L¤i câ: DC?AD v DC ?S A n¶n DC ?(S AD ). Tø â, DC?AH .Vªy AH?(S DC ), suy rad[ A; ( S C D )] =AH=S D 2=ap 22: B AC DSHChån ¡p ¡n A dC¥u 37. X¡c su§t mët x¤ thõ b­n tróng hçng t¥m l  0;3 . Ng÷íi â b­n 3l¦n. T½nh x¡c su§tº ng÷íi â b­n tróng ½t nh§t 1l¦n. A 0;027 . B 0;657 . C 0;973 . D 0;343 . ÊLíi gi£i.154/305 154/305 pGV: L¶ Quang Xe  Ô0967.003.131PHT TRIšN — THAM KHƒO 2022 NÌI N€O CÂ Þ CH, NÌI  C CON ×ÍNG155ÌX¡c su§t c£ ba l¦n ·u b­n tr÷ñt l  (0;7) 3.Ì X¡c su§t b­n tróng ½t nh§t 1l¦n l  1� (0;7) 3= 0 ;657 :Chån ¡p ¡n B dC¥u 38. Trong khæng gian vîi h» tröc tåa ë Oxyzvi¸t ph÷ìng tr¼nh ÷íng th¯ng giao tuy¸ncõa hai m°t ph¯ng (P ) : x+ 3 y� z+ 1 = 0 ,(Q ) : 2 x� y+ z� 7 = 0 . A x+ 2 2=y �3 =z+ 3 �7 . B x� 2 2=y 3=z� 3 �7 . C x�2 =y� 3 �3 =z� 10 7. D x� 2 �2 =y 3=z� 3 7. ÊLíi gi£i.Gåi (d ) l  ÷íng th¯ng giao tuy¸n cõa hai m°t ph¯ng (P )v  (Q ).X²t h» ph÷ìng tr¼nh ¨x + 3 y� z+ 1 = 02 x � y+ z� 7 = 0 :Cho x= 2 , ta câ y= 0 ; z= 3 , ta câ A(2; 0; 3) 2(d ).M°t kh¡c #u d = [ #n (P ); #n (Q )] = (2;�3; �7) ta câ thº chån #u 1 = (�2; 3; 7) l m v²c-tì ch¿ ph÷ìng cõa( d ).Vªy (d ) : x� 2 �2 =y 3=z� 3 7.Chån ¡p ¡n D dC¥u 39. Câ bao nhi¶u sè nguy¶n d÷ìng ysao cho ùng vîi méi ycâ khæng qu¡ 5 sè nguy¶n xthäa m¢n €3x+2�p 3Š(y � 3x) > 0? A 79. B 80. C 81. D 82. ÊLíi gi£i.°t t= 3 x;(t > 0), b§t ph÷ìng tr¼nh ¢ cho trð th nh€9t � p 3Š(y � t) > 0, ‚t� p 39Œ(t � y) < 0: (1)V¼ y2 Z+n¶n y > p 39, do â b§t ph÷ìng tr¼nh(1) ,p 39< t < y)p 39<3x< y , � 3 2< x <log3y:Do méi y2 Z+câ khæng qu¡ 5 sè nguy¶n x2 �� 3 2; log3y‹n¶n� 1 log3y 4, 1 3y 34, 1 3y 81:Vªy y2 f 1; 2; 3; 4; : : :; 81 gn¶n câ 81 gi¡ trà nguy¶n d÷ìng cõa y.Chån ¡p ¡n C dC¥u 40. Cho h m sè y= x4� 3x 2� 2. T¼m sè thüc d÷ìng mº ÷íng th¯ng y= m c­t çthà h m sè t¤i 2 iºm ph¥n bi»t A; Bsao cho tam gi¡c OABvuæng t¤i O, trong â Ol  gèc tåaë.155/305 155/305 pGV: L¶ Quang Xe  Ô0967.003.131— SÈ 10 NÌI N€O CÂ Þ CH, NÌI  C CON ×ÍNG156A m= 2 . B m= 3 2. C m= 3 . D m= 1 . ÊLíi gi£i.D¹ th§y tam gi¡c OABc¥n t¤i O, do â n¸u tam gi¡c OABvuæng th¼nâ vuæng c¥n t¤i O.Tø â suy ra ÕOAB =ÕOB A = 45. Gi£ sû Al  iºm câ ho nh ëd÷ìng, th¸ th¼ A(jm j; m )v  B(�j mj; m ).Do Athuëc ç thà h m sè y= x4� 3x 2� 2n¶n m4� 3m 2� m �2 = 0 ,tø â suy ra m= 2 l  ¡p ¡n óng. O xyy= m B AChån ¡p ¡n A dC¥u 41. Cho h m sè y= f(x ) x¡c ành tr¶n Rn f 1g thäa m¢n f0( x ) = 1 x� 1,f(0) = 1 v f (2) = 2 . Khi â f(� 3) + f(3) b¬ng A 3 + 3 ln 2. B 4 + 3 ln 2. C 2 + 3 ln 2. D 1 + 3 ln 2. ÊLíi gi£i.Ta câf(x ) = ¨ln( x� 1) + C1;khi x > 1ln(1 �x) + C2;khi x < 1:Do f(0) = 1 v f(2) = 2 n¶nC1= 2v C2= 1. Do â f(� 3) + f(3) = 3 ln 2 + 3 .Chån ¡p ¡n A dC¥u 42. Cho h¼nh châp S:ABC Dcâ t§t c£ c¡c c¤nh ·u b¬ng a. Gåi Fl  trung iºm cõac¤nh S A. T½nh kho£ng c¡ch tø S¸n m°t ph¯ng (F C D ). A 12a. B É15a. C É211a. D É219a. ÊLíi gi£i.Gåi Ol  t¥m cõa h¼nh vuæng ABC D. Ta c⨠S A =S C =aAC =ap 2) 4S AC vuæng c¥n t¤i S:Khi â, S O=ap 22.H¼nh châp S:ABC Dcâ t§t c£ c¡c c¤nh ·u b¬ng an¶n S O?( ABC D ).Suy ra VS:AC D =1 3S OSAC D =1 3ap 22a2 2=a3p 212.Ta câ VS:C DF VS:C DA =S F S A=1 2)VS:C DF =a3p 224: O AB SCDF156/305 156/305 pGV: L¶ Quang Xe  Ô0967.003.131PHT TRIšN — THAM KHƒO 2022 NÌI N€O CÂ Þ CH, NÌI  C CON ×ÍNG157X²t4DF C câDC =a; DF =ap 32;C F 2= AC2+ S C 2 2�S A2 4=5a 2 4)C F =ap 52:Suy ra SDF C =a2p 118.Vªyd(S; (F C D )) =3VS:C DF SDF C =3a3p 224a2p 118=ap 2211:Chån ¡p ¡n C dC¥u 43. Cho sè phùc wv  hai sè thüc a; b. Bi¸t z1 =w� 2� 3i v  z2 = 2w� 5l  hai nghi»mphùc cõa ph÷ìng tr¼nh z2+ az +b= 0 :T½nh T= jz 21 j+ jz 22 j: A T= 4 p 13. B T= 10 . C T= 5 . D T= 25 . ÊLíi gi£i.Theo ành l½ Vi-²t ¨z1 +z2 =�a (1)z 1 z2 =b (2):Theo gi£ thi¸t )2z1 �z2 = 1�6i (3):Tø (1) v (3) )8><>: z1 = 1� a 3�2iz 2 =�1 + 2a 3+ 2i:Thay v o (2))�1� a 3�2i‹ ��1 + 2a 3+ 2i‹= b, (a � 1)(1 + 2 a) 9+ 4 +4 + 2a 3i= b:V¼ a; b 2R n¶n 8><>: (a � 1)(1 + 2 a) 9+ 4 =b4 + 2 a 3= 0 ,¨a = �2b = 5 :Khi â z1 = 1�2i, z2 = 1 + 2i) T= jz 21 j+ jz 22 j= 5 :Chån ¡p ¡n C dC¥u 44. X²t c¡c sè phùc z, w thäa m¢n w= iz v  j(1 + i) z + 2 �2ij = p 2. Gi¡ trà lîn nh§tcõa jz � wjb¬ng A 3. B 2p 3. C 3p 2. D 3p 3. ÊLíi gi£i.Ta câ j(1 + i) z + 2 �2ij = p 2) p 2 j (1 + i) z j � j 2� 2ij , p 2 p 2jz j � 2p 2) j zj  3.L¤i câ jz � wj= jz � izj= jz j  j 1� ij = p 2jz j  3p 2.Vªy jz � wjmax = 3p 2.Chån ¡p ¡n C dC¥u 45.157/305 157/305 pGV: L¶ Quang Xe  Ô0967.003.131— SÈ 10 NÌI N€O CÂ Þ CH, NÌI  C CON ×ÍNG158Cho h m sèf(x ) = mx4+ nx 3+ px 2+ qx +rvîi (m; n; p; q; r 2R).H m sè f0( x ) câ ç thà nh÷ h¼nh b¶n. Tªp nghi»m cõa ph÷ìngtr¼nh f(x ) = rcâ sè ph¦n tû l  A 4. B 2. C 3. D 1. xyO�1 354ÊLíi gi£i.Ta câ f0( x ) = 4 mx3+ 3 nx3+ 2 px+q (1).Tø ç thà h m sè f0( x ), ta câ ph÷ìng tr¼nh f0( x ) = 0 câ ba nghi»mìn x= �1, x = 5 4v x= 3 .Do â f0( x ) = m(x + 1) �x � 5 4‹(x � 3), vîi m <0.S 1 = 5 4Z0 m(x + 1) �x � 5 4‹(x � 3) dx = 8575jm j 3072.S 2 = 3Z5 4 m(x + 1) �x � 5 4‹(x � 3) dx = 8575jm j 3072. xyO�1 354S1 S2 S1 =S2 , � 5 4Z0 f0( x ) d x= 3Z5 4f0( x ) d x, f(3) = f(0) = r.B£ng bi¸n thi¶n xf0( x ) f(x ) �1�1 54 3 +1 +0 �0 +0 ��1�1 rr�1�10rSuy ra ph÷ìng tr¼nhf(x ) = rcâ t§t c£ 3nghi»m.Chån ¡p ¡n C dC¥u 46. Trong khæng gian vîih» tåa ë Oxyz, cho m°t ph¯ng (P ) : x+ y+ z� 3 = 0 v  ÷íngth¯ng d: 8><>: x= 1 �ty = 1 + tz = 1 . Ph÷ìng tr¼nh ÷íng th¯ngn¬m trong m°t ph¯ng (P ), çng thíi c­t v 158/305 158/305 pGV: L¶ Quang Xe  Ô0967.003.131PHT TRIšN — THAM KHƒO 2022 NÌI N€O CÂ Þ CH, NÌI  C CON ×ÍNG159vuæng gâc vîi ÷íng th¯ngdl   : 8><>: x= 1 + aty = 1 + btz = c+ 2 tvîia; b; c 2R. Gi¡ trà a+ b+ cb¬ng A �1. B 0. C 4. D 1. ÊLíi gi£i.Gåi A= d\ (P )) A(1; 1; 1) .Ta câ #n (P ) = (1; 1; 1),#u (d ) = (�1; 1; 0) )#u 4 = #n (P ); #u (d )= ( �1; �1; 2) .Suy ra  :8><>: x= 1 �ty = 1 �tz = 1 + 2 t:Suy ra a+ b+ c= ( �1) + ( �1) + 1 = �1.Chån ¡p ¡n A dC¥u 47.Cho mët h¼nh trö trán xoay v  h¼nh vuæng ABC Dc¤nhacâ 2 ¿nh li¶nti¸p A; B n¬m tr¶n ÷íng trán ¡y thù nh§t cõa h¼nh trö, hai ¿nh cán l¤in¬m tr¶n ÷íng trán ¡y thù hai cõa h¼nh trö. M°t ph¯ng (ABC D )t¤ovîi ¡y cõa h¼nh trö gâc 45. T½nh thº t½ch khèi trö. A 3a 3 16. B p2a 3 16. C a3 16. D 3p 2a 3 16. DCNABM IOO0 ÊLíi gi£i. Gåi M; N l¦n l÷ñt l  trung iºm cõa ABv C D ,O v  O0l¦n l÷ñt l  t¥m hai m°t ¡y. Gåi Il  giaoiºm cõa M Nv OO 0. Gâc giúa m°t ph¯ng (ABC D )v  m°t ¡y l  gâc (M N; OM ) =ÕI M O . Do â ÕI M O = 45. Suy ra4 I M O vuæng c¥n t¤i O. Ta câ M N=BC =an¶n I M =a 2v AM =1 2AB=a 2. Suy raOM=OI = a 2p 2)OO 0=2 OI =a p2.4 OM A vuæng t¤i Mn¶n OA2= OM 2+ AM 2= a2 8+a2 4=3a 2 8. Suy raR= OA =ap 32p 2. Ta câ Vtrö =R 2h = 3a 2 8a p2=3p 2a 3 16.Chån ¡p ¡n D dC¥u 48. Trong c¡c nghi»m (x ;y ) thäa m¢n b§t ph÷ìng tr¼nh logx2+2 y2(2x+ y)  1. Gi¡ tràlîn nh§t cõa biºu thùc T= 2 x+ yb¬ng A 94. B 92. C 98. D 9. ÊLíi gi£i.159/305 159/305 pGV: L¶ Quang Xe  Ô0967.003.131— SÈ 10 NÌI N€O CÂ Þ CH, NÌI  C CON ×ÍNG160TH1:x2+ 2 y2> 1. °t z= yp 2, suy ra x2+ z2> 1 (1) . Khi â:log x2+2 y2(2x+ y)  1, 2x + y x2+ 2 y2, 2x + z p2x2+ z2, (x � 1)2+ �z � 1 2p 2‹2 9 8(2).Tªp hñp c¡c iºm M(x ;y ) l  mi·n (H )bao gçm mi·n ngo i cõa h¼nh trán (C1) :x2+ z2= 1 v  mi·ntrong cõa h¼nh trán (C2) : (x� 1)2+ �z � 1 2p 2‹2= 9 8.H» 8>>>>><>>>>>: T= 2 x+ z p2( x � 1)2+ �z � 1 2p 2‹2 9 8x 2+ z2> 1 câ nghi»m khi ÷íng th¯ngd: 2 x+ z p2�T = 0 câ iºm chung vîimi·n (H ).º T¤t gi¡ trà lîn nh§t th¼ ÷íng th¯ng dph£i ti¸p xóc vîi ÷íng trán (C2), ngh¾a l  ta câd( I ; d ) = 3 2p 2, T� 9 4 = 9 4,T= 9 2vîiI�1; 1 2p 2‹l  t¥m cõa ÷íng trán (C2).TH2: 0< x 2+ 2 y2< 1ta câlog x2+2 y2(2x+ y)  1, 2x + y x2+ 2 y2, T= 2 x+ y < 1(lo¤i).Vªy max T= 9 2.Chån ¡p ¡n B dC¥u 49. Cho ba h¼nh c¦u ti¸p xóc ngo i nhau tøng æi mët v  còng ti¸p xóc vîi mët m°tph¯ng. C¡c ti¸p iºm cõa c¡c h¼nh c¦u tr¶n m°t ph¯ng lªp th nh tam gi¡c câ c¡c c¤nh b¬ng 4, 2v  3. T½ch b¡n k½nh cõa ba h¼nh c¦u tr¶n l  A 12. B 3. C 6. D 9. ÊLíi gi£i.Gåi I ; J; K l¦n l÷ñt l  t¥m cõa ba h¼nh c¦u v  A; B; Cl¦n l÷ñtl  h¼nh chi¸u cõa I ; J; Ktr¶n m°t ph¯ng ¢ cho.°t I A=R1; J B=R2; K C=R3 l  b¡n k½nh cõa ba h¼nh c¦u(khæng m§t t½nh têng qu¡t gi£ sû R1> R2> R3).Trong h¼nh thang vuæng I ABJk´J H ?I A th¼ ta câ J H=ABv  I H =R1�R2.Trong tam gi¡c I J Hvuæng ta câI J 2= J H 2+ I H 2, (R1+R2)2= AB 2+ ( R1�R2)2, R1R2= AB2 4: ABC IJKHT÷ìng tü ta câR2R3= BC2 4; R1R3= AC2 4)R1R2R3= ABBC C A 8= 3.Chån ¡p ¡n B dC¥u 50. Cho h m sè y= f(x ) câ ¤o h m l  f0( x ) = x(x � 1)2(x � 2);8 x 2 R. X²t h m sèg (x ) = f�5x x2+ 4 ‹. Trong c¡c kh¯ng ành sau kh¯ng ành n o l  óng? A H m sè çng bi¸n tr¶n(0; 1). B H m sè nghàch bi¸n tr¶n(0; 4). C H m sè ¤t cüc ¤i t¤ix= 0 . D H m sè ¤t gi¡ trà nhä nh§t t¤ix= 1 . ÊLíi gi£i.160/305 160/305 pGV: L¶ Quang Xe  Ô0967.003.131MÖC LÖCNÌI N€O CÂ Þ CH, NÌI  C CON ×ÍNG161�p döng cæng thùc[f (u )] 0= u0 f 0( u ) ta câg 0( x ) = •f �5x x2+ 4 ‹˜0= �5x x2+ 4 ‹0 f 0�5x x2+ 4 ‹= 5(4�x2) (x 2+ 4) 2 5x x2+ 4 �5x x2+ 4 �1‹2�5x x2+ 4 �2‹= 5(4�x2) 5x (5 x� x2� 4)2(5 x� 2x 2� 8) (x 2+ 4) 6 :g 0( x ) = 0 ,x= �2; x= 0; x= 1; x= 2; x= 4 .V¼ (5x� 2x 2� 8) <0;8 x 2 R v  (5x� x2� 4)2= ( x� 1)2(x � 4)2 0;8 x 2 R n¶n ta câ b£ng bi¸nthi¶n sau xg0( x ) g(x ) �1�2 0 1 2 4 +1 �0 +0 �0 �0 +0 +Düa v o b£ng bi¸n thi¶n ta th§y h m sè ¤t cüc ¤i t¤ix= 0 .Chån ¡p ¡n C H˜T161/305 161/305 pGV: L¶ Quang Xe  Ô0967.003.131— SÈ 11 NÌI N€O CÂ Þ CH, NÌI  C CON ×ÍNG162BË GIO DÖC & €O T„OTR×ÍNG THPT NGUY™N T‡T TH€NH GV: L– QUANG XE - 0967.003.131 — SÈ 11 PHT TRIšN — THAM KHƒO 2022N‹M HÅC 2021 - 2022Mæn:To¡nThíi gian l m b i: 90 phót — THAM KHƒO PTMH2022dC¥u 1. T¼m sè phùc zbi¸t r¬ng z+ 2 z= 3 �i. A z= 1 + i. B z= 1 �1 3i. C z= �1 + i. D z= �1� i. ÊLíi gi£i.Ta th§y z+ 2 z= 3 �i, z+ 2 z= 3� i, z+ 2 z= 3 + i:Ta ÷ñc ¨z+ 2 z= 3 �i4 z + 2 z= 6 + 2 i)z= 1 + i:Chån ¡p ¡n A dC¥u 2. Trong khæng gian Oxyz, cho m°t c¦u (S )câ ph÷ìng tr¼nh (x � 1)2+ y2+ ( z+ 2) 2= 16 .Tåa ë t¥m Iv  b¡n k½nh rcõa m°t c¦u (S ) l  A I(� 1; 0; 2) ,r = 4 . B I(1; 0; �2) ,r = 16 . C I(1; 0; �2) ,r = 4 . D I(� 1; 0; 2) ,r = 16 . ÊLíi gi£i.Tåa ë t¥m I(1; 0; �2) v  b¡n k½nh r= 4 .Chån ¡p ¡n C dC¥u 3. iºm n o sau ¥y thuëc ç thà (C )cõa h m sè y= x2+ 3 x+ 3 x+ 1 ? A (3; 0). B (2; 1). C (0; 3). D (� 2; 1) . ÊLíi gi£i.Ta câ y(0) = 3 n¶n iºm câ to¤ ë (0; 3)thuëc ç thà (C )cõa h m sè â cho.Chån ¡p ¡n C dC¥u 4. Câ bao nhi¶u m°t c¦u i qua ba ¿nh cõa mët tam gi¡c? A Khæng câ. B Câ væ sè. C Câ hai. D Câ mët.ÊLíi gi£i.Gåi Il  t¥m ÷íng trán ngo¤i ti¸p tam gi¡c ABC.Gåi l  ÷íng th¯ng qua Iv  vuæng gâc vîi m°t ph¯ng (ABC )v  l  tröc võa ÷íng trán ngo¤iti¸p tam gi¡c ABC. Méi iºm J2 , ta ·u câ J A=J B =J C )Jl  t¥m m°t c¦u i qua ba ¿nhcõa tam gi¡c ABC.Vªy câ væ sè m°t c¦u i qua ba ¿nh cõa mët tam gi¡c.Chån ¡p ¡n B 162/305 162/305 pGV: L¶ Quang Xe  Ô0967.003.131PHT TRIšN — THAM KHƒO 2022 NÌI N€O CÂ Þ CH, NÌI  C CON ×ÍNG163dC¥u 5. Cho hai h m sè f(x ) v  g(x ) li¶n töc tr¶n Kv a; b 2K . Kh¯ng ành n o sau ¥yl  kh¯ng ành sai? A bZa [f (x ) + g(x )] d x= bZa f(x ) d x+ bZa g(x ) d x. B bZa kf(x ) d x= k bZa f(x ) d x. C bZa [f (x )g (x )] d x= bZa f(x ) d x bZa g(x ) d x. D bZa [f (x ) � g(x )] d x= bZa f(x ) d x� bZa g(x ) d x. ÊLíi gi£i.Düa v o t½nh ch§t cõa t½ch ph¥n.Chån ¡p ¡n C dC¥u 6. Cho h m sè y= f(x ) câ b£ng x²t d§u cõa ¤o h m nh÷ sau. xf0( x ) 0 �2 0 1 3 +1 +0 �0 +0 �0 �H m sè ¢ cho câ bao nhi¶u iºm cüc trà?A 3. B 1. C 2. D 4. ÊLíi gi£i.Ta câ f0( x ) êi d§u qua ba iºm x= �2, x = 0 v x= 1 . N¶n y= f(x ) câ 3iºm cüc trà.Chån ¡p ¡n A dC¥u 7. Nghi»m cõa b§t ph÷ìng tr¼nh log1 2(x � 3) >2l  A 36 x6 13 4. B 3< x 613 4. C x6 13 4. D x> 13 4. ÊLíi gi£i.Ta câ log1 2(x � 3) >2, 0< x �3 1 4,3< x 13 4:Chån ¡p ¡n B dC¥u 8. T½nh thº t½ch khèi châp câ di»n t½ch ¡y b¬ng 4v  chi·u cao b¬ng 3. A 12. B 36. C 4. D 16. ÊLíi gi£i.Ta câ V= 1 3h S = 1 33 4 = 4 .Chån ¡p ¡n C 163/305 163/305 pGV: L¶ Quang Xe  Ô0967.003.131— SÈ 11 NÌI N€O CÂ Þ CH, NÌI  C CON ×ÍNG164dC¥u 9. H m sèy= (2 x� 1)�4câ tªp x¡c ành l  A (0; +1). B Rn§1 2ª. C R. D [0; +1). ÊLíi gi£i.H m sè x¡c ành khi 2x � 16= 0 ,x6= 1 2.Chån ¡p ¡n B dC¥u 10. T½ch t§t c£ c¡c nghi»m cõa ph÷ìng tr¼nh 2x2+ x= 4 b¬ng A 2. B 3. C �2. D �1. ÊLíi gi£i.Ta câ 2x2+ x= 4 ,x2+ x� 2 = 0 ,x= �2_ x= 1 .Chån ¡p ¡n C dC¥u 11. T½ch ph¥n I=  3Z0 sinxdx b¬ng A p32. B �p 32. C 12. D �1 2. ÊLíi gi£i.Ta câ I=  3Z0 sinxdx = �cos x  30 =1 2.Chån ¡p ¡n C dC¥u 12. Thu gån sè phùc z= i+ (2 �4i) � (3 �2i) , ta ÷ñc: A z= �1� i. B z= 1 �i. C z= �1� 2i. D z= 1 + i. ÊLíi gi£i.Ta câ z= i+ (2 �4i) � (3 �2i) = i+ 2 �4i � 3 + 2 i= �1� i.Chån ¡p ¡n A dC¥u 13. T¼m mët v²c-tì ph¡p tuy¸n cõa m°t ph¯ng 2x � y+ 3 z� 2 = 0 . A #n = (1; 2; 3) . B #n = (2; �1; 3) . C #n = ( �2; �1; 3) . D #n = (2; �1; �3) . ÊLíi gi£i.Mët v²c-tì ph¡p tuy¸n cõa m°t ph¯ng 2x � y+ 3 z� 2 = 0 l #n = (2; �1; 3) .Chån ¡p ¡n B dC¥u 14. Trong khæng gian vîi h» tåa ë OxyzchoA(1; 2; 3) ,B (� 4; 4; 6) . Tåa ë trång t¥mG cõa tam gi¡c OABl  A G(1; �2; �3) . B G(� 1; 2; 3) . C G(� 3; 6; 9) . D G�� 3 2; 3;9 2‹. ÊLíi gi£i.164/305 164/305 pGV: L¶ Quang Xe  Ô0967.003.131PHT TRIšN — THAM KHƒO 2022 NÌI N€O CÂ Þ CH, NÌI  C CON ×ÍNG165Gi£ sûG(xG ;yG ;zG )) 8>>>>><>>>>>: xG = xO +xA +xB 3=0 + 1 + (�4) 3=�1y G = yO +yA +yB 3=0 + 2 + 4 3= 2z G = zO +zA +zB 3=0 + 3 + 6 3= 3 .Vªy G(� 1; 2; 3) .Chån ¡p ¡n B dC¥u 15. Trong m°t ph¯ng tåa ë Oxy, tåa ë iºm Mbiºu di¹n sè phùc z= 4 �il  A M(4; 1) . B M(� 4; 1) . C M(4; �1) . D M(� 4; �1) . ÊLíi gi£i.iºm biºu di¹n sè phùc z= 4 �il  M(4; �1) .Chån ¡p ¡n C dC¥u 16. ç thà h m sè y= 3 + 2x 2x � 2câ ÷íng ti»m cªn ùng l  A y= �1. B y= 1 . C x= �1. D x= 1 . ÊLíi gi£i.Ta câ limx ! 1� y= �1 v limx ! 1+ y= + 1 ) n¶n ti»m cªn ùng l  x= 1 .Chån ¡p ¡n D dC¥u 17. Cho c¡c sè thüc a, b, c thäa m¢n a >0, a 6= 1 ,> 0, c > 0. Kh¯ng ành n o sau ¥ysai ? A logab c= logab� logac. B loga b= logab. C logabc= logab+ logac. D logab = logab. ÊLíi gi£i.Kh¯ng ành sai l  loga b= logab.Chån ¡p ¡n B dC¥u 18.H m sè y= bx�c x� a (a 6= 0; a; b; c 2R)câ ç thà nh÷ h¼nh v³ b¶n. M»nh· n o sau ¥y óng? A a >0, b < 0, c � ab < 0. B a >0, b > 0, c � ab > 0. C a >0, b > 0, c � ab = 0 . D a >0, b > 0, c � ab < 0. xyOÊLíi gi£i.Tªp x¡c ành D=Rn f ag.y 0= �ab +c (x � a)2 .Düa v o ç thà suy ra165/305 165/305 pGV: L¶ Quang Xe  Ô0967.003.131— SÈ 11 NÌI N€O CÂ Þ CH, NÌI  C CON ×ÍNG166Ì÷íng ti»m cªn ùng x= a > 0;Ì ÷íng ti»m cªn ngang y= b > 0;Ì y0< 0) c� ab < 0.Chån ¡p ¡n D dC¥u 19. Trong khæng gian vîi h» tåa ë Oxyz, cho ÷íngth¯ng d: x� 1 1=y 2=z+ 2 1. iºmn o thuëc ÷íng th¯ng d? A P(2; 2; �1) . B Q(0; �2; �1) . C N(1; 0; 2) . D M(� 1; 0; 2) . ÊLíi gi£i.Thay tåa ë iºm Pv o ph÷ìng tr¼nh ÷íng th¯ng dta ÷ñc 2� 1 1=2 2=�1 + 2 1(thäa m¢n).Vªy iºm Pthuëc ÷íng th¯ng d.Chån ¡p ¡n A dC¥u 20. Câ bao nhi¶u c¡ch x¸p 5håc sinh theo mët h ng ngang ?A 10. B 24. C 5. D 120. ÊLíi gi£i.Méi c¡ch x¸p 5håc sinh theo mët h ng ngang l  mët ho¡n và cõa 5ph¦n tû n¶n sè c¡ch x¸p l P 5 = 120.Chån ¡p ¡n D dC¥u 21. Thº t½ch khèi lªp ph÷ìng câ c¤nh ab¬ng A 3a 2. B a2. C 3a . D a3. ÊLíi gi£i.Thº t½ch cõa khèi lªp ph÷ìng l  V=a3. A0 D0 AB CB0 C0 DaChån ¡p ¡n D dC¥u 22. ¤o h m cõa h m sè y= log3(4x+ 1) l  A y0= ln 3 4x + 1 . B y0= 4 (4x+ 1) ln 3 . C y0= 4 ln 3 4x + 1 . D y0= 1 (4x+ 1) ln 3 . ÊLíi gi£i.Ta câ y0= (4x+ 1) 0 (4x+ 1) ln 3 =4 (4x+ 1) ln 3 .Chån ¡p ¡n B 166/305 166/305 pGV: L¶ Quang Xe  Ô0967.003.131PHT TRIšN — THAM KHƒO 2022 NÌI N€O CÂ Þ CH, NÌI  C CON ×ÍNG167dC¥u 23.÷íng cong ð h¼nh b¶n l  ç thà cõa h m sè y= ax+b cx+dvîia; b; c; dl  c¡c sè thüc. M»nh · n o d÷îi ¥y óng? A y0< 0;8 x 6= 2 . B y0< 0;8 x 6= 1 . C y0> 0;8 x 6= 2 . D y0> 0;8 x 6= 1 . xyO 21ÊLíi gi£i.Theo h¼nh v³ ta câ h m sè nghàch bi¸n tr¶n c¡c kho£ng x¡c ành v  câ ti»m cªn ùng l  x= 2 )y 0< 0;8 x 6= 2 .Chån ¡p ¡n A dC¥u 24. T½ch ph¥n  4Z0 cosxdx b¬ng A p22�1. B p22. C �p 22. D 1� p 22. ÊLíi gi£i.Ta câ  4Z0 cosxdx = sin x  40 = p 22.Chån ¡p ¡n B dC¥u 25. Cho c§p sè cëng (un)câ sè h¤ng thù hai u2 = 2v  cæng sai d= 3 . Gi¡ trà cõa u4b¬ng A 8. B 11. C 14. D 5. ÊLíi gi£i.Ta câ u4 =u1 + 3d= u2 + 2d= 8 .Chån ¡p ¡n A dC¥u 26. T¼m nguy¶n h m cõa h m sè f(x ) = xp 5. A Zf(x )d x= 1 p5� 1xp 5� 1+ C. B Zf(x )d x= xp 5+1+C. C Zf(x )d x= 1 p5 + 1xp 5+1+C. D Zf(x )d x= p 5x p 5� 1+ C. ÊLíi gi£i.�p döng cæng thùc nguy¶n h m cì b£n suy ra Zf(x )d x= 1 p5 + 1xp 5+1+C.Chån ¡p ¡n C 167/305 167/305 pGV: L¶ Quang Xe  Ô0967.003.131— SÈ 11 NÌI N€O CÂ Þ CH, NÌI  C CON ×ÍNG168dC¥u 27. ç thà h m sè y= x+ 1 x� 2câ t¥m èi xùngIl  A I(� 2; 1) . B I(2; 1) . C I(2; �1) . D I(� 2; �1) . ÊLíi gi£i.ç thà h m sè y= x+ 1 x� 2câ t¥m èi xùng l I(2; 1) .Chån ¡p ¡n B dC¥u 28. Gi¡ trà nhä nh§t cõa h m sè y= x3+ xtr¶n [0; 1] l  A 4. B 1. C 2. D 0. ÊLíi gi£i.y 0= 3 x2+ 1 >0;8 x 2 [0; 1] )minx 2 [0;1] y= y(0) = 0 :Chån ¡p ¡n D dC¥u 29. Cho h m sè y= x+ 1 x� 1. Kh¯ng ành n o sau ¥y l  óng? A H m sè nghàch bi¸n tr¶n c¡c kho£ng(�1 ; 1)v (1; + 1). B H m sè nghàch bi¸n tr¶nRnf 1g . C H m sè çng bi¸n tr¶nRnf 1g . D H m sè nghàch bi¸n tr¶n kho£ng(�1 ; 1)v  çng bi¸n tr¶n (1; +1). ÊLíi gi£i.V¼ y0= � 2 (x � 1)2;8 x 6= 1 n¶n h m sè nghàch bi¸n tr¶n c¡c kho£ng (�1 ; 1)v (1; + 1).Chån ¡p ¡n A dC¥u 30. Cholog35 =a;log57 =b, khi â log45175b¬ng A a+ b 2 +a. B a(2 + b) 2 +a. C a(a + b) 2 +a. D 2a + b 2 +a. ÊLíi gi£i.Ta câ log45175 = log5175 log545 =log5�7 52 log5�5 32= log57 + 2 2 log53 + 1 =2 +b 21 a+ 1 =a(2 + b) 2 +a.Chån ¡p ¡n B dC¥u 31. Cho l«ng trö ·u ABC:A0B 0C 0. Bi¸t r¬ng gâc giúa (A 0BC )v  (ABC )l  30, tamgi¡c A0BC câ di»n t½ch b¬ng 2. T½nh thº t½ch khèi l«ng trö ABC:A0B 0C 0. A 2p 6. B p62. C 2. D p3. ÊLíi gi£i.168/305 168/305 pGV: L¶ Quang Xe  Ô0967.003.131PHT TRIšN — THAM KHƒO 2022 NÌI N€O CÂ Þ CH, NÌI  C CON ×ÍNG169GåiMl  trung iºm BC.Ta câ ¨BC ?AMBC ?AA 0)BC ?(A 0AM )) BC ?A0M .Do â ¨BC ?A0MBC ?AM )((A0BC ); (ABC )) = ( A0M ;AM ))ÖA 0M A = 30.°t BC=al  c¤nh cõa tam gi¡c ·u 4ABC . Ta suy ra ë d i÷íng trung tuy¸n AM=ap 32. A0 C0 A CMB0 BX²t tam gi¡c4A0M A vuæng t¤i Ata câ:Ì tan ÖA 0M A =AA0 AM)AA 0= AM tan 30 = a 2.Ì cos ÖA 0M A =AM A0M )A0M = AM cos 30=a.Khi â S4 A0BC =1 2A 0M BC )1 2a2= 2 )a= 2 .Vªy VABC:A 0B 0C 0= AA 0 S4 ABC =p 38a3= p 3.Chån ¡p ¡n D dC¥u 32. Cho h m sè f(x ) li¶n töc tr¶n Rv  2Z0 (3f(x ) + 2 x) d x= 7 . T½nh 2Z0 f(x ) d x. A 3. B 4. C 2. D 1. ÊLíi gi£i.Ta câ 2Z0 (3f(x ) + 2 x) d x= 3 2Z0 f(x ) d x+ 2Z0 2x dx = 3 2Z0 f(x ) d x+ 4 .Theo gi£ thi¸t 2Z0 (3f(x ) + 2 x) d x= 7 suy ra 32Z0 f(x ) d x+ 4 = 7 .Do â 2Z0 f(x ) d x= 1 .Chån ¡p ¡n D dC¥u 33. Trong khæng gian Oxyz, cho m°t c¦u (S ) : x2+ y2+ z2� 2x � 4y � 6z = 0 c­t c¡ctia Ox,Oy ,Oz l¦n l÷ñt t¤i c¡c iºm A,B ,C (kh¡c O). Ph÷ìng tr¼nh m°t ph¯ng (ABC )l  A x2�y 4�z 6= 1. B x2+y 4+z 6= 1. C x2+y 4+z 6= 0. D x2+y 4�z 6= 1. ÊLíi gi£i.Do cho m°t c¦u (S ) : x2+ y2+ z2� 2x � 4y � 6z = 0 c­t c¡c tia Ox,Oy ,Oz l¦n l÷ñt t¤i c¡c iºm A,B ,C (kh¡c O) n¶n A(2; 0; 0) ,B (0; 4; 0) ,C (0; 0; 6) .Ph÷ìng tr¼nh m°t ph¯ng (ABC )l  x 2+y 4+z 6.Chån ¡p ¡n B 169/305 169/305 pGV: L¶ Quang Xe  Ô0967.003.131— SÈ 11 NÌI N€O CÂ Þ CH, NÌI  C CON ×ÍNG170dC¥u 34. T¼mcbi¸t a, b, c l  c¡c sè nguy¶n d÷ìng thäa m¢n c= ( a+ bi)3� 107 i A 198. B 6. C 1. D 7. ÊLíi gi£i.Ta câ c= ( a+ bi)3� 107 i= ( a3� 3ab 2) + (3 a2b � b3� 107) i.Do c2 Z+n¶n ta câ,3a 2b � b3� 107 = 0 ,b(3 a2� b2) = 107 = 1 107 = 107 1, 26664 ¨b= 13 a 2� b2= 107¨b= 1073 a 2� b2= 1 (do 107 l  sè nguy¶n tè ), 2666664 ¨a = 6b = 18<: a= 11450 3b = 107 :V¼ a, b, c l  c¡c sè nguy¶n d÷ìng n¶n a= 6 ,b = 1 .Khi â c= a3� 3ab 2= 198 .Chån ¡p ¡n A dC¥u 35. Cho l«ng trö tam gi¡c ·u ABC:A1B1C1 câAB =a, AA1=h. T½nh kho£ng c¡ch tøA ¸n m°t ph¯ng (BC A1) A p3a 2+ 4 h2 4. B 2p 3a 2+ 4 h2 4. C ahp 32p 3a 2+ 4 h2. D ahp 3p3a 2+ 4 h2. ÊLíi gi£i.L§y Mtrung iºm BC, k´ AH ?A1M:(1)Ta câ tam gi¡c ABC·u n¶nAM ?BC v A1A? (ABC )) A1A? BC .Suy ra BC?(A1AM)) AH ?BC: (2)Tø (1) v  (2): d(A; ( BC A1)) =AH.Ta câ: AM=ap 32.1 AH2= 1 AA21 +1 AM2= 1 h2 + 4 3a 2 )AH = ahp 3p3a 2+ 4 h2.Vªy d(A; (BC A1)) = ahp 3p3a 2+ 4 h2. B C C1 AA1 B1 HMChån ¡p ¡n D dC¥u 36. Tø c¡c chú sè 0, 1, 2, 3, 5 câ thº lªp th nh bao nhi¶u sè tü nhi¶n khæng chia h¸t cho5 gçm bèn chú sè æi mët kh¡c nhau? A 120. B 54. C 72. D 69. ÊLíi gi£i.170/305 170/305 pGV: L¶ Quang Xe  Ô0967.003.131PHT TRIšN — THAM KHƒO 2022 NÌI N€O CÂ Þ CH, NÌI  C CON ×ÍNG171C¡ch 1:Ta x²t c¡c tr÷íng hñp sau:Ì Sè c¦n lªp câ d¤ng abc1vîi a6= 0 . Tr÷íng hñp n y câ 3c¡ch chån chú sè a, 3 c¡ch chån chú sèb v  2c¡ch chån chú sè c. Vªy câ 33 2 = 18 sè.Ì T÷ìng tü sè c¦n lªp câ d¤ng abc2ho°c abc3vîi a6= 0 th¼ méi tr÷íng hñp lªp luªn nh÷ tr¶n câ18 sè.Vªy câ t§t c£ 318 = 54 sè thäa m¢n y¶u c¦u.C¡ch 2:Ta i t¼m sè c¡c sè chia h¸t cho 5câ bèn chú sè kh¡c nhau. X²t c¡c tr÷íng hñp:Ì Sè c¦n lªp câ d¤ng abc0. Tr÷íng hñp n y câ 4c¡ch chån chú sè a, 3 c¡ch chån chú sè bv  2c¡ch chån chú sè c. Vªy câ 43 2 = 24 sè.Ì Sè c¦n lªp câ d¤ng abc5v  a6= 0 . Tr÷íng hñp n y câ 3c¡ch chån chú sè a, 3 c¡ch chån chú sèb v  2c¡ch chån chú sè c. Vªy câ 33 2 = 18 sè.Vªy câ 24 + 18 = 42 sè chia h¸t cho 5.M°t kh¡c sè c¡c sè câ bèn chú sè kh¡c nhau ÷ñc lªp tø 0, 1, 2, 3, 5 l  A45 �A34 = 96. Vªy sè c¡c sèthäa m¢n y¶u c¦u ban ¦u cõa · b i l  96�42 = 54 sè.Chån ¡p ¡n B dC¥u 37. Trong khæng gian vîi h» tåa ë Oxyz, chol  ÷íng th¯ng i qua gèc tåa ë, vuænggâc vîi tröc ho nh v  c­t ÷íng th¯ng d: x� 1 1=y� 2 �1 =z� 1 �3 . Ph÷ìng tr¼nhl  A x2=y �1 =z 1. B x1=y �2 =z 1. C 8><>: x= 0y = 3 + tz = 4 + t. D 8><>: x= 0y = 3 tz = 4 t. ÊLíi gi£i.Gåi A=  \d) A(1 + t; 2 �t; 1 �3t) ) # OA = (1 + t; 2 �t; 1 �3t) . vuæng gâc vîi tröc ho nh # OA: #i = 0 ,1:(1 + t) + 0(2 �t) + 0 (1 �3t) = 0 ,t= �1. i qua iºm O(0; 0; 0) v  câ VTCP # OA = (0; 3; 4) ) : 8><>: x= 0y = 3 tz = 4 t.Chån ¡p ¡n D dC¥u 38. Chologb(a + 1) >0. Kh¯ng ành n o sau ¥y l  óng? A (b � 1)a > 0. B a+ b < 1. C a+ b > 1. D a(b + 1) >0. ÊLíi gi£i.Ta câ logb(a + 1) >0, 8><>: 0< b 6= 1a + 1 >0( a + 1 �1)( b� 1) >0, 8><>: b >0a > �1a (b � 1) >0.Chån ¡p ¡n A dC¥u 39. Cho h m sè y= 2x � 1 x� 1 câ ç thà(C ). Sè c¡c gi¡ trà nguy¶n cõa tham sè m2[ � 2020; 2020] º ÷íng th¯ng d:y = �x + m c­t (C )t¤i hai iºm ph¥n bi»t l  A 4035. B 4036. C 4037. D 2020.171/305 171/305 pGV: L¶ Quang Xe  Ô0967.003.131— SÈ 11 NÌI N€O CÂ Þ CH, NÌI  C CON ×ÍNG172ÊLíi gi£i.X²t ph÷ìng tr¼nh ho nh ë giao iºm2x � 1 x� 1 =�x + m , ¨x 6= 1x 2� (m �1)x+ m �1 = 0 :(1)Do x= 1 khæng thäa m¢n ph÷ìng tr¼nh (1) n¶n dc­t (C )t¤i 2iºm ph¥n bi»t khi v  ch¿ khi ph÷ìngtr¼nh (1) câ 2nghi»m ph¥n bi»t,(m �1)2� 4(m �1) >0, –m �1< 0m �1> 4, –m < 1m > 5:K¸t hñp vîi i·u ki»n m2Z v  m2[� 2020; 2020] suy ra câ4041�5 = 4036 gi¡ trà nguy¶n mthäa· b i.Chån ¡p ¡n B dC¥u 40. Cho h m sè f(x ) thäa m¢n f(1) = 4 v f(x ) = xf0( x ) � 2x 3� 3x 2vîi måi x >0.Gi¡ trà cõa f(2) b¬ng A 5. B 10. C 20. D 15. ÊLíi gi£i.V¼ x > 0n¶nf(x ) = xf0( x ) � 2x 3� 3x 2, xf0( x ) � f(x ) x2 = 2x+ 3) Zxf0( x ) � f(x ) x2 dx = Z(2x+ 3) d x) f(x ) x=x2+ 3 x+ C) f(x ) = x3+ 3 x2+ C x:Tø f(1) = 4 suy ra4 = 4 + C, C= 0 .Vªy f(2) = 20 .Chån ¡p ¡n C dC¥u 41. Cho khèi châp S:ABC Dcâ ¡yABC D l  h¼nh chú nhªt v  BC= 2AB = 2S B = 2a,gâc giúa S Bv  m°t ph¯ng (ABC D )b¬ng 45. Thº t½ch Vkhèi châp S:ABC Dl  A V= p 2a 3 3. B V= p 2a 3 2. C V=p 2a 3. D V= p 2a 3 6. ÊLíi gi£i.Gåi Hl  h¼nh chi¸u cõa Str¶n (ABC D ). Khi â gâc giúaS B v  m°t ph¯ng (ABC D )b¬ng gâc ÕS B H = 45. Suy ra vuæng c¥n t¤i H. L¤i câ S B=an¶n S H=ap 22.Do â thº t½ch khèi châp S:ABC Dl V = 1 3S HSABC D =1 3ap 22a2a = p 2a 3 3: SABCH DChån ¡p ¡n A 172/305 172/305 pGV: L¶ Quang Xe  Ô0967.003.131PHT TRIšN — THAM KHƒO 2022 NÌI N€O CÂ Þ CH, NÌI  C CON ×ÍNG173dC¥u 42. T¼m mæun cõa sè phùc z, bi¸t z+ 2 z= 3 �2i A p10. B p2. C p5. D p13. ÊLíi gi£i.Ta câ z+ 2 z= 3 �2i ) z+ 2 z= 3 + 2 i) 4z + 2 z= 6 + 4 i) z+ 3 z= 3 + 6 i) z= 1 + 2 i) j zj = p 5.Chån ¡p ¡n C dC¥u 43. GåiA,B ,C l¦n l÷ñt l  iºm biºu di¹n cõa c¡c sè phùc z, iz ,2z . Bi¸t di»n t½ch tamgi¡c ABC b¬ng4. Mæ-un cõa sè phùc zb¬ng A p2. B 8. C 2. D 2p 2. ÊLíi gi£i.Gåi z= a+ bi vîi a; b 2R ) A(a ;b), iz =�b+ ai ) B(� b; a ), C (2 a; 2 b).Ta th§y OB?AC n¶n SABC =1 2d(B; AC )AC .Ta câ ph÷ìng tr¼nh ACl �bx +ay = 0 )d(B; AC ) =p a2+ b2= jz j = AC) 1 2jz j  j zj = 4 ) jzj = 2 p 2.Chån ¡p ¡n D dC¥u 44.Cho h m sè y= f(x ). ç thà h m sè y= f0( x ) nh÷ h¼nh b¶n. T¼m sèiºm cüc trà cõa h m sè g(x ) = f(x 2� 3). A 2. B 3. C 4. D 5. O xy�1 4�2 1ÊLíi gi£i.Ta câ g0( x ) = 2 xf0( x 2� 3)g 0( x ) = 0 ,–x = 0f 0( x 2� 3) = 0 ,264 x= 0x 2� 3 = �2x 2� 3 = 1 (nghi»m k²p) ,264 x= 0x = 1x = 2 (nghi»m k²p) :B£ng bi¸n thi¶n xg0( x ) g(x ) �1�2 �1 0 1 2 +1 �0 �0 +0 �0 +0 +Düa v o b£ng bi¸n thi¶n ta th§y h m sè câ 3 iºm cüc trà.173/305 173/305 pGV: L¶ Quang Xe  Ô0967.003.131— SÈ 11 NÌI N€O CÂ Þ CH, NÌI  C CON ×ÍNG174Chån ¡p ¡n B dC¥u 45. Trong khæng gian vîi h» tåa ë Oxyz, cho hai m°t ph¯ng ( ) : x� 2y + z� 4 = 0 ,( ) : x+ 2 y� 2z + 4 = 0 v  hai iºm M(� 2; 5; �1) ; N (6; 1; 7) . iºm Itr¶n giao tuy¸n cõa ( ) v ( ) sao cho # I M +# I N nhä nh§t câ tåa ë l  A �62 29;35 29;124 29‹. B (2; 3; 3). C (0;�2; 0) . D �12 29;35 29;24 29‹. ÊLíi gi£i.Ta câ VTPT cõa ( ), ( ) l¦n l÷ñt l  #n = (1;�2; 1) v #n = (1; 2;�2) .Khi â v²c-tì ch¿ ph÷ìng cõa giao tuy¸n ( ) vîi ( ) l  #u = [ #n ; #n ] = (2; 3; 4)v K(0; �2; 0) n¬mtr¶n giao tuy¸n ( ) vîi ( ) .ph÷ìng tr¼nh tham sè cõa giao tuy¸n ( ) vîi ( ) l 8><>: x= 2 ty = �2 + 3 tz = 4 t:Gåi J(2; 3; 3) l  trung iºm cõa M N # I M +# I N = 2 # I J = 2I J.Vªy # I M +# I N nhä nh§t khiI Jnhä nh§t. M°t kh¡c,I 2 ( )\ ( ) n¶n I Jnhä nh§t khi I J?( )\ ( ): Gåi I(2 t; � 2 + 3 t; 4 t) ) # I J = (2 �2t; 5 �3t; 3 �4t) .Do # I J #u = 0 )t= 31 29:Vªy I�62 29;35 29;124 29‹.Chån ¡p ¡n A dC¥u 46. Mët khèi nân câ thi¸t di»n qua tröc l  mët tam gi¡c vuæng c¥n v  ÷íng sinh câ ëd i b¬ng 3p 2cm. Mët m°t ph¯ng i qua ¿nh v  t¤o vîi ¡y mët gâc 60chia khèi nân th nh 2ph¦n. T½nh thº t½ch ph¦n nhä (T½nh g¦n óng ¸n h ng ph¦n tr«m). A 4;36 cm3. B 4;53 cm3. C 5;37 cm3. D 5;61 cm3. ÊLíi gi£i.X²t khèi nân nh÷ h¼nh v³.Ta câ 4S B O l  nûa tam gi¡c vuæng c¥n ¿nh Sn¶nBO =S Bp 22= 3.M°t ph¯ng qua ¿nh Sc­t ¡y theo d¥y cung AB, t¥m ÷íngtrán ¡y l  O.Gåi Il  h¼nh chi¸u vuæng gâc cõa Ol¶n AB )ÔS I O = 60.X²t 4S OB ; S O =p S B2� OB 2= 3 .X²t 4S I O; OI =S O tan 60= p 3.X²t 4I OA; cosÔI OA =I O OA=1 p3.Suy ra ÕAOB = 2ÔI OA 109 ;47 . SOBAIGåiSl  di»n t½ch h¼nh ph¯ng t¤o bði d¥y cung ABv  ÷íng trán (O )(ph¦n nhä).S = 1 2109 ;47   180OB 2� 1 2OAOB sin 109 ;47  4;36 cm2:Gåi Vn l  thº t½ch ph¦n nhä,Vn = 1 3S S O 4;36 cm3.174/305 174/305 pGV: L¶ Quang Xe  Ô0967.003.131PHT TRIšN — THAM KHƒO 2022 NÌI N€O CÂ Þ CH, NÌI  C CON ×ÍNG175Chån ¡p ¡n A dC¥u 47. Trong t§t c£ c¡c c°p sè thüc (x ;y ) thäa m¢n logx2+ y2+3 (2x+ 2 y+ 5) 1, câ baonhi¶u gi¡ trà thüc cõa mº tçn t¤i duy nh§t c°p (x ;y ) sao cho x2+ y2+ 4 x+ 6 y+ 13 �m = 0 ? A 1. B 2. C 3. D 0. ÊLíi gi£i.Bi¸n êi gi£ thi¸t b i to¡n ta ÷ñclogx2+ y2+3 (2x+ 2 y+ 5) 1, 2x + 2 y+ 5 x2+ y2+ 3 ,(x � 1)2+ ( y� 1)2 4:(1)X²t iºm M(x ;y ) trong m°t ph¯ng tåa ë Oxy, ta suy ra Mthuëc h¼nh trán (H )t¥m I(1; 1) , b¡nk½nh R1= 2.M°t kh¡c ta câx2+ y2+ 4 x+ 6 y+ 13 �m = 0 ,(x + 2) 2+ ( y+ 3) 2= m: (2)Ì Vîi m= 0 , ta câ c°p (x ;y ) duy nh§t thäa m¢n ( ) l  (� 2; �3) , nh÷ng c°p sè n y khæng thäam¢n (1), n¶n lo¤i m= 0 .Ì Vîi m > 0, M n¬m thuëc ÷íng trán (C )t¥m J(� 2; �3) , b¡n k½nh R2= p m.Tø â suy ra º câ duy nh§t c°p sè (x ;y ) thäa b i to¡n th¼ (C )xóc ngo i vîi (H )ho°c (C )ti¸pxóc trong vîi (H )v  (H )n¬m trong (C )., –I J =R1+R2R 2�R1=I J ,–p m+ 2 = 5p m�2 = 5 ,–m = 9m = 49 :Vªy câ hai gi¡ trà thüc cõa mthäa b i to¡n.Chån ¡p ¡n B dC¥u 48. Trong khæng gian Oxyzcho ÷íng th¯ng d:x� 3 2=y� 2 3=z 6v  m°t c¦u(S ) :( x � 1)2+ ( y� 1)2+ z2= 9 . Bi¸t ÷íng th¯ng dc­t m°t c¦u (S ) theo d¥y cung AB. ë d i ABl  A 4. B 2p 5. C 2p 3. D 4p 2. ÊLíi gi£i.Gåi Hl  trung iºm cõa AB.Khi â AB= 2p I B2� I H 2= 2 p R2� d ( I ; d ).d i qua iºm M(2; 3; 0) v #u d = (2; 3; 6).Vªy d (I ; d ) = ”# I M ;#u d— j#u dj .Ta câ # I M = (2; 1; 0) )”# I M ;#u d—= (6; �12; 4) .Vªy ”# I M ;#u d— = 14.m  j#u dj= p 22+ 3 2+ 6 2= 7 )d (I ; d ) = 2 .Vªy AB= 2p 32� 22= 2 p 5. IHABChån ¡p ¡n B dC¥u 49. Câ bao nhi¶u gi¡ trà nguy¶n cõa tham sè mº h m sè y= jx 4� 8x 3+ 18 x2+ mjcâ3 iºm cüc trà?175/305 175/305 pGV: L¶ Quang Xe  Ô0967.003.131MÖC LÖCNÌI N€O CÂ Þ CH, NÌI  C CON ×ÍNG176A 1. B væ sè. C 2. D khæng câ.ÊLíi gi£i.X²t f(x ) = x4� 8x 3+ 18 x2+ m ) f0( x ) = 4 x3� 24x2+ 36 x= 4 x(x 2� 6x + 9) = 4 x(x � 3)2.Gi£i f0( x ) = 0 ,–x = 0x = 3 . B£ng bi¸n thi¶n xf0( x ) f(x ) �10 3 +1 �0 +0 ++1 +1 mm +1 +1 ÌN¸u m0th¼ h m sè y= jf (x )j = f(x ) câ 1iºm cüc trà.Ì N¸u m < 0th¼ h m sè ç thà h m sè y= f(x ) c­t tröc ho nh t¤i 2iºm ph¥n bi»t.Tø ç thà h m sè y= f(x ) suy ra ç thà h m sè y= jf (x )j, ta th§y h m sè y= jf (x )j câ 3iºmcüc trà.Vªy câ væ sè gi¡ trà nguy¶n cõa mº h m sè ¢ cho câ 3iºm cüc tri.Chån ¡p ¡n B H˜T176/305 176/305 pGV: L¶ Quang Xe  Ô0967.003.131PHT TRIšN — THAM KHƒO 2022 NÌI N€O CÂ Þ CH, NÌI  C CON ×ÍNG177BË GIO DÖC & €O T„OTR×ÍNG THPT NGUY™N T‡T TH€NH GV: L– QUANG XE - 0967.003.131 — SÈ 12 PHT TRIšN — THAM KHƒO 2022N‹M HÅC 2021 - 2022Mæn:To¡nThíi gian l m b i: 90 phót — THAM KHƒO HKIIdC¥u 1. Cho sè phùc zcâ iºm biºu di¹n trong m°t ph¯ng tåa ë Oxyl  iºm M(3; �5) . X¡cành sè phùc li¶n hñp zcõa z. A z= �5 + 3 i. B z= 5 + 3 i. C z= 3 + 5 i. D z= 3 �5i. ÊLíi gi£i.V¼ sè phùc zcâ iºm biºu di¹n trong m°t ph¯ng tåa ë Oxyl  iºm M(3; �5) n¶n z= 3 �5i.Do â sè phùc li¶n hñp cõa sè phùc zl  z= 3 + 5 i.Chån ¡p ¡n C dC¥u 2. Trong khæng gian Oxyz, cho m°t c¦u (S ) : x2+ y2+ z2� 4x + 6 y+ 8 z+ 4 = 0 . X¡cành tåa ë t¥m Iv  b¡n k½nh Rcõa m°t c¦u (S ). A I(2; �3; �4) ,R = 25 . B I(� 2; 3; 4) ,R = 5 . C I(2; �3; �4) ,R = 5 . D I(2; �3; �4) ,R = p 5. ÊLíi gi£i.Ta câx2+ y2+ z2� 4x + 6 y+ 8 z+ 4 = 0 ,(x � 2)2+ ( y+ 3) 2+ ( z+ 4) 2= 25 :Vªy m°t c¦u (S ) câ t¥m I(2; �3; �4) v  b¡n k½nh R= 5 .Chån ¡p ¡n C dC¥u 3. H m sè n o sau ¥y câ tªp x¡c ành D=R? A y= p x� 1 x2+ 1 . B y= x� 1 2x � 1. C y= x3� 2x 2+ 1 . D y= p x3+ 1 . ÊLíi gi£i.H m sè y= x3� 2x 2+ 1 câ tªp x¡c ành D=R.Chån ¡p ¡n C dC¥u 4. Khèi c¦u b¡n k½nh R= 2 acâ thº t½ch l  A 32a 3 3. B 6a 3. C 16a 2. D 8a 3 3. ÊLíi gi£i.V = 4 3R3= 4 3(2 a)3= 32 3a3.Chån ¡p ¡n A dC¥u 5. T¼m hå nguy¶n h m cõa h m sè f(x ) = ( x� 1)3. A 3(x� 1) + C. B 14(x � 1)4+ C. C 4(x� 1)4+ C. D 14(x � 1)3+ C.177/305 177/305 pGV: L¶ Quang Xe  Ô0967.003.131— SÈ 12 NÌI N€O CÂ Þ CH, NÌI  C CON ×ÍNG178ÊLíi gi£i.Ta câ Zf(x ) d x= Z(x � 1)3dx = 1 4(x � 1)4+ C.Chån ¡p ¡n B dC¥u 6.Cho h m sè y= f(x ) câ b£ng bi¸nthi¶n nh÷ h¼nh v³ b¶n. M»nh · n od÷îi ¥y óng? xf0( x ) f(x ) �10 2 +1 +0 �0 +�1�1 5511 +1 +1 A H m sè khæng câ cüc trà. B H m sè ¤t cüc ¤i t¤ix= 0 . C H m sè ¤t cüc tiºu t¤ix= 1 . D H m sè ¤t cüc ¤i t¤ix= 5 . ÊLíi gi£i.Düa v o b£ng bi¸n thi¶n ta th§y h m sè ¤t cüc tiºu t¤i x= 2 v  ¤t cüc ¤i t¤i x= 0 .Chån ¡p ¡n B dC¥u 7. Tªp nghi»m cõa b§t ph÷ìng tr¼nh 32x � 1> 27 l  A �1 2; +1‹. B (2; +1). C (3; +1). D �1 3; +1‹. ÊLíi gi£i.Ta câ 32x�1> 27 , 2x � 1> 3, x > 2.Vªy tªp nghi»m cõa b§t ph÷ìng tr¼nh ¢ cho l  (2; +1).Chån ¡p ¡n B dC¥u 8. Cho h¼nh châp S:ABC Dcâ ¡yABC D l  h¼nh vuæng c¤nh a, c¤nh b¶n S Avuænggâc m°t ¡y, S A=ap 6. T½nh thº t½ch khèi châp S:ABC D. A a3p 64. B a3p 6. C a3p 63. D a3p 62. ÊLíi gi£i.Di»n t½ch h¼nh vuæng ABC Db¬nga2.Thº t½ch khèi châp S:ABC Dl V = 1 3a 2a p 6 =a3p 63: SB C DAChån ¡p ¡n C dC¥u 9. Tªp x¡c ành cõa h m sè y= ( x+ 1) 1 3l  A D= ( �1 ;� 1) . B D=Rn f� 1g . C D=R. D D= ( �1; + 1). ÊLíi gi£i.º h m sè x¡c ành i·u ki»n l  x+ 1 >0, x > �1. Vªy tªp x¡c ành h m sè D= ( �1; + 1).178/305 178/305 pGV: L¶ Quang Xe  Ô0967.003.131PHT TRIšN — THAM KHƒO 2022 NÌI N€O CÂ Þ CH, NÌI  C CON ×ÍNG179Chån ¡p ¡n D dC¥u 10. Ph÷ìng tr¼nh log2(x � 2) = 1 câ nghi»m l  A x= 1 . B x= 4 . C x= 3 . D x= 2 . ÊLíi gi£i.Ta câ log2(x � 2) = 1 ,x� 2 = 2 ,x= 4 :Chån ¡p ¡n B dC¥u 11. Cho h m sè y= f(x ) li¶n töc tr¶n o¤n [a ;b]. M»nh · n o d÷îi ¥y sai? A bZa f(x ) d x= bZa f(t) d t. B bZa f(x ) d x= � aZb f(x ) d x. C bZa kdx = k(a � b), 8k 2 R. D bZa f(x ) d x= cZa f(x ) d x+ bZc f(x ) d x, 8c 2 (a ;b). ÊLíi gi£i.Ta câ bZa kdx = kx ba =kb �ka =k(b � a).Chån ¡p ¡n C dC¥u 12. Cho hai sè phùc z1 = 2�2i; z2=�3 + 3 i. Khi â z1 �z2 b¬ng A 5� 5i. B �5i. C �5 + 5 i. D �1 + i. ÊLíi gi£i.z 1 �z2 = 2�2i � (3 + 3 i) = 5 �5i.Chån ¡p ¡n A dC¥u 13. Trong khæng gian Oxyz, cho m°t ph¯ng (P ) : 3 x� y+ 2 = 0 . V²c-tì n o trong c¡cv²c-tì d÷îi ¥y l  mët v²c-tì ph¡p tuy¸n cõa (P )? A (3;�1; 0) . B (� 1; 0; �1) . C (3; 0;�1) . D (3;�1; 2) . ÊLíi gi£i.M°t ph¯ng (P )câ mët v²c-tì ph¡p tuy¸n l  #n = (3; �1; 0) .Chån ¡p ¡n A dC¥u 14. Trong khæng gian vîi h» tåa ë Oxyz, t¼m tåa ë iºm Hl  h¼nh chi¸u vuæng gâccõa iºm A(2; �1; 3) l¶n m°t ph¯ng (Oxz ). A H(2; 0; 3) . B H(2; �1; 0) . C H(2; 1; 3) . D H(0; �1; 0) .179/305 179/305 pGV: L¶ Quang Xe  Ô0967.003.131— SÈ 12 NÌI N€O CÂ Þ CH, NÌI  C CON ×ÍNG180ÊLíi gi£i.Tåa ë iºm Hl  h¼nh chi¸u vuæng gâc cõa iºm A(2; �1; 3) l¶n m°t ph¯ng (Oxz )l  H(2; 0; 3) .Chån ¡p ¡n A dC¥u 15. Trong m°t ph¯ng vîi h» tåa ë Oxy, cho c¡c iºm A(4; 0) ,B (1; 4) v C(1; �1) . GåiG l  trång t¥m cõa tam gi¡c ABC. Bi¸t r¬ng Gl  iºm biºu di¹n sè phùc z. M»nh · n o sau¥y l  óng? A z= 3 �3 2i. B z= 3 + 3 2i. C z= 2 �i. D z= 2 + i. ÊLíi gi£i.G l  trång t¥m cõa tam gi¡c ABCsuy ra G�4 + 1 + 1 3;0 + 4 + (�1) 3‹= (2; 1) .Vªy Gl  iºm biºu di¹n cõa sè phùc z= 2 + i.Chån ¡p ¡n D dC¥u 16. T¼m sè ti»m cªn cõa ç thà h m sè y= 3x + 1 x� 2. A 2. B 3. C 1. D 0. ÊLíi gi£i.Ì limx ! +1 y= limx! +1 3x + 1 x� 2 = 3v limx !�1 y= limx!�1 3x + 1 x� 2 = 3.N¶n y= 3 l  ti»m cªn ngang cõa ç thà h m sè.Ì limx ! 2+ y= limx! 2+ 3x + 1 x� 2 = +1v  limx ! 2� y= limx! 2� 3x + 1 x� 2 =�1 .N¶n x= 2 l  ti»m cªn ùng cõa ç thà h m sè.Vªy ç thà h m sè câ hai ti»m cªn.Chån ¡p ¡n A dC¥u 17. Choa >0, a 6= 1 ,b > 0, c > 0sao cho logab= 3 ,logac= �2. T½nh loga(a 3b 2p c). A 6. B �18 . C �9. D 8. ÊLíi gi£i.Ta câ loga(a 3b 2p c) = logaa3+ log ab2+ log ap c= 3 + 2 logab+ 1 2logac= 3 + 2 3 + 1 2(� 2) = 8 .Chån ¡p ¡n D dC¥u 18.÷íng cong trong h¼nh v³ b¶n l  ç thà cõa h m sè n o d÷îi ¥y? A y= x3� 2x 2+ 1 . B y= x3+ x2� 1. C y= x3+ x2+ 1 . D y= x3� 2x 2� 1. xyOÊLíi gi£i.ç thà h m sè c­t tröc tung t¤i iºm câ tung ë d÷ìng v  câ mët cüc trà ¥m.X²t y= x3+ x2+ 1 cây0= 3 x2+ 2 x= 0 ,x= �2 3<0v  x= 0 (thäa m¢n).180/305 180/305 pGV: L¶ Quang Xe  Ô0967.003.131PHT TRIšN — THAM KHƒO 2022 NÌI N€O CÂ Þ CH, NÌI  C CON ×ÍNG181Chån ¡p ¡n C dC¥u 19. Trong khæng gian Oxyz, ÷íng th¯ng d: x� 1 1=y� 2 1=z� 3 1i qua iºm n od÷îi ¥y? A M(� 1; 2; 3) . B N(3; 2; 1) . C P(1; 2; 3) . D Q(0; 0; 0) . ÊLíi gi£i.X²t iºm P2d, 1� 1 1=2� 2 1=3� 3 1óng.Chån ¡p ¡n C dC¥u 20. Sè c¡c sè tü nhi¶n câ 4chú sè ph¥n bi»t l§y tø tªp hñp M=f1; 2; 3; 4; 5; 6; 7; 8; 9 gl  A 4!. B A49 . C 49. D C49 . ÊLíi gi£i.Méi c¡ch s­p thù tü 4ph¦n tû cõa Mcho ta mët sè tho£ m¢n y¶u c¦u b i to¡n.Do â sè c¡c sè tho£ m¢n y¶u c¦u b i to¡n l  sè ch¿nh hñp chªp 4cõa 9ph¦n tû.Chån ¡p ¡n B dC¥u 21. Cho l«ng trö ùng ABC:A0B 0C 0câ ¡y ABCl  tam gi¡c ·u c¤nh a, AA 0= 4 a. Thºt½ch khèi l«ng trö ABC:A0B 0C 0l  A a3. B 2p 3a 3. C p3a 3. D p3a 3 3. ÊLíi gi£i.Do l«ng trö ABC:A0B 0C 0l  l«ng trö ùng )AA 0? (ABC ), suy ra AA0=4 a l  chi·u cao cõa l«ng trö ¢ cho.Do 4ABC ·u, suy ra di»n t½ch ¡y S4 ABC =a2p 34.Thº t½ch khèi l«ng trö V=AA 0 S4 ABC = 4aa2p 34=a3p 3: B0 BA0 A C0 CChån ¡p ¡n C dC¥u 22. ¤o h m cõa h m sè y= e x2+ xl  A y0= ( x2+ x)e 2x +1. B y0= (2 x+ 1)e x. C y0= (2 x+ 1)e 2x +1. D y0= (2 x+ 1)e x2+ x. ÊLíi gi£i.y 0= ( x2+ x)0e x2+ x= (2 x+ 1)e x2+ x:Chån ¡p ¡n D dC¥u 23. Cho h m sè y= f(x ) câ b£ng bi¸n thi¶n nh÷ sau181/305 181/305 pGV: L¶ Quang Xe  Ô0967.003.131— SÈ 12 NÌI N€O CÂ Þ CH, NÌI  C CON ×ÍNG182xy0 y �1�p 2 0 p2 +1 �0 +0 �0 ++1 +1 �2 �2 22�2 �2 +1 +1 H m sèy= f(x ) çng bi¸n tr¶n kho£ng n o d÷îi ¥y A (� 1; 0) . B (� 2; + 1). C (� 2; 2) . D (�1 ; 0). ÊLíi gi£i.H m sè çng bi¸n tr¶n kho£ng (� p 2; 0)n¶n çng bi¸n tr¶n kho£ng (� 1; 0) .Chån ¡p ¡n A dC¥u 24. T½nh t½ch ph¥n I= 2Z1 x+ 1 xdx . A I= 1 �ln 2 . B I= 2 ln 2 . C I= 1 + ln 2 . D I= 7 4. ÊLíi gi£i.Ta câ I= 2Z1 x+ 1 xdx = 2Z1 �1 + 1 x‹dx = ( x+ ln jx j) 21 = 1 + ln 2.Chån ¡p ¡n C dC¥u 25. Cho c§p sè cëng un, bi¸tu1 =�5, u2 =�3. Trong c¡c k¸t qu£ sau, k¸t qu£ n oóng? A u5 =�1. B u5 = 5. C u5 = 3. D u5 = 1. ÊLíi gi£i.Ta câ cæng sai d= u2 �u1 = 2. Do â u5 =u1 + 4d= 3 .Chån ¡p ¡n C dC¥u 26. Hå nguy¶n h m cõa h m sè f(x ) = 2 2xl  A 4xln 4 + C. B 14xln 4 +C. C 4x+ C. D 4x ln 4+C. ÊLíi gi£i.Ta câ Z22xdx = Z4xdx = 4x ln 4+C.Chån ¡p ¡n D dC¥u 27. Bi¸tA(0; a); B (b ; 1) thuëc ç thà h m sè y= x3+ x2� 1, khi â gi¡ trà a+ bl  A �1. B 0. C 1. D 2. ÊLíi gi£i.+ A(0; a) thuëc ç thà h m sè y= x3+ x2� 1th¼ a= �1.182/305 182/305 pGV: L¶ Quang Xe  Ô0967.003.131PHT TRIšN — THAM KHƒO 2022 NÌI N€O CÂ Þ CH, NÌI  C CON ×ÍNG183+B(b ; 1) thuëc ç thà h m sè y= x3+ x2� 1th¼ b3+ b2� 1 = 1 ,b= 1 . Khi â a+ b= 0 :Chån ¡p ¡n B dC¥u 28. T¼mmº gi¡ trà nhä nh§t cõa h m sè y= x3� 3x 2� 9x + m tr¶n o¤n [0; 4]b¬ng�25 ,khi â h¢y t½nh gi¡ trà cõa biºu thùc P= 2 m+ 1 . A 1. B 7. C 5. D 3. ÊLíi gi£i.Ta câ y0= 3 x2� 6x � 9; y0= 0 ,–x = 3x = �1 =2 [0; 4] :Khi â y(0) = m,y(4) = �20 + m,y(3) = �27 + m, suy ra min[0;4] y= y(3) = �27 + m.Do â min[0;4] y= �25 , � 27 + m=�25 , m= 2 )P= 2 2 + 1 = 5 .Chån ¡p ¡n C dC¥u 29. C¡c kho£ng çng bi¸n cõa h m sè y= �x3+ 3 x2+ 4 l  A (� 1; 1) . B (�1 ; 0); (2; + 1). C (0; 2). D (�1 ; 1); (0; + 1). ÊLíi gi£i.Ta câ y0= �3x 2+ 6 x; y 0= 0 ,–x = 0x = 2 . B£ng bi¸n thi¶n xf0( x ) f(x ) �10 2 +1 �0 +0 �+1 +1 44 88�1�1Vªy h m sè çng bi¸n tr¶n kho£ng(0; 2).Chån ¡p ¡n C dC¥u 30. Choa, b, c l  c¡c sè nguy¶n d÷ìng. Gi£ sû log182430 =alog183 +blog185 +c. Gi¡trà cõa biºu thùc 3a + b+ 1 b¬ng A 9. B 11. C 1. D 7. ÊLíi gi£i.V¼ 2430 = 2 355 n¶n log182430 = log1833+ log 185 + log1818 = 3 log183 + log185 + 1.Suy ra a= 3 ,b = 1 ,c = 1 , do â 3a + b+ 1 = 11 .Chån ¡p ¡n B dC¥u 31. Vîi mët t§m b¼a h¼nh vuæng, ng÷íi ta c­t bä ð méi gâc t§m b¼a mët h¼nh vuæng c¤nh12 cm rçi g§p l¤i th nh mët h¼nh hëp chú nhªt khæng câ n­p (h¼nh v³). Gi£ sû thº t½ch cõa c¡ihëp â l  4800cm3th¼ c¤nh cõa t§m b¼a ban ¦u câ ë d i l  bao nhi¶u?183/305 183/305 pGV: L¶ Quang Xe  Ô0967.003.131— SÈ 12 NÌI N€O CÂ Þ CH, NÌI  C CON ×ÍNG184A 44cm. B 42cm. C 36cm. D 38cm. ÊLíi gi£i.Gåi x(cm) (x > 24)l  ë d i c¤nh h¼nh vuæng ban ¦u.Do âV= 12 (x � 24) (x � 24), 4800 = 12( x� 24) 2, x2� 48x+ 176 = 0, –x = 44x = 4 (lo¤i).Vªy x= 44 cm.Chån ¡p ¡n A dC¥u 32. T¼m sè gi¡ trà cõa tham sè mº mZ0 (2x+ 1) d x= 2 . A 2. B 1. C 0. D 3. ÊLíi gi£i.Ta câ mZ0 (2x+ 1) d x= 2 ,(x 2+ x) m0 = 2,m2+ m = 2 ,–m = 1m =�2:Vªy câ 2gi¡ trà cõa mthäa m¢n.Chån ¡p ¡n A dC¥u 33. Trong khæng gian Oxyz, m°t ph¯ng (P )i qua iºm A(2; 3; 3) v  song song vîi gi¡cõa hai v²c-tì #a = (1; 0; 2) v #b = ( �1; 3; 1) câ ph÷ìng tr¼nh l  A (P ) : x+ 2 y+ 3 z+ 14 = 0 . B (P ) : x+ 2 y� 12 = 0 . C (P ) : 2 x+ y� z� 4 = 0 . D (P ) : 2 x+ y� z� 2 = 0 . ÊLíi gi£i.Ta câ ”#a ; #b —= ( �6; �3; 3) .Suy ra m°t ph¯ng (P )i qua A(2; 3; 3) v  câ v²c-tì ph¡p tuy¸n #n = (2; 1; �1) .) (P ) : 2 ( x� 2) + 1 ( y� 3) �1 ( z� 3) = 0 hay(P ) : 2 x+ y� z� 4 = 0 .Chån ¡p ¡n C dC¥u 34. Câ bao nhi¶u sè thùc thäa m¢n z+ jz j2i � 1� 3 4i= 0 ? A 1. B 3. C 2. D 0.184/305 184/305 pGV: L¶ Quang Xe  Ô0967.003.131PHT TRIšN — THAM KHƒO 2022 NÌI N€O CÂ Þ CH, NÌI  C CON ×ÍNG185ÊLíi gi£i.°t z= a+ bi (a; b 2R): Thay v o biºu thùc cõa b i to¡n ta câ:( a � 1) + �a2+ b2+ b� 3 4‹i= 0 )8<: a= 1b 2+ b+ 1 4= 0 )8<: a= 1b = �1 2:Vªy ch¿ câ óng mët sè phùc thäa m¢n b i to¡n.Chån ¡p ¡n A dC¥u 35. Cho h¼nh châp tù gi¡c ·u S:ABC Dcâ c¤nh ¡y b¬ng av  chi·u cao b¬ng h. Gåi Ol  t¥m cõa ¡y ABC D. T½nh kho£ng c¡ch tø O¸n (S AB ). A ahp2a 2+ 4 h2. B ahpa2+ 4 h2. C ahpa2+ h2. D ah2p a2+ h2. ÊLíi gi£i.Ta câ S O?(ABC D ).K´ OK ?AB; (K 2AB ), OH ?S K; (H 2S K ).V¼ ¨OH ?S K (S AB )OH ?AB (S AB ); (AB ?(S OK )) OH ?(S AB ).Suy ra: kho£ng c¡ch tø O¸n (S AB )l :d (O; (S AB ))=OH .X²t S OK vuæng t¤i O, ta câ:1 OH2= 1 OK2+ 1 S O2) OH = OKS O pOK2+ S O 2= ah 2É a2 4+h2 =ah pa2+ 4 h2.Vªy kho£ng c¡ch c¦n t¼m l d(O; (S AB ))=OH = ah pa2+ 4 h2: HB C DSAK OChån ¡p ¡n B dC¥u 36. Câ bao nhi¶u sè tü nhi¶n câ 6chú sè æi mët kh¡c nhau trong â chùa c¡c chú sè3 ;4 ;5 v  chú sè 4ùng c¤nh chú sè 3v  chú sè 5? A 1470. B 750. C 2940. D 1500. ÊLíi gi£i.X²t sè câ 6chú sè æi mët kh¡c nhau câ d¤ng x= a1a2a3a4a5a6. V¼ c¡c chú sè3;4 ;5 ùng c¤nh nhauv  chú sè 4ùng c¤nh chú sè 3v  chú sè 5n¶n câ hai kh£ n«ng 345ho°c 543. Ta coi 345v 543 l chú sè °c bi»t A(câ 2chú sè °c bi»t n y). º ¸m sè c¡c sè xthäa m¢n y¶u c¦u b i to¡n, ta x²thai tr÷íng hñp:Tr÷íng hñp ch§p nhªn c£ a1 = 0:Ì Chån và tr½ cho A, câ 4c¡ch chån.Ì Chån 3chú sè cán trong tªp f0; 1; 2; 6; 7; 8; 9 gv  s­p x¸p v o c¡c và tr½ cán l¤i, câ A37 c¡ch.Tr÷íng hñp n y câ 24 A 37 sè.185/305 185/305 pGV: L¶ Quang Xe  Ô0967.003.131— SÈ 12 NÌI N€O CÂ Þ CH, NÌI  C CON ×ÍNG186Tr÷íng hñpa1 = 0:Ì Chån và tr½ cho A, câ 3c¡ch chån.Ì Chån 2chú sè cán trong tªp f1; 2; 6; 7; 8; 9 gv  s­p x¸p v o c¡c và tr½ cán l¤i, câ A26 c¡ch. Tr÷ínghñp n y câ 23 A 26 sè.Vªy, sè c¡c sè c¦n t¼m l  24 A 37 �23 A 26 = 1500.Chån ¡p ¡n D dC¥u 37. Trong khæng gian vîi h» tåa ë Oxyz, cho iºm M(2; 1; 0) v  ÷íng th¯ng  :x� 1 2=y + 1 1=z �1. Ph÷ìng tr¼nh tham sè cõa ÷íng th¯ngdi qua M, c­t v  vuæng gâc vîi l  A d: 8><>: x= 2 + ty = 1 �4tz = �2t: B d: 8><>: x= 2 + 2 ty = 1 + tz = �t: C d: 8><>: x= 2 �ty = 1 + tz = t: D d: 8><>: x= 1 + ty = �1� 4tz = 2 t: ÊLíi gi£i.Ta câ  :8><>: x= 1 + 2 ty = �1 + tz = �t:Gåi N=d\  ) N(1 + 2 t; � 1 + t; � t) ) # M N = (�1 + 2 t; � 2 + t; � t) .÷íng th¯ng câ v²c-tì ch¿ ph÷ìng #u = (2; 1; �1) .÷íng th¯ng di qua Mv d?  , # M N #u = 0 ,2(�1 + 2 t) + 1( �1 + t) � 1(�t) = 0 ,t= 2 3.Suy ra # M N =�1 3;� 4 3;� 2 3‹, suy ra v²c-tì ch¿ ph÷ìng cõa dl  #u d= (1;�4; �2) .n¶n d: 8><>: x= 2 + ty = 1 �4tz = �2t:Chån ¡p ¡n A dC¥u 38. Cho d¢y(un)thäa m¢n u1 = 5;un+1n +1 =unn + 2 n+ 2 3n, 8n  1. T¼m sè nguy¶n nhänh§t thäa m¢n unn �2n> 5100. A 233. B 146. C 232. D 147. ÊLíi gi£i.°t vn =unn �2n) v1 =u1 �2 = 3 . Ta câu n+1n +1 =unn + 2 n+ 2 3n, un+1n +1 �2n+1=unn �2n+ 2 3n, vn+1 =vn + 23n:Suy ra vn =v1 + 2(3 1+ 3 2+    + 3 n� 1) = 3 n) un �2n= 3 n.Ta câ un �2n> 5100,3n> 5100,n > 100 log35 146 ;5 ) n 147 .Chån ¡p ¡n D dC¥u 39. T¼m gi¡ trà thüc cõa tham sè kbi¸t ÷íng th¯ng d:y = x+ 2 k+ 1 c­t ç thà h m sèy = 2x + 1 x+ 1 t¤i hai iºmA; Bsao cho kho£ng c¡ch tø hai iºm â ¸n tröc ho nh b¬ng nhau. A k= �1. B k= 2 . C k= �2. D k= 1 . ÊLíi gi£i.186/305 186/305 pGV: L¶ Quang Xe  Ô0967.003.131PHT TRIšN — THAM KHƒO 2022 NÌI N€O CÂ Þ CH, NÌI  C CON ×ÍNG187X²t ph÷ìng tr¼nh ho nh ë giao iºmx+ 2 k+ 1 = 2x + 1 x+ 1 , i·u ki»nx6= �1.Vîi i·u ki»n ph÷ìng tr¼nh t÷ìng ÷ìng vîi (x + 2 k+ 1) ( x+ 1) = 2 x+ 1 ,x2+ 2 kx+ 2 k= 0 : (1)÷íng th¯ng dc­t ç thà t¤i hai iºm ph¥n bi»t ,ph÷ìng tr¼nh (1)câ hai nghi»m ph¥n bi»t kh¡c� 1,¨ 0= k2� 2k > 01 � 2k + 2 k6= 0 ,k2 (1 ; 0) [(2; + 1): ( )Khi â ph÷ìng tr¼nh (1)câ hai nghi»m ph¥n bi»t x1;x2 )A(x1;x1 + 2k+ 1) v B(x2;x2 + 2k+ 1) .Hai iºm A; Bc¡ch ·u hai tröc ho nh , jx1 + 2k+ 1 j= jx2 + 2k+ 1 j , x1 +x2 =�4k � 2.Theo ành lþ Vi-²t ta câ x1 +x2 =�2k , do â �2k = �4k � 1, k= �1 thäa m¢n ( ).Chån ¡p ¡n A dC¥u 40. GåiF(x ) l  mët nguy¶n h m cõa h m sè f(x ) = 4 x3� 3x + 2 thäa m¢n F(� 1) = �3 2.Khi â ph÷ìng tr¼nh F(x ) = 2 x+ 1 câ sè nghi»m thüc l  A 0. B 1. C 2. D 3. ÊLíi gi£i.Ta câ F(x ) = Zf(x )d x= Z�4 x 3� 3x + 2 dx = x4� 3 2x2+ 2 x+ C.Do F(� 1) = �3 2n¶n suy ra(� 1) 4� 3 2(� 1) 2+ 2 (� 1) + C=�3 2)C= 1 .Tø â F(x ) = x4� 3 2x2+ 2 x+ 1 . Bði vªyF (x ) = 2 x+ 1 ,x4� 3 2x2+ 2 x+ 1 = 2 x+ 1, x4� 3 2x2= 0, 24 x= 0x = p 62:Do â ph÷ìng tr¼nh F(x ) = 2 x+ 1 câ3nghi»m ph¥n bi»t.Chån ¡p ¡n D dC¥u 41. Cho l«ng trö ùng tam gi¡c ABC:A0B 0C 0câ ¡y ABCl  tam gi¡c vuæng c¥n t¤i Bvîi BA =BC =a, bi¸t m°t ph¯ng (A 0BC )hñp vîi m°t ph¯ng ¡y (ABC )mët gâc 60. Thº t½chcõa khèi l«ng trö ¢ cho l  A p3a 3. B a3 2. C 2p 3a 3 3. D a3p 32. ÊLíi gi£i.187/305 187/305 pGV: L¶ Quang Xe  Ô0967.003.131— SÈ 12 NÌI N€O CÂ Þ CH, NÌI  C CON ×ÍNG188Ta câBA?BC (1). M°t kh¡c ta câ BC?AA 0(do AA0? (ABC )),suy ra BC?BA 0(2) .Tø (1) v (2) suy ra ÕA 0BA l  gâc giúa hai m°t ph¯ng (A 0BC )v  (ABC ).Theo gi£ thi¸t ta câ ÕA 0BA = 60 .Ta câ S4 ABC =1 2BABC =1 2a a = a2 2.AA 0= AB tan ÕA 0BA =atan 60 = ap 3.) VABC:A 0B 0C 0= S4 ABC AA 0= a2 2a p 3 =a3p 32(vtt). a 60 BAA0 C C0 B0 Chån ¡p ¡n D dC¥u 42. Häi câ bao nhi¶u sè phùc zthäa çng thíi c¡c i·u ki»n jz � ij = 5 v z2l  sè thu¦n£o? A 2. B 3. C 0. D 4. ÊLíi gi£i.°t z= x+ iy (vîi x, y 2 R).Ta câ jz � ij = 5 ,x2+ ( y� 1)2= 25 :( )z 2l  sè thu¦n £o, suy ra x2� y2= 0 ,–x = yx = �y:Vîi x= ythay v o ( ) ta ÷ñc x2+ ( x� 1)2= 25 ,2x 2� 2x � 24 = 0 ,–x = 4x = �3:Vîi x= �y thay v o ( ) ta ÷ñc x2+ ( x+ 1) 2= 25 ,2x 2+ 2 x� 24 = 0 ,–x = �4x = 3 :Vªy câ 4sè phùc c¦n t¼m l  4 + 4i, � 3� 3i, � 4 + 4 i, 3 � 3i.Chån ¡p ¡n D dC¥u 43. Cho sè phùc zthäa m¢n j2 z � 1j = j z+ 1 + ij v  iºm biºu di¹n cõa ztr¶n m°t ph¯ngtåa ë thuëc ÷íng trán câ t¥m I(1; 1) , b¡n k½nh R= p 5. Khi â t½ch mæun cõa t§t c£ c¡c sèphùc zthäa m¢n c¡c y¶u c¦u tr¶n l ? A p5. B 3. C 3p 5. D 1. ÊLíi gi£i.Gåi z= a+ bi(a; b 2R).Theo gi£ thi¸t ta câj2( a+ bi)� 1j = ja � bi+ 1 + ij, j (2a� 1) + 2 bij= j( a + 1) �(b � 1)ij, (2a� 1)2+ (2 b)2= ( a+ 1) 2+ ( b� 1)2, 3a 2+ 3 b2� 6a + 2 b� 1 = 0 : (1)V¼ iºm biºu di¹n cu© ztr¶n m°t ph¯ng tåa å thuëc ÷íng trán t¥m I(1; 1) ; R=p 5n¶n ta câ( a � 1)2+ ( b� 1)2= 5, a2+ b2� 2a � 2b = 3, a2� 2a = 3 �b2+ 2 b: (2)188/305 188/305 pGV: L¶ Quang Xe  Ô0967.003.131PHT TRIšN — THAM KHƒO 2022 NÌI N€O CÂ Þ CH, NÌI  C CON ×ÍNG189Th¸(2)v o (1)ta ÷ñc 3(3�b2+ 2 b) + 3 b2+ 2 b� 1 = 0 ,b= �1.Khi â, thay v o (2)ta suy ra –a = 0a = 2 )–z1 =�1z 2 = 2�i) jz1j  jz2j= p 5.Chån ¡p ¡n A dC¥u 44. Cho h m sè y= f(x ) câ ¤o h m tr¶n Rv  f0( x ) câ b£ng x²t d§u nh÷ sau xf0( x ) �1�2 2 +1 �0 +0 �Sè iºm cüc trà cõa h m sèg(x ) = f(x 2� j xj) l  A 7. B 5. C 3. D 9. ÊLíi gi£i.Ta câ g(x ) = f(jx j2� j xj) .X²t h m sè h(x ) = f(x 2� x) ) g(x ) = h(jx j) .Ta câ h0( x ) = f0(x 2� x) = (2 x� 1)f0(x 2� x). Khi âh 0( x ) = 0 ,–2x � 1 = 0f 0�x 2� x= 0 ,2664 x= 1 2x 2� x= �2x 2� x= 2 ,2664 x= 1 2x = �1x = 2 :Ta câ b£ng bi¸n thi¶n cõa h m sè h(x ) = f(x 2� x). xh0( x ) h(x ) �1�1 12 2 +1 +0 �0 +0 ��1�1 f(� 1) f(� 1) f�1 2‹ f�1 2‹ f(2) f(2) �1�1Düa v o b£ng bi¸n thi¶n ta th§y h m sèf(x ) câ 2iºm cüc trà d÷ìng n¶n h m sè g(x ) = h(jx j) câ 5iºm cüc trà.Chån ¡p ¡n B dC¥u 45. Cho h m sè y= f(x ) = x3+ 3 x� 4. Câ bao nhi¶u gi¡ trà cõa tham sè mº ph÷ìngtr¼nh (f (x )) 3= 3p f(x ) + m+m câ óng 2nghi»m ph¥n bi»t? A væ sè. B 2. C 4. D 5. ÊLíi gi£i.°t u= 3p f(x ) + m) u3= f(x ) + m. Khi â (f (x )) 3= u+ m ) u3+ u= ( f(x )) 3+ f(x ). (*)X²t h m sè g(x ) = x3+ x) g0( x ) = 3 x2+ 1 >0;8 x 2 R.Suy ra h m sè g(x ) luæn çng bi¸n tr¶n R.Suy ra (?) , u= f(x ) , (f (x )) 3� m =f(x ) , (f (x )) 3� f(x ) = m. (**)°t t= f(x ) ) ( ), t3� t= m.X²t h m sè y= f(x ) = x3+ 3 x� 4) f0( x ) = 3 x2+ 3 >0;8 x 2 R.Suy ra h m sè f(x ) luæn çng bi¸n tr¶n R.Suy ra méi gi¡ trà cõa tcho duy nh§t 1nghi»m cõa ph÷ìng tr¼nh x3+ 3 x� 4 = t.189/305 189/305 pGV: L¶ Quang Xe  Ô0967.003.131— SÈ 12 NÌI N€O CÂ Þ CH, NÌI  C CON ×ÍNG190)(f (x )) 3= 3p f(x ) + m+m câ óng 2nghi»m ph¥n bi»t th¼ ph÷ìng tr¼nh t3� t= m câ óng 2nghi»m ph¥n bi»t.X²t h m sè f(t) = t3� t) f0( t) = 3 t2� 1, f0( t) = 0 ,t=  1 p3.B£ng bi¸n thi¶n xy0 y �1 �1 p3 1p3 +1 +0 �0 +�1�1 2p 392p 39�2p 39�2p 39 +1 +1 y= m T÷ b£ng bi¸n thi¶n ta câ ph÷ìng tr¼nht3� t= m câ óng 2nghi»m ph¥n bi»t khi v  ch¿ khim =2p 39.Chån ¡p ¡n B dC¥u 46. Cho h¼nh chú nhªt ABC DcâAB = 2,AD = 2p 3v  n¬m trong m°t ph¯ng (P ).Quay (P )mët váng quanh ÷íng th¯ng BD. Khèi trán xoay ÷ñc t¤o th nh câ thº t½ch b¬ng A 28 9. B 28 3. C 56 9. D 56 3. ÊLíi gi£i.Gåi c¡c iºm E,F ,I, J ,H ,L ,K nh÷ h¼nh v³.Gåi V1,V2,V3 l¦n l÷ñt l  thº t½ch c¡c khâi nân nhªn ÷ñc khiquay c¡c tam gi¡c ABH,ADH ,I DL mët váng quanh ÷íngth¯ng BD.Ta câ AH=p 3, I L = 2 p3,BH = 1,H D = 3,LD = 2. Khi âthº t½ch khèi trán xoay ÷ñc t¤o th nh l V = 2 ( V1 +V2 �V3)= 2 •1 3AH 2BH +1 3AH 2DH �1 3I L 2LD ˜= 2 3�3 1 + 3 3 � 4 32‹= 56 9: A FIE CJ DBH KLChån ¡p ¡n C dC¥u 47. Cho h m sè y= f(x ) li¶n töc tr¶n o¤n [� 1; 9] v  câ ç thà l  ÷íng cong trong h¼nhv³ d÷îi ¥y190/305 190/305 pGV: L¶ Quang Xe  Ô0967.003.131PHT TRIšN — THAM KHƒO 2022 NÌI N€O CÂ Þ CH, NÌI  C CON ×ÍNG191xy�1 1 2 3 4 5 6 7 8 9�4 �3 �2 �1 12OCâ t§t c£ bao nhi¶u gi¡ trà nguy¶n cõa tham sèmº b§t ph÷ìng tr¼nh 163 f(x )� [f 2(x ) + 2 f(x )�8] 4f(x ) (m 2� 3m )6f(x )nghi»m óng vîi måi gi¡ trà 2[� 1; 9] ? A 32. B 31. C 5. D 6. ÊLíi gi£i.+) Tø ç thà h m sè suy ra �4 f(x )  2,8 x 2 [� 1; 9] , °t t= f(x ) ) t2 [� 4; 2] .+) Ta t¼m m º 163t� [t 2+ 2 t� 8]4t (m 2� 3m )6tóng 8t 2 [� 4; 2], 16 2t �[t 2+ 2 t� 8]�2 3‹t m2� 3m; 8t 2 [� 4; 2] .Ta câ 16 2t 4; 8t 2 [� 4; 2] , d§u b¬ng x£y ra khi t= 2 .M  t2+ 2 t� 8 0; 8t 2 [� 4; 2] )[t 2+ 2 t� 8]�2 3‹t 0; 8t 2 [� 4; 2] , d§u b¬ng x£y ra khi t= 2 .) 16 2t �[t 2+ 2 t� 8]�2 3‹t 4; 8t 2 [� 4; 2] .Vªy m2� 3m 4, � 1 m 4, m  mnguy¶n n¶n m2 f� 1; 0; 1; 2; 3; 4 g.Chån ¡p ¡n DdC¥u 48. Trong khæng gian Oxyz, cho m°t ph¯ng (Pm ) : (m�1)x+ (2 �m)y � mz + 2m�1 = 0thay êi. H¼nh chi¸u vuæng gâc cõa iºm A(1; 0; 3) l¶n m°t ph¯ng (Pm )luæn thuëc ÷íng trán cèành câ b¡n k½nh l  A R= 1 . B R= 1 2. C R= p 22. D R= p 32. ÊLíi gi£i.Gåi dm l  ÷íng th¯ng i quaAv  vuæng gâc vîi (Pm ).Gåi Am l  h¼nh chi¸u vuæng gâc cõaAl¶n (Pm )suy ra Am =dm \(Pm ). Ta x²tÌ Khi m= 0 ;(P0) :�x+ 2 y� 1 = 0 suy ra d0: 8><>: x= 1 �ty = 2 tz = 3 (t 2 R):Ta câ A0(1�t;2t; 3) vîi tthäa �(1 �t) + 2 2t � 1 = 0 ,t= 2 5.Vªy A0�3 5;4 5; 3 ‹.191/305 191/305 pGV: L¶ Quang Xe  Ô0967.003.131— SÈ 12 NÌI N€O CÂ Þ CH, NÌI  C CON ×ÍNG192ÌKhi m= 1 ;(P1) :y� z+ 1 = 0 suy rad1: 8><>: x= 1y = kz = 3 �k (k 2 R):Ta câ A1(1; k; 3� k) vîi kthäa k� (3 �k) + 1 = 0 ,k= 1 .Vªy A1(1; 1; 2).Ì Khi m= 2 ;(P2) :x� 2z + 3 = 0 suy rad2: 8><>: x= 1 + ly = 0z = 3 �2l (l 2 R):Ta câ A2(1 +l;0;3 � 2l) vîi lthäa (1 +l) � 2(3 �2l) + 3 = 0 ,l= 2 5.Vªy A2�7 5; 0;11 5‹.V¼ Am luæn thuëc mët ÷íng trán cè ành n¶n ÷íng trán â l  ÷íng trán ngo¤i ti¸p tam gi¡cA 0A1A2. GåiRl  b¡n k½nh cõa ÷íng trán n y. Ta °tÌ a= A0A1= Ê �1 � 3 5‹2+ �1 � 4 5‹2+ (2 �3)2= p 305.Ì b= A0A2= Ê �7 5�3 5‹2+ �0 � 4 5‹2+ �11 5�3‹2= 4p 35.Ì c= A1A2= Ê �7 5�1‹2+ (0 �1)2+ �11 5�2‹2= p 305.Ì p= a+ b+ c 2=p 30 + 2p 35.Ta câ di»n t½ch 4A0A1A2l S= p p(p � a)( p� b)( p� c) = 6p 625.M°t kh¡c S= abc 4R )R= abc 4S = p 22.Chån ¡p ¡n C dC¥u 49. Câ bao nhi¶u gi¡ trà nguy¶n d÷ìng cõa tham sè m º h m sè y= j3 x 4� 4x 3� 12 x2+ m jcâ 5iºm cüc trà? A 16. B 44. C 26. D 27. ÊLíi gi£i.X²t h m sè f(x ) = 3 x4� 4x 3� 12x2+ mCâ f0( x ) = 12 x3� 12x2� 24x;f 0( x ) = 0 ,264 x= 0x = �1x = 2 :B£ng bi¸n thi¶n xf0( x ) f(x ) �1�1 0 2 +1 �0 +0 �0 ++1 +1 m�5 m�5 mmm�32 m�32 +1 +1192/305 192/305 pGV: L¶ Quang Xe  Ô0967.003.131MÖC LÖCNÌI N€O CÂ Þ CH, NÌI  C CON ×ÍNG193Ta th§y, h m sèy= f(x ) luæn câ 3 iºm cüc trà n¶n h m sè y= jf (x )j câ 5 iºm cüc trà khi v  ch¿khi ph÷ìng tr¼nh f(x ) = 0 câ hai nghi»m ph¥n bi»t kh¡c �1; 0; 2 ho°c câ 3 nghi»m ph¥n bi»t v  câ 1nghi»m b¬ng �1 (ho°c 0 ; 2).Tø b£ng bi¸n thi¶n, ta câ –m �32 <06 m �5m 60 ,–5 6 m < 32m 60:V¼ m nguy¶n, d÷ìng n¶n m2 f 5; 6; ; 31 g ) câ27 gi¡ trà mc¦n t¼m.Chån ¡p ¡n D H˜T193/305 193/305 pGV: L¶ Quang Xe  Ô0967.003.131— SÈ 13 NÌI N€O CÂ Þ CH, NÌI  C CON ×ÍNG194BË GIO DÖC & €O T„OTR×ÍNG THPT NGUY™N T‡T TH€NH GV: L– QUANG XE - 0967.003.131 — SÈ 13 PHT TRIšN — THAM KHƒO 2022N‹M HÅC 2021 - 2022Mæn:To¡nThíi gian l m b i: 90 phót — THAM KHƒO HKIIdC¥u 1. Cho sè phùc z= 4 �3i. T½nh mæ-un cõa sè phùc z. A jz j = p 7. B jz j = 25 . C jz j = 5 . D jz j = 7 . ÊLíi gi£i.Ta câ jz j = j4 � 3ij = p 42+ ( �3) 2= 5 .Chån ¡p ¡n C dC¥u 2. Trong khæng gian Oxyz, cho m°t c¦u (S ) : x2+ y2+ z2� 4x + 6 y� 10 z+ 13 = 0 . T¥mI v  b¡n k½nh Rcõa (S ) l  A I(� 2; �3; �5) ; R = 25 . B I(2; 3; 5) ; R= 5 . C I(� 2; 3; �5) ; R = 25 . D I(2; �3; 5) ; R= 5 . ÊLíi gi£i.Gåi I(a; b; c )l  t¥m cõa (S ), ta câ 8><>: 2a = �42 b = 62 c = �10 )8><>: a= �2b = 3c = �5:T¥m cõa (S ) câ tåa ë l  I(2; �3; 5) , b¡n k½nh R= p 22+ ( �3) 2+ 5 2� 13 = 5 .Chån ¡p ¡n D dC¥u 3. Cho h m sè y= f(x ) câ limx ! +1 f(x ) = 0 v limx !�1 f(x ) = + 1. M»nh · n o sau ¥y l m»nh · óng? A ç thà h m sè n¬m ph½a tr¶n tröc ho nh.B ç thà h m sè câ mët ti»m cªn ùng l  ÷íng th¯ngy= 0 . C ç thà h m sè câ mët ti»m cªn ngang l  tröc ho nh.D ç thà h m sè khæng câ ti»m cªn ngang.ÊLíi gi£i.Tø limx ! +1 f(x ) = 0 suy ra ç thà h m sè câ mët ti»m cªn ngang l  tröc ho nh.Chån ¡p ¡n C dC¥u 4. Cho khèi c¦u câ b¡n k½nh R= 6 . Thº t½ch cõa khèi c¦u b¬ng A 144. B 36. C 288. D 48. ÊLíi gi£i.V = 4 3R3= 288 .Chån ¡p ¡n C 194/305 194/305 pGV: L¶ Quang Xe  Ô0967.003.131PHT TRIšN — THAM KHƒO 2022 NÌI N€O CÂ Þ CH, NÌI  C CON ×ÍNG195dC¥u 5. Trong c¡c m»nh · sau, m»nh · n o sai?A Ze2xdx = 1 2e2x+ C. B Z3x 2dx = x3+ C. C Z1 2x dx = lnjx j 2+C. D Zsin 2 xdx = 2 cos 2 x+ C. ÊLíi gi£i.V¼ Zsin 2 xdx = �1 2cos 2x+ C.Chån ¡p ¡n D dC¥u 6. Cho h m sè y= f(x ) câ b£ng bi¸n thi¶n xy0 y �10 2 +1 �0 +0 �+1 +1 11 55�1�1H m sè ¤t cüc tiºu t¤i iºmA x= 0 . B x= 2 . C x= 1 . D x= 5 . ÊLíi gi£i.H m sè y0êi d§u tø ¥m sang d÷ìng khi xi qua iºm 0(theo chi·u t«ng cõa x) n¶n x= 0 l  iºmcüc tiºu cõa h m sè ¢ cho.Chån ¡p ¡n A dC¥u 7. T¼m tªp nghi»m cõa b§t ph÷ìng tr¼nh log3(x � 2) >2. A (�1 ; 11) . B (2; +1). C [11; +1). D (11; +1). ÊLíi gi£i.i·u ki»n: x� 2> 0, x > 2:V¼ 3> 1n¶n log3(x � 2) >2, x� 2> 32, x> 11.Vªy tªp nghi»m cõa b§t ph÷ìng tr¼nh l  [11; +1).Chån ¡p ¡n C dC¥u 8. Thº t½ch cõa khèi châp câ chi·u cao b¬ng hv  di»n t½ch ¡y b¬ng Bl  A V= 1 3Bh. B V= 1 6Bh. C V=Bh . D V= 1 2Bh. ÊLíi gi£i.Thº t½ch cõa khèi châp câ chi·u cao b¬ng hv  di»n t½ch ¡y b¬ng Bl V= 1 3Bh.Chån ¡p ¡n A dC¥u 9. T¼m tªp x¡c ành cõa h m sè y= (4 x� x2) 1 3. A D= (0; 4) . B D= [0; 4] .195/305 195/305 pGV: L¶ Quang Xe  Ô0967.003.131— SÈ 13 NÌI N€O CÂ Þ CH, NÌI  C CON ×ÍNG196C D= ( �1 ; 0)[(4; + 1). D D=R. ÊLíi gi£i.H m sè y= (4 x� x2) 1 3x¡c ành khi v  ch¿ khi 4x � x2> 0, x2 (0; 4) .Tªp x¡c ành cõa h m sè ¢ cho l  D= (0; 4) .Chån ¡p ¡n A dC¥u 10. T¼m tªp nghi»m Scõa ph÷ìng tr¼nh 3x2= 9 . A S= fp 2; 2g. B S= f� p 2;p 2g . C S= f� p 2; 2g. D S= f� 2; 2 g. ÊLíi gi£i.PT ,3x2= 3 2, x2= 2 ,x= p 2.Chån ¡p ¡n B dC¥u 11. Cho2Z� 1 f(x ) d x= 2 ,7Z� 1 f(t) d t= 9 . Gi¡ trà cõa 7Z2 f(z ) d zl  A 7. B 3. C 11. D 5.ÊLíi gi£i.Ta câ 7Z2 f(z ) d z= 7Z2 f(x ) d x= 7Z� 1 f(x ) d x� 2Z� 1 f(x ) d x= 7Z� 1 f(t) d t� 2Z� 1 f(x ) d x= 9 �2 = 7 :Chån ¡p ¡n A dC¥u 12. T¼m ph¦n thüc v  ph¦n £o cõa sè phùc zthäa m¢n z= 2 �i+ �1 3�2i‹. A 73v �3i. B 73v �3. C 73v 2. D 53v 1 2. ÊLíi gi£i.Ta câ z= 2 �i+ �1 3�2i‹= 7 3�3i.Vªy ph¦n thüc v  ph¦n £o cõa sè phùc zl¦n l÷ñt l  7 3v �3.Chån ¡p ¡n B dC¥u 13. Trong khæng gian vîi h» tåa ë Oxyz, v²c-tì n o d÷îi ¥y l  mët v²c-tì ph¡p tuy¸ncõa m°t ph¯ng (Oxy )? A #j (� 5; 0; 0) . B #k (0; 0; 1) . C #i = (1; 0; 0) . D #m = (1; 1; 1) . ÊLíi gi£i.M°t ph¯ng (Oxy ) :z= 0 câ mët v²c-tì ph¡p tuy¸n l  #k = (0; 0; 1) .Chån ¡p ¡n B 196/305 196/305 pGV: L¶ Quang Xe  Ô0967.003.131PHT TRIšN — THAM KHƒO 2022 NÌI N€O CÂ Þ CH, NÌI  C CON ×ÍNG197dC¥u 14. Trong khæng gian vîi h» tåa ë Oxyz, iºm n o d÷îi ¥y thuëc tröc Oy? A N(2; 0; 0) . B Q(0; 3; 2) . C P(2; 0; 3) . D M(0; �3; 0) . ÊLíi gi£i.iºm A(x ;y ;z ) 2 Oy ,¨x = 0z = 0 . Suy ra, trong4iºm ¢ cho, iºm M(0; �3; 0) 2Oy .Chån ¡p ¡n D dC¥u 15. Sè phùc n o sau ¥y câ iºm biºu di¹n l  M(1; �2) ? A 1 + 2i. B 1� 2i. C �2 + i. D �1� 2i. ÊLíi gi£i.Sè phùc z= 1 �2i câ iºm biºu di¹n l  M(1; �2) .Chån ¡p ¡n B dC¥u 16. ÷íng ti»m cªn ùng cõa ç thà h m sè y= 2x � 3 2x + 1 l  ÷íng th¯ng A x= 3 2. B x= �1 2. C y= 1 . D y= �1 2. ÊLíi gi£i.Ta câ: limx ! � 1 2� 2x � 3 2x + 1 = +1n¶n x= �1 2l  ÷íng ti»m cªn ùng.Chån ¡p ¡n B dC¥u 17. Vîial  sè thüc d÷ìng tòy þ, log3(3a) b¬ng A 1 + log3a. B 3 log3a. C log3a. D 1� log3a. ÊLíi gi£i.Ta câ: log3(3a) = log33 + log3a = 1 + log3aChån ¡p ¡n A dC¥u 18.ç thà trong h¼nh b¶n l  ç thà cõa h m sè n o? A y= �x3+ 4 x2+ 9 x+ 1 . B y= x3+ 6 x2+ 9 x+ 1 . C y= x4� 5x 2+ 1 . D y= x3+ 5 x2+ 8 x+ 1 . xyO�3 �1 �3 1ÊLíi gi£i.ç thà h m sè b¶n l  ç thà cõa h m sè bªc 3 v  câ h» sè tr÷îc x3l  sè d÷ìng.Khi x= �3 th¼ y= 1 n¶n ç thà trong h¼nh b¶n l  ç thà cõa h m sè y= x3+ 6 x2+ 9 x+ 1 .Chån ¡p ¡n B 197/305 197/305 pGV: L¶ Quang Xe  Ô0967.003.131— SÈ 13 NÌI N€O CÂ Þ CH, NÌI  C CON ×ÍNG198dC¥u 19. Trong khæng gian OxyzchoM(� 1; 2; 3) . H¼nh chi¸u vuæng gâc cõa Mtr¶n tröc Oxl  iºm câ tåa ë? A P(� 1; 0; 0) . B Q(0; 2; 3) . C K(0; 2; 0) . D E(0; 0; 3) . ÊLíi gi£i.Tröc Oxcâ ph÷ìng tr¼nh l  8><>: x= ty = 0z = 0 . H¼nh chi¸u cõaMl¶n tröc Oxl  iºm P(� 1; 0; 0) .Chån ¡p ¡n A dC¥u 20. Mët nhâm håc sinh câ 5håc sinh nam v  7håc sinh nú. Sè c¡ch chån 4håc sinh cõanhâm º tham gia mët buêi lao ëng l  A A412 . B C45 + C 47 . C 4!. D C412 . ÊLíi gi£i.Sè c¡ch chån 4håc sinh cõa nhâm º tham gia mët buêi lao ëng l  C412 .Chån ¡p ¡n D dC¥u 21. Cho khèi l«ng trö câ ¡y l  tam gi¡c ·u c¤nh av  chi·u cao b¬ng 4a . Thº t½ch cõakhèi l«ng trö ¢ cho b¬ng A a3p 33. B 2a 3p 3. C a3p 32. D a3p 3. ÊLíi gi£i.V =hS¡y = 4a1 2ap 32a= a3p 3(vtt).Chån ¡p ¡n D dC¥u 22. T½nh ¤o h m cõa h m sè y= ( x2� 2x + 2) e x. A y0= x2e x. B y0= 2e x. C y0= �2x ex. D y0= (2 x� 2)e x. ÊLíi gi£i.Ta câ y0= (2 x� 2)e x+ ( x2� 2x + 2) e x= x2e x.Chån ¡p ¡n A dC¥u 23. Cho h m sè y= f(x ) li¶n töc tr¶n Rcâ b£ng bi¸n thi¶n nh÷ sau xy0 y �1�1 0 1 +1 +0 � +0 ��1�1 22�1 �1 3322M»nh · n o d÷îi ¥y óng ?A H m sè çng bi¸n tr¶n kho£ng(�1 ; 2). B H m sè çng bi¸n tr¶n kho£ng(� 1; 3) . C H m sè nghàch bi¸n tr¶n kho£ng(� 2; 1) . D H m sè nghàch bi¸n tr¶n kho£ng(1; 2). ÊLíi gi£i.H m sè nghàch bi¸n tr¶n (1; +1)n¶n nghàch bi¸n tr¶n kho£ng (1; 2).198/305 198/305 pGV: L¶ Quang Xe  Ô0967.003.131PHT TRIšN — THAM KHƒO 2022 NÌI N€O CÂ Þ CH, NÌI  C CON ×ÍNG199Chån ¡p ¡n D dC¥u 24. Cho hai h m sè f(x ), g(x ) li¶n töc tr¶n [a ;b] v  a < c < b . M»nh · n o d÷îi ¥ysai ? A bZa [f (x ) + g(x )] d x= bZa f(x ) d x+ bZa g(x ) d x. B bZa kf(x ) d x= k bZa f(x ) d xvîi kl  h¬ng sè. C bZa f(x ) g(x ) dx = bZa f(x ) d x bZa g(x ) d x. D bZa f(x ) d x= cZa f(x ) d x+ bZc f(x ) d x. ÊLíi gi£i.Theo t½nh ch§t cõa t½ch ph¥n x¡c ành ta câ bZa [f (x ) + g(x )] d x= bZa f(x ) d x+ bZa g(x ) d xbZa kf(x ) d x= k bZa f(x ) d xvîi k l  h¬ng sè.bZa f(x ) d x= cZa f(x ) d x+ bZc f(x ) d x.Ta khæng câ t½nh ch§t bZa f(x ) g(x ) dx = bZa f(x ) d x bZa g(x ) d x.Chån ¡p ¡n C dC¥u 25. Cho c§p sè cëng (un)thäa m¢n ¨u4 = 10u 4 +u6 = 26 câ cæng sai l  A d= �3. B d= 3 . C d= 5 . D d= 6 . ÊLíi gi£i.Gåi dl  cæng sai cõa c§p sè cëng. Ta câ:¨u4 = 10u 4 +u6 = 26 ,¨u1 + 3d= 102 u1 + 8d= 26 ,¨u1 = 1d = 3 :Vªy cæng sai d= 3 .199/305 199/305 pGV: L¶ Quang Xe  Ô0967.003.131— SÈ 13 NÌI N€O CÂ Þ CH, NÌI  C CON ×ÍNG200Chån ¡p ¡n B dC¥u 26. Choy= f(x ), y = g(x ) l  c¡c h m sè li¶n töc tr¶n R. T¼m kh¯ng ành saitrong c¡ckh¯ng ành sau A Zkf(x ) d x= kZf(x ) d xvîi k2 Rnf 0g . B Z[f (x ) + g(x )] d x= Zf(x ) d x+ Zg(x ) d x. C Z[f (x ) g (x )] d x= Zf(x ) d xZg(x ) d x. D •Zf(x ) d x˜0= f(x ) . ÊLíi gi£i.Ta câ Z[f (x ) g (x )] d x= Zf(x ) d xZg(x ) d xl  sai .Chån ¡p ¡n C dC¥u 27. Cho h m sè y= x+ 2 m x� m câ ç thà l (Cm). T¼m mº ç thà (Cm)i qua iºmA (2; �1) . A m= 0 . B m=�4. C m= 4 . D m=�1 4. ÊLíi gi£i.Ta câ A(2; �1) 2(Cm), � 1 =2 + 2m 2� m ,2 + 2 m=m �2, m=�4.Chån ¡p ¡n B dC¥u 28. GåiMv ml¦n l÷ñt l  gi¡ trà lîn nh§t, gi¡ trà nhä nh§t cõa h m sè y= x3� 3x 2� 9x +1tr¶n o¤n [0; 4]. T½nh têng m+ 2 M. A m+ 2 M= 17 . B m+ 2 M=�37 . C m+ 2 M= 51 . D m+ 2 M=�24 . ÊLíi gi£i.Ta câ y0= 3 x2� 6x � 9, y0= 0 ,–x = �1 =2 [0; 4]x = 3 2[0; 4] :y (0) = 1 ,y(3) = �26 ,y(4) = �19 .Vªy M= max[0;4] y= 1 ,m = min[0;4] y= �26 n¶n m+ 2 M=�24 .Chån ¡p ¡n D dC¥u 29. H m sèy= �x3+ 3 x� 5çng bi¸n tr¶n kho£ng n o sau ¥y? A (1; +1). B (�1 ; 1). C (�1 ;� 1) . D (� 1; 1) . ÊLíi gi£i.Ta câ y0= �3x 2+ 3 :Gi£i y0= 0 ,x= 1:B£ng bi¸n thi¶n200/305 200/305 pGV: L¶ Quang Xe  Ô0967.003.131PHT TRIšN — THAM KHƒO 2022 NÌI N€O CÂ Þ CH, NÌI  C CON ×ÍNG201xy0 y �1�1 1 +1 �0 +0 �+1 +1 �7 �7 �3 �3 �1�1Vªy h m sè çng bi¸n tr¶n(� 1; 1) :Chån ¡p ¡n D dC¥u 30. Cho c¡c sè thüc d÷ìng a, b thäa m¢n 3 loga+ 2 log b= 1 . M»nh · n o sau ¥yóng? A a3+ b2= 1 . B 3a + 2 b= 10 . C a3b 2= 10 . D a3+ b2= 10 . ÊLíi gi£i.Ta câ: 3 loga+ 2 log b= 1 ,log a3+ log b2= 1 ,log ( a3b 2) = 1 ,a3b 2= 10 :Chån ¡p ¡n C dC¥u 31. Cho khèi châp S:ABC Dcâ ¡yABC D l  h¼nh chú nhªt, bi¸t AB=a, AD = 2av S A vuæng gâc vîi m°t ph¯ng ABC D, c¤nhS C= 3a. T½nh chi·u cao cõa khèi S:ABC D. A ap 3. B 3a . C ap 5. D 2a . ÊLíi gi£i.X²t 4ABC vuæng t¤i B, ta câAC =p AB2+ BC 2= p a2+ 4 a2= ap 5:X²t 4S AC vuæng t¤i A, ta câS A =p S C2� AC 2= p 9a 2� 5a 2= 2 a:Vªy chi·u cao cõa khèi châp S A= 2a. SB C DAChån ¡p ¡n D dC¥u 32. T½nhI= 1Z0 e3xdx . A I= e 3� 1. B I= e �1. C I= e3� 1 3. D I= e 3+ 1 2. ÊLíi gi£i.Ta câ I= 1 3e3x 10 =e3� 1 3.Chån ¡p ¡n C 201/305 201/305 pGV: L¶ Quang Xe  Ô0967.003.131— SÈ 13 NÌI N€O CÂ Þ CH, NÌI  C CON ×ÍNG202dC¥u 33. Trong khæng gian Oxyz, m°t ph¯ng i qua t¥m cõa m°t c¦u (x � 1) 2+( y+2) 2+ z2= 12v  song song vîi m°t ph¯ng (Oxz )câ ph÷ìng tr¼nh l  A y+ 1 = 0 . B y� 2 = 0 . C y+ 2 = 0 . D x+ z� 1 = 0 . ÊLíi gi£i.M°t c¦u câ t¥m I(1; �2; 0) .M°t ph¯ng song song m°t ph¯ng (Oxz )n¶n câ d¤ng y+ D = 0 , qua I(1; �2; 0) n¶nD= 2 .Vªy m°t ph¯ng c¦n t¼m l  y+ 2 = 0 .Chån ¡p ¡n C dC¥u 34. Cho sè phùc z= 1 + ip 3. Sè phùc li¶n hñp cõa zl  A z= 1 �ip 3. B z= �p 3� i. C z= �1 + ip 3. D z= p 3 +i. ÊLíi gi£i.z = a+ ib ) z= a� bi.Chån ¡p ¡n A dC¥u 35. Cho tù di»n ·u ABC Dc¤nh b¬ng a. Gåi Ml  trung iºm C D. T½nh kho£ng c¡chgiúa hai ÷íng th¯ng ACv BM . A ap 2211. B ap 23. C ap 33. D a. ÊLíi gi£i.Gåi Nl  trung iºm AD, suy ra M NAC , do âd (AC ;BM ) =d(AC ; (BM N )) =d(D ; (BM N )):Gåi Il  h¼nh chi¸u cõa Ntr¶n (ABC ), suy ra N IAH v N I =AH 2.Tø â suy ra VN:BM D =1 3N I S4 BM D =1 4VABC D =a3p 248.Ta câ S4 BM N =a2p 1116.V N:BM D =VD:BM N =1 3d(D ; (BM N )):S4BM N )d (D ; (BM N )) =ap 2211 BC M DNIH AChån ¡p ¡n A dC¥u 36. Th¦y gi¡o D÷ìng câ 30c¥u häi kh¡c nhau gçm 5c¥u khâ, 10c¥u trung b¼nh v  15c¥u d¹. Tø 30c¥u häi â câ thº lªp ÷ñc bao nhi¶u · kiºm tra, méi · gçm 5c¥u häi kh¡c nhau,sao cho trong méi · nh§t thi¸t ph£i câ õ 3lo¤i c¥u häi v  sè c¥u d¹ khæng ½t hìn 2. A 56875. B 42802. C 41811. D 32023. ÊLíi gi£i.Ta câ c¡c tr÷íng hñp:Ì Tr÷íng hñp 1. Mët · câ 2c¥u d¹, 1c¥u khâ v  2c¥u trung b¼nh )sè · t¤o ÷ñc l :C 152 C 51 C 102 = 23625.202/305 202/305 pGV: L¶ Quang Xe  Ô0967.003.131PHT TRIšN — THAM KHƒO 2022 NÌI N€O CÂ Þ CH, NÌI  C CON ×ÍNG203ÌTr÷íng hñp 2. Mët · câ 2c¥u d¹, 2c¥u khâ v  1c¥u trung b¼nh )sè · t¤o ÷ñc l :C 152 C 52 C 101 = 10500.Ì Tr÷íng hñp 3. Mët · câ 3c¥u d¹, 1c¥u khâ v  1c¥u trung b¼nh )sè · t¤o ÷ñc l :C 153 C 51 C 101 = 22750.) sè · câ thº t¤o ÷ñc l : 23625 + 10500 + 22750 = 56875 .Chån ¡p ¡n A dC¥u 37. Trong khæng gian Oxyz, cho ÷íng th¯ng d: 8><>: x= 1 + 3 ty = 1 + 4 tz = 1 . Gåil  ÷íng th¯ng iqua iºm A(1; 1; 1) v  câ v²c-tì ch¿ ph÷ìng #u = ( �2; 1; 2) . ÷íng ph¥n gi¡c cõa gâc nhån t¤o bðid v  câ ph÷ìng tr¼nh l  A 8><>: x= 1 + 27 ty = 1 + tz = 1 + t. B 8><>: x= �18 + 19 ty = �6 + 7 tz = 11 �10t . C 8><>: x= �18 + 19 ty = �6 + 7 tz = �11 �10t. D 8><>: x= 1 �ty = 1 + 17 tz = 1 + 10 t. ÊLíi gi£i.Ph÷ìng tr¼nh tham sè cõa  :8><>: x= 1 �2ty = 1 + tz = 1 + 2 t:D¹ th§y A= d\ . Chån B(� 1; 2; 3) 2 ) AB = 3.Gåi C2dthäa m¢n AB=AC )C�14 5;17 5; 1 ‹ho°c C�� 4 5;� 7 5; 1 ‹.Ta th§y vîi C�� 4 5;� 7 5; 1 ‹th¼ ÕBAC nhån.Trung iºm cõa o¤n BCl I�� 9 10;3 10; 1‹. ÷íng ph¥n gi¡c c¦n t¼m l  I Acâ v²c-tì ch¿ ph÷ìng l # u = (19; 7; �10) .Suy ra ÷íng ph¥n gi¡c câ ph÷ìng tr¼nh 8><>: x= 1 + 19 ty = 1 + 7 tz = 1 �10td¹ th§y ÷íng th¯ng n y tròng vîi ÷íngth¯ng 8><>: x= �18 + 19 ty = �6 + 7 tz = 11 �10t:Chån ¡p ¡n B dC¥u 38. B§t ph÷ìng tr¼nh (3x� 1)( x2+ 3 x� 4) >0câ bao nhi¶u nghi»m nguy¶n nhä hìn6 ? A 9. B 5. C 7. D Væ sè.ÊLíi gi£i.Vîi x > 0, b§t ph÷ìng tr¼nh t÷ìng ÷ìng ¨3x� 1> 0x 2+ 3 x� 4> 0,x > 1:Vîi x < 0, b§t ph÷ìng tr¼nh t÷ìng ÷ìng ¨3x� 1< 0x 2+ 3 x� 4< 0,x < �4:Chån ¡p ¡n D 203/305 203/305 pGV: L¶ Quang Xe  Ô0967.003.131— SÈ 13 NÌI N€O CÂ Þ CH, NÌI  C CON ×ÍNG204dC¥u 39. GåiSl  tªp hñp t§t c£ c¡c gi¡ trà cõa tham sè mº ÷íng th¯ng d:y = x+ m �1c­t ç thà h m sè (C ) : y= x3+ ( m�3)x2+ x+ 1 t¤i ba iºm ph¥n bi»t A(1; yA ), B ,C sao choBC = 2p 3. Têng b¼nh ph÷ìng t§t c£ c¡c ph¦n tû cõa tªp hñp Sl  A 64. B 40. C 52. D 32. ÊLíi gi£i.Ph÷ìng tr¼nh ho nh ë giao iºm cõa ÷íng th¯ng dv  ç thà h m sè (C )l x 3+ ( m�3)x2+ x+ 1 = x+ m �1, x3+ ( m�3)x2+ 2 �m = 0 ,(x � 1)[ x2+ ( m�2)x+ m �2] = 0 :Khi â ta câ –x � 1 = 0x 2+ ( m�2)x+ m �2 = 0 ,–x = 1x 2+ ( m�2)x+ m �2 = 0 ( ):º ÷íng th¯ng dc­t ç thà h m sè (C )t¤i ba iºm ph¥n bi»t A,B ,C n¶n ( ) câ hai nghi»m ph¥nbi»t x1,x2 kh¡c1.Vªy ¨(m �2)2� 4(m �2) >02 m �36= 0 ,8<: (m �2)( m�6) >0m 6= 3 2,8>><>>: –m < 2m > 6m 6= 3 2,–m < 2m > 6:Theo ành l½ Vi-²t, ta câ x1 +x2 =�(m �2) v  x1x2 =m �2.Gåi B(x1;x1 +m �1) v  C(x2;x2 +m �1).N¶n BC2= 2( x2 �x1)2= 12 ,(x1 +x2)2� 4x1x2 = 6,m2� 8m + 6 = 0 ,–m = 4 + p 10m = 4 �p 10:So vîi i·u ki»n th¼ m= 4 �p 10v m= 4 + p 10thäa y¶u c¦u b i to¡n.Vªy têng b¼nh ph÷ìng c¡c ph¦n tû cõa tªp hñp Sl  52.Chån ¡p ¡n C dC¥u 40. Cho h m sè y= f(x ) thäa m¢n f0( x ) = 2018 xln 2018 �cos xv  f(0) = 2 . Ph¡t biºun o sau ¥y óng? A f(x ) = 2018 x+ sin x+ 1 . B f(x ) = 2018x ln 2018+ sinx+ 1 . C f(x ) = 2018x ln 2018�sin x+ 1 . D f(x ) = 2018 x� sin x+ 1 . ÊLíi gi£i.V¼ Z(2018 xln 2018 �cos x) d x= 2018 x� sin x+ C. Do f(0) = 2 n¶nC= 1 .Vªy f(x ) = 2018 x� sin x+ 1 .Chån ¡p ¡n D dC¥u 41. Cho h¼nh l«ng trö tam gi¡c ·u ABC:A0B 0C 0câ AB =a, ÷íng th¯ng AB0t¤o vîim°t ph¯ng (BC C 0B 0) mët gâc 30. T½nh thº t½ch Vcõa khèi l«ng trö ¢ cho. A V= a3p 64. B V= a3p 612. C V= a3 4. D V= 3a 3 4. ÊLíi gi£i.204/305 204/305 pGV: L¶ Quang Xe  Ô0967.003.131PHT TRIšN — THAM KHƒO 2022 NÌI N€O CÂ Þ CH, NÌI  C CON ×ÍNG205GåiMl  trung iºm cõa BC.Düng AM?BC , m°t kh¡c AM?BB 0) AM ?(BC C 0B 0) .Khi â ÖAB 0M = 30 , l¤i câ AM=ap 32) AB 0= AM sinÖAB 0M =AM sin 30=ap 3:) BB 0= p AB02� AB 2= ap 2:Tø â V=SABC BB 0= a3p 64 AB CMA0 B0 C0 Chån ¡p ¡n A dC¥u 42. GåiA,B ,C l¦n l÷ñt l  iºm biºu di¹n cõa c¡c sè phùc z, iz ,2z . Bi¸t di»n t½ch tamgi¡c ABC b¬ng4. Mæ-un cõa sè phùc zb¬ng A p2. B 8. C 2. D 2p 2. ÊLíi gi£i.Gåi z= a+ bi vîi a; b 2R ) A(a ;b), iz =�b+ ai ) B(� b; a ), C (2 a; 2 b).Ta th§y OB?AC n¶n SABC =1 2d(B; AC )AC .Ta câ ph÷ìng tr¼nh ACl �bx +ay = 0 )d(B; AC ) =p a2+ b2= jz j = AC) 1 2jz j  j zj = 4 ) jzj = 2 p 2.Chån ¡p ¡n D dC¥u 43. Câ bao nhi¶u sè thùc thäa m¢n z+ jz j2i � 1� 3 4i= 0 ? A 1. B 3. C 2. D 0.ÊLíi gi£i.°t z= a+ bi (a; b 2R): Thay v o biºu thùc cõa b i to¡n ta câ:( a � 1) + �a2+ b2+ b� 3 4‹i= 0 )8<: a= 1b 2+ b+ 1 4= 0 )8<: a= 1b = �1 2:Vªy ch¿ câ óng mët sè phùc thäa m¢n b i to¡n.Chån ¡p ¡n A dC¥u 44.Cho h m sè bªc bèn f(x ) câ f(0) = 3 2. H m sèy= f0( x )câ ç thà trong h¼nh v³ b¶n. Sè iºm cüc trà cõa h m sèy = j4 f (x + 1) + x2+ 2 xj l  A 3. B 5. C 4. D 6. xyO�2 14�2 ÊLíi gi£i.°t t= x+ 1 , ta x²t h m sè h(t) = 4 f(t) + ( t� 1)2+ 2( t� 1) = 4 f(t) + t2� 1.205/305 205/305 pGV: L¶ Quang Xe  Ô0967.003.131— SÈ 13 NÌI N€O CÂ Þ CH, NÌI  C CON ×ÍNG206Ta câh0( t) = 4 f0( t) + 2 t, f0( t) = 0 ,f0( t) = �t 2.Tr¶n h» tröc tåa ë Oty, v³ ç thà y= f0( t) v  ÷íngth¯ng y= �t 2, quan s¡t ç thà ta suy raf0( t) = �t 2, 264 t= �2t = 0t = 4 :Ta t½nh h(0) = 4 f(0) �1 = 5 . Ta câ b£ng bi¸n thi¶n tyO�2 14�2 B ADCFE 2�1 th0( t) h(t) �1�2 0 4 +1 �0 +0 �0 ++1 +1 h(� 2) h(� 2) 55h(4) h(4) +1 +1 Gåi c¡c iºmA(� 2; 1) ,B (� 2; 0) ,C (4; �2) ,D (4; 0) ,E (2; 0) ,F (2; �1) . Tø h¼nh v³, ta th§y0Z� 2 •f 0( t) � �t 2˜dt < S4OAB = 1, 0Z� 2 h0( t) d t < 4, h(0) �h(� 2) <4, h(2) >1> 0� 4Z0 •f 0( t) � �t 2˜dt > 1 2SC DE F =3 2, � 4Z0 h0( t) d t > 6, h(4) <�1< 0:Nh÷ vªy suy ra h m sè y= jh (t) j câ 5iºm cüc trà.Chån ¡p ¡n B dC¥u 45. Trong khæng gian Oxyz, cho hai iºm A(1; 2; �3) ,M (� 2; �2; 1) v  ÷íng th¯ng dcâ ph÷ìng tr¼nh x+ 1 2=y� 5 2=z �1. Ph÷ìng tr¼nh ÷íng th¯ngd0i qua iºm Mv  vuæng gâcvîi dsao cho kho£ng c¡ch tø iºm A¸n d0nhä nh§t l  A 8><>: x= �2 + ty = �2z = 1 + 2 t. B 8><>: x= �2y = �2 + tz = 1 + 2 t. C 8><>: x= �2 + ty = �2z = 1 + t. D 8><>: x= �2 + ty = �2� tz = 1 . ÊLíi gi£i.Ta câ d0i qua iºm Mv  vuæng gâc vîi d) d0 (P ), vîi (P )l  ph÷ìng tr¼nh m°t ph¯ng i quaiºm Mv  vuæng gâc vîi d.Khi â kho£ng c¡ch tø iºm A¸n d0nhä nh§t khi d0i qua iºm H, vîi Hl  h¼nh chi¸u cõa iºmA l¶n m°t ph¯ng (P ).Ta câ VTPT cõa (P )l  #n P = (2; 2;�1) .Ph÷ìng tr¼nh m°t ph¯ng (P )l  2x + 2 y� z+ 9 = 0 .Gåi l  ph÷ìng tr¼nh ÷íng th¯ng qua A(1; 2; �3) v  vuæng gâc vîi (P ). Ph÷ìng tr¼nh ÷íng th¯ng206/305 206/305 pGV: L¶ Quang Xe  Ô0967.003.131PHT TRIšN — THAM KHƒO 2022 NÌI N€O CÂ Þ CH, NÌI  C CON ×ÍNG207l 8><>: x= 1 + 2 ty = 2 + 2 tz = 1 �t:Ta câ H=  \(P ). X²t ph÷ìng tr¼nh2(1 + 2 t) + 2(2 + 2 t) � (� 3� t) + 9 = 0 ,9t + 18 = 0 ,t= �2:Ta suy ta H(� 3; �2; �1) .Ph÷ìng tr¼nh ÷íng th¯ng d0qua M(� 2; �2; 1) v  câ VTCP #u d= # H M = (1; 0; 2) l 8><>: x= �2 + ty = �2z = 1 + 2 t:Chån ¡p ¡n A dC¥u 46.Cho khèi nân câ ë lîn gâc ð ¿nh l   3. Mët khèi c¦u(S1)nëi ti¸p trong khèi nân. Gåi S2 l  khèi c¦u ti¸p xóc vîi t§tc£ c¡c ÷íng sinh cõa nân v  vîi S1,S3 l  khèi ti¸p xóc vîit§t c£ c¡c ÷íng sinh cõa nân v  vîi S2;: : : ;Sn l  khèi c¦uti¸p xóc vîi t§t c£ c¡c ÷íng sinh cõa nân v  vîi Sn� 1. GåiV 1,V2,V3; : : : ; Vn� 1,Vn l¦n l÷ñt l  thº t½ch cõa khèi c¦uS1,S 2,S3; : : : ; Sn� 1,Sn v Vl  thº t½ch cõa khèi nân. T½nh gi¡trà biºu thùc T= limn! +1 V1 +V2 +   +Vn V: A 79. B 35. C 12. D 613. ÊLíi gi£i.Gåi ÷íng k¼nh ¡y l  a. B¡n k½nh ¡y l  R= a 2. V¼ gâc ð ¿nh l  3n¶n thi¸t di»n qua tröc l  mëttam gi¡c ·u, suy ra l= a.÷íng cao h= p l2� R2= É a2� a2 4=ap 32)a= 2h p3.B¡n k½nh cõa khèi c¦u S1 l R1= h 3, b¡n k½nh cõa khèi c¦uS2 l R2= h� 2R1 3=h 9=h 32 , b¡n k½nhcõa khèi c¦u Sn l  h 3n .Nh÷ vªy ta câV1 +V2 +   +Vn V=4 3�h3 33 + h3 36 +   +h3 33n ‹ 3ha 22 =4h 3 27�1 + 1 27+   + 1 27n� 1‹ hh2 3=4 91� 1 27n 1� 1 27:Suy ra T= limn! +1 V1 +V2 +   +Vn V= limn! +1 4 91� 1 27n 1� 1 27=6 13:Chån ¡p ¡n D 207/305 207/305 pGV: L¶ Quang Xe  Ô0967.003.131— SÈ 13 NÌI N€O CÂ Þ CH, NÌI  C CON ×ÍNG208dC¥u 47. Sè gi¡ trà nguy¶n cõa tham sè mº b§t ph÷ìng tr¼nh ln (x2+ 2 x+ m)> 2 ln(2 x� 1)chùa óng 2sè nguy¶n l  A 4. B 8. C 3. D 9. ÊLíi gi£i.Ta câln�x 2+ 2 x+ m> 2 ln(2 x� 1), 8<: ln�x 2+ 2 x+ m> ln(2 x� 1)2x > 1 2, 8<: x2+ 2 x+ m > (2x� 1)2x > 1 2, 8<: 3x 2� 6x + 1 < m (1)x > 1 2:X²t h m sè y= 3 x2� 6x + 1 < m vîix > 1 2câ b£ng bi¸n thi¶n xy 12 1 2 3 +1 �5 4�2 101 +1 º b§t ph÷ìng tr¼nh(1)câ nghi»m th¼ m >�2, khi â (1)luæn câ mët nghi»m nguy¶n x= 1 .Do â, º (1)câ óng 2nghi»m nguy¶n th¼ 2nghi»m nguy¶n â l  x= 1 v x= 2 . Khi â 2 m 10.Vªy câ 9gi¡ trà nguy¶n d÷ìng cõa tham sè mº b§t ph÷ìng tr¼nh ¢ cho chùa óng 2sè nguy¶n.Chån ¡p ¡n D dC¥u 48. Trong khæng gian vîi h» to¤ ë Oxyz, x²t c¡c iºm A(0; 0; 1) ,B (m ; 0; 0) ,C (0; n; 0) ,D (1; 1; 1) vîim > 0; n > 0v  m+n= 1 . Bi¸t r¬ng khi m; nthay êi, tçn t¤i m°t c¦u cè ànhti¸p xóc vîi m°t ph¯ng (ABC )v  i qua D. T½nh b¡n k½nh m°t c¦u â. A R= 1 . B R= p 22. C R= p 32. D R= 3 2. ÊLíi gi£i.208/305 208/305 pGV: L¶ Quang Xe  Ô0967.003.131MÖC LÖCNÌI N€O CÂ Þ CH, NÌI  C CON ×ÍNG209°tB1(1; 0; 0),C1(0; 1; 0)v E(1; 1; 0) .Ta ÷ñc ¨OB =C C1OC =BB1:Tø â, ta th§y 4OAB =4C C1E) C E =AB .T÷ìng tü, ta ÷ñc AC=E B .Do vªy, 4ABC =4E C B .Gåi Il  trung iºm AE, ta ÷ñc I�1 2;1 2;1 2‹.Ta câ VI ABC =VI E BC) d(I; ( ABC )) = d( I; ( E BC )) =p 32=I D .Vªy m°t c¦u S‚I; p 32Œluæn qua Dv  ti¸p xóc vîi( ABC ). B1 C1 EAO zxy BCChån ¡p ¡n C dC¥u 49.Cho h m sè y= f(x ) câ ç thà nh÷ h¼nh v³ b¶n b¶n. Câ bao nhi¶usè nguy¶n d÷ìng cõa tham sè mº h m sè g(x ) = jf (x + 2018) + mjcâ 7iºm cüc trà ? A 2. B 3. C 4. D 6. O xy�3 �6 2ÊLíi gi£i.V¼ h m f(x ) ¢ cho câ 3iºm cüc trà n¶n f(x + 2018) + mcông luæn câ 3iºm cüc trà (do ph²p tànhti¸n khæng l m £nh h÷ðng ¸n sè cüc trà).Do â y¶u c¦u b i to¡n ,sè giao iºm cõa ç thà f(x + 2018) + mvîi tröc ho nh l  4.º sè giao iºm cõa ç thà f(x + 2018) + mvîi tröc ho nh l  4, ta c¦n çng thíiTành ti¸n ç thà f(x ) xuèng d÷îi nhä hìn 2ìn và )m > �2Tành ti¸n ç thà f(x ) l¶n tr¶n nhä hìn 3ìn và )m < 3.Vªy �2< m < 3) m2 f 1; 2g(do m2Z+).Chån ¡p ¡n A H˜T209/305 209/305 pGV: L¶ Quang Xe  Ô0967.003.131— SÈ 14 NÌI N€O CÂ Þ CH, NÌI  C CON ×ÍNG210BË GIO DÖC & €O T„OTR×ÍNG THPT NGUY™N T‡T TH€NH GV: L– QUANG XE - 0967.003.131 — SÈ 14 PHT TRIšN — THAM KHƒO 2022N‹M HÅC 2021 - 2022Mæn:To¡nThíi gian l m b i: 90 phót — THAM KHƒO HKIIdC¥u 1. Ph¦n thüc v  ph¦n £o cõa sè sè phùc z= �2� 3i l¦n l÷ñt l  A �2; � 3i. B �2; � 3. C �3; � 2. D �3i; 2. ÊLíi gi£i.Sè phùc z= �2� 3i câ ph¦n thüc v  ph¦n £o l¦n l÷ñt l  �2; � 3.Chån ¡p ¡n B dC¥u 2. Trong khæng gian vîi h» tröc tåa ë Oxyz, cho m°t c¦u (S ) : ( x� 3) 2+( y+1) 2+( z+2) 2=8 . Khi â t¥m Iv  b¡n k½nh Rcõa m°t c¦u l  A I(3; �1; �2) ; R = 2 p 2. B I(3; �1; �2) ; R = 4 . C I(� 3; 1; 2) ; R= 2 p 2. D I(� 3; 1; 2) ; R= 4 . ÊLíi gi£i.T¥m Iv  b¡n k½nh Rcõa m°t c¦u l  I(3; �1; �2) ; R = 2 p 2.Chån ¡p ¡n A dC¥u 3. ç thà h m sè y= x3+ 3 x2� 2nhªn : A Tröc tung l m tröc èi xùng. B Gèc tåa ë O l m t¥m èi xùng.C iºmI(� 1; 0) l m t¥m èi xùng. D ÷íng th¯ngx= 1 l m tröc èi xùng. ÊLíi gi£i.Ta câ: y0= 3 x2+ 6 x= 0y 00= 6 x+ 6 = 0 ,x= �1) y= 0 .H m sè y= x3+ 3 x2� 2l  h m a thùc bªc ba n¶n nhªn iºm I(� 1; 0) l m t¥m èi xùng.Chån ¡p ¡n C dC¥u 4. Thº t½ch khèi c¦u ÷íng k½nh 2a b¬ng A 323a3. B 243a3. C 43a3. D 83a3. ÊLíi gi£i.V = 4 3 �2a 2‹3= 4 3a3.Chån ¡p ¡n C dC¥u 5. Cho hai h m sè f(x ) v  g(x ) li¶n töc tr¶n I(vîi Il  kho£ng ho°c o¤n ho°c núakho£ng cõa R). M»nh · n o d÷îi ¥y sai? A Z[f (x ) � g(x )] d x= Zf(x ) d x� Zg(x ) d x.210/305 210/305 pGV: L¶ Quang Xe  Ô0967.003.131PHT TRIšN — THAM KHƒO 2022 NÌI N€O CÂ Þ CH, NÌI  C CON ×ÍNG211B Zf(x ) g (x ) d x= Zf(x ) d xZg(x ) d x. C Zkf(x ) d x= kZf(x ) d xvîi kl  h¬ng sè kh¡c 0. D Z[f (x ) + g(x )] d x= Zf(x ) d x+ Zg(x ) d x. ÊLíi gi£i.Vîi hai h m sè f(x ) v  g(x ) b§t k¼ khæng tçn t¤i ¯ng thùc Zf(x )g (x ) d x= Zf(x ) d xZg(x ) d x.Chån ¡p ¡n B dC¥u 6. Cho h m sè y= f(x ) x¡c ành v  li¶n töc tr¶n R, b£ng x²t d§u cõa f0( x ) nh÷ sau: xf0( x ) �10 1 2 +1 � +0 �0 +Sè iºm cüc trà cõa h m sè ¢ cho l A 0. B 1. C 2. D 3. ÊLíi gi£i.Düa v o b£ng x²t d§u f0( x ) ta th§y f0( x ) câ 3l¦n êi d§u n¶n h m sè f(x ) câ 3iºm cüc trà.Chån ¡p ¡n D dC¥u 7. Gi£i b§t ph÷ìng tr¼nh log3(2x� 3) >2. A 3< x < 6. B 32< x <6. C x >3 2. D x >6. ÊLíi gi£i.log 3(2x� 3) >2, ¨2x � 3> 02 x � 3> 32, 8<: x >3 2x > 6,x > 6.Chån ¡p ¡n D dC¥u 8. Cæng thùc t½nh thº t½ch Vkhèi l«ng trö câ di»n t½ch ¡y Bv  chi·u cao hl  A V=Bh . B V= 1 3Bh. C V= 1 6Bh. D V=Bh . ÊLíi gi£i.Cæng thùc t½nh thº t½ch Vkhèi l«ng trö câ di»n t½ch ¡y Bv  chi·u cao hl  V=Bh .Chån ¡p ¡n A dC¥u 9. T¼m tªp x¡c ành cõa h m sè y= (2 x� 1)�5 3. A D=R. B D=Rn§1 2ª. C D=•1 2; +1‹. D D=�1 2; +1‹. ÊLíi gi£i.Ta câ sè mô �5 3=2 Z n¶n h m sè ¢ cho x¡c ành khi v  ch¿ khi 2x � 1> 0, x > 1 2.Vªy tªp x¡c ành cõa h m sè ¢ cho l  D=�1 2; +1‹.211/305 211/305 pGV: L¶ Quang Xe  Ô0967.003.131— SÈ 14 NÌI N€O CÂ Þ CH, NÌI  C CON ×ÍNG212Chån ¡p ¡n D dC¥u 10. Gi¡ trà n o sau ¥y l  mët nghi»m cõa ph÷ìng tr¼nh log3(2x2+ 1) = 2 ? A x= 2 . B x= 4 . C x= 3 . D x= 1 . ÊLíi gi£i.KX: x2 R.Ta câ log3(2x2+ 1) = 2 ,2x 2+ 1 = 9 ,x2= 4 ,x= 2.Chån ¡p ¡n A dC¥u 11. Bi¸t2Z1 dx 3x + 1 =aln 7 + bln 2 ( a; b2Q). Khi â têng a+ bb¬ng A 13. B 1. C �1 3. D �1. ÊLíi gi£i.�p döng cæng thùc Z1 ax+bdx = 1 alnjax +bj + C:Ta câ 2Z1 dx 3x + 1 =1 3lnj3 x + 1 j 21 =1 3ln 7�2 3ln 2. Do â a= 1 3; b=�2 3)a+ b= �1 3:Chån ¡p ¡n C dC¥u 12. Choz1; z2l  hai sè phùc tòy þ. Kh¯ng ành n o d÷îi ¥ysai? A z z= jz j2. B jz1 +z2j= jz1j+ jz2j. C z1 +z2 = z1 + z2. D jz1 z2j= jz1j  jz2j. ÊLíi gi£i.Kh¯ng ành jz1 +z2j= jz1j+ jz2jsai v¼ jz1 +z2j  jz1j+ jz2j.Chån ¡p ¡n B dC¥u 13. Trong khæng gian Oxyz, m°t ph¯ng (P ) : 3 x� y+ z� 5 = 0 câ mët v²c-tì ph¡ptuy¸n l  A #n = (3; 1; �1) . B #n = (1; �1; 3) . C #n = ( �1; 1; 3) . D #n = (3; �1; 1) . ÊLíi gi£i.M°t ph¯ng (P ) : 3 x� y+ z� 5 = 0 câ mët v²c-tì ph¡p tuy¸n l  #n = (3; �1; 1) .Chån ¡p ¡n D dC¥u 14. Trong khæng gian Oxyz, cho h¼nh b¼nh h nh ABC D, bi¸tA(1; 1; 1) ,B (� 2; 2; 3) ,C (� 5; �2; 2) . Tåa ë iºm Dl  A D(� 2; �3; 0) . B D(2; 3; 4) . C D(� 2; 3; 0) . D D(� 8; �1; 4) . ÊLíi gi£i.ABC D l  h¼nh b¼nh h nh ,# AB =# DC ,8><>: �3 = �5� xD1 = �2� yD2 = 2 �zD ,8><>: xD =�2y D =�3z D = 0:Chån ¡p ¡n A 212/305 212/305 pGV: L¶ Quang Xe  Ô0967.003.131PHT TRIšN — THAM KHƒO 2022 NÌI N€O CÂ Þ CH, NÌI  C CON ×ÍNG213dC¥u 15.Cho bèn sè phùc câ iºm biºu di¹n l¦n l÷ñt l  M,N ,P ,Q nh÷h¼nh v³ b¶n. Sè phùc câ mæ-un lîn nh§t l  sè phùc câ iºm biºudi¹n l  A N. B P. C Q. D M. y�2 23x�3 �1 2O MNPQ 1�2 ÊLíi gi£i.Düa v o h¼nh biºu di¸n suy ra jzM j= p 5;jzN j= p 10;jzP j= p 13;jzQ j= p 8.Vªy sè phùc câ mæ un lîn nh§t l  sè phùc câ iºm biºu di¹n l  iºm P.Chån ¡p ¡n B dC¥u 16. Sè ÷íng ti»m cªn cõa ç thà h m sè y= 1 x� 1l  A 2. B 1. C 3. D 0. ÊLíi gi£i.Tªp x¡c ành: D=Rn f 1g .Ta câ limx ! 1+ y= + 1v  limx ! 1� y= �1 n¶n ÷íng th¯ng x= 1 l  ti»m cªn ùng cõa ç thà h m sè.L¤i câ limx ! +1 y= limx!�1 y= 0 n¶n ÷íng th¯ng y= 0 l  ti»m cªn ngang cõa ç thà h m sè.Vªy ç thà h m sè câ 2÷íng ti»m cªn.Chån ¡p ¡n A dC¥u 17. Choal  sè thüc d÷ìng kh¡c 1. Gi¡ trà cõa biºu thùc loga‚a23p a25p a4 15p a7 Œb¬ng A 125. B 95. C 2. D 3. ÊLíi gi£i.Ta câ loga‚a23p a25p a4 15p a7 Œ= log a‚a2a 2 3a 4 5a7 15Œ= log aa52 15a7 15= logaa 52 15� 7 15= logaa3= 3 .Chån ¡p ¡n D dC¥u 18.H m sè n o d÷îi ¥y câ ç thà nh÷ h¼nh v³ b¶n? A y= x3� 2x 2+ x� 1. B y= x3� x2+ x� 1. C y= x3� x2� 1. D y= x3+ 3 x2+ x� 1. xyOÊLíi gi£i.Ta x²t sü t÷ìng giao giúa ç thà h m sè vîi tröc ho nh.Ì Ph÷ìng tr¼nh x3� 2x 2+ x� 2 = 0 câ óng mët nghi»m n¶n ç thà h m sè y= x3� 2x 2+ x� 2ch¿ c­t tröc ho nh t¤i mët iºm.213/305 213/305 pGV: L¶ Quang Xe  Ô0967.003.131— SÈ 14 NÌI N€O CÂ Þ CH, NÌI  C CON ×ÍNG214ÌPh÷ìng tr¼nh x3� x2+ x� 1 = 0 câ óng mët nghi»m n¶n ç thà h m sè y= x3� x2+ x� 1ch¿ c­t tröc ho nh t¤i mët iºm.Ì Ph÷ìng tr¼nh x3� x2� 1 = 0 câ óng mët nghi»m n¶n ç thà h m sè y= x3� x2� 1ch¿ c­ttröc ho nh t¤i mët iºm.Ì Ph÷ìng tr¼nh x3+ 3 x2+ x� 1 = 0 ,264 x= �1x = �1� p 2x = �1 + p 2n¶n ç thà h m sèy= x3+ 3 x2+ x� 1c­t tröc ho nh t¤i ba iºm.Vªy h m sè y= x3+ 3 x2+ x� 1câ ç thà nh÷ h¼nh v³.Chån ¡p ¡n D dC¥u 19. Trong khæng gian Oxyz, cho ÷íng th¯ng d: 8><>: x= 1 + ty = 2 �4tz = 3 �5t ; t2R. Häi di qua iºmn o d÷îi ¥y? A (3; 6; 8). B (1;�4; �5) . C (� 1; 2; 3) . D (0; 6; 8). ÊLíi gi£i.Chån t= �1 ta câ di qua iºm (0; 6; 8).Chån ¡p ¡n D dC¥u 20. Cho tªp hñp Mcâ20 ph¦n tû. Sè tªp con gçm 5ph¦n tû cõa Ml  A C520 . B 5!. C A520 . D 205. ÊLíi gi£i.Méi tªp con gçm 5ph¦n tû cõa Ml  mët tê hñp chªp 5cõa 20n¶n sè tªp con gçm 5ph¦n tû cõa Ml  C520 .Chån ¡p ¡n A dC¥u 21. Cho h¼nh châp S:ABC Dcâ ¡yABC D l  h¼nh chú nhªt, AB=a, AD = 2a, S Avuæng gâc vîi m°t ph¯ng (ABC D )v  S A =ap 3. T½nh thº t½ch cõa khèi châp S:ABC. A 2a 3p 33. B 2a 3p 3. C a3p 3. D a3p 33. ÊLíi gi£i.Ta câ 4ABC vuæng t¤i Bn¶n SABC =1 2BABC =1 2BAAD =a2.Do â VS:ABC =1 3S ASABC =a3p 33. BCAD S214/305 214/305 pGV: L¶ Quang Xe  Ô0967.003.131PHT TRIšN — THAM KHƒO 2022 NÌI N€O CÂ Þ CH, NÌI  C CON ×ÍNG215Chån ¡p ¡n D dC¥u 22. ¤o h m cõa h m sè y= log8(x 3� 3x � 4) l  A x2� 1 (x 3� 3x � 4) ln 2 . B 3x 3� 3 (x 3� 3x � 4) ln 2 . C 3x 3� 3 x3� 3x � 4. D 1(x 3� 3x � 4) ln 8 . ÊLíi gi£i.Ta câ y0= 3x 2� 3 (x 3� 3x � 4) ln 8 =x2� 1 (x 3� 3x � 4) ln 2 .Chån ¡p ¡n A dC¥u 23. H¼nh b¶n l  ç thà cõa h m sè y= f0( x ).Häi h m sè y= f(x ) çng bi¸n tr¶n kho£ng n o d÷îi ¥y? A (0; 1)v (2; + 1). B (1; 2). C (2; +1). D (0; 1). O xy1 2ÊLíi gi£i.Düa v o ç thà h m sè y= f0( x ), ta th§y f0( x ) > 0khi ch¿ khi x2 (2; + 1).Vªy h m sè y= f(x ) çng bi¸n tr¶n kho£ng (2; +1).Chån ¡p ¡n C dC¥u 24. T½nh t½ch ph¥n 1Z0 (ex+ 1) d x. A e +C, vîi C2R. B 2;718 . C e. D 2e�3. ÊLíi gi£i.1Z0 (ex+ 1) d x= (e x+ x) 10 = (e + 1)�(1 + 0) = e .Chån ¡p ¡n C dC¥u 25. Cho d¢y sè (un)l  mët c§p sè cëng, bi¸t u2 +u21 = 50. T½nh têng cõa 22sè h¤ng¦u ti¶n cõa d¢y. A 550. B 2018. C 1100. D 50. ÊLíi gi£i.Ta câ u1 +u22 =u2 +u21 = 50)S22 = 22 2(u1 +u22 ) = 550.Chån ¡p ¡n A dC¥u 26. T¼m nguy¶n h m cõa h m sè f(x ) = e �x �2 + ex cos2x ‹. A F(x ) = �2 ex + tanx+ C. B F(x ) = 2e x� tan x+ C. C F(x ) = �2 ex �tan x+ C. D F(x ) = 2e �x+ tan x+ C. ÊLíi gi£i.215/305 215/305 pGV: L¶ Quang Xe  Ô0967.003.131— SÈ 14 NÌI N€O CÂ Þ CH, NÌI  C CON ×ÍNG216Tªp x¡c ànhD=Rnn 2+k; k 2Zo.Ta câ Ze�x �2 + ex cos2x ‹dx = Z�2e �x+ 1 cos2x ‹dx = � 2 ex + tanx+ C:Chån ¡p ¡n A dC¥u 27. X¡c ành tåa ë iºm Il  t¥m èi xùng cõa ç thà h m sè y= x� 3 x� 2. A I(3; 2) . B I(2; 1) . C I(2; 3) . D I(1; 2) . ÊLíi gi£i.T¥m èi xùng cõa ç thà h m sè ¢ cho l  giao iºm cõa ti»m cªn ùng v  ti»m cªn ngang.H m sè ¢ cho câ ti»m cªn ùng l  ÷íng th¯ng x= 2 v  ti»m cªn ngang l  ÷íng th¯ng y= 1 .Vªy t¥m èi xùng c¦n t¼m l  I(2; 1) .Chån ¡p ¡n B dC¥u 28. Gi¡ trà nhä nh§t cõa h m sè f(x ) = x2+ x+ 4 x+ 1 tr¶n o¤n[0; 2]b¬ng A 3. B �5. C 4. D 103. ÊLíi gi£i.H m sè li¶n töc v  x¡c ành tr¶n [0; 2].Ta câ f0( x ) = x2+ 2 x� 3 (x + 1) 2;f0( x ) = 0 ,x2+ 2 x� 3 = 0 ,–x = 1 2[0; 2]x = �362 [0; 2] :Ta t½nh ÷ñc f(0) = 4 ; f(1) = 3 ; f(2) = 10 3:Vªy minx 2 [0;2] f(x ) = 3 khix= 1 :Chån ¡p ¡n A dC¥u 29. Trong c¡c h m sè sau, h m sè n o nghàch bi¸n tr¶n R? A y= 1 3x3� x2+ x� 1. B y= 1 3x3+ x� 1. C y= �1 3x3+ x2� x+ 1 . D y= �1 3x3+ 3 x2� 2x + 1 . ÊLíi gi£i.X²t h m sè y= �1 3x3+ x2� x+ 1 .Ta th§y y0= �x2+ 2 x� 1 = �(x � 1)2 0; 8x 2 R.Vªy h m sè y= �1 3x3+ x2� x+ 1 nghàch bi¸n tr¶n R.Chån ¡p ¡n C dC¥u 30. Cholog23 =a;log27 =b. Biºu di¹n log22016theoav  b. A log22016 = 5 + 2a+ b. B log22016 = 5 + 3a+ 2 b. C log22016 = 2 + 2a+ 3 b. D log22016 = 2 + 3a+ 2 b. ÊLíi gi£i.216/305 216/305 pGV: L¶ Quang Xe  Ô0967.003.131PHT TRIšN — THAM KHƒO 2022 NÌI N€O CÂ Þ CH, NÌI  C CON ×ÍNG217V¼log22016 = log2(2 5327) = log225+ log 232+ log 27 = 5 + 2 log23 + log27n¶nlog 22016 = 5 + 2a+ b:Chån ¡p ¡n A dC¥u 31. Cho h¼nh châp S:ABC Dcâ ¡yABC D l  h¼nh b¼nh h nh. Thº t½ch cõa khèi châpS:ABC D b¬ng3a 3. Bi¸t di»n t½ch cõa tam gi¡c S ADb¬ng2a 2. T½nh kho£ng c¡ch htø B¸n m°tph¯ng (S AD ). A h= a. B h= 9a 4. C h= 3a 2. D h= 4a 9. ÊLíi gi£i.Ta câ VB:S AD =1 2VS:ABC D =3a 3 2:M  VB:S AD =1 3d( B; ( S AD ))S4 S AD :Suy ra d(B; ( S AD )) =3VB:S AD S4 S AD =9a 4: AB C DSChån ¡p ¡n B dC¥u 32. Cho2Z1 f(u ) d u= 4 v 4Z1 f(v ) d v= 18 . T½nh t½ch ph¥n I= 4Z2 •f (x ) � 3 x˜dx . A I= 22 �3 ln 2 . B I= 14 + 3 ln 2 . C I= 14 �3 ln 2 . D I= 22 + 3 ln 2 . ÊLíi gi£i.Ta câI= 4Z2 •f (x ) � 3 x˜dx = 4Z2 f(x ) d x� 4Z2 3 xdx= 4Z1 f(x ) d x� 2Z1 f(x ) d x� 3 ln xj42= 18 �4� 3 (ln 4 �ln 2) :Vªy I= 14 �3 ln 2 .Chån ¡p ¡n C dC¥u 33. Trong khæng gian Oxyz, ph÷ìng tr¼nh m°t ph¯ng (P ) i qua iºm M(0; �1; 4) v nhªn #u = (3; 2; 1) ,#v = ( �3; 0; 1) l m v²c-tì ch¿ ph÷ìng l  A x� y� z� 12 = 0 . B x+ y+ z� 3 = 0 . C 3x + 3 y� z= 0 . D x� 3y + 3 z� 15 = 0 . ÊLíi gi£i.Ta câ #n (P ) = [ #u ; #v ] = (2; �6; 6) = 2(1; �3; 3) .L¤i câ (P )i qua M(0; �1; 4) n¶n câ ph÷ìng tr¼nh x� 3(y+ 1) + 3( z� 4) = 0 ,x� 3y + 3 z� 15 = 0 .Chån ¡p ¡n D 217/305 217/305 pGV: L¶ Quang Xe  Ô0967.003.131— SÈ 14 NÌI N€O CÂ Þ CH, NÌI  C CON ×ÍNG218dC¥u 34. Cho hai sè phùc z1; z2thuëc tªp hñpS= z 2 C : iz �2� 3i = 2 v  thäa m¢nz 1 +z2 = 4�2i. T½nh A= jz1j2+ jz2j2. A A= 6 . B A= 14 . C A= 8 . D A= 12 . ÊLíi gi£i.Gåi M; N l¦n l÷ñt l  iºm biºu di¹n cõa z1; z2.Gåi El  trung iºm M N.Ta câ iz �2� 3i = 2, i (z � 3 + 2 i) = 2, z � 3 + 2 i = 2 : (1)Tø (1) ta th§y M; Nthuëc ÷íng trán t¥m I(3; �2) b¡n k½nhR = 2 . xyOINM ETa câE(2; �1) ) # E I = (1; �1) ) E I =p 2) M N = 2p 2.Trong 4OM N , ta câOE2= OM2+ ON 2 2�M N2 4) OM 2+ ON 2= 14 :Chån ¡p ¡n B dC¥u 35. Cho h¼nh châp S:ABC Dcâ ¡yABC D l  h¼nh vuæng c¤nh a, c¤nh S Avuæng gâcvîi m°t ph¯ng ¡y, gâc giúa S Cv  m°t ph¯ng (S AD )b¬ng 30. Thº t½ch cõa khèi châp S:ABC Db¬ng A a3p 23. B a3p 22. C a3p 33. D a3p 32. ÊLíi gi£i.S ABC D =a2.S C \(S AD ) =fS g.V¼ C D ?AD ;C D ?S A )C D ?(S AD )t¤i D.Suy ra S Dl  h¼nh chi¸u cõa S Cl¶n m°t ph¯ng (S AD )) (S C; (S AD )) = ( S C; S D ) =ÕC S D = 30.tan ÕC S D =DC S D)S D = DC tan 30=ap 3.Trong 4S AD :S A 2+ AD 2= S D 2) S A 2= S D 2� AD 2= ( ap 3)2� a2= 2 a2) S A =ap 2.Khi â: V= 1 3SS:ABC D S A =1 3a 2a p 2 =a3p 23. B AC DSChån ¡p ¡n A dC¥u 36. Mët nhâm håc câ 25håc sinh. Gi¡o vi¶n c¦n chån ra mët nhâm v  ch¿ ành mët emtrong nhâm l m nhâm tr÷ðng. Sè håc sinh trong nhâm ph£i lîn hìn 1v  nhä hìn 25. Häi câ baonhi¶u c¡ch lªp nhâm thäa m¢n c¡c y¶u c¦u tr¶n?218/305 218/305 pGV: L¶ Quang Xe  Ô0967.003.131PHT TRIšN — THAM KHƒO 2022 NÌI N€O CÂ Þ CH, NÌI  C CON ×ÍNG219A 419430400. B 419430350. C 201326568. D 201326592. ÊLíi gi£i.Sè c¡ch chån nhâm gçm khåc sinh l  Ck25 k .Theo b i ra sè th nh vi¶n trong nhâm ph£i lîn hìn 1v  nhä hìn 25.Suy ra câ 24Xk =2 Ck25 k = 25 (1 + 1) 24� C125 �25 C 2525 = 419430350c¡ch.Chån ¡p ¡n B dC¥u 37. Trong khæng gian Oxyz, cho ba iºm A(1; 0; 1) ; B(3; 2; �1) ; C (� 3; �2; 3) . ÷íngth¯ng i qua t¥m ÷íng trán ngo¤i ti¸p tam gi¡c ABC v  vuæng gâc vîi m°t ph¯ng (ABC )câph÷ìng tr¼nh A 8><>: x= �4y = 3 + tz = t . B 8><>: x= �8y = 11 + tz = t . C 8><>: x= �2y = 5 + tz = t . D 8><>: x= 2y = tz = 5 �t. ÊLíi gi£i.Tåa ë cõa c¡c v²c-tì # AB; # AC l # AB (2; 2; �2) ;# AC (� 4; �2; 2)Gåi #n l  v²c-tì ph¡p tuy¸n cõa m°t ph¯ng (ABC )) #n = ”# AB; # AC —= (0; 4; 4) .Ph÷ìng tr¼nh m°t ph¯ng (ABC )l  4 (y� 0) + 4 ( z� 1) = 0 ,y+ z� 1 = 0 .Gåi I(a ;b; c) l  t¥m ÷íng trán ngo¤i ti¸p tam gi¡c ABC . Ta câ h» ph÷ìng tr¼nh8><>: I2 (ABC )I A =I BI A =I C ,8><>: b+ c� 1 = 0(1 �a)2+ b2+ (1 �c)2= (3 �a)2+ (2 �b)2+ (1 + c)2(1 �a)2+ b2+ (1 �c)2= (3 + a)2+ (2 + b)2+ (3 �c)2, 8><>: b+ c= 1a + b� c= 32 a + b� c= �5 , 8><>: x= �8y = 6z = �5:) I(� 8; 6; 5) .Ph÷ìng tr¼nh ÷íng th¯ng qua Iv  vuæng gâc vîi (ABC )l 8><>: x= �8y = 6 + tz = �5 + thay 8><>: x= �8y = 11 + tz = t:Chån ¡p ¡n B dC¥u 38. T¼m sè nghi»m nguy¶n cõa b§t ph÷ìng tr¼nh log3x2+ 4 x 2x � 3 <1: A 3. B 4. C 2. D 1. ÊLíi gi£i.Ta câ log3x2+ 4 x 2x � 3 <1, 8>><>>: x2+ 4 x 2x � 3 >0x 2+ 4 x 2x � 3 <3, 8>><>>: x2+ 4 x 2x � 3 >0x 2� 2x + 9 2x � 3 <0, ¨2x � 3< 0x 2+ 4 x < 0, �4< x < 0:Vªy b§t ph÷ìng tr¼nh câ 3nghi»m nguy¶n.Chån ¡p ¡n A 219/305 219/305 pGV: L¶ Quang Xe  Ô0967.003.131— SÈ 14 NÌI N€O CÂ Þ CH, NÌI  C CON ×ÍNG220dC¥u 39. T¼m t§t c£ gi¡ trà thüc cõa tham sè mº ÷íng th¯ng (d ) : y= mx �m �1c­t çthà (C ) : y= x3� 3x 2+ 1 t¤i3iºm A; B; C ph¥n bi»t ( Bthuëc o¤n AC), sao cho tam gi¡c AOCc¥n t¤i O(vîi Ol  gèc to¤ ë). A m=�2. B m= 2 . C m=�1. D m= 1 . ÊLíi gi£i.X²t ph÷ìng tr¼nh ho nh ë giao iºm:x 3� 3x 2+ 1 = mx�m �1, x3� 3x 2+ 2 = m(x � 1) , (x � 1)( x2� 2x � 2� m) = 0º ÷íng th¯ng (d ) : y= mx �m �1c­t ç thà (C ) : y= x3� 3x 2+ 1 t¤i 3 iºm A; B; Cph¥n bi»t th¼ph÷ìng tr¼nh x2� 2x � 2� m = 0 (1) ph£i câ hai nghi»m ph¥n bi»t kh¡c 1, ¨ 0= 1 + 2 + m >0� 3� m 6= 0 ,¨ m > �3m 6= �3 ,m > �3.�p döng ành lþ Viet v o ph÷ìng tr¼nh (1)ta câ: ¨x1 +x2 = 2x 1 x2 =�2� m v Bthuëc o¤n ACn¶n:x B = 1)xA +xC = 2.V¼ tam gi¡c AOCc¥n t¤i On¶n:OA =OC ,x2A +y2A =x2C +y2C ,x2A + (mxA�m �1)2= x2C + (mxC�m �1)2, (x 2A �x2C ) +m(xA �xC ) [m (xA +xC )� 2m �2] = 0, (xA �xC ) [(xA +xC ) +m2(xA +xC )� 2m 2� 2m ] = 0 ,(xA +xC ) +m2(xA +xC )� 2m 2� 2m =0 , 2 + 2 m2� 2m 2� 2m = 0 ,m= 1 .Chån ¡p ¡n D dC¥u 40. Cho hai h m sè F(x ) = ( x2+ ax +b)e �xv  f(x ) = ( �x2+ 3 x+ 6)e �x. T¼m av  bºF (x ) l  mët nguy¶n h m cõa h m sè f(x ). A a= 1 ,b = �7. B a= 1 ,b = 7 . C a= �1, b = 7 . D a= �1, b = �7. ÊLíi gi£i.f (x ) = F0( x ) = (2 x+ a)e �x� (x 2+ ax +b)e �x= ( �x2+ (2 �a)x + ( a� b)e �x, ta c⨠2� a= 3a � b= 6 ,¨a = �1b = �7:Chån ¡p ¡n D dC¥u 41. Cho khèi châp S:ABCcâ ¡y l  tam gi¡c c¥n t¤i A,AB =a, b=120 , ÕS B A =ÕS C A =90 . Gåi l  gâc giúa hai m°t ph¯ng (S AB )v  (S AC ). Khi cos = 3 4th¼ thº t½ch khèi châp ¢cho b¯ng A 3a 3. B a3. C 3a 3 4. D a3 4. ÊLíi gi£i.220/305 220/305 pGV: L¶ Quang Xe  Ô0967.003.131PHT TRIšN — THAM KHƒO 2022 NÌI N€O CÂ Þ CH, NÌI  C CON ×ÍNG221SB A CH KK´S H ?(ABC ); H 2(ABC )suy ra S H?AB v S H ?AC .Khi â ta câ ¨S H ?ABS B ?AB )AB ?(S B H )) AB ?BH .Chùng minh t÷ìng tü ta câ AC?C H suy ra tù gi¡c ABH Cnëi ti¸p trong ÷íng trán ÷íng k½nhAH . Do â gâc BH Cb¬ng60.D¹ th§y 4AH B =4AH C )H B =H C n¶n4H B C ·u.4 ABC c¥n t¤i Acâ AB =a; ÕBAC = 120suy ra BC2= 3 a2.Do â H B2= H C 2= BC 2= 3 a2.D¹ th§y 4S H B =4S H C )S B =S C n¶n 4S AB =4S AC .Trong m°t ph¯ng (S AB )k´ BK ?S A; (K 2S A ).Trong m°t ph¯ng (S AC )k´ C K1?S A; (K12S A ).X²t hai tam gi¡c vuæng 4K AB v 4K1ACcâAB =AC; ÕBAK =ÖC AK 1(v¼4S AB =4S AC ) suyra 4K AB =4K1AC)AK =AK1m Kv  K1n¬m giúaSv  An¶n ÒK =ÒK 1Tø â ta câ C K?S A v BH =C K .Do â cos = cos ÕBK C , BK2+ C K 2� BC 2 2BK C K = 3 4, 2BK 2� BC 2 2BK 2 = 3 4. (1)°t S H=x;(x > 0).X²t 4S H B câS B 2= S H 2+ H B 2= 3 a2+ x2.X²t 4S AB vuæng t¤i Bcâ 1 BK2= 1 BA2+ 1 BS2) 1 BK2= 1 a2 + 1 3a 2+ x2) BK 2= a2(3 a2+ x2) 4a 2+ x2Thay v o (1)ta câ 2a 2(3 a2+ x2) 4a 2+ x2 �3a 2 2a 2(3 a2+ x2) 4a 2+ x2 =3 4,x= ap 3Vªy thº t½ch khèi châp S:ABCl 1 3S H 1 2AB AC sin ÕBAC =1 3a p 31 2a2sin 120 = a3 4Chån ¡p ¡n D dC¥u 42. T¼mcbi¸t a, b, c l  c¡c sè nguy¶n d÷ìng thäa m¢n c= ( a+ bi)3� 107 i A 198. B 6. C 1. D 7. ÊLíi gi£i.Ta câ c= ( a+ bi)3� 107 i= ( a3� 3ab 2) + (3 a2b � b3� 107) i.221/305 221/305 pGV: L¶ Quang Xe  Ô0967.003.131— SÈ 14 NÌI N€O CÂ Þ CH, NÌI  C CON ×ÍNG222Doc2 Z+n¶n ta câ,3a 2b � b3� 107 = 0 ,b(3 a2� b2) = 107 = 1 107 = 107 1, 26664 ¨b= 13 a 2� b2= 107¨b= 1073 a 2� b2= 1 (do 107 l  sè nguy¶n tè ), 2666664 ¨a = 6b = 18<: a= 11450 3b = 107 :V¼ a, b, c l  c¡c sè nguy¶n d÷ìng n¶n a= 6 ,b = 1 .Khi â c= a3� 3ab 2= 198 .Chån ¡p ¡n A dC¥u 43. Häi câ bao nhi¶u sè phùc zthäa çng thíi c¡c i·u ki»n jz � ij = 5 v z2l  sè thu¦n£o? A 2. B 3. C 0. D 4. ÊLíi gi£i.°t z= x+ iy (vîi x, y 2 R).Ta câ jz � ij = 5 ,x2+ ( y� 1)2= 25 :( )z 2l  sè thu¦n £o, suy ra x2� y2= 0 ,–x = yx = �y:Vîi x= ythay v o ( ) ta ÷ñc x2+ ( x� 1)2= 25 ,2x 2� 2x � 24 = 0 ,–x = 4x = �3:Vîi x= �y thay v o ( ) ta ÷ñc x2+ ( x+ 1) 2= 25 ,2x 2+ 2 x� 24 = 0 ,–x = �4x = 3 :Vªy câ 4sè phùc c¦n t¼m l  4 + 4i, � 3� 3i, � 4 + 4 i, 3 � 3i.Chån ¡p ¡n D dC¥u 44.Cho h m sè y= f(x ) câ ç thà nh÷ h¼nh v³ b¶n. Sè iºm cüc trà cõah m sè g(x ) = f(x 2� 2jx j) l  A 5. B 3. C 7. D 2. O xy�1 14ÊLíi gi£i.Ì ç thà cõa h m sè g(x ) c­t tröc tung t¤i iºm A(0; 2) (h¼nh v³).Ì Sè iºm cüc trà cõa g(x ) b¬ng 2n + 1 trong â nl  sè iºm cüc trà d÷ìng cõa h m sè h(x ) =f (x 2� 2x ).222/305 222/305 pGV: L¶ Quang Xe  Ô0967.003.131PHT TRIšN — THAM KHƒO 2022 NÌI N€O CÂ Þ CH, NÌI  C CON ×ÍNG223ÌTa câ h0( x ) = (2 x� 2)f0( x 2� 2x ). Suy ra h0( x ) = 0 ,–x = 1f 0( x 2� 2x ) = 0 :Ì Câ f0( x 2� 2x ) = 0 ,–x 2� 2x = �1x 2� 2x = 1 ,264 x= 1 ( nghi»m k²p - nhªn )x = 1 + p 2 (nghi»m ìn - nhªn )x = 1 �p 2 (nghi»m ìn - lo¤i ):Ì Do â n= 2 v  khi â sè iºm cüc trà cõa g(x ) b¬ng 5.Chån ¡p ¡n A dC¥u 45. Trong khæng gian vîi h» tåa ë Oxyz, cho hai ÷íng th¯ng d1 :x� 2 1=y� 2 2=z + 1 �1 v d2 :x� 1 �1 =y �1 =z 2. Vi¸t ph÷ìng tr¼nh ÷íng ph¥n gi¡c gâc nhån t¤o bðid1 v d2. A d2 :x+ 1 2=y �3 =z 3. B d2 :x� 1 1=y 1=z 1. C d2 :x� 1 2=y 3=z �3. D d2 :x+ 1 1=y 1=z 1. ÊLíi gi£i.Tåa ë giao iºm cõa d1 v d2 l A(1; 0; 0) :Tr¶n d1 l§yMsao cho # AM =�1 p6;2 p6;� 1 p6‹, tr¶n d2 l§yNsao cho # AN =��1 p6;�1 p6;2 p6‹.Khi â ta câ tam gi¡c AM Nc¥n t¤i Av  câ AM=AN = 1.cos ÖM AN =# AM # AN AMAN <0. Do â ph¥n gi¡c ùng vîi gâc nhån câ mët v²c-tì ch¿ ph÷ìng l  #u =# AM �# AN =�2 p6;3 p6;� 3 p6‹. Suy ra (2; 3;�3) l  mët v²c-tì ch¿ ph÷ìng cõa ÷íng ph¥n gi¡c gâcnhån. Vªy ph÷ìng tr¼nh ph¥n gi¡c l  d2 :x� 1 2=y 3=z �3.Chån ¡p ¡n C dC¥u 46. Cho m°t c¦u t¥m O, b¡n k½nh R. H¼nh trö (H )câ b¡n k½nh ¡y l  rnëi ti¸p m°tc¦u. Thº t½ch khèi trö ÷ñc t¤o n¶n bði (H )câ thº t½ch lîn nh§t khi rb¬ng A r= p 3R . B r= p 22R. C r= p 6R . D r= p 63R. ÊLíi gi£i.H¼nh trö nëi ti¸p trong m°t c¦u câ t¥m ¡y l  E, câ b¡n k½nh E A=r(0 <r < R ), ÷íng cao K E= 2E I.X²t tam gi¡c vuæng I E AcâI E =p I A2� E A 2= p R2� r2.Thº t½ch khèi trö l  V=r 2h = r 22I E = 2r2p R2� r2.X²t h m sè y= r2p R2� r2vîi ( 0< r < R ).Câ y0= 2 rp R2� r2+ r2 �2r 2p R2� r2 = 2rp R2� r2� r3 pR2� r2 =2 rR 2� 3r3 pR2� r2 . KIEAy0= 0 ,2rR 3� 3r3= 0 ,r(2 R2� 3r2) = 0 ,r= p 63R.B£ng bi¸n thi¶n223/305 223/305 pGV: L¶ Quang Xe  Ô0967.003.131— SÈ 14 NÌI N€O CÂ Þ CH, NÌI  C CON ×ÍNG224ry0 y 0 p63R R+0 �Nh¼n b£ng bi¸n thi¶n ta th§yy y‚p 63RŒ) ymax =y‚p 63RŒ.D§u b¬ng x£y ra ,r= p 63R. Vªy thº t½ch h¼nh trö lîn nh§t ,ymax ,r= p 63R.Chån ¡p ¡n D dC¥u 47. Bi¸tal  sè thüc d÷ìng b§t k¼ º b§t ph÷ìng tr¼nh ax> 9x + 1 nghi»m óng vîi måix 2 R. M»nh · n o sau ¥y l  óng? A a2 �10 3; 10 4. B a2 �10 2; 10 3. C a2 �0; 10 2. D �10 4; + 1. ÊLíi gi£i.B§t ph÷ìng tr¼nh ax> 9x + 1 óng vîi måi x2 R n¶n óng vîi x= 1 )a> 10.Do a > 1n¶n h m sè y= axçng bi¸n tr¶n Rv  ç thà h m sè y= axcâ b· lãm quay l¶n tr¶n.Hai ç thà h m sè y= axv  y= 9 x+ 1 luæn i qua iºm A(0; 1) n¶n b§t ph÷ìng tr¼nh ax> 9x + 1nghi»m óng vîi måi x2 R khi ÷íng th¯ng y= 9 x+ 1 l  ti¸p tuy¸n cõa ç thà h m sè t¤i iºm A.Ph÷ìng tr¼nh ti¸p cõa ç thà h m sè y= axt¤i Al  y= xln a+ 1 .Suy ra lna= 9 ,a= e 9) a2 �10 3; 10 4.Chån ¡p ¡n A dC¥u 48. Trong khæng gian vîi h» tröc tåa ë Oxyz, cho m°t c¦u (S ) : x2+ y2+ z2+ 4 x�6 y + m = 0 v  ÷íng th¯ng l  giao tuy¸n cõa hai m°t ph¯ng ( ) : x+ 2 y� 2z � 4 = 0 v ( ) : 2 x� 2y � z+ 1 = 0 . ÷íng th¯ng c­t m°t c¦u (S ) t¤i hai iºm ph¥n bi»t A; Bthäa m¢nAB = 8 khi A m= 12 . B m=�12 . C m=�10 . D m= 5 . ÊLíi gi£i.M°t ph¯ng ( ) v  ( ) l¦n l÷ñt câ v²c-tì ph¡p tuy¸n l  #n = (1; 2;�2)v  #n = (2;�2; �1) .Ta t½nh ÷ñc [#n ; #n ] = (�6; �3; �6) . Suy ra v²c-tì ch¿ ph÷ìng cõa l  #u = (2; 1; 2) .Ta câ i qua M(2; 2; 1) n¶n ph÷ìng tr¼nh ch½nh t­c cõa l x � 2 2=y� 2 1=z� 1 2:M°t c¦u (S ) câ t¥m I(� 2; 3; 0) v  b¡n k½nh R= p 13�m; (m < 13).Gåi Kl  h¼nh chi¸u vuæng gâc cõa Itr¶n . IA BKKhi â,K(2 + 2 t; 2 + t; 1 + 2 t) v  # I K = (2 t+ 4; t� 1; 2 t+ 1) ?#u .Suy ra, 2(2t+ 4) + t� 1 + 2(2 t+ 1) = 0 ,9t + 9 = 0 ,t= �1.Nh÷ th¸, I K= 3.Tam gi¡c I AKvuæng t¤i Kn¶n I K2+ K A 2= I A 2, 32+ 4 2= 13 �m , m=�12 .Vªy m=�12 .Chån ¡p ¡n B 224/305 224/305 pGV: L¶ Quang Xe  Ô0967.003.131MÖC LÖCNÌI N€O CÂ Þ CH, NÌI  C CON ×ÍNG225dC¥u 49. Câ bao nhi¶u gi¡ trà nguy¶n cõa tham sè mº ç thà h m sè y= jx 4� 2mx 2+ 2 m2+m �12jcâ 7iºm cüc trà? A 1. B 4. C 0. D 2. ÊLíi gi£i.Nhªn x²t sè iºm cüc trà cõa h m sè y= jx 4� 2mx 2+ 2 m2+ m �12jb¬ng têng sè iºm cüc tràcõa h m sè y= f(x ) = x4� 2mx 2+ 2 m2+ m �12 cëng vîi sè nghi»m ph¥n bi»t cõa ph÷ìng tr¼nhf (x ) = 0 . D¹ th§y h m sè y= f(x ) câ nhi·u nh§t 3iºm cüc trà v  ph÷ìng tr¼nh f(x ) = 0 câ nhi·unh§t 4nghi»m ph¥n bi»t do â sè iºm cüc trà cõa h m sè y= jx 4� 2mx 2+ 2 m2+ m �12jl  7khiv  ch¿ khi h m sè y= f(x ) câ 3iºm cüc trà v  ph÷ìng tr¼nh f(x ) = 0 câ4nghi»m ph¥n bi»t.Ta câ f0( x ) = 4 x3� 4mx )f0( x ) = 0 ,–x = 0x 2= m:H m sè y= f(x ) câ ba iºm cüc trà ,m > 0, khi â f0( x ) = 0 ,–x = 0x = p m:B£ng bi¸n thi¶n cõa h m sè y= f(x ) l  xy0 y �1�p m0 pm +1 �0 +0 �0 +�1�1m2+ m �12 m2+ m �12 2m 2+ m �12 2m 2+ m �12 m2+ m �12 m2+ m �12 +1 +1 Düa v o b£ng bi¸n thi¶n ta th§y ph÷ìng tr¼nhf(x ) = 0 câ4nghi»m ph¥n bi»t khi v  ch¿ khi¨ 2m 2+ m �12 >0m 2+ m �12 <0 ,2664 �1 + p 974< m <3� 4< m < �1� p 974:K¸t hñp vîi m >0) �1 + p 974< m <3) khæng câ gi¡ trà nguy¶n n o cõa mthäa m¢n.Chån ¡p ¡n C H˜T225/305 225/305 pGV: L¶ Quang Xe  Ô0967.003.131— SÈ 15 NÌI N€O CÂ Þ CH, NÌI  C CON ×ÍNG226BË GIO DÖC & €O T„OTR×ÍNG THPT NGUY™N T‡T TH€NH GV: L– QUANG XE - 0967.003.131 — SÈ 15 PHT TRIšN — THAM KHƒO 2022N‹M HÅC 2021 - 2022Mæn:To¡nThíi gian l m b i: 90 phót — THAM KHƒO HKIIdC¥u 1. Sè phùc li¶n hñp cõa sè phùc z= 8 �9i l  A z = 8 �9i. B z = �8 + 9 i. C z = 8 + 9 i. D z = �8� 9i. ÊLíi gi£i.Sè phùc li¶n hñp cõa sè phùc z= 8 �9i l  z = 8 + 9 i.Chån ¡p ¡n C dC¥u 2. Trong khæng gian vîi h» tåa ë Oxyz, cho iºm A(2; �3; 1) . Vi¸t ph÷ìng tr¼nh m°tc¦u t¥m Av  câ b¡n k½nh R= 5 . A (x + 2) 2+ ( y� 3)2+ ( z+ 1) 2= 5 . B (x � 2)2+ ( y+ 3) 2+ ( z� 1)2= 25 . C (x � 2)2+ ( y+ 3) 2+ ( z� 1)2= 5 . D (x + 2) 2+ ( y� 3)2+ ( z+ 1) 2= 25 . ÊLíi gi£i.M°t c¦u t¥m A(2; �3; 1) v  b¡n k½nh R= 5 câ ph÷ìng tr¼nh l ( x � 2)2+ ( y+ 3) 2+ ( z� 1)2= 25 :Chån ¡p ¡n B dC¥u 3. T¥m èi xùng cõa ç thà h m sè y= x� 2 2x � 1l  A I�� 1 2;1 2‹. B I�1 2;1 2‹. C I�1 2;� 1‹. D I�� 1 2; 2 ‹. ÊLíi gi£i.ç thà h m sè y= ax+b cx+dvîiad�bc 6= 0 nhªn iºm I�� d c;a c‹l m t¥m èi xùng. Do â I�1 2;1 2‹l  t¥m èi xùng cõa ç thà h m sè ¢ cho.Chån ¡p ¡n B dC¥u 4. Di»n t½ch cõa m°t c¦u b¡n k½nh 2a l  A 4a 2. B 16a 2. C 16a2. D 4a 2 3. ÊLíi gi£i.Di»n t½ch cõa m°t c¦u b¡n k½nh 2a l  S= 4 R2= 16 a2:Chån ¡p ¡n B dC¥u 5. T¼m hå nguy¶n h m cõa h m sè f(x ) = cos 2 x. A Zcos 2 xdx = 2 sin 2 x+ C. B Zcos 2 xdx = �1 2sin 2x+ C.226/305 226/305 pGV: L¶ Quang Xe  Ô0967.003.131PHT TRIšN — THAM KHƒO 2022 NÌI N€O CÂ Þ CH, NÌI  C CON ×ÍNG227C Zcos 2 xdx = �2 sin 2 x+ C. D Zcos 2 xdx = 1 2sin 2x+ C. ÊLíi gi£i.�p döng cæng thùc Zcos( ax+b) d x= 1 asin(ax+b) + C.Vªy Zcos 2 xdx = 1 2sin 2x+ C.Chån ¡p ¡n D dC¥u 6.Cho h m sè y= f(x ) x¡c ành li¶n töc v li¶n töc tr¶n Rv  câ b£ng bi¸n thi¶n nh÷sau. Kh¯ng ành n o sau ¥y l  kh¯ng ànhóng? A H m sè câ gi¡ trà cüc tiºu b¬ng0. B H m sè ¤t cüc ¤i t¤ix= 0 . C H m sè câ hai cüc trà.D H m sè câ gi¡ trà cüc ¤i b¬ng�4. xy0 y= f(x ) �1�2 0 +1 +0 �0 +�1�1 00�4 �4 +1 +1 ÊLíi gi£i.Düa v o b£ng bi¸n thi¶n, ta th§y h m sè câ hai cüc trà.Chån ¡p ¡n C dC¥u 7. Tªp nghi»m cõa b§t ph÷ìng tr¼nh �1 3‹3x> �1 3‹2x +6l  A (0; 6). B (�1 ; 6). C (0; 64). D (6; +1). ÊLíi gi£i.Ta câ �1 3‹3x> �1 3‹2x +6, 3x < 2x + 6 ,x < 6.Chån ¡p ¡n B dC¥u 8. Cho khèi châp câ ¡y l  h¼nh vuæng c¤nh ap 2v  chi·u cao b¬ng 3a . Thº t½ch Vcõakhèi châp ¢ cho b¬ng A V=a3p 2. B V= 2 a3. C V= 6 a3. D V= a3p 23. ÊLíi gi£i.Thº t½ch khèi châp ¢ cho l  V= 1 3S h=1 3€ap 2Š23a = 2 a3.Chån ¡p ¡n B dC¥u 9. Tªp x¡c ành cõa h m sè y= (3 x� x2)�3 2l  A R. B (0; 3). C (�1 ; 0)[(3; + 1). D Rnf 0; 3 g. ÊLíi gi£i.H m sè y= (3 x� x2)�3 2x¡c ành khi 3x � x2> 0, 0< x < 3.227/305 227/305 pGV: L¶ Quang Xe  Ô0967.003.131— SÈ 15 NÌI N€O CÂ Þ CH, NÌI  C CON ×ÍNG228Vªy tªp x¡c ành cõa h m sè ¢ cho l (0; 3).Chån ¡p ¡n B dC¥u 10. Ph÷ìng tr¼nh 52x � 1= 125 câ nghi»m l  A x= 2 . B x= 1 . C x= 3 . D x= 6 . ÊLíi gi£i.5 2x � 1= 125 ,52x � 1= 5 3, 2x � 1 = 3 ,x= 2 .Chån ¡p ¡n A dC¥u 11. Cho h m sè y= f(x ) li¶n töc tr¶n [a ;b] v  c2 [a ;b]. M»nh · n o sau ¥y óng? A bZa f(x ) d x= aZc f(x ) d x+ bZc f(x ) d x. B bZa f(x ) d x= cZa f(x ) d x+ cZb f(x ) d x. C bZa f(x ) d x= aZc f(x ) d x+ cZb f(x ) d x. D bZa f(x ) d x= cZa f(x ) d x+ cZb f(x ) d x. ÊLíi gi£i.Sû döng t½nh ch§t cì b£n, ta câ bZa f(x ) d x= cZa f(x ) d x+ cZb f(x ) d x.Chån ¡p ¡n D dC¥u 12. T¼m sè phùc zthäa m¢n z+ 2 �3i = 3 �2i. A z= 1 + i. B z= 1 �i. C z= 5 �5i. D z= 1 �5i. ÊLíi gi£i.z + 2 �3i = 3 �2i , z= 1 + i.Chån ¡p ¡n A dC¥u 13. Trong khæng gian tåa ë Oxyz, cho m°t ph¯ng (P )câ ph÷ìng tr¼nh 2x � 4z � 5 = 0 .Mët v²c-tì ph¡p tuy¸n cõa (P )l  A #n = (1; 0; �2) . B #n = (2; �4; �5) . C #n = (0; 2; �4) . D #n = (1; �2; 0) . ÊLíi gi£i.Ta câ (P ) : 2 x� 4z � 5 = 0 )(P ) : x� 2z � 5 2= 0.Mët v²c-tì ph¡p tuy¸n cõa (P )l  #n = (1; 0; �2) .Chån ¡p ¡n A dC¥u 14. Trong khæng gian Oxyz, cho iºm A(3; �1; 1) . H¼nh chi¸u vuæng gâc cõa Atr¶n m°tph¯ng (Oyz )l  iºm A M(0; �1; 0) . B N(3; 0; 0) . C P(0; �1; 1) . D Q(0; 0; 1) . ÊLíi gi£i.H¼nh chi¸u vuæng gâc cõa Atr¶n m°t ph¯ng (Oyz )l  P(0; �1; 1) .Chån ¡p ¡n C 228/305 228/305 pGV: L¶ Quang Xe  Ô0967.003.131PHT TRIšN — THAM KHƒO 2022 NÌI N€O CÂ Þ CH, NÌI  C CON ×ÍNG229dC¥u 15.Cho iºm Ml  iºm biºu di¹n cõa sè phùc z. T¼m ph¦nthüc v  ph¦n £o cõa sè phùc z. A Ph¦n thüc�4 v  ph¦n £o l  3i. B Ph¦n thüc3v  ph¦n £o l  �4. C Ph¦n thüc�4 v  ph¦n £o l  3. D Ph¦n thüc4v  ph¦n £o l  �4i. �4 O x3yMÊLíi gi£i.Ph¦n thüc �4 v  ph¦n £o l  3.Chån ¡p ¡n C dC¥u 16. Ti»m cªn ùng cõa ç thà h m sè y= x+ 1 x+ 2 câ ph÷ìng tr¼nh l  A y= 1 . B x= �2. C x= 1 . D y= �2. ÊLíi gi£i.Ta câ limx !� 2+ y= �1 n¶n ç thà h m sè câ ti»m cªn ùng l  ÷íng th¯ng x= �2.Chån ¡p ¡n B dC¥u 17. Cho c¡c sè d÷ìng a; b; cv a6= 1 . Kh¯ng ành n o sau ¥y óng? A logab+ logac= loga(b + c). B logab+ logac= logajb � cj. C logab+ logac= loga(bc ). D logab+ logac= loga(b � c). ÊLíi gi£i.Theo t½nh ch§t logarit ta câ logab+ logac= loga(bc ).Chån ¡p ¡n C dC¥u 18.Häi av  bthäa m¢n i·u ki»n n o º h m sè y= ax4+ bx2+ c( a 6= 0) câ ç thà d¤ng nh÷ h¼nh v³ b¶n? A a >0;b < 0. B a <0;b > 0. C a >0;b > 0. D a <0;b > 0. xy0ÊLíi gi£i.Tø ç thà ta th§y a >0. Ngo i ra y0= 4 ax3+ 2 bxcâ ba nghi»m ph¥n bi»t )b < 0.Chån ¡p ¡n A dC¥u 19. Trong khæng gian Oxyz, iºm n o d÷îi ¥y thuëc ÷íng th¯ng d: x+ 2 3=y� 1 4=229/305 229/305 pGV: L¶ Quang Xe  Ô0967.003.131— SÈ 15 NÌI N€O CÂ Þ CH, NÌI  C CON ×ÍNG230z+ 1 2? A P(4; 9; 3) . B Q(2; �1; 1) . C N(3; 4; 2) . D M(4; 7; 2) . ÊLíi gi£i.Thay tåa ë iºm P(4; 9; 3) v o ph÷ìng tr¼nh ÷íng th¯ng d: x+ 2 3=y� 1 4=z+ 1 2ta ÷ñc4 + 2 3=9� 1 4=3 + 1 2= 2(óng).Vªy iºm Pthuëc ÷íng th¯ng d.Chån ¡p ¡n A dC¥u 20. Câ bao nhi¶u c¡ch chån 3 håc sinh tø mët nhâm 15 håc sinh? A 153. B 315. C A3515 . D C315 . ÊLíi gi£i.Câ C315 c¡ch chån 3 håc sinh tø mët nhâm 15 håc sinh.Chån ¡p ¡n D dC¥u 21. Cho h¼nh châp S:ABC Dcâ ¡yABC D l  h¼nh chú nhªt, AB=a, AD = 2a, S Avuæng gâc vîi m°t ph¯ng (ABC D ), S A =ap 3. T½nh thº t½ch cõa khèi châp S:ABC D. A a3p 33. B 2a 3p 3. C a3p 3. D 2a 3p 33. ÊLíi gi£i.Gåi Vl  thº t½ch cõa khèi châp S:ABC D, ta câV = 1 3S AAB AD =2p 3a 3 3. ABC DSChån ¡p ¡n D dC¥u 22. T½nh ¤o h m cõa h m sè y= 2018 x. A y0= x2018 x� 1. B y0= 2018 x. C y0= 2018x ln 2018. D y0= 2018 xln 2018 . ÊLíi gi£i.Ta câ y= 2018 x) y0= 2018 xln 2018 .Chån ¡p ¡n D dC¥u 23. Cho h m sè y= f(x ) x¡c ành tr¶n Rn f 2g v  câ b£ng bi¸n thi¶n nh÷ h¼nh d÷îi ¥y.230/305 230/305 pGV: L¶ Quang Xe  Ô0967.003.131PHT TRIšN — THAM KHƒO 2022 NÌI N€O CÂ Þ CH, NÌI  C CON ×ÍNG231xy0 y �12 +1 � �11�1 +1 11H¢y chån m»nh · óng.A f(x ) nghàch bi¸n tr¶n tøng kho£ng (�1 ; 2)v (2; + 1). B f(x ) çng bi¸n tr¶n tøng kho£ng (�1 ; 2)v (2; + 1). C f(x ) nghàch bi¸n tr¶n R. D f(x ) çng bi¸n bi¸n tr¶n R. ÊLíi gi£i.Tø b£ng bi¸n thi¶n, ta th§y h m sè ¢ cho nghàch bi¸n tr¶n tøng kho£ng (�1 ; 2)v (2; + 1).Chån ¡p ¡n A dC¥u 24. Cho h m sè f(x ) li¶n töc tr¶n o¤n [1; 2]v  thäa m¢n 2Z1 f(x ) d x= 3 . T½nh t½ch ph¥nI = 2Z1 2f (x ) d x. A I= 1 . B I= 2 . C I= 5 . D I= 6 . ÊLíi gi£i.Ta câ I= 2Z1 2f (x ) d x= 2 2Z1 f(x ) d x= 2 3 = 6 .Chån ¡p ¡n D dC¥u 25. Cho c§p sè cëng (un)bi¸t u1 =�5, d = 2 . Sè 93l  sè h¤ng thù bao nhi¶u? A 50. B 100. C 44. D 75. ÊLíi gi£i.Ta câ un =u1 + (n� 1)d) 93 = �5 + ( n� 1):2 ) n� 1 = 49 )n= 50 .Chån ¡p ¡n A dC¥u 26. Nguy¶n h m cõa h m sè f(x ) = sin x+ xl  A �cos x+ 1 + C. B cosx+ 1 2x2+ C. C �cos x+ 1 2x2+ C. D cosx+ x2+ C. ÊLíi gi£i.Ta câ Z(sin x+ x)d x= �cos x+ 1 2x2+ C.Chån ¡p ¡n C 231/305 231/305 pGV: L¶ Quang Xe  Ô0967.003.131— SÈ 15 NÌI N€O CÂ Þ CH, NÌI  C CON ×ÍNG232dC¥u 27.Cho h m sè y= f(x ) câ ç thà nh÷ h¼nh b¶n. M»nh · n o d÷îi ¥yóng? A H m sè câ gi¡ trà cüc tiºu b¬ng2. B H m sè câ gi¡ trà lîn nh§t b¬ng2v  gi¡ trà nhä nh§t b¬ng �2. C H m sè ¤t cüc ¤i t¤ix= 0 v  ¤t cüc tiºu t¤i x= 2 . D H m sè câ ba cüc trà. xyO22�2 ÊLíi gi£i.Düa v o ç thà ta câ nhªn x²t: h m sè ¤t cüc ¤i t¤i x= 0 v  ¤t cüc tiºu t¤i x= 2 .Chån ¡p ¡n C dC¥u 28. Cho h m sè y= x4� 4x 2+ 3 . Gåi Mv ml¦n l÷ñt l  gi¡ trà lîn nh§t v  gi¡ trà nhänh§t cõa h m sè ¢ cho tr¶n o¤n [� 1; 2] . Gi¡ trà cõa M+m l  A 2. B �1. C 0. D 3. ÊLíi gi£i.Ta câ y0= 4 x3� 8x ) y0= 0 ,264 x= 0 2[� 1; 2]x = �p 2=2 [� 1; 2]x = p 22 [� 1; 2] :y (� 1) = 0 ,y(2) = 3 ,y(0) = 3 ,y(p 2) =�1.Do â M= max[� 1;2] y= 3 ; m = min[� 1;2] y= �1. Suy ra M+m = 2 .Chån ¡p ¡n A dC¥u 29. C¡c kho£ng nghàch bi¸n cõa h m sè y= 2x + 1 x� 1 l  A (�1 ; 1)v (1; + 1). B (�1 ; +1)n f 1g . C (2; +1). D (�1 ; 2). ÊLíi gi£i.Tªp x¡c ành D=Rn f 1g .f 0( x ) = � 3 (x � 1)2<0; 8x 2 D n¶n h m sè nghàch bi¸n tr¶n c¡c kho£ng (�1 ; 1)v (1; + 1).Chån ¡p ¡n A dC¥u 30. Chologax= �1 v  logay= 4 . Gi¡ trà cõa biºu thùc loga(x 2y 3) b¬ng A 3. B 10. C �14 . D 65. ÊLíi gi£i.Ta câ loga(x 2y 3) = log ax2+ log ay3= 2 log ax+ 3 logay= �2 + 12 = 10 .Chån ¡p ¡n B dC¥u 31. Cho tù di»n ·u S:ABCcâ c¤nh b¬ng a. M l  mët iºm b§t k¼ b¶n trong tù di»n.Têng kho£ng c¡ch tø Mtîi c¡c m°t cõa khèi tù di»n l 232/305 232/305 pGV: L¶ Quang Xe  Ô0967.003.131PHT TRIšN — THAM KHƒO 2022 NÌI N€O CÂ Þ CH, NÌI  C CON ×ÍNG233A Mët ¤i l÷ñng phö thuëc và tr½ cõaM. B aÉ 23. C ap2. D ap3. ÊLíi gi£i.Do S:ABC l  tù di»n ·u câ c¤nh b¬ng an¶n ta câS ABC =SS AB =SS AC =SS BC =a2p 34:Gåi Gl  trång t¥m tam gi¡c ABC.Ta câ: AG=ap 33v S G =p S A2� AG 2= Ê a2� a2 3=aÉ 23: SMBA CGDoMn¬m trong tù di»n n¶n ta câVM:ABC +VM:S AC +VM:S AB +VM:S BC =VS:ABC, 1 3SABC (d( M; (ABC )) + d( M;(S AC )) + d( M;(S AB )) + d( M;(S B C ))) = 1 3S G SABC, d(M; (ABC )) + d( M;(S AC )) + d( M;(S AB )) + d( M;(S B C )) =S G=aÉ 23:Chån ¡p ¡n B dC¥u 32. ChoZ10 f(x ) d x= �1 v  Z10 g(x ) d x= 1 . Khi â Z10 [f (x ) � 7g (x )] d xb¬ng A �8. B 6. C �6. D 8. ÊLíi gi£i.Ta câ Z10 [f (x ) � 7g (x )] d x= Z10 f(x ) d x� 7Z10 g(x ) d x= �8.Chån ¡p ¡n A dC¥u 33. Trong khæng gian Oxyz cho hai iºm A(1; �3; 2) v B(3; 1; 4) . Khi â, m°t ph¯ngtrung trüc cõa o¤n th¯ng ABcâ ph÷ìng tr¼nh l  A x� 2y + z� 7 = 0 . B 2x � y+ 3 z� 4 = 0 . C 2x + 4 y+ 2 z� 3 = 0 . D x+ 2 y+ z� 3 = 0 . ÊLíi gi£i.Ta câ ¨M (2; �1; 3) l  trung iºm c£ o¤n th¯ng AB# AB = (2; 4; 2) :M°t ph¯ng trung trüc cõa o¤n th¯ng ABcâ ph÷ìng tr¼nh l  x+ 2 y+ z� 3 = 0 .Chån ¡p ¡n D dC¥u 34. Cho sè phùc zcâ mæun b¬ng 2018v wl  sè phùc thäa m¢n biºu thùc 1 z+1 w=233/305 233/305 pGV: L¶ Quang Xe  Ô0967.003.131— SÈ 15 NÌI N€O CÂ Þ CH, NÌI  C CON ×ÍNG2341z+ w. Mæun cõa sè phùcwb¬ng A 2018. B 2019. C 2017. D p2019. ÊLíi gi£i.Tø gi£ thi¸t ta câ 1 z+1 w=1 z+ w )(z + w)2� zw zw(z + w) = 0, suy ra �z + 1 2w ‹2= ‚� ip 3w 2Œ2.Khi â z= ‚� 1 2�ip 32Œw ho°c z= ‚� 1 2+ip 32Œw ) j wj= � 2018 É14+3 4= 2018.Chån ¡p ¡n A dC¥u 35. Cho h¼nh hëp chú nhªt ABC D:A0B 0C 0D 0câ ¡y ABC D l  h¼nh vuæng c¤nh ap 2,AA 0= 2 a. T½nh kho£ng c¡ch giúa hai ÷íng th¯ng BDv C D 0. A 2a . B ap 2. C ap 55. D 2a p 55. ÊLíi gi£i.Ta câ BDB0D 0, B 0D 0 (C D 0B 0) ) BD (C D 0B 0)) d(C D 0; BD ) = d ( D;(C D 0B 0)) .Gåi I= DC 0\ D0C ) I= DC 0\ (C D 0B 0) m  Il  trungiºm cõa DC0) d (D; (C D 0B 0)) = d ( C0; (C D 0B 0)) .V¼ A0B 0C 0D 0l  h¼nh vuæng t¥m O0c¤nh ap 2.) C0O 0= a) C O 0= p C C02+ C0O 02= ap 5.Ta câ di»n t½ch S4 C B 0D 0= 1 2C O0 B 0D 0= 1 2ap 52 a = a2p 5.Ta câ VC0:C D 0B 0= VC:C 0B 0D 0= 1 6C C0 C 0B 0 C 0D 0= 2 3a3.) d (C0; (C B 0D 0)) = 3VC0:C B 0D 0 S4 C B 0D 0 = 32 3a3 a2p 5=2a p 55. AA0 BCB0 C0 DD0 IO0 Chån ¡p ¡n D dC¥u 36. T½nh têng cõa t§t c£ c¡c sè câ 5chú sè æi mët kh¡c nhau ÷ñc lªp th nh tø tªpA = f1; 2; 3; 4; 5 g. A 333330. B 7999920. C 1599984. D 3999960. ÊLíi gi£i.Sè c¡c sè tü nhi¶n câ 5chú sè kh¡c nhau l§y tø tªp Al  5! = 120 sè.Trong 120sè t¼m ÷ñc, luæn chån ÷ñc mët c°p sè (x ;y ) sao cho x+ y= 66666 . Vªy têng cõa 120sèt¼m ÷ñc l  6066666 = 3999960 .Chån ¡p ¡n D dC¥u 37. Trong khæng gian vîi h» tåa ë Oxyz, cho2÷íng th¯ng d1: x� 2 1=y� 2 2=z+ 1 �1 ,d 2: x� 1 �1 =y �1 =z 2. Vi¸t ph÷ìng tr¼nh ÷íng ph¥n gi¡c gâc nhån t¤o bðid1,d2. A x� 1 2=y 3=z �3. B x� 1 2=y 1=z 1. C x+ 1 2=y 3=z 3. D x� 1 2=y 1=z �1. ÊLíi gi£i.234/305 234/305 pGV: L¶ Quang Xe  Ô0967.003.131PHT TRIšN — THAM KHƒO 2022 NÌI N€O CÂ Þ CH, NÌI  C CON ×ÍNG235GåiMl  giao iºm cõa hai ÷íng th¯ng d1 v d2. Khi âM(1; 0; 0) .Gåi #u 1, #u 2 l¦n l÷ñt l  v²c-tì ch¿ ph÷ìng cõa c¡c ÷íng th¯ngd1 v d2. Khi â #u 1 = (1; 2;�1) ,# u 2= (�1; �1; 2) . Suy ra j#u 1j= j#u 2j= p 6v  cos( #u 1; #u 2) = #u 1#u 2 j#u 1j  j #u 2j =�5 6<0.V¼ gâc giúa hai v²c-tì #u 1 v  #u 2 l  gâc tò, n¶n mët v²c-tì ch¿ ph÷ìng cõa ÷íng ph¥n gi¡c gâc nhånt¤o bði d1 v d2 l  #u = #u 1 � #u 2 = (2; 3;�3) .Vªy ph÷ìng tr¼nh ph¥n gi¡c c¦n t¼m l  x� 1 2=y 3=z �3.Chån ¡p ¡n A dC¥u 38. Häi câ bao nhi¶u gi¡ trà nguy¶n mº ph÷ìng tr¼nh 9x� (m + 1)3 x+ 2 m�2 = 0 câhai nghi»m ph¥n bi»t x1; x2thäa m¢n(x1 + 1)(x2 + 1)3? A 0. B 1. C 2. D 3. ÊLíi gi£i.Ta th§y9x� (m + 1)3 x+ 2 m�2 = 0 (1), –3x= 23 x= m �1:Ph÷ìng tr¼nh (1)câ hai nghi»m x1; x2, ¨m > 1m 6= 3 : (2)Ta th§y(x1 + 1)(x2 + 1)3) [log3(m �1) + 1] (log32 + 1)3, log36log3[3(m �1)] log327, log3[3(m �1)] log627) log3[3(m �1)] <2) 3(m �1) <9(2)) ¨1 < m < 4m 6= 3) m= 2 :Chån ¡p ¡n B dC¥u 39.235/305 235/305 pGV: L¶ Quang Xe  Ô0967.003.131— SÈ 15 NÌI N€O CÂ Þ CH, NÌI  C CON ×ÍNG236Cho h m sèf(x ) = x3+ ax 2+ bx +ccâ ç thà h m sè nh÷ h¼nhb¶n. H m sè g(x ) = f(� x2+ 3 x) çng bi¸n tr¶n kho£ng n o? A (0; 1). B (1; 2). C (4; +1). D (�1 ; 0). xyO�2 �2 2ÊLíi gi£i.Ta câ g0( x ) = ( �2x + 3) f 0( � x2+ 3 x).Suy ra g0( x ) = 0 ,–� 2x + 3 = 0f 0( � x2+ 3 x) = 0 ,2664 x= 3 2� x2+ 3 x= �2� x2+ 3 x= 0 ,266666664 x= 3 2x = 3 p 172x = 0x = 3 :B£ng bi¸n thi¶n xg0 g �1 3� p 172 0 1;5 3 3 +p 172 +1 +0 �0 +0 �0 +0 �Düa v o b£ng bi¸n thi¶n suy ra h m sè çng bi¸n tr¶n(0; 1).Chån ¡p ¡n A dC¥u 40. H m sè n o d÷îi ¥y khængl  nguy¶n h m cõa h m sè y= x(2 + x) (x + 1) 2? A y= x2+ x� 1 x+ 1 . B y= x2� x� 1 x+ 1 . C y= x2+ x+ 1 x+ 1 . D y= x2 x+ 1 . ÊLíi gi£i.Ta câ bi¸n êi: x(2 + x) (x + 1) 2= x2+ 2 x (x + 1) 2= 1� 1 (x + 1) 2.Suy ra, Zx(2 + x) (x + 1) 2dx = Z‚1� 1 (x + 1) 2Œdx = x+ 1 x+ 1 +C = x2+ ( C+ 1) x+ C + 1 x+ 1 .Ì Vîi C=�2, ta ÷ñc y= x2� x� 1 x+ 1 ;Ì Vîi C= 0 , ta ÷ñc y= x2+ x+ 1 x+ 1 ;236/305 236/305 pGV: L¶ Quang Xe  Ô0967.003.131PHT TRIšN — THAM KHƒO 2022 NÌI N€O CÂ Þ CH, NÌI  C CON ×ÍNG237ÌVîi C=�1, ta ÷ñc y= x2 x+ 1 .Vªy h m sè khæng ph£i l  nguy¶n h m cõa h m sè ¢ cho l  y= x2+ x� 1 x+ 1 .Chån ¡p ¡n A dC¥u 41. Cho khèi châp S:ABCcâ ¡yABCl  tam gi¡c vuæng c¥n t¤i B,S A ?(ABC ),AC = 3ap 2, S B = 2ap 3. T½nh thº t½ch Vcõa khèi châp S:ABC. A V= 3a 3p 32. B V= 3a 3p 212. C V= a3p 212. D V= a3p 32. ÊLíi gi£i.Tam gi¡c ABCvuæng c¥n t¤i B,AC = 3ap 2) BA =BC = 3a.Di»n t½ch tam gi¡c ABCl SABC =1 2BA BC =9a 2 2.4 S AB vuæng t¤i A) S A =p S B2� AB 2= ap 3.Thº t½ch Vcõa khèi châp S:ABCl V = 1 3S A SABC =1 3a p 39a 2 2=3p 3a 3 2. AB CSChån ¡p ¡n A dC¥u 42. Cho tªpX=f1; 3; 5; 7; 9 g. Câ bao nhi¶u sè phùc z= x+ yi câ ph¦n thüc, ph¦n £o·u thuëc Xv  câ têng x+ y 10? A 20. B 10. C 15. D 24. ÊLíi gi£i.X²t sè phùc z= x+ yi (x; y 2X).V¼ sè phùc z= x+ yi thäa m¢n x+ y 10 n¶n ta x²t c¡c tr÷íng hñp saua) (x ;y ) 2 f (1; 3) ;(1; 5) ;(1; 7) ;(1; 9) ;(3; 5) ;(3; 7) g, câ 2 6 = 12 sè phùc thäa m¢n.b) (x ;y ) 2 f (1; 1) ;(3; 3) ;(5; 5) g, câ 3sè phùc thäa m¢n.Vªy câ 12 + 3 = 15 sè phùc thäa m¢n · b i.Chån ¡p ¡n C dC¥u 43. Cho sè phùc zcâ mæun b¬ng 2018v wl  sè phùc thäa m¢n biºu thùc 1 z+1 w=1 z+ w. Mæun cõa sè phùcwb¬ng A 2018. B 2019. C 2017. D p2019. ÊLíi gi£i.Tø gi£ thi¸t ta câ 1 z+1 w=1 z+ w )(z + w)2� zw zw(z + w) = 0, suy ra �z + 1 2w ‹2= ‚� ip 3w 2Œ2.Khi â z= ‚� 1 2�ip 32Œw ho°c z= ‚� 1 2+ip 32Œw ) j wj= � 2018 É14+3 4= 2018.Chån ¡p ¡n A 237/305 237/305 pGV: L¶ Quang Xe  Ô0967.003.131— SÈ 15 NÌI N€O CÂ Þ CH, NÌI  C CON ×ÍNG238dC¥u 44. Cho h m sè f(x ), b£ng bi¸n thi¶n cõa h m sè f0( x ) nh÷ sau: xf0( x ) f(x ) �1�3 1 3 +1 �0 +0 �0 ++1 +1 �3 �3 33�2 �2 +1 +1 Sè iºm cüc trà cõa h m sèy= f(6 �3x ) l  A 1. B 2. C 3. D 4. ÊLíi gi£i.Ta câ y0= �3f 0(6 �3x ). Cho y0= 0 ,264 6� 3x = �36 � 3x = 16 � 3x = 3 ,2664 x= 3x = 5 3x = 1 :B£ng bi¸n thi¶n xf0( x ) �11 53 3 +1 �0 +0 �0 +Nhªn x²t:y0êi d§u 3 l¦n khi i qua c¡c nghi»m n¶n ph÷ìng tr¼nh y0= 0 câ 3 nghi»m ph¥n bi»t. Vªyh m sè y= f(6 �3x ) câ 3 cüc trà.Chån ¡p ¡n C dC¥u 45. Trong khæng gian vîi h» tröc tåa ë Oxyz, cho ÷íng th¯ng d: 8><>: x= 0y = tz = 2 �t, bi¸t r¬ngd v  tröc Oxch²o nhau. Lªp ph÷ìng tr¼nh ÷íng vuæng gâc chung cõa dv  tröc Ox. A 8><>: x= 1y = tz = t. B 8><>: x= 0y = 2 tz = t. C 8><>: x= 0y = 2 �tz = t . D 8><>: x= 0y = tz = t. ÊLíi gi£i.Gåi H K l  o¤n vuæng gâc chung cõa dv  tröc Ox(H 2d; K 2Ox ).Khi â H(0; t; 2 �t) ; K (t0; 0; 0) )# H K = (t0; � t; � 2 + t) .Ta câ ¨# H K #u d= 0# H K #i = 0 ,¨0t0+ 1 ( �t) � 1 ( �2 + t) = 0t 0 1 + 0 ( �t) + 0 ( �2 + t) = 0 ,¨t= 1 )H(0; 1; 1)t 0= 0 )K(0; 0; 0) O.Ta suy ra # H K = (0; �1; �1) .÷íng th¯ng (H K )qua Ov  câ VTCP # H K = (0; �1; �1) . Vªy (H K ) :8><>: x= 0y = tz = t.Chån ¡p ¡n D 238/305 238/305 pGV: L¶ Quang Xe  Ô0967.003.131PHT TRIšN — THAM KHƒO 2022 NÌI N€O CÂ Þ CH, NÌI  C CON ×ÍNG239dC¥u 46.Cho mët h¼nh trö trán xoay v  h¼nh vuæng ABC Dc¤nhacâ 2 ¿nh li¶nti¸p A; B n¬m tr¶n ÷íng trán ¡y thù nh§t cõa h¼nh trö, hai ¿nh cán l¤in¬m tr¶n ÷íng trán ¡y thù hai cõa h¼nh trö. M°t ph¯ng (ABC D )t¤ovîi ¡y cõa h¼nh trö gâc 45. T½nh thº t½ch khèi trö. A 3a 3 16. B p2a 3 16. C a3 16. D 3p 2a 3 16. DCNABM IOO0 ÊLíi gi£i. Gåi M; N l¦n l÷ñt l  trung iºm cõa ABv C D ,O v  O0l¦n l÷ñt l  t¥m hai m°t ¡y. Gåi Il  giaoiºm cõa M Nv OO 0. Gâc giúa m°t ph¯ng (ABC D )v  m°t ¡y l  gâc (M N; OM ) =ÕI M O . Do â ÕI M O = 45. Suy ra4 I M O vuæng c¥n t¤i O. Ta câ M N=BC =an¶n I M =a 2v AM =1 2AB=a 2. Suy raOM=OI = a 2p 2)OO 0=2 OI =a p2.4 OM A vuæng t¤i Mn¶n OA2= OM 2+ AM 2= a2 8+a2 4=3a 2 8. Suy raR= OA =ap 32p 2. Ta câ Vtrö =R 2h = 3a 2 8a p2=3p 2a 3 16.Chån ¡p ¡n D dC¥u 47. Gåial  sè thüc lîn nh§t º b§t ph÷ìng tr¼nh x2� x+ 2 + aln ( x2� x+ 1) >0nghi»m óng vîi måi x2 R. M»nh · n o sau ¥y óng? A a2 (2; 3] . B a2 (8; + 1). C a2 (6; 7] . D a2 (� 6; �5]. ÊLíi gi£i.°t t= x2� x+ 1 = �x � 1 2‹2+ 3 4suy rat> 3 4.B§t ph÷ìng tr¼nh ¢ cho trð th nhx2� x+ 2 + aln �x 2� x+ 1 > 0, t+ aln t+ 1 >0, aln t> �t� 1:Ì Tr÷íng hñp 1: t= 1 khi â aln 1 >�1� 1luæn óng vîi måi a.Ì Tr÷íng hñp 2: 3 46t < 1.Ta câ aln t> �t� 1;8 t 2 •3 4; 1 ‹, a6 �t� 1 lnt ;8 t 2 •3 4; 1 ‹.X²t h m sè f(t) = �t� 1 lnt )f0( t) = �lnt� 1� 1 tln2t >0;8 t 2 •3 4; 1 ‹.Do â a6 �t� 1 lnt ;8 t 2 •3 4; 1 ‹, a6 �7 4 ln3 4.239/305 239/305 pGV: L¶ Quang Xe  Ô0967.003.131— SÈ 15 NÌI N€O CÂ Þ CH, NÌI  C CON ×ÍNG240ÌTr÷íng hñp 3: t >1.Ta câ aln t> �t� 1;8 t 2 (1; + 1), a> �t� 1 lnt ;8 t 2 (1; + 1).X²t h m sè f(t) = �t� 1 lnt )f0( t) = �lnt� 1� 1 tln2t ;8 t 2 (1; + 1).X²t h m sè g(t) = ln t� 1� 1 t,g0( t) = 1 t+1 t2 >0n¶n h m sè çng bi¸n.Suy ra ph÷ìng tr¼nh g(t) = 0 câ tèi a mët nghi»m.V¼ g(1) = �2; limt! +1 g(t) = + 1n¶n g(t) = 0 câ duy nh§t mët nghi»m tr¶n (1; +1).Do â f0( t) = 0 câ duy nh§t mët nghi»m l  t0 . Khi âlnt0 = t0 + 1 t0 suy raf(t0 ) =�t0 . B£ngbi¸n thi¶n tf0( t) f(t) 1 t0 +1 +0 ��1�1 �t0 �t0 �1�1Suy raa> �t� 1 lnt ;8 t 2 (1; + 1), a> �t0 .Do â �t0 6a6 �7 4 ln3 46;08 .Vªy sè thüc athäa m¢n y¶u c¦u b i to¡n l : a2 (6; 7] .Chån ¡p ¡n C dC¥u 48. Trong khæng gian Oxyz, cho ÷íng th¯ng d: x+ 3 2=y+ 1 1=z� 3 1v  m°t ph¯ng( P ) : x+ 2 y� z+ 5 = 0 . GåiAl  giao iºm cõa ÷íng th¯ng dv  m°t ph¯ng (P ); B l  iºmthuëc dcâ ho nh ë d÷ìng v  AB=p 6. Gåi C(x ;y ;z ) l  iºm thuëc m°t ph¯ng (P ) sao choAC =3p 22v ÕABC = 60. T½nh gi¡ trà S= x+ y+ z. A S= 0 . B S= 7 . C S= 5 . D S= 6 . ÊLíi gi£i.V¼ Al  giao iºm cõa ÷íng th¯ng dv  m°t ph¯ng (P )n¶n A(� 3 + 2 a;� 1 + a; 3 + a).Tåa ë Athäa m¢n ph÷ìng tr¼nh (� 3 + 2 a) + 2( �1 + a) � (3 + a) + 5 = 0 ,a= 1 . Vªy A(� 1; 0; 4) .Gåi B(� 3 + 2 t; � 1 + t; 3 + t) 2 d; �t > 3 2‹.Ta câ AB=p 6, (� 2 + 2 t) 2+ ( �1 + t) 2+ ( �1 + t) 2= 6 ,–t = 0 (lo¤i)t = 2 (nhªn).Suy ra B(1; 1; 5) .X²t 4ABC ta câ AC sinB=AB sinC,sin C= ABsin B AC=p 6sin 60  3p 22= 1)ÕAC B = 90.Do â Cl  h¼nh chi¸u vuæng gâc cõa Bl¶n (P ).BC quaB(1; 1; 5) , câ v²c-tì ch¿ ph÷ìng #u = #n P = (1; 2;�1) , câ ph÷ìng tr¼nh 8><>: x= 1 + t0y = 1 + 2 t0z = 5 �t0 .240/305 240/305 pGV: L¶ Quang Xe  Ô0967.003.131MÖC LÖCNÌI N€O CÂ Þ CH, NÌI  C CON ×ÍNG241X²t ph÷ìng tr¼nh(1 +t0) + 2(1 + 2 t0) � (5 �t0) + 5 = 0 ,t0= �1 2.Suy ra C�1 2; 0;11 2‹, do â x+ y+ z= 1 2+ 0 +11 2= 6.Chån ¡p ¡n D dC¥u 49. Câ bao nhi¶u gi¡ trà nguy¶n d÷ìng cõa tham sè mº h m sè y= j3 x 4� 4x 3� 12 x2+ m jcâ 5iºm cüc trà? A 27. B 16. C 26. D 44. ÊLíi gi£i.°t f(x ) = 3 x4� 4x 3� 12x2+ m th¼ y= jf (x )j = p f2(x ).Ta câ y0= f0( x ) f (x ) jf (x )j . M f0( x ) = 12 x3� 12x2� 24x; f 0( x ) = 0 ,264 x= 0x = �1x = 2 :Ta câ y0êi d§u qua c¡c nghi»m cõa f0( x ) = 0 v  khæng x¡c ành t¤i c¡c iºm l m f(x ) = 0 .Vªy sè iºm cüc trà cõa h m sè ythuëc tªp nghi»m cõa f0( x ) = 0 ; f(x ) = 0 .Ta lªp b£ng bi¸n thi¶n cõa h m sè f(x ) = 3 x4� 4x 3� 12x2+ m. xf0( x ) f(x ) �1�1 0 2 +1 �0 +0 �0 ++1 +1 �5 + m �5 + m mm�32 + m �32 + m +1 +1 º h m sèycâ 5iºm cüc trà câ c¡c kh£ n«ng sauÌ f(x ) = 0 câ hai nghi»m kh¡c 0;� 1;2 . Khi â 264 m <0¨� 32 + m <0� 5 + m > 0,–m < 05 < m < 32:Ì f(x ) = 0 câ ba nghi»m trong â câ mët nghi»m l  �1;0 ho°c 2. Khi â ta câ m= 0 ho°c m= 5 .Nh÷ng v¼ mnguy¶n d÷ìng n¶n m2 f 5; 6; 7; : : :; 31 g. Vªy câ t§t c£ 27gi¡ trà thäa m¢n.Chån ¡p ¡n A H˜T241/305 241/305 pGV: L¶ Quang Xe  Ô0967.003.131— SÈ 16 NÌI N€O CÂ Þ CH, NÌI  C CON ×ÍNG242BË GIO DÖC & €O T„OTR×ÍNG THPT NGUY™N T‡T TH€NH GV: L– QUANG XE - 0967.003.131 — SÈ 16 PHT TRIšN — THAM KHƒO 2022N‹M HÅC 2021 - 2022Mæn:To¡nThíi gian l m b i: 90 phót — THAM KHƒO HKIIdC¥u 1. T¼m c¡c sè thüc x, y thäa m¢n (2x+ 5 y) + (4 x+ 3 y)i = 5 + 2 i. A x= 5 14v y= �8 7. B x= 8 7v y= � 5 14. C x= � 5 14v y= 8 7. D x= � 5 14v y= �8 7. ÊLíi gi£i.Ta câ (2x+ 5 y) + (4 x+ 3 y)i = 5 + 2 i, ¨2x + 5 y= 54 x + 3 y= 2 ,8><>: x= � 5 14y = 8 7:Chån ¡p ¡n C dC¥u 2. Trong khæng gian vîi h» tåa ë Oxyz, cho m°t c¦u (S ) câ ph÷ìng tr¼nh x2+ y2+ z2�2 x + 4 y� 6z � 2 = 0 . Tåa ë t¥m Icõa m°t c¦u l  A T¥mI(1; �2; 3) . B T¥mI(1; �2; 1) . C T¥mI(� 1; 2; 3) . D T¥mI(1; 2; �3) . ÊLíi gi£i.Gåi I(a ;b; c) l  t¥m m°t c¦u. Khi â ta câ a= 1; b= �2; c= 3 .Vªy t¥m m°t c¦u l  I(1; �2; 3) .Chån ¡p ¡n A dC¥u 3. H m sè n o sau ¥y câ tªp x¡c ành D=R? A y= p x� 1 x2+ 1 . B y= x� 1 2x � 1. C y= x3� 2x 2+ 1 . D y= p x3+ 1 . ÊLíi gi£i.H m sè y= x3� 2x 2+ 1 câ tªp x¡c ành D=R.Chån ¡p ¡n C dC¥u 4. Khèi c¦u (S ) b¡n k½nh Rcâ thº t½ch b¬ng A 4R 2. B 43R3. C 13R3. D R3. ÊLíi gi£i.Ta câ cæng thùc t½nh thº t½ch khèi c¦u (S ) b¡n k½nh Rl  4 3R3.Chån ¡p ¡n B dC¥u 5. Cho h m sè f(x ) li¶n töc tr¶n Rv  a2 R, khi â aZa f(x ) d xb¬ng242/305 242/305 pGV: L¶ Quang Xe  Ô0967.003.131PHT TRIšN — THAM KHƒO 2022 NÌI N€O CÂ Þ CH, NÌI  C CON ×ÍNG243A a2. B 1. C 2a . D 0. ÊLíi gi£i.Gåi F(x ) l  mët nguy¶n h m cõa h m sè f(x ).Ta câ aZa f(x ) d x= F(a ) � F(a ) = 0 .Chån ¡p ¡n D dC¥u 6. Cho h m sè f(x ) câ b£ng bi¸n thi¶n nh÷ sau xf0( x ) f(x ) �1�2 0 2 +1 �0 +0 �0 ++1 +1 �1 �1 22�1 �1 +1 +1 H m sè ¢ cho ¤t cüc ¤i t¤i iºm n o d÷îi ¥y?A x= 2 . B x= �22 . C x= 0 . D x= �1. ÊLíi gi£i.Düa v o b£ng bi¸n thi¶n ta th§y h m sè ¤t cüc ¤i t¤i iºm x= 0 .Chån ¡p ¡n C dC¥u 7. Tªp nghi»m cõa b§t ph÷ìng tr¼nh log0;3 (3x� 2) 0l  A �2 3; +1‹. B �2 3; 1 ‹. C �2 3;;1 ˜. D (2; +1). ÊLíi gi£i.Ta câ: log0;3 (3x� 2) 0, ¨3x � 2> 03 x � 2 1, 8<: x >2 3x  1,2 3< x1.Tªp nghi»m cõa b§t ph÷ìng tr¼nh l  �2 3; 1 ˜.Chån ¡p ¡n C dC¥u 8. L«ng trö tam gi¡c ·u câ ë d i t§t c£ c¡c c¤nh b¬ng 3. Thº t½ch khèi l«ng trö ¢ chob¬ng A 27p 34. B 9p 34. C 27p 32. D 9p 32. ÊLíi gi£i.243/305 243/305 pGV: L¶ Quang Xe  Ô0967.003.131— SÈ 16 NÌI N€O CÂ Þ CH, NÌI  C CON ×ÍNG244Gåi khèi l«ng trö tam gi¡c ·u â l ABC:A0B 0C 0ta câV ABC:A 0B 0C 0= AA 0 S4 ABC = 332p 34=27p 34: B CB0 C0 AA0 Chån ¡p ¡n A dC¥u 9. Cho 2 R, tªp x¡c ành cõa h m sè y= (1 + x) l  A (� 1; + 1). B Rn f� 1g . C R. D [� 1; + 1). ÊLíi gi£i.H m sè x¡c ành khi 1 +x > 0, x > �1.Vªy tªp x¡c ành l  D= ( �1; + 1).Chån ¡p ¡n A dC¥u 10. Nghi»m cõa ph÷ìng tr¼nh (0;3) x� 2= 1 l  A x= 3 . B x= 1 . C x= 0 . D x= 2 . ÊLíi gi£i.Ta câ (0;3) x� 2= 1 ,x� 2 = 0 ,x= 2 .Chån ¡p ¡n D dC¥u 11. T½nh t½ch ph¥n  4Z0 sinxdx . A I= 2� p 22. B I= p 22. C I= �p 22. D I= 2 + p 22. ÊLíi gi£i.Ta câ:  4Z0 sinxdx = �cos x  40 =�p 22+ 1 =2� p 22.Chån ¡p ¡n A dC¥u 12. Cho hai sè phùc z1 = 4�3i v  z2 = 7 + 3i. T¼m sè phùc z= z1 �z2. A z= 11 . B z= 3 + 6 i. C z= �1� 10i. D z= �3� 6i. ÊLíi gi£i.z = z1 �z2 = (4�3i) � (7 + 3 i) = (4 �7) + ( �3i � 3i) = �3� 6i.Chån ¡p ¡n D 244/305 244/305 pGV: L¶ Quang Xe  Ô0967.003.131PHT TRIšN — THAM KHƒO 2022 NÌI N€O CÂ Þ CH, NÌI  C CON ×ÍNG245dC¥u 13. Trong khæng gian Oxyz, cho m°t ph¯ng (P ) : x 1+y 2+z 3�1 = 0 . V²c-tì n o d÷îi¥y l  mët vector ph¡p tuy¸n cõa m°t ph¯ng (P )? A #n 1= (1; 2; 3). B #n 2= (1; 2;�1) . C #n 3= �1; 1 2;� 1‹. D #n 4= �1; 1 2;1 3‹. ÊLíi gi£i.Câ (P ) : x 1+y 2+z 3�1 = 0 , mët v²c-tì ph¡p tuy¸n l  #n (P ) = �1 1;1 2;1 3‹.Chån ¡p ¡n D dC¥u 14. Trong khæng gian vîi h» tröc tåa ë Oxyz, cho hai iºm M(� 2; 6; 1) v M0( a ;b; c)èi xùng nhau qua m°t ph¯ng (Oyz ). T½nh S= 7 a� 2b + 2017 c� 1. A S= 2017 . B S= 2042 . C S= 0 . D S= 2018 . ÊLíi gi£i.Gåi Hl  h¼nh chi¸u cõa Ml¶n (Oyz ), suy ra H(0; 6; 1) .Do M0èi xùng vîi Mqua (Oyz )n¶n M M 0nhªn Hl m trung iºm, suy ra M0(2; 6; 1) .Vªy T= 7 2� 2 6 + 2017 1� 1 = 2018 .Chån ¡p ¡n D dC¥u 15. Tr¶n m°t ph¯ng tåa ë, iºm biºu di¹n cõa sè phùc z= ‚1 + ip 31 +iŒ3l  iºm n od÷îi ¥y? A D(2; 2) . B C€1; 3 p 3Š. C B‚1 2;p 32Œ. D A(2; �2) . ÊLíi gi£i.Ta câ z= 1 + 3ip 3 + 9i2+ 3 p 3i3 1 + 3i+ 3 i2+ i3 =4 1� i= 2 + 2i. Vªy iºm biºu di¹n cõa zl  D(2; 2) .Chån ¡p ¡n A dC¥u 16. ÷íng th¯ng câ ph÷ìng tr¼nh n o sau ¥y l  ti»m cªn ùng cõa ç thà h m sèy = x� 3 x+ 2 ? A x= 3 . B x= �2. C y= 1 . D y= �3 2. ÊLíi gi£i.Ta câ limx !� 2+ y= �1 ;limx !� 2� y= + 1 ) x= �2 l  ti»m cªn ùng.Chån ¡p ¡n B dC¥u 17. Cho c¡c sè thüc d÷ìng a, b kh¡c 1. Kh¯ng ành n o sau ¥y sai? A logab= �logba. B loga3b= 1 3logab. C logab2= 2 log ab. D logablogba= 1 . ÊLíi gi£i.Ta câ logab= 1 logba n¶n kh¯ng ànhlogab= �logbasai.245/305 245/305 pGV: L¶ Quang Xe  Ô0967.003.131— SÈ 16 NÌI N€O CÂ Þ CH, NÌI  C CON ×ÍNG246Chån ¡p ¡n A dC¥u 18. H m sè n o câ b£ng bi¸n thi¶n nh÷ h¼nh v³. xy0 y �1�1 0 1 +1 �0 +0 �0 ++1 +1 �4 �4 �3 �3 �4 �4 +1 +1 A y= x4� 2x 2� 3. B y= �x4+ 2 x2� 3. C y= x4� 2x 2+ 3 . D y= �x4+ 2 x2+ 3 . ÊLíi gi£i.D¹ th§y b£ng bi¸n thi¶n tr¶n l  b£ng bi¸n thi¶n cõa h m sè tròng ph÷ìng câ a >0.M°t kh¡c ç thà h m sè c­t tröc tung t¤i iºm (0;�3) n¶n h m sè c¦n t¼m l  y= x4� 2x 2� 3.Chån ¡p ¡n A dC¥u 19. Trong khæng gian vîi h» tåa ë Oxyz, ÷íng th¯ng  :8><>: x= 2 �ty = 1z = �2 + 3 tkhæng i quaiºm n o sau ¥y? A M(2; 1; �2) . B P(4; 1; �4) . C Q(3; 1; �5) . D N(0; 1; 4) . ÊLíi gi£i.Kiºm tra th§y iºm P(4; 1; �4) khæng thäa m¢n ph÷ìng tr¼nh ÷íng th¯ng .Chån ¡p ¡n B dC¥u 20. Cho tªp hñp Mcâ10 ph¦n tû. Sè tªp con gçm hai ph¦n tû cõa Ml  A A810 . B A210 . C C210 . D 102. ÊLíi gi£i.Sè tªp con câ 2ph¦n tû tø tªp hñp câ 10ph¦n tû b¬ng sè c¡ch l§y ra 2ph¦n tû tø tªp hñp câ 10ph¦n tû v  b¬ng C210 .Chån ¡p ¡n C dC¥u 21. Cho h¼nh hëp ùng ABC D:A0B 0C 0D 0câ c¤nh b¶n AA0= hv  di»n t½ch cõa tam gi¡cABC b¬ngS. Thº t½ch cõa khèi hëp ABC D:A0B 0C 0D 0b¬ng A V= 1 3S h. B V= 2 3S h. C V=S h . D V= 2 S h. ÊLíi gi£i.V¼ SABC =Sn¶n suy ra SABC D = 2S.Thº t½ch khèi hëp ABC D:A0B 0C 0D 0l : VABC D:A 0B 0C 0D 0= SABC D AA 0= 2 S h.Chån ¡p ¡n D 246/305 246/305 pGV: L¶ Quang Xe  Ô0967.003.131PHT TRIšN — THAM KHƒO 2022 NÌI N€O CÂ Þ CH, NÌI  C CON ×ÍNG247dC¥u 22. H m sèy= ln(cos x) câ ¤o h m tr¶n tªp x¡c ành cõa nâ l  A y0= 1 cosx. B y0= �sinx cosx. C y0= sinx cosx. D y0= 1 sinx. ÊLíi gi£i.Ta câ y0= [ln(cos x)] 0= (cosx)0 cosx=�sinx cosx.Chån ¡p ¡n B dC¥u 23.Cho h m sè f(x ) câ ç thà nh÷ h¼nh v³. H m sè ¢ cho nghàch bi¸ntr¶n kho£ng n o d÷îi ¥y? A (0; 1). B (� 1; 0) . C (1; +1). D (� 1; 1) . xyO�1 11�1 �2 ÊLíi gi£i.Düa v o ç thà ta th§y h m sè nghàch bi¸n tr¶n kho£ng (0; 1).Chån ¡p ¡n A dC¥u 24. Cho h m sè y= f(x ) li¶n töc tr¶n Rthäa m¢n 3Z1 f(x ) d x= 8 ,3Z2 f(x ) d x= 5 . Gi¡trà cõa t½ch ph¥n 2Z1 f(x ) d xb¬ng A �3. B 40. C 3. D 13. ÊLíi gi£i.3Z1 f(x ) d x= 2Z1 f(x ) d x+ 3Z2 f(x ) d x, 2Z1 f(x ) d x= 3Z1 f(x ) d x� 3Z2 f(x ) d x= 8 �5 = 3 .Chån ¡p ¡n C dC¥u 25. Cho d¢y sè ¨u1 = 1u n =un� 1 + 2 (n >1). Kh¯ng ành n o sau ¥y óng? A u5 = 9. B u3 = 4. C u2 = 2. D u6 = 13. ÊLíi gi£i.D¢y sè l  c§p sè vîi cëng cæng sai d= 2 ,un =u1 + 2(n� 1) suy ra u5 = 9.Chån ¡p ¡n A dC¥u 26. Hå nguy¶n h m cõa h m sè f(x ) = e x+ xl  A ex+ 1 2x2+ C. B 1x+ 1 ex+ 1 2ex+ 1 2ex+ C. C ex+ x2+ C. D ex+ 1 + C. ÊLíi gi£i.247/305 247/305 pGV: L¶ Quang Xe  Ô0967.003.131— SÈ 16 NÌI N€O CÂ Þ CH, NÌI  C CON ×ÍNG248Zf(x ) d x= Z(ex+ x) d x= e x+ 1 2x2+ C.Chån ¡p ¡n A dC¥u 27. Cho h m sè y= x� 1 x+ m;(m 6= �1) câ ç thà l  (C ). T¼m mº (C )nhªn iºm I(2; 1)l m t¥m èi xùng. A m= 1 2. B m=�1 2. C m= 2 . D m=�2. ÊLíi gi£i.ç thà h m sè y= x� 1 x+ m;(m 6= �1) câ t¥m èi xùng I(� m ; 1) .Vªy m=�2.Chån ¡p ¡n D dC¥u 28. T¼m gi¡ trà nhä nh§t cõa h m sè y= 4 x3� 3x 4tr¶n o¤n [0; 2]. A min[0;2] y= �24 . B min[0;2] y= 1 . C min[0;2] y= 0 . D min[0;2] y= �16 . ÊLíi gi£i.H m sè ¢ cho li¶n töc tr¶n [0; 2].Ta câ: y0= 12 x2� 12x3; 8x 2 R.y 0= 0 ,–x = 0x = 1 :Hìn núa, y(0) = 0 ;y(1) = 1 ;y(2) = �16 .Do â min[0;2] y= y(2) = �16 .Chån ¡p ¡n D dC¥u 29. H m sèy= �x3+ 3 x� 2nghàch bi¸n tr¶n c¡c kho£ng n o sau ¥y? A (�1 ;� 1) v  (1; + 1). B (� 1; + 1). C (� 1; 1) . D (�1 ;� 1) [(1; + 1). ÊLíi gi£i.Ta câ y0= �3x 2+ 3 ,y0= 0 ,x= 1. B£ng x²t d§u y0 xy0 y �1�1 1 +1 �0 +0 �H m sè nghàch bi¸n tr¶n c¡c kho£ng(�1 ;� 1) v  (1; + 1).Chån ¡p ¡n A dC¥u 30. Rót gån biºu thùc R= logab3 2+ loga2b5 2(vîi a >0, a 6= 1 v b > 0). A R= 15 4logab. B R= logab. C R= 11 4logab. D R= 15 8logab.248/305 248/305 pGV: L¶ Quang Xe  Ô0967.003.131PHT TRIšN — THAM KHƒO 2022 NÌI N€O CÂ Þ CH, NÌI  C CON ×ÍNG249ÊLíi gi£i.Ta câ R= 3 2logab+ 5 4logab= 11 4logab.Chån ¡p ¡n C dC¥u 31. Cho h¼nh châp S:ABC Dcâ ¡y l  h¼nh thoi t¥m Oc¤nh av  câ gâc ÕBAD = 60.÷íng th¯ng S Ovuæng gâc vîi m°t ¡y (ABC D )v  S O =3a 4. Kho£ng c¡ch tøA¸n m°t ph¯ng( S B C )l  A ap 32. B 3a 2. C 2a 3. D 3a 4. ÊLíi gi£i. CAD O BSTa câVS:ABC D =1 3S OSABC D =1 33a 4a a sin 60 = a3p 38:Tam gi¡c S B Cl  tam gi¡c câ BC=a, S B =p S O2+ OB 2= Ê �3a 4‹2+ a 22= ap 134v S C =p OC2+ OS 2= s ‚ap 32Œ2+ �3a 4‹2= ap 214.Do â SS BC =a2p 34)d(A; (S B C )) =3VS:ABC D SS BC =3a 2.Chån ¡p ¡n D dC¥u 32. Cho2Z� 1 f(x ) d x= 3 v �1Z2 g(x ) d x= 1 . T½nh 2Z� 1 [x + 2 f(x ) � 3g (x )] d x A 52. B 212. C 262. D 72. ÊLíi gi£i.2Z� 1 [x + 2 f(x ) � 3g (x )] d x = 2Z� 1 xdx + 2 2Z� 1 f(x ) d x� 3 2Z� 1 g(x ) d x= x2 2 2� 1 + 2 2Z� 1 f(x ) d x+ 3 �1Z2 g(x ) d x= 3 2+ 6 + 3 =21 2:249/305 249/305 pGV: L¶ Quang Xe  Ô0967.003.131— SÈ 16 NÌI N€O CÂ Þ CH, NÌI  C CON ×ÍNG250Chån ¡p ¡n B dC¥u 33. Trong khæng gian Oxyz, cho ba iºm A(� 1; 1; 1) ,B (2; 1; 0) ,C (1; �1; 2) . M°t ph¯ngi qua Av  vuæng gâc vîi ÷íng th¯ng BCcâ ph÷ìng tr¼nh l  A x+ 2 y� 2z + 1 = 0 . B x+ 2 y� 2z � 1 = 0 . C 3x + 2 z� 1 = 0 . D 3x + 2 z+ 1 = 0 . ÊLíi gi£i.Ta câ # BC = (�1; �2; 2) l  mët v²c-tì ph¡p tuy¸n cõa m°t ph¯ng (P )c¦n t¼m.) #n = �# BC = (1; 2; �2) công l  mët v²c-tì ph¡p tuy¸n cõa m°t ph¯ng (P ).Vªy ph÷ìng tr¼nh m°t ph¯ng (P )l  1(x+ 1) + 2( y� 1) �2(z� 1) , x+ 2 y� 2z + 1 = 0 .Chån ¡p ¡n A dC¥u 34. Häi câ bao nhi¶u sè phùc zthäa çng thíi c¡c i·u ki»n jz � ij = 5 v z2l  sè thu¦n£o? A 2. B 3. C 0. D 4. ÊLíi gi£i.°t z= x+ iy (vîi x, y 2 R).Ta câ jz � ij = 5 ,x2+ ( y� 1)2= 25 :( )z 2l  sè thu¦n £o, suy ra x2� y2= 0 ,–x = yx = �y:Vîi x= ythay v o ( ) ta ÷ñc x2+ ( x� 1)2= 25 ,2x 2� 2x � 24 = 0 ,–x = 4x = �3:Vîi x= �y thay v o ( ) ta ÷ñc x2+ ( x+ 1) 2= 25 ,2x 2+ 2 x� 24 = 0 ,–x = �4x = 3 :Vªy câ 4sè phùc c¦n t¼m l  4 + 4i, � 3� 3i, � 4 + 4 i, 3 � 3i.Chån ¡p ¡n D dC¥u 35. Cho h¼nh châp S:ABC Dcâ ¡yABC D l  h¼nh vuæng c¤nh b¬ng ap 3; tam gi¡cS B C vuæng t¤i Sv  n¬m trong m°t ph¯ng vuæng gâc vîi ¡y, ÷íng th¯ng S Dt¤o vîi m°t ph¯ng( S B C )mët gâc 60. Thº t½ch cõa khèi châp ¢ cho b¬ng A a3p 3. B a3p 6. C a3p 66. D a3p 63. ÊLíi gi£i.K´ S H ?BC . Tø gi£ thi¸t suy ra S H?(ABC D ).X¡c ành ÷ñc h¼nh chi¸u cõa Dl¶n (S B C )l  iºm C.Do â: Û�( S D; (S B C )) =Ù�( S D; S C ) =ÕDS C = 60.Tam gi¡c vuæng S C Dvuæng t¤i Ccâ S C =DC cot ÕDS C =a.Tam gi¡c vuæng S B Cvuæng t¤i ScâS B =p BC2� S C 2= ap 2; S H =S BS C BC=ap 63Vªy thº t½ch khèi châp:VS:ABC D =1 3SABC D S H =1 3AB2S H =a3p 63. B CA DH S250/305 250/305 pGV: L¶ Quang Xe  Ô0967.003.131PHT TRIšN — THAM KHƒO 2022 NÌI N€O CÂ Þ CH, NÌI  C CON ×ÍNG251Chån ¡p ¡n D dC¥u 36. Mët nhâm câ 7håc sinh trong â câ 3nam v  4nú. Häi câ bao nhi¶u c¡ch x¸p c¡chåc sinh tr¶n th nh mët h ng ngang sao cho c¡c håc sinh nú ùng c¤nh nhau? A 144. B 5040. C 576. D 1200. ÊLíi gi£i.Ì Sè c¡ch x¸p 3håc sinh nam th nh mët h ng l  3!c¡ch.Ì Ùng vîi méi c¡ch x¸p 3håc sinh nam th nh mët h ng, t¤o ra 4và tr½ trèng m  t¤i méi và tr½trèng â n¸u x¸p c¡c håc sinh nú v o. Khi â c¡c håc sinh nú ùng c¤nh nhau trong h ng.Ì Ùng vîi méi c¡ch x¸p tr¶n câ 4!c¡ch x¸p c¡c håc sinh núVªy câ 3!4! 4 = 576 c¡ch x¸p.Chån ¡p ¡n C dC¥u 37. Trong khæng gian Oxyz, cho m°t ph¯ng (P ) : x+ y+ z� 3 = 0 v  ÷íng th¯ngd : x 1=y+ 1 2=z� 2 �1 . H¼nh chi¸u vuæng gâc cõadtr¶n (P )câ ph÷ìng tr¼nh l  A x+ 1 �1 =y+ 1 �4 =z+ 1 5. B x� 1 3=y� 1 �2 =z� 1 �1 . C x� 1 1=y� 1 4=z� 1 �5 . D x� 1 1=y� 4 1=z+ 5 1. ÊLíi gi£i.Ph÷ìng tr¼nh tham sè cõa ÷íng th¯ng dl  8><>: x= ty = �1 + 2 tz = 2 �t:Gåi Al  giao iºm cõa ÷íng th¯ng dv  m°t ph¯ng (P ). Khi â, ta câ h» ph÷ìng tr¼nh 8>>><>>>: x= ty = �1 + 2 tz = 2 �tx + y+ z� 3 = 0 )t + ( �1 + 2 t) + (2 �t) � 3 = 0 ,t= 1 )A(1; 1; 1) .Ta câ ÷íng th¯ng dcâ v²c-tì ch¿ ph÷ìng l  #u d= (1; 2;�1) , m°t ph¯ng (P )câ v²c-tì ph¡p tuy¸n l # n (P ) = (1; 1; 1).Gåi (Q )l  m°t ph¯ng chùa ÷íng th¯ng dv  vuæng gâc vîi (P ). Khi â (Q )câ v²c-tì ph¡p tuy¸n l # n (Q ) = #u d; #n (P )= (3; �2; �1) .Gåi ÷íng th¯ng l  h¼nh chi¸u vuæng gâc cõa dl¶n (P ). Khi â l  giao tuy¸n cõa hai m°t ph¯ng( P )v  (Q ).Suy ra v²c-tì ch¿ ph÷ìng cõa l  #u  = #n (P ); #n (Q )= (1; 4; �5) .Vªy h¼nh chi¸u vuæng gâc cõa dtr¶n (P )câ ph÷ìng tr¼nh l  x� 1 1=y� 1 4=z� 1 �5 .Chån ¡p ¡n C dC¥u 38. Gi£i b§t ph÷ìng tr¼nh log2�log 1 2�2x� 15 16‹‹62. A x> 0. B log215 16< x <log231 16. C 06 x < log231 16. D log231 16< x60.251/305 251/305 pGV: L¶ Quang Xe  Ô0967.003.131— SÈ 16 NÌI N€O CÂ Þ CH, NÌI  C CON ×ÍNG252ÊLíi gi£i.Ta câlog2�log 1 2�2x� 15 16‹‹62, 0< log 1 2�2x� 15 16‹6 4, 1 1662x� 15 16<1, 16 2x< 31 16, 06 x < log231 16:Chån ¡p ¡n C dC¥u 39. Bi¸t r¬ng câ hai gi¡ trà thüc cõa tham sè mº ç thà h m sè y= 2x + 1 x� 1(C ) v ÷íng th¯ng d:y = mx + 3 c­t nhau t¤i hai iºm ph¥n bi»t A,B sao cho tam gi¡c OABvuængt¤i O(vîi Ol  gèc tåa ë). Têng cõa hai gi¡ trà â b¬ng A 0. B 4. C 8. D 6. ÊLíi gi£i.Vîi i·u ki»n x6= 1 ta câ ph÷ìng tr¼nh ho nh ë giao iºm cõa (C )v  dl 2 x + 1 x� 1 =y= mx + 3 )2x + 1 = ( mx+ 3)( x� 1) , mx 2� (m �1)x� 4 = 0 : ( )Rã r ng x= 1 khæng l  nghi»m cõa ph÷ìng tr¼nh ( ). Nh÷ vªy ÷íng th¯ng dc­t (C )t¤i hai iºmph¥n bi»t A,B khi ph÷ìng tr¼nh ( ) câ hai nghi»m ph¥n bi»t, tùc l ¨ m 6= 0( m �1)2+ 16 m > 0,¨m 6= 0m 2+ 14 m+ 1 >0 ,8><>: m6= 0–m < �7� 4p 3m > �7 + 4 p 3( )Gåi c¡c giao iºm cõa dv  (C )l  A(x1;mx1+ 3)v B(x2;mx2+ 3), trong â x1,x2 l  hai nghi»mcõa ph÷ìng tr¼nh ( ).Theo gi£ thi¸t, tam gi¡c OABvuæng t¤i On¶n# OA # OB = 0,x1x2 + (mx1+ 3)(mx2+ 3) = 0,x1x2 +m2x 1x2 + 3m(x1 +x2) + 9 = 0, (m 2+ 1) x1x2 + 3m(x1 +x2) + 9 = 0,�4( m2+ 1) m+3m (m �1) m+ 9 = 0, � 4(m2+ 1) + 3 m(m �1) + 9 m= 0 , � m2+ 6 m�4 = 0, m= 3 p 5: (thäa m¢ i·u ki»n ( ))Vªy m= 3 p 5l  c¡c gi¡ trà thäa m¢n y¶u c¦u b i to¡n v  têng cõa chóng b¬ng 6.Chån ¡p ¡n D dC¥u 40. Cho h m sè f(x ) x¡c ành tr¶n o¤n [� 1; 2] thäa m¢n f(0) = 1 v f2(x ) f 0( x ) =3 x 2+ 2 x� 2. Sè nghi»m cõa ph÷ìng tr¼nh f(x ) = 1 tr¶n o¤n [� 1; 2] l  A 1. B 3. C 0. D 2. ÊLíi gi£i.V¼ h m sè f(x ) x¡c ành tr¶n o¤n [� 1; 2] n¶n ta câZf2(x ) f 0( x )d x= Zf2(x )d( f(x )) = Z(3x2+ 2 x� 2)d x, 1 3f3(x ) = x3+ x2� 2x + C.252/305 252/305 pGV: L¶ Quang Xe  Ô0967.003.131PHT TRIšN — THAM KHƒO 2022 NÌI N€O CÂ Þ CH, NÌI  C CON ×ÍNG253V¼f(0) = 1 suy raC= 1 3.Ta câ f(x ) = 1 ,f3(x ) = 1 ,3(x3+ x2� 2x ) + 1 = 1 ,x3+ x2� 2x = 0 ,x= 0 ho°c x= 1 ho°cx = �2 (lo¤i v¼ x= �2 =2 [� 1; 2] ).Chån ¡p ¡n D dC¥u 41. Cho h¼nh châp S:ABCcâS A vuæng gâc vîi ¡y, S C= 2,ÕBC S = 45, gâc giúa haim°t ph¯ng (S AB )v  (S B C )b¬ng 90, gâc giúa hai m°t ph¯ng (S AC )v  (S B C )b¬ng 60. Thºt½ch khèi châp S:ABCl  A V= p 215. B V= 2 p 3. C V= 2 p 2. D V= 2p 315. ÊLíi gi£i.Thº t½ch khèi châp V= 1 3S ASABC .K´ AH ?S B suy ra AH?(S B C ).Do BC ?S A v BC ?AH n¶nBC ?(S AB ), do â tam gi¡c ABCvuæng t¤i B.K´ BI ?AC )BI ?S C v  k´ BK?S C )S C ?(BI K ).Do â gâc giúa hai m°t ph¯ng (S AC )v  (S B C )l  ÕBK I = 60.Do ÕBC S = 45n¶n S B=BC =p 2v  Kl  trung iºm cõa S Cn¶nBK =S C 2= 1. SAB CHI KTrong tam gi¡c vuængBI KcâBI =BK sin 60 = p 32.Trong tam gi¡c vuæng ABCcâ1 BI2= 1 AB2+ 1 BC2)AB = BIBC pBC2� BI 2= p 305.S ABC =1 2ABBC =p 155;S A =p S B2� AB 2= 2p 55.Vªy V= 1 3S ASABC =2p 315.Chån ¡p ¡n D dC¥u 42. Cho sè phùc z= 1 + ip 3. Sè phùc li¶n hñp cõa zl  A z= 1 �ip 3. B z= �p 3� i. C z= �1 + ip 3. D z= p 3 +i. ÊLíi gi£i.z = a+ ib ) z= a� bi.Chån ¡p ¡n A dC¥u 43. Cho tªpX=f1; 3; 5; 7; 9 g. Câ bao nhi¶u sè phùc z= x+ yi câ ph¦n thüc, ph¦n £o·u thuëc Xv  câ têng x+ y 10? A 20. B 10. C 15. D 24. ÊLíi gi£i.X²t sè phùc z= x+ yi (x; y 2X).V¼ sè phùc z= x+ yi thäa m¢n x+ y 10 n¶n ta x²t c¡c tr÷íng hñp saua) (x ;y ) 2 f (1; 3) ;(1; 5) ;(1; 7) ;(1; 9) ;(3; 5) ;(3; 7) g, câ 2 6 = 12 sè phùc thäa m¢n.253/305 253/305 pGV: L¶ Quang Xe  Ô0967.003.131— SÈ 16 NÌI N€O CÂ Þ CH, NÌI  C CON ×ÍNG254b)(x ;y ) 2 f (1; 1) ;(3; 3) ;(5; 5) g, câ 3sè phùc thäa m¢n.Vªy câ 12 + 3 = 15 sè phùc thäa m¢n · b i.Chån ¡p ¡n C dC¥u 44. Cho h m sè y= f(x ) li¶n töc v  câ ¤o h m f0( x ) tr¶n R. H¼nh b¶n d÷îi l  ç thàh m sè y= f0( x ). O xy�1 1 2�2 �2 2Häi h m sèg(x ) = f(x ) + x2 2�2019 câ tèi a bao nhi¶u iºm cüc trà? A 8. B 7. C 5. D 9. ÊLíi gi£i.X²t h m sè h(x ) = f(x ) + x2 2�2019 .Ì Ta t¼m sè cüc trà cõa h m sè h(x ).Ta câ: h0( x ) = f0( x ) + x.Düa v o ç thà ta v³ th¶m ÷íng th¯ng y= �x. O xy�1 1 2�2 �2 2Oh0( x ) = 0 ,f0( x ) + x= 0 ,f0( x ) = �x , 264 x= �2x = 0x = 2 :B£ng bi¸n thi¶n cõa h(x ) xh0 h �1�2 0 2 +1 +0 �0 +0 ��1�1 h(� 2) h(� 2) h(0) h(0) h(2) h(2) �1�1254/305 254/305 pGV: L¶ Quang Xe  Ô0967.003.131PHT TRIšN — THAM KHƒO 2022 NÌI N€O CÂ Þ CH, NÌI  C CON ×ÍNG255H m sèy= h(x ) câ ba iºm cüc trà.Ì T¼m sè nghi»m cõa ph÷ìng tr¼nh h(x ) = 0 .Tø b£ng bi¸n thi¶n ta câ h(x ) = 0 câ tèi a 4 nghi»m ph¥n bi»t.Vªy h m sè g(x ) = jh (x )j câ tèi a: 3 + 4 = 7iºm cüc trà.Chån ¡p ¡n B dC¥u 45. Trong khæng gian vîi h» tåa ë Oxyz, cho hai ÷íng th¯ng d1: x� 2 3=y+ 1 1=z+ 3 2v  d2: x� 3 1=y� 7 �2 =z� 1 �1 . Vi¸t ph÷ìng tr¼nh ÷íng th¯ngc­t d1 v d2 çng thíi i quaiºm M(3; 10; 1) . A  :x� 3 �1 =y� 10 5=z� 1 1. B  :x� 3 �1 =y� 10 �5 =z� 1 1. C  :x� 3 1=y� 10 �5 =z� 1 1. D  :x� 3 �1 =y� 10 �5 =z� 1 �1 . ÊLíi gi£i.Gåi #u 1, #u 2 l¦n l÷ñt l  VTCP cõad1 v d2.L§y A(2; �1; �3) 2d1,B (3; 7; 1) 2d2.Gåi (P )l  m°t ph¯ng chùa v  d1. Mët VTPT cõa(P )l  #n 1 = ”#u 1; # AM —= ( �18; �10; 32) .Gåi (Q )l  m°t ph¯ng chùa v  d2. Mët VTPT cõa(Q )l  #n 2 = ”#u 2; # BM —= (3; 0; 3) .Suy ra mët VTCP cõa ÷íng th¯ng l  #u = [ #n 1; #n 2] = (�30; 150; 30) .Vªy ph÷ìng tr¼nh ÷íng th¯ng l  x� 3 �1 =y� 10 5=z� 1 1.Chån ¡p ¡n A dC¥u 46. Cho tam gi¡c ABCvuæng t¤i A. Gåi Va; Vb; Vct÷ìng ùng l  thº t½ch cõa c¡c h¼nhtrán xoay t¤o bði tam gi¡c ABCkhi cho nâ l¦n l÷ñt quay xung quanh c¡c c¤nh BC,C A ,AB . H»thùc n o sau ¥y óng? A 1V2a =1 V2b +1 V2c . B V2a =V2b +V2c . C V2a =V2b V 2c . D 1V2a =V2b V 2c V2b +V2c . ÊLíi gi£i.°t AB=c; BC =a; AC =b. Ta câ, Vb = bc2 3,Vc = cb2 3,Va = b2c 2 3a . Khi â,1 V2a =1 V2b +1 V2c .Chån ¡p ¡n A dC¥u 47. Câ bao nhi¶u gi¡ trà nguy¶n cõa m2[0; 2018] º b§t ph÷ìng tr¼nh m+ e x 2 4p e2x+ 1óng vîi måi x2 R. A 2017. B 2018. C 2019. D 2016. ÊLíi gi£i.Ta câ m+ e x 2 4p e2x+ 1 ,m 4p e2x+ 1 �ex 2(1).X²t h m sè y= f(x ) = 4p e2x+ 1 �ex 2.Câ y0= 1 2‚e2x 4p (e2x+ 1) 3�ex 2Œ= 1 2e2x� ex 24p (e2x+ 1) 3 4p (e2x+ 1) 3 .Suy ra y0= 0 ,e2x� ex 24p (e2x+ 1) 3= 0 ,e3x 2= 4p (e2x+ 1) 3, (e3x)2= (e 2x+ 1) 3(2).°t t= e 2x, (t > 0). Khi â (2),3t2+ 3 t+ 1 = 0 , ph÷ìng tr¼nh n y væ nghi»m n¶n ph÷ìng tr¼nh(2) væ nghi»m hay y0= 0 væ nghi»m.Suy ra y0khæng êi d§u tr¶n R, d¹ th§y y0(0) <0) y0< 0, 8x 2 R.255/305 255/305 pGV: L¶ Quang Xe  Ô0967.003.131— SÈ 16 NÌI N€O CÂ Þ CH, NÌI  C CON ×ÍNG256Do â h m sèy= f(x ) = 4p e2x+ 1 �ex 2nghàch bi¸n tr¶n R.Câ limx !�1 f(x ) = limx!�1 €4p e2x+ 1 �ex 2Š= 1 .Khi â b§t ph÷ìng tr¼nh (1)câ nghi»m vîi måi x2 R , m1.Suy ra câ 2018gi¡ trà nguy¶n cõa mthäa m¢n · b i.Chån ¡p ¡n B dC¥u 48. Trong khæng gian tåa ë Oxyz, x²t ba iºm A(a ; 0; 0) ; B(0; b; 0) ; C(0; 0; c) vîi a; b; c l c¡c sè thüc thay êi thäa m¢n 1 a�1 b+1 c= 1. Bi¸t r¬ng m°t c¦u (S ) : ( x� 2) 2+( y� 1) 2+( z� 3) 2= 25c­t m°t ph¯ng (ABC )theo giao tuy¸n l  mët ÷íng trán câ b¡n k½nh b¬ng 4. Gi¡ trà cõa biºuthùc a+ b� cb¬ng A 1. B 4. C 2. D 3. ÊLíi gi£i. M H IPPh÷ìng tr¼nh m°t ph¯ng(ABC ) :x a+y b+z c= 1.Do 1 a�1 b+1 c= 1n¶n m°t ph¯ng (ABC )luæn i qua iºm cè ành H(1; �1; 1) .M°t c¦u (S ) câ t¥m I(2; 1; 3) , b¡n k½nh R= 5 .Ta câ # I H = (1; 2; 2) )I H = 3.Gåi Kl  h¼nh chi¸u vuæng gâc cõa Itr¶n (ABC ).M°t ph¯ng (ABC )c­t m°t c¦u theo giao tuy¸n l  ÷íng trán câ b¡n k½nh r= 4 n¶nI K=p R2� r2=3 ) KH.) M°t ph¯ng (ABC )nhªn # I H l m v²c-tì ph¡p tuy¸n.) ¨# I H # AB = 0# I H # AC = 0 ,¨� a+ 2 b= 0� a+ 2 c= 0 )b= c= a 2.K¸t hñp 1 a�1 b+1 c= 1)a= 1 ; b=c= 1 2)a+ b� c= 1 .Chån ¡p ¡n A dC¥u 49. Cho h m sè y= jx 4� 2mx 2+ 2 m�1j vîi ml  tham sè thüc. Sè gi¡ trà nguy¶n trongkho£ng [� 2; 2] cõamº h m sè ¢ cho câ 3iºm cüc trà l  A 1. B 2. C 3. D 4. ÊLíi gi£i.256/305 256/305 pGV: L¶ Quang Xe  Ô0967.003.131MÖC LÖCNÌI N€O CÂ Þ CH, NÌI  C CON ×ÍNG257�p döng cæng thùc(ju j) 0= €p u2Š0= uu 0 ju j ta ÷ñcy 0= (x 4� 2mx 2+ 2 m�1) (4 x3� 4mx ) jx 4� 2mx 2+ 2 m�1j= (x 2� 1)( x2� 2m + 1) 4x (x 2� m) jx 4� 2mx 2+ 2 m�1jTa th§y y0luæn câ nghi»m x= 0 ; 1 n¶n º h m sè câ 3cüc trà th¼ ph£i x£y ra mët trong c¡c tr÷ínghñp sauÌ (x 2� 2m + 1)( x2� m)væ nghi»m ,m < 0.Ì x= 0 l  nghi»m (x 2� 2m + 1)( x2� m), m= 0 ho°c m= 1 2. Thû l¤i ch¿ câm= 0 thäa m¢n.Ì x= 1 l  nghi»m (x 2� 2m + 1)( x2� m), m= 1 . Thû l¤i m= 1 thäa m¢n.Vªy câ 4 gi¡ trà nguy¶n khæng ¥m cõa m2[� 2; 2] thäa m¢n.Chån ¡p ¡n D H˜T257/305 257/305 pGV: L¶ Quang Xe  Ô0967.003.131— SÈ 17 NÌI N€O CÂ Þ CH, NÌI  C CON ×ÍNG258BË GIO DÖC & €O T„OTR×ÍNG THPT NGUY™N T‡T TH€NH GV: L– QUANG XE - 0967.003.131 — SÈ 17 PHT TRIšN — THAM KHƒO 2022N‹M HÅC 2021 - 2022Mæn:To¡nThíi gian l m b i: 90 phót — THAM KHƒO HKIIdC¥u 1. Sè phùc li¶n hñp cõa sè phùc z= 2 �3i l  A z= 3 + 2 i. B z= 3 �2i. C z= 2 + 3 i. D z= �2 + 3 i. ÊLíi gi£i.Sè phùc li¶n hñp cõa sè phùc z= 2 �3i l  z= 2 + 3 i.Chån ¡p ¡n C dC¥u 2. Trong khæng gian Oxyz, cho m°t c¦u (S ) : ( x� 1)2+ ( y+ 1) 2+ ( z� 2)2= 9 . Tåa ët¥m Iv  b¡n k½nh Rcõa (S ) l¦n l÷ñt l  A I(1; �1; 2) ,R = 3 . B I(� 1; 1; �2) ,R = 3 . C I(1; �1; 2) ,R = 9 . D I(� 1; 1; �2) ,R = 9 . ÊLíi gi£i.Ph÷ìng tr¼nh m°t c¦u vi¸t l¤i nh÷ sau (S ) : ( x� 1)2+ [ y� (� 1)] 2+ ( z� 2)2= 3 2. Vªy tåa ë t¥m Iv  b¡n k½nh Rcõa (S ) l¦n l÷ñt l  I(1; �1; 2) ,R = 3 .Chån ¡p ¡n A dC¥u 3. Cho h m sè y= f(x ) câ limx ! +1 f(x ) = 0 v limx !�1 f(x ) = + 1. M»nh · n o sau ¥y l m»nh · óng? A ç thà h m sè n¬m ph½a tr¶n tröc ho nh.B ç thà h m sè câ mët ti»m cªn ùng l  ÷íng th¯ngy= 0 . C ç thà h m sè câ mët ti»m cªn ngang l  tröc ho nh.D ç thà h m sè khæng câ ti»m cªn ngang.ÊLíi gi£i.Tø limx ! +1 f(x ) = 0 suy ra ç thà h m sè câ mët ti»m cªn ngang l  tröc ho nh.Chån ¡p ¡n C dC¥u 4. Cho m°t c¦u câ b¡n k½nh b¬ng ap 3. Di»n t½ch xung quanh Scõa m°t c¦u b¬ng A S= 36 a2. B S= 4 a2p 3. C S= 4 a2. D S= 12 a2. ÊLíi gi£i.Di»n t½ch cõa m°t c¦u l  S= 4 R2= 12 a2.Chån ¡p ¡n D dC¥u 5. Ph¡t biºu n o sau ¥y l  óng? A Zsin xdx = �sin x+ C. B Zsin xdx = �cos x+ C.258/305 258/305 pGV: L¶ Quang Xe  Ô0967.003.131PHT TRIšN — THAM KHƒO 2022 NÌI N€O CÂ Þ CH, NÌI  C CON ×ÍNG259C Zsin xdx = sin x+ C. D Zsin xdx = cos x+ C. ÊLíi gi£i.Ta câ Zsin xdx = �cos x+ C.Chån ¡p ¡n B dC¥u 6. Cho h m sè y= f(x ) câ b£ng bi¸n thi¶n nh÷ d÷îi ¥y xf0( x ) f(x ) �10 1 +1 +0 �0 +�1�1 55�1 �1 +1 +1 M»nh · n o sau ¥y óng?A H m sè câ iºm cüc tiºux= 0 . B H m sè câ iºm cüc ¤ix= 5 . C H m sè câ iºm cüc tiºux= �1. D H m sè câ iºm cüc tiºux= 1 . ÊLíi gi£i.Düa v o b£ng bi¸n thi¶n ta th§y h m sè câ iºm cüc tiºu x= 1 .Chån ¡p ¡n D dC¥u 7. Tªp nghi»m cõa b§t ph÷ìng tr¼nh (0;8) x< 3l  A �log 0;8 3; +1. B ��1 ; log0;8 3. C �log 34 5; +1‹. D ��1 ; log34 5‹. ÊLíi gi£i.Ta câ (0;8) x< 3, x > log0;8 3.Do â, tªp nghi»m cõa b§t ph÷ìng tr¼nh (0;8) x< 3l  �log 0;8 3; +1Chån ¡p ¡n A dC¥u 8. T½nh thº t½ch khèi lªp ph÷ìng câ c¤nh ap 3. A 3a 3. B a3p 3. C a3p 27. D a3 3. ÊLíi gi£i.Thº t½ch khèi lªp ph÷ìng c¤nh ap 3l  (a p 3)3= a3p 27.Chån ¡p ¡n C dC¥u 9. T¼m tªp x¡c ành Dcõa h m sè y= ( x2� 3x + 2) 2016. A D=Rn f 1; 2 g. B D= ( �1 ; 1)[(2; + 1). C D=R. D D= (1; 2) . ÊLíi gi£i.V¼ 2016 nguy¶n d÷ìng n¶n h m sè câ tªp x¡c ành l  D=R.Chån ¡p ¡n C 259/305 259/305 pGV: L¶ Quang Xe  Ô0967.003.131— SÈ 17 NÌI N€O CÂ Þ CH, NÌI  C CON ×ÍNG260dC¥u 10. Nghi»m cõa ph÷ìng tr¼nh log2(3x� 2) = 3 l  A x= 4 . B x= 10 3. C x= 11 2. D x= 5 . ÊLíi gi£i.Ta câ log2(3x� 2) = 3 ,3x � 2 = 2 3, 3x = 10 ,x= 10 3.Chån ¡p ¡n B dC¥u 11. Bi¸t3Z0 f(x ) d x= 6 v 10Z0 f(x ) d x= 10 . T½nh I= 10Z3 f(x ) d x. A 16. B 6. C 4. D �4. ÊLíi gi£i.Câ 10Z0 f(x ) d x= 3Z0 f(x ) d x+ 10Z3 f(x ) d x, 10Z3 f(x ) d x= 10Z0 f(x ) d x� 3Z0 f(x ) d x= 4 :Chån ¡p ¡n C dC¥u 12. Cho sè phùc z= 3 �2i. Ph¦n thüc cõa sè phùc w= iz � zl  A i. B 1. C �1. D 4. ÊLíi gi£i.Ta câ z= 3 + 2 i) w= iz � z= i(3 �2i) � (3 + 2 i) = �1 + i.Vªy sè phùc w= iz � zcâ ph¦n thüc l  �1.Chån ¡p ¡n C dC¥u 13. Trong khæng gian Oxyz, cho m°t ph¯ng (P ) : 2 x� y+ 2 z� 3 = 0 . V²c-tì n o d÷îi¥y khæng ph£i l  v²c-tì ph¡p tuy¸n cõa m°t ph¯ng (P )? A #n 1= (2;�1; 2) . B #n 3= (4;�2; 4) . C #n 2= (�2; 1; �2) . D #n 4= (6; 3; 6). ÊLíi gi£i.V²c-tì #n 4= (6; 3; 6)khæng l  v²c-tì ph¡p tuy¸n cõa mp (P ).Chån ¡p ¡n A dC¥u 14. Trong khæng gian Oxyz, cho hai iºm A(2; �4; 3) v B(2; 2; 7) . Trung iºm cõa o¤nth¯ng ABcâ tåa ë l  A (1; 3; 2). B (2;�1; 5) . C (2;�1; �5) . D (2; 6; 4). ÊLíi gi£i.Trung iºm Icõa o¤n th¯ng ABcâ tåa ë l  8>>>>><>>>>>: xI = xA +xB 2=2 + 2 2= 2y I = yA +yB 2=�4 + 2 2=�1z I = zA +zB 2=3 + 7 2= 5:Vªy I(2; �1; 5) .Chån ¡p ¡n B 260/305 260/305 pGV: L¶ Quang Xe  Ô0967.003.131PHT TRIšN — THAM KHƒO 2022 NÌI N€O CÂ Þ CH, NÌI  C CON ×ÍNG261dC¥u 15.Tr¶n m°t ph¯ng tåa ë, sè phùc z= 3 �4i ÷ñc biºu di¹n bði iºmn o trong c¡c iºm A,B ,C ,D ? A iºmD. B iºmB. C iºmA. D iºmC. x�4 3y�3 �4 �3 43�4 34OCD ABÊLíi gi£i.Ta câ z= 3 �4i n¶n iºm biºu di¹n sè phùc zl  D(3; �4) .Chån ¡p ¡n A dC¥u 16. ÷íng ti»m cªn ùng cõa ç thà h m sè y= x+ 1 x� 2câ ph÷ìng tr¼nh l  A y= 2 . B y= 1 . C x= 2 . D x= �2. ÊLíi gi£i.V¼ limx ! 2+ (x � 2) = 0 , khix! 2+n¶n x� 2> 0v  limx ! 2+ (x + 1) = 3 >0n¶n limx ! 2+ x+ 1 x� 2= +1.V¼ limx ! 2� (x � 2) = 0 , khix! 2�n¶n x� 2< 0v  limx ! 2� (x + 1) = 3 >0n¶n limx ! 2+ x+ 1 x� 2=�1 .ç thà h m sè y= x+ 1 x� 2câ ÷íng ti»m cªn ùng l x= 2 .Chån ¡p ¡n C dC¥u 17. Choa; b > 0;a; b 6= 1 v x; y l  hai sè thüc d÷ìng. Trong c¡c m»nh · d÷îi ¥y, m»nh· n o sai? A loga(xy ) = logax+ logay. B logbalogax= logbx. C loga1 x=1 logax . D logax y= logax� logay. ÊLíi gi£i.Ta câ m»nh · loga1 x=1 logax l  m»nh · sai.Chån ¡p ¡n C dC¥u 18.261/305 261/305 pGV: L¶ Quang Xe  Ô0967.003.131— SÈ 17 NÌI N€O CÂ Þ CH, NÌI  C CON ×ÍNG262Cho h m sèy= ax 3+ bx2+ cx +dcâ ç thà nh÷ h¼nh v³ b¶n. M»nh ·n o d÷îi ¥y óng? A a >0; b > 0; c = 0 ; d > 0. B a >0; b < 0; c > 0; d > 0. C a >0; b < 0; c = 0 ; d > 0. D a >0; b < 0; c < 0; d > 0. O xyÊLíi gi£i.ç thà h m sè tr¶n l  ç thà h m sè bªc ba câ a >0.ç thà h m sè c­t tröc tung t¤i iºm câ tung ë d÷ìng n¶n d >0.Câ f0( x ) = 3 ax2+ 2 bx+c;H m sè câ 2 iºm cüc trà trong â câ 1 cüc trà x1 = 0v  1 cüc trà x2 =kvîi k > 0.Khi â ph÷ìng tr¼nh f0( x ) = 0 câ hai nghi»m ph¥n bi»t trong â 1 nghi»m x1 = 0v  1 nghi»m x2 =kvîi k > 0.Suy ra 3a 02+ 2 b0 + c= 0 ,c= 0 .�p döng ành lþ Vi-²t: x1 +x2 =�2b 3a .Câ x1 +x2 =k > 0n¶n �2b 3a >0m  a > 0n¶n b <0.Chån ¡p ¡n C dC¥u 19. Trong khæng gian Oxyz, cho iºm A(3; �2; 1) . ÷íng th¯ng n o sau ¥y i quaA ? A 1: x� 3 1=y+ 2 1=z� 1 2. B 2: x� 3 4=y+ 2 �2 =z+ 1 �1 . C 3: x+ 3 1=y+ 2 1=z� 1 2. D 4: x� 3 4=y� 2 �2 =z� 1 �1 . ÊLíi gi£i.Ì V¼ 3� 3 1=�2 + 2 1=1� 1 2= 0n¶n ÷íng th¯ng 1: x� 3 1=y+ 2 1=z� 1 2i quaA.Ì V¼ 3� 3 4=�2 + 2 �2 6= 1 + 1 �1 n¶n ÷íng th¯ng2: x� 3 4=y+ 2 �2 =z+ 1 �1 khæng i quaA.Ì V¼ 3 + 3 16= �2 + 2 1n¶n ÷íng th¯ng3: x+ 3 1=y+ 2 1=z� 1 2khæng i quaA.Ì V¼ 3� 3 46= �2� 2 �2 n¶n ÷íng th¯ng4: x� 3 4=y� 2 �2 =z� 1 �1 khæng i quaA.Chån ¡p ¡n A dC¥u 20. Tø c¡c sè 1, 2, 3, 4, 5, 6 lªp ÷ñc bao nhi¶u sè tü nhi¶n gçm ba chú sè æi mët kh¡cnhau? A A36 . B C36 . C 6!. D 3!. ÊLíi gi£i.Sè c¡c sè tü nhi¶n gçm ba chú sè æi mët kh¡c nhau câ c¡c chú sè ÷ñc l§y trong bë sè 1, 2, 3, 4, 5,6 l  A36 .262/305 262/305 pGV: L¶ Quang Xe  Ô0967.003.131PHT TRIšN — THAM KHƒO 2022 NÌI N€O CÂ Þ CH, NÌI  C CON ×ÍNG263Chån ¡p ¡n A dC¥u 21. T½nh thº t½ch Vcõa h¼nh lªp ph÷ìng câ c¤nh b¬ng 2cm l  A V= 8 cm3. B V= 24 cm3. C V= 8 3cm3. D V= 4 cm3. ÊLíi gi£i.Ta câ: V=a3= 2 3= 8 cm3.Chån ¡p ¡n A dC¥u 22. T½nh ¤o h m cõa h m sè f(x ) = 2 x. A 2xln 2 . B x2x� 1. C 2x ln 2. D 2x. ÊLíi gi£i.Theo cæng thùc ta câ f0( x ) = 2 xln 2 .Chån ¡p ¡n A dC¥u 23.Cho h m sè y= f(x ) x¡c ành v  li¶n töc tr¶n R, câ ç thà ðh¼nh b¶n. H m sè y= f(x ) nghàch bi¸n tr¶n kho£ng n o d÷îi¥y? A (0; 1). B (�1 ; 0). C (1; 2). D (2; +1). xy�1 O 1 2ÊLíi gi£i.Nhªn th§y ç thà i xuèng trong kho£ng (0; 1), suy ra h m sè y= f(x ) nghàch bi¸n tr¶n kho£ng (0; 1).Chån ¡p ¡n A dC¥u 24. T½nh t½ch ph¥n I= ln 2Z0 �e4x+ 1 dx . A I= 15 4+ ln 2. B I= 4 + ln 2 . C I= 17 4+ ln 2. D I= 15 2+ ln 2. ÊLíi gi£i.Ta câ I= �1 4e4x+ x‹ ln 20 =�1 4e4 ln 2+ ln 2 ‹� 1 4=15 4+ ln 2.Chån ¡p ¡n A dC¥u 25. Cho c§p sè cëng câ sè h¤ng thù 3v  sè h¤ng thù 7l¦n l÷ñt l  6v  �2. T¼m sè h¤ngthù 5. A u5 = 4. B u5 =�2. C u5 = 0. D u5 = 2. ÊLíi gi£i.Ta câ ¨u3 = 6u 7 =�2 , ¨u1 + 2d= 6u 1 + 6d= �2 , ¨u1 = 10d = �2:263/305 263/305 pGV: L¶ Quang Xe  Ô0967.003.131— SÈ 17 NÌI N€O CÂ Þ CH, NÌI  C CON ×ÍNG264Do âu5 =u1 + 4d= 2 .Chån ¡p ¡n D dC¥u 26. T¼m nguy¶n h m cõa h m sè f(x ) = 2 4x � 3. A Z2 4x � 3dx = 1 4lnj4 x � 3j + C. B Z2 4x � 3dx = 1 2ln �2x � 3 2‹+ C. C Z2 4x � 3dx = 1 2ln 2x � 3 2 +C. D Z2 4x � 3dx = 2 ln 2x � 3 2 +C. ÊLíi gi£i.Ta câ Z2 4x � 3dx = 2 4lnj4 x � 3j + C1= 1 2�ln 2 + ln 2x � 3 2 ‹+ C1= 1 2ln 2x � 3 2 +C.Chån ¡p ¡n C dC¥u 27. Cho h m sè y= f(x ) câ ç thà nh÷ h¼nh b¶n.M»nh · n o d÷îi ¥y óng? A H m sè câ gi¡ trà cüc tiºu b¬ng2. B H m sè câ gi¡ trà lîn nh§t b¬ng2v  gi¡ trà nhä nh§t b¬ng �2. C H m sè ¤t cüc ¤i t¤ix= 0 v  ¤t cüc tiºu t¤i x= 2 . D H m sè câ ba cüc trà.xyO 1 2�2 2ÊLíi gi£i.Düa v o ç thà ta k¸t luªnÌ H m sè câ hai cüc trà.Ì H m sè câ gi¡ trà cüc tiºu b¬ng �2.Ì H m sè ¤t cüc ¤i t¤i x= 0 v  ¤t cüc tiºu t¤i x= 2 .Chån ¡p ¡n C dC¥u 28. GåiMv ml¦n l÷ñt l  gi¡ trà lîn nh§t, gi¡ trà nhä nh§t cõa h m sè y= x4 2�4x 2+ 1tr¶n [� 1; 3] . T½nh gi¡ trà cõa T= 2 M+m. A T= 4 . B T= �5. C T= 12 . D T= �6. ÊLíi gi£i.Ì Ta câ y0= 2 x2� 8x = 2 x(x 2� 4),y0= 0 ,2x (x 2� 4) = 0 ,264 x= 0 tho£ m¢nx = 2 tho£ m¢nx = �2 lo¤i :Ì Ta câ: y(� 1) = �5 2,y(0) = 1 ,y(2) = �7, y(3) = 11 2.Ì Gi¡ trà lîn nh§t, gi¡ trà nhä nh§t cõa h m sè tr¶n [� 1; 3] l M = maxx2 [� 1;3] y= f(3) = 11 2; m= minx2 [� 1;3] y= f(2) = �7:264/305 264/305 pGV: L¶ Quang Xe  Ô0967.003.131PHT TRIšN — THAM KHƒO 2022 NÌI N€O CÂ Þ CH, NÌI  C CON ×ÍNG265Suy ra ta câ2M +m = 2 11 2+ (�7) = 4 .Chån ¡p ¡n A dC¥u 29. Kho£ng nghàch bi¸n cõa h m sè y= x3+ 3 x2+ 4 l  A (�1 ; 0). B (�1 ;� 2) v  (0; + 1). C (2; +1). D (� 2; 0) . ÊLíi gi£i.Ta câ y0= 3 x2+ 6 x= 0 ,–x = 0x = �2:B£ng bi¸n thi¶n xy0 y �1�2 0 +1 +0 �0 +�1�1 8844 +1 +1 Suy ra h m sè nghàch bi¸n tr¶n(� 2; 0) .Chån ¡p ¡n D dC¥u 30. Cho c¡c sè thüc d÷ìng a; b;vîia6= 1 . Kh¯ng ành n o sau ¥y l  kh¯ng ànhóng? A loga2(ab ) = 1 2logab. B loga2(ab ) = 2 + 2 logab. C loga2(ab ) = 1 4logab. D loga2(ab ) = 1 2+1 2logab. ÊLíi gi£i.Ta câ loga2(ab ) = 1 2loga(ab ) = 1 2(1 + logab) = 1 2+1 2logab, n¶n c¥u D óng.Chån ¡p ¡n D dC¥u 31. Cho(H )l  khèi l«ng trö tam gi¡c ·u câ t§t c£ c¡c c¤nh b¬ng nhau, thº t½ch cõa( H )b¬ng p 34. ë d i c¤nh cõa khèi l«ng trö(H )l  A 3p 3. B p34. C 1. D 3p 163. ÊLíi gi£i.°t ë d i c¤nh cõa khèi l«ng trö (H )l  x, x > 0. Chi·u cao cõa l«ng tröb¬ng x.Di»n t½ch ¡y cõa l«ng trö b¬ng S= x2p 34.Suy ra thº t½ch khèi l«ng trö b¬ng V= x3p 34=p 34)x3= 1 ,x= 1(TM). B0 BA0 A C0 C265/305 265/305 pGV: L¶ Quang Xe  Ô0967.003.131— SÈ 17 NÌI N€O CÂ Þ CH, NÌI  C CON ×ÍNG266Chån ¡p ¡n C dC¥u 32. Cho1Z0 f(x ) d x= 3 ,2Z1 f(x ) d x= 2 . Khi â 2Z0 f(x ) d xb¬ng A 6. B �1. C 1. D 5. ÊLíi gi£i.2Z0 f(x ) d x= 1Z0 f(x ) d x+ 2Z1 f(x ) d x= 5 .Chån ¡p ¡n D dC¥u 33. Trong khæng gian Oxyz, m°t ph¯ng (P )song song vîi m°t ph¯ng (Oyz )v  i quaiºm A(1; 2; 3) câ ph÷ìng tr¼nh A x= 1 . B z= 3 . C y= 2 . D x+ y+ z� 6 = 0 . ÊLíi gi£i.M°t ph¯ng (Oyz )câ ph÷ìng tr¼nh l  x= 0 .M°t ph¯ng (P )song song vîi m°t ph¯ng (Oyz )s³ câ ph÷ìng tr¼nh têng qu¡t l  x= m, m  A(1; 2; 3) 2( P )n¶n m= 1 .Vªy ph÷ìng tr¼nh m°t ph¯ng (P )c¦n t¼m l  x= 1 .Chån ¡p ¡n A dC¥u 34. Cho sè phùc zthäa m¢n j2 z � 1j = j z+ 1 + ij v  iºm biºu di¹n cõa ztr¶n m°t ph¯ngtåa ë thuëc ÷íng trán câ t¥m I(1; 1) , b¡n k½nh R= p 5. Khi â t½ch mæun cõa t§t c£ c¡c sèphùc zthäa m¢n c¡c y¶u c¦u tr¶n l ? A p5. B 3. C 3p 5. D 1. ÊLíi gi£i.Gåi z= a+ bi(a; b 2R).Theo gi£ thi¸t ta câj2( a+ bi)� 1j = ja � bi+ 1 + ij, j (2a� 1) + 2 bij= j( a + 1) �(b � 1)ij, (2a� 1)2+ (2 b)2= ( a+ 1) 2+ ( b� 1)2, 3a 2+ 3 b2� 6a + 2 b� 1 = 0 : (1)V¼ iºm biºu di¹n cu© ztr¶n m°t ph¯ng tåa å thuëc ÷íng trán t¥m I(1; 1) ; R=p 5n¶n ta câ( a � 1)2+ ( b� 1)2= 5, a2+ b2� 2a � 2b = 3, a2� 2a = 3 �b2+ 2 b: (2)Th¸ (2)v o (1)ta ÷ñc 3(3�b2+ 2 b) + 3 b2+ 2 b� 1 = 0 ,b= �1.Khi â, thay v o (2)ta suy ra –a = 0a = 2 )–z1 =�1z 2 = 2�i) jz1j  jz2j= p 5.Chån ¡p ¡n A 266/305 266/305 pGV: L¶ Quang Xe  Ô0967.003.131PHT TRIšN — THAM KHƒO 2022 NÌI N€O CÂ Þ CH, NÌI  C CON ×ÍNG267dC¥u 35. Cho h¼nh châp S:ABCcâ ¡yABCl  tam gi¡c vuæng c¥n t¤i A, câ AB=AC = 1,H¼nh chi¸u vuæng gâc cõa Sl¶n m°t ph¯ng ¡y l  trung iºm Mcõa c¤nh BC, gâc giúa S Mv ( S AB )b¬ng 60. Thº t½ch khèi châp S:ABCb¬ng A p336. B p312. C p618. D p636. ÊLíi gi£i.Gåi Nl  trung iºm AB) AB ?M N (M N AC ); M N =1 2AC=1 2:Do vªy AB?(S M N )(V¼ AB vuæng gâc vîi hai ÷íng th¯ng c­tnhau S M; M N trong m°t ph¯ng (S M N )).K´ M H ?S N )M H ?(S AB ). Do M H vuæng gâc vîi hai ÷íngth¯ng c­t nhau S N; ABtrong m°t ph¯ng (S AB ).) (S M; (S AB )) = ( S M; S H ) =ÖM S H =ÖM S N = 60:tan ÖM S N =M N S M)S M =p 36:) VS:ABC =1 3S M S4 ABC =p 336: A CB MN SHChån ¡p ¡n A dC¥u 36. Câ bao nhi¶u sè tü nhi¶n câ 5 chú sè câ d¤ng abcdethäa m¢n a6 b6 c6 d6 e? A 126. B 1287. C 27216. D 90000. ÊLíi gi£i.TH1. a < b < c < d < eV¼ chú sè ùng tr÷îc luæn nhä hìn chú sè ùng sau n¶n sè â khæng thº câ chú sè 0.Tø tªp f1; 2; 3; 4; 5; 6; 7; 8; 9 gta chia l m c¡c tªp con câ 5 ph¦n tû. Câ t§t c£ C59 tªp. X²t 1 tªpgçm 5 chú sè a;b; c; d ;e b§t k¼. Khi â, tø 5 chú sè a;b; c; d ;e ta lªp ÷ñc duy nh§t mët sè l  abcdethäa m¢n y¶u c¦u · b i. Vªy câ t§t c£ C59 sè.Khi c¡c chú sè khæng æi mët kh¡c nhau th¼ x£y ra nhúng tr÷íng hñp sau:TH2.Câ 1 d§u  = x£y ra: Câ C14 C 49 sè thäa m¢n.TH3.Câ 2 d§u  = x£y ra: Câ C24 C 39 sè thäa m¢n.TH4.Câ 3 d§u  = x£y ra: Câ C34 C 29 sè thäa m¢n.TH5.Câ 4 d§u  = x£y ra: Câ C44 C 19 sè thäa m¢n.Vªy câ t§t c£ c¡c sè thäa m¢n y¶u c¦u l  C59 + C 14 C 49 + C 24 C 39 + C 34 C 29 + C 44 C 19 = 1287.Chån ¡p ¡n B dC¥u 37. Trong khæng gian Oxyz, cho hai iºm A(2; �2; �1) ; B �� 4 3;� 8 3;8 3‹. ÷íng th¯ng i qua t¥m ÷íng trán nëi ti¸p tam gi¡c OABv  vuæng gâc vîi m°t ph¯ng (OAB ). Häi i quaiºm n o d÷îi ¥y?267/305 267/305 pGV: L¶ Quang Xe  Ô0967.003.131— SÈ 17 NÌI N€O CÂ Þ CH, NÌI  C CON ×ÍNG268A Q(5; �1; 5) . B N(3; 0; 2) . C M(1; �1; 1) . D P(� 5; �4; 5) . ÊLíi gi£i.Ta câ: OA= 3; OB = 4; AB = 5.Gåi Il  t¥m ÷íng trán nëi ti¸p tam gi¡c OAB.) 8>>>>>>>>>>>><>>>>>>>>>>>>: xI = ABxO +OB xA +OA xB AB+OB +OA =50 + 4 2 + 3 �� 4 3‹ 5 + 4 + 3=1 3y I = AByO +OB yA +OA yB AB+OB +OA =50 + 4 (� 2) + 3 �� 8 3‹ 5 + 4 + 3=�4 3z I = ABzO +OB zA +OA zB AB+OB +OA =50 + 4 (� 1) + 3 8 35 + 4 + 3=1 3)I�1 3;� 4 3;1 3‹.” # OA; # OB —= ( �8; �4; �8) = �4(2; 1; 2) .Ph÷ìng tr¼nh ÷íng th¯ng  :x� 1 32=y+ 4 31=z� 1 32.÷íng th¯ng i qua iºm M(1; �1; 1) .Chån ¡p ¡n C dC¥u 38. Cho d¢y sè (an)thäa m¢n a1 = 1v an = 10an� 1 �1, 8n  2. T¼m gi¡ trà nhä nh§tcõa nº log an >100 . A 100. B 101. C 102. D 103. ÊLíi gi£i.Ta câ an = 10an� 1 �1, an � 1 9= 10 �an� 1 � 1 9‹. (1).°t bn =an � 1 9)b1 =a1 � 1 9=8 9. Tø(1))bn = 10bn � 1;8n  2D¢y (bn )l  c§p sè nh¥n vîi cæng bëi l  q= 10 . Suy ra bn =b1 qn� 1= 8 910 n� 1.Do â an =bn + 1 9=8 910n� 1+ 1 9;8n = 1 ;2 ; : : : .Ta câ logan >100 ,an >10100,8 910n� 1+ 1 9>10100.Vªy gi¡ trà nhä nh§t cõa nº log an >100 l n= 102 .Chån ¡p ¡n C dC¥u 39. Bi¸t ÷íng th¯ng y= 2 x+ 2 mluæn c­t ç thà h m sè y= x2+ 3 x+ 1 t¤i hai iºm ph¥nbi»t A,B vîi måi gi¡ trà tham sè m. T¼m ho nh ë trung iºm cõa AB? A m+ 1 . B �m �1. C �2m �2. D �2m + 1 . ÊLíi gi£i.Ph÷ìng tr¼nh ho nh ë giao iºm giúa ÷íng th¯ng y= 2 x+ 2 mv  ç thà h m sè y= x2+ 3 x+ 12 x + 2 m= x2+ 3 x+ 1 ,¨(2 x+ 2 m)(x + 1) = x2+ 3x + 1 6= 0 ,¨x2+ 2( m+ 1) x+ 2 m�3 = 0( )x 6= �1 .Gåi xA ,xB l  hai nghi»m ph¥n bi»t cõa ph÷ìng tr¼nh( ).Theo ành lþ Vi-et xA +xB =�2( m + 1) .268/305 268/305 pGV: L¶ Quang Xe  Ô0967.003.131PHT TRIšN — THAM KHƒO 2022 NÌI N€O CÂ Þ CH, NÌI  C CON ×ÍNG269Khi â ho nh ë trung iºm cõaABb¬ng: xA +xB 2=�2( m + 1) 2=�m �1.Chån ¡p ¡n B dC¥u 40. Cho h m sè f0( x ) li¶n töc tr¶n R. Bi¸t x4l  mët nguy¶n h m cõa h m sè f0( x )e x, håt§t c£ c¡c nguy¶n h m cõa h m sè f00(x )e xl  A 4x 3� x4+ C. B 4x 3+ x2+ C. C x3� 4x 4+ C. D x3� x4+ C. ÊLíi gi£i.Do x4l  mët nguy¶n h m cõa h m sè f0( x )e xn¶n f0( x )e x= ( x4)0= 4 x3. ( ).¤o h m hai v¸ cõa ( ) ta ÷ñc f00(x )e x+ f0( x )e x= 12 x2.K¸t hñp vîi ( ) thu ÷ñc f00(x )e x= 12 x2� 4x 3.Do â Zf00(x )e xdx = Z(12 x2� 4x 3) d x= 4 x3� x4+ C.Chån ¡p ¡n A dC¥u 41.Cho l«ng trö tam gi¡c ABC:A0B 0C 0câ ¡y ABCl  tam gi¡c ·uc¤nh a:H¼nh chi¸u vuæng gâc cõa A0l¶n m°t ph¯ng (ABC )l trung iºm cõa AB:N¸uAC0v  A0B vuæng gâc vîi nhau th¼ khèil«ng trö ABC:A0B 0C 0câ thº t½ch l  A p6a 3 2. B p6a 3 4. C p6a 3 8. D p6a 3 24. AC BA0 B0 C0 ÊLíi gi£i.Di»n t½ch ¡y l  SABC =a2p 34.Gåi H,H 0l¦n l÷ñt l  trung iºm cõa AB,A 0B 0v  K=AH 0\ A0B .Ta câ C H?AB; C H ?A0H ) C H ?(AA 0B 0B ) ) C0H 0?( AA 0B 0B )) C0H 0? A0B .L¤i câ A0B ?C0H 0; A 0B ?AC 0) A0B ?(AC 0H )) A0B ?AH 0(t¤iK ).°t A0H =x) H0B =x. AC BA0 B0 C0 H0 H KTa câAB= 2A0H 0) K B =2 3A0B = 2 3É x2+ a2 4; K A=2 3AH0= 2 3p x2+ a2.4 K AB vuæng t¤i Kn¶nK B2+ K A 2= a2, 4 9�2x 2+ 5a 2 4‹= a2, 8x 2+ 5 a2= 9 a2, x= ap 22:Vªy V=SABC A 0H =a2p 34ap 22=a3p 68.Chån ¡p ¡n C dC¥u 42. Cho sè phùc zthäa m¢n z(2 �i) + 13 i= 1 . T½nh mæ-un cõa sè phùc z269/305 269/305 pGV: L¶ Quang Xe  Ô0967.003.131— SÈ 17 NÌI N€O CÂ Þ CH, NÌI  C CON ×ÍNG270A jz j = p 34. B jz j = p 343. C jz j = 5p 343. D jz j = 34 . ÊLíi gi£i.C¡ch 1 Ta câz(2 �i) + 13 i= 1 ) jz(2 �i) j = j1 � 13ij , j zj  p 5 =p 170, jzj = p 34:C¡ch 2 Ta câz(2 �i) + 13 i= 1 ,z(2 �i) = 1 �13i, z(2 �i)(2 + i) = (1 �13i)(2 + i) , 5z = 15 �25i, z= 3 �5i) j zj = p 32+ 5 2= p 34.Chån ¡p ¡n A dC¥u 43. Cho sè phùc z= m+ 1 + mivîi m2R. Häi câ bao nhi¶u gi¡ trà nguy¶n cõam 2(� 5; 5) sao cho jz � 2ij > 1? A 5. B 4. C 0. D 9. ÊLíi gi£i.Ta câjz � 2ij > 1 , j m+ 1 + mi�2ij > 1, È (m + 1) 2+ ( m�2)2> 1, 2m 2� 2m + 5 >1, 2m 2� 2m + 4 >0, m2R:V¼ m2(� 5; 5) v ml  sè nguy¶n n¶n m2 f� 4;�3; �2; �1; 0; 1; 2; 3; 4 g.Chån ¡p ¡n D dC¥u 44.Cho h m sè y= f(x ) câ ç thà nh÷ h¼nh v³ b¶n. Sè gi¡trà nguy¶n cõa tham sè mº h m sè y= jf (x ) + mjcâ baiºm cüc tiºu l  A 2. B 4. C 5. D 3. xyO�4 ÊLíi gi£i.Nhªn th§y h m sè y= f(x ) + mcâ gi¡ trà cüc ¤i yC =m v  gi¡ trà cüc tiºu yCT =�4 + m. Ta v³ç thà h m sè y= jf (x ) + mj(ph¦n ÷íng cong n¬m ph½a tr¶n tröc ho nh ) trong c¡c tr÷íng hñp sauÌ yC =m 0270/305 270/305 pGV: L¶ Quang Xe  Ô0967.003.131PHT TRIšN — THAM KHƒO 2022 NÌI N€O CÂ Þ CH, NÌI  C CON ×ÍNG271xyO�4 ÌyCT =�4 + m0 xyO�4 ̨yC =m > 0y CT =�4 + m < 0 xyO�4 Tø â suy ra, vîi0< m < 4th¼ h m sè y= jf (x ) + mjcâ ba iºm cüc tiºu.Vªy câ 3gi¡ trà nguy¶n cõa tham sè mº h m sè y= jf (x ) + mjcâ ba iºm cüc tiºu.Chån ¡p ¡n D dC¥u 45. Trong khæng gian Oxyz, cho ÷íng th¯ng vuæng gâc m°t ph¯ng271/305 271/305 pGV: L¶ Quang Xe  Ô0967.003.131— SÈ 17 NÌI N€O CÂ Þ CH, NÌI  C CON ×ÍNG272(P ) : 7 x+ y� 4z = 0 , c­t c£ hai ÷íng th¯ng d1: x 2=y� 1 �1 =z+ 2 1v d2: 8><>: x= �1 + 2 ty = 1 + tz = 3câ ph÷ìng tr¼nh ch½nh t­c l  A  :x� 2 �7 =y �1 =z+ 1 4: B 8><>: x= 2 �7ty = �tz = �1 + 4 t: C  :x+ 2 �7 =y� 3 �1 =z+ 1 4: D  :x+ 7 �5 =y+ 1 �1 =z� 4 3: ÊLíi gi£i.Mp (P )câ mët VTPT l  #n = (7; 1; 4) .d 1 câ ph÷ìng tr¼nh tham sè l  8><>: x= 2 my = 1 �mz = �2 + m:Gi£ sû c­t d1 t¤iA,) A(2 m; 1 �m;� 2 + m).Gi£ sû c­t d2 t¤iB,) B(� 1 + 2 t; 1 + t; 3) .Suy ra # AB = (2 t� 1� 2m ;t + m; 5 �m).V¼ ?(P )n¶n # AB còng ph÷ìng vîi #n .Do â2 t � 1� 2m 7=t+ m 1=5� m �4 =k, 8><>: 2t � 1� 2m = 7 kt + m =k5 � m =�4k ,8><>: t= �2m = 1k = �1:Suy ra A(2; 0; �1) ,# AB = (�7; �1; 4) .Vªy ph÷ìng tr¼nh ch½nh t­c cõa l  x� 2 �7 =y �1 =z+ 1 4. d1 d2 #n P ABChån ¡p ¡n A dC¥u 46. Cho tù di»n ABC DcâAD ?(ABC ), ABC l  tam gi¡c vuæng t¤i B. Bi¸t BC=a,AB =ap 3, AD = 3a. Quay c¡c tam gi¡c ABCv ABD (bao gçm c£ iºm b¶n trong 2tam gi¡c)xung quanh ÷íng th¯ng ABta ÷ñc 2khèi trán xoay. Thº t½ch ph¦n chung cõa 2khèi trán xoayâ b¬ng A 5p 3a 3 16. B 3p 3a 3 16. C 8p 3a 3 3. D 4p 3a 3 16. ÊLíi gi£i.272/305 272/305 pGV: L¶ Quang Xe  Ô0967.003.131PHT TRIšN — THAM KHƒO 2022 NÌI N€O CÂ Þ CH, NÌI  C CON ×ÍNG273ADB C D1 D2 C1 C2 N MAB HTù di»nABC Dcâ¨DA ?(ABC )BC ?(ABD ), do â khi quay hai tam gi¡cABCv ABD quanh tröc ABta s³÷ñc hai h¼nh nân trán xoay, v  ph¦n chung cõa hai h¼nh nân n y l  ph¦n tæ m u x¡m ð h¼nh tr¶n.Khi â x²t m°t ph¯ng qua tröc ABcõa hai h¼nh nân (C1C2D2D1), gåi M=BD2\AC2;N =BD 1\AC1.Ta câ 4BM C2v4D2M A, n¶nBM D2M =BC2 AD2=1 3)H M AD2=1 4)H M BC2=3 4)H M =3a 4:Suy ra Ì Thº t½ch khèi nân (AN M )l  V1 = 1 3H M2AH =1 39a 2 16AH .Ì Thº t½ch khèi nân (BN M )l  V2 = 1 3H M2BH =1 39a 2 16BH .Vªy thº t½ch khèi trán xoay c¦n t¼m l V1 +V2 = 1 39a 2 16(AH +BH ) = 1 39a 2 16AH =3p 3a 3 16:Chån ¡p ¡n B dC¥u 47. Câ t§t c£ bao nhi¶u bë sè nguy¶n (x ;y ;z ) thäa m¢n çng thíi e2x + y+ z� 3 2x + y+ z� 2v  z2� yz +x= 0 ? A 5. B 2. C 4. D 7. ÊLíi gi£i.°t t= 2 x+ y+ z� 3. Ta câe 2x + y+ z� 3 2x + y+ z� 2trð th nh et t+ 1 ,et� t� 1 0. ( )X²t h m sè f(t) = e t� t� 1tr¶n R.Ta câ f0( t) = e t� 1; f0( t) = 0 ,t= 1 . tf0( t) f(t) �10 +1 �0 ++1 +1 00 +1 +1273/305 273/305 pGV: L¶ Quang Xe  Ô0967.003.131— SÈ 17 NÌI N€O CÂ Þ CH, NÌI  C CON ×ÍNG274Tø b£ng bi¸n thi¶n ta câf(t)  0, 8t 2 R.Do â ( ) , t= 0 .B i to¡n trð th nh: T¼m t§t c£ bao nhi¶u bë sè nguy¶n (x ;y ;z ) thäa m¢n çng thíi¨ 2x + y+ z� 3 = 0 (1)z 2� yz +x= 0 : (2)Ta câ (1),x= �(y + z� 3) 2.Th¸ v o (2)ta ÷ñcz2� yz �y+ z� 3 2= 0, 2z 2� 2yz �y� z+ 3 = 0, y(2 z+ 1) = 2 z2� z+ 3, y= 2z 2� z+ 3 2z + 1, y= z� 1 + 4 2z + 1 :M  z2 Z n¶n2666666666666664 2z + 1 = 1 )z= 0 )y= 3 )x= 02 z + 1 = �1 )z= �1) y= �6) x= 52 z + 1 = 2 )z= 1 2(lo¤i)2 z + 1 = �2 )z= �3 2(lo¤i)2 z + 1 = 4 )z= 3 2(lo¤i)2 z + 1 = �4 )z= �5 2(lo¤i).Vªy câ 2bë sè nguy¶n thäa m¢n l  (0; 3; 0)v (5; �6; �1) .Chån ¡p ¡n B dC¥u 48. Trong khæng gian vîi h» tröc tåa ë Oxyz, cho hai iºm M(1; 2; 1) ; N(� 1; 0; �1) .Câ bao nhi¶u m°t ph¯ng qua M; Nc­t c¡c tröc Ox,Oy l¦n l÷ñt t¤i Av  B,(A 6= B)sao choAM =p 3BN ? A 1. B 2. C 4. D Væ sè.ÊLíi gi£i.Gåi #n = ( a;b; c) l  mët ph¡p v²c-tì cõa mp( ) qua M; N thäa m¢n · b i (vîi a2+ b2+ c2> 0).Ta câ ¨# M N = (�2; �2; �2)# n ? # M N )a+ b+ c= 0 )c= �a� b6= 0 :V¼ mp ( ) c­t Ox; Oy l¦n l÷ñt t¤i A; Bn¶nab 6= 0 .Khi â, ta ÷ñc ph÷ìng tr¼nh mp( ) : ax +by �(a + b)z � b= 0 .Ta t½nh ÷ñc 8<: A�b a; 0; 0 ‹B (0; 1; 0) )8<: # AM =�1 � b a; 2; 1 ‹# BN = (�1; �1; �1) :274/305 274/305 pGV: L¶ Quang Xe  Ô0967.003.131PHT TRIšN — THAM KHƒO 2022 NÌI N€O CÂ Þ CH, NÌI  C CON ×ÍNG275Theo · b i ta câAM=p 3BN, AM2= 3 BN 2, �1 � b a‹2+ 5 = 3 3, 264 b a=�1b a= 3:Vîi b a=�1 ta chån 8><>: a= 1b = �1c = 0 , ta ÷ñc( ) : x� y� 1 = 0 .Vîi b a= 3ta chån 8><>: a= 1b = 3c = �4, ta ÷ñc( ) : x+ 3 y� 4z � 3 = 0 .Chån ¡p ¡n B dC¥u 49.Cho hai h m a thùc y= f(x ), y = g(x ) câ ç thà l  hai ÷íng congð h¼nh v³ d÷îi. Bi¸t r¬ng ç thà h m sè y= f(x ) câ óng mët iºmcüc trà l  B, ç thi h m sè y= g(x ) câ óng mët iºm cüc trà l A v  AB =7 4. Câ bao nhi¶u gi¡ trà nguy¶n cõa tham sèmthuëckho£ng (� 5; 5) º h m sè y= jjf (x ) � g(x )j + mjcâ óng 5iºmcüc trà? A 1. B 3. C 4. D 6. O xyAB y= f(x ) y= g(x ) ÊLíi gi£i.Ì X²t h m h(x ) = f(x ) � g(x ) câ h0( x ) = f0( x ) � g0( x ). Gåi ho nh ë cõa Av  Bl x0 th¼ th§yr¬ng Vîi x2 (�1 ;x0)th¼ f0( x ) < 0; g0( x ) > 0) h0( x ) < 0. Vîi x2 (x0; +1)th¼ f0( x ) > 0; g0( x ) < 0) h0( x ) > 0. T¤i x= x0 th¼h0( x ) = 0Vªy h m h0( x ) ch¿ câ mët cüc trà t¤i x0 v  â l  cüc tiºu.Ì B£ng bi¸n thi¶n cõa h(x ) xh0( x ) h(x ) �1 x0 +1 �0 ++1 +1 �7 4�7 4 +1 +1275/305 275/305 pGV: L¶ Quang Xe  Ô0967.003.131MÖC LÖCNÌI N€O CÂ Þ CH, NÌI  C CON ×ÍNG276Tø â, ta câ b£ng bi¸n thi¶n cõau(x ) = jh (x )j l  (vîi x1,x2 l  hai giao iºm cõay= f(x ) v y = g(x )) xu0( x ) u(x ) �1 x1 x0 x2 +1 � +0 � ++1 +1 00 747400 +1 +1 ÌKhi mthay êi th¼ ç thà cõa h m jh (x )j + m v¨n câ sè iºm cüc trà b¬ng sè iºm cüc trà cõah m jh (x )j v  b¬ng 3. Khi â sè iºm cüc trà cõa ç thà h m jjh (x )j + mjs³ b¬ng 3cëng vîi sègiao iºm cõa nâ vîi tröc Ox(khæng tròng vîi iºm cüc trà).Ì Do â h m sè jjh (x )j + mjcâ 5iºm cüc trà , jh(x )j + m = 0 , jh(x )j = �m câ óng 2nghi»mph¥n bi»t kh¡c x1; x2, �m > 7 4,m < �7 4. Suy ra câ3gi¡ trà nguy¶n cõa mthuëc (� 5; 5)thäa m¢n.Chån ¡p ¡n B H˜T276/305 276/305 pGV: L¶ Quang Xe  Ô0967.003.131PHT TRIšN — THAM KHƒO 2022 NÌI N€O CÂ Þ CH, NÌI  C CON ×ÍNG277BË GIO DÖC & €O T„OTR×ÍNG THPT NGUY™N T‡T TH€NH GV: L– QUANG XE - 0967.003.131 — SÈ 18 PHT TRIšN — THAM KHƒO 2022N‹M HÅC 2021 - 2022Mæn:To¡nThíi gian l m b i: 90 phót — THAM KHƒO HKIIdC¥u 1. Cho sè phùc zthäa m¢n: 3z = 9 �6i. Sè phùc li¶n hñp cõa zl  A z= 9 + 6 i. B z= 3 . C z= 27 + 18 i. D z= 3 + 2 i. ÊLíi gi£i.Ta câ 3z = 9 �6i , z= 3 �2i. Suy ra z= 3 + 2 i.Chån ¡p ¡n D dC¥u 2. Trong khæng gian Oxyz, b¡n k½nh cõa m°t c¦u (S ) : x2+ y2+ ( z+ 3) 2= 25 b¬ng A 3. B 5. C 25. D 4. ÊLíi gi£i.M°t c¦u (S ) câ b¡n k½nh R= p 25 = 5.Chån ¡p ¡n B dC¥u 3. ç thà h m sè y= x3+ 3 x2� 2nhªn : A Tröc tung l m tröc èi xùng. B Gèc tåa ë O l m t¥m èi xùng.C iºmI(� 1; 0) l m t¥m èi xùng. D ÷íng th¯ngx= 1 l m tröc èi xùng. ÊLíi gi£i.Ta câ: y0= 3 x2+ 6 x= 0y 00= 6 x+ 6 = 0 ,x= �1) y= 0 .H m sè y= x3+ 3 x2� 2l  h m a thùc bªc ba n¶n nhªn iºm I(� 1; 0) l m t¥m èi xùng.Chån ¡p ¡n C dC¥u 4. Thº t½ch Vcõa khèi c¦u b¡n k½nh 6cm l  A V= 216 (cm 3). B V= 288 (cm 3). C V= 432 (cm 3). D V= 864 (cm 3). ÊLíi gi£i.Thº t½ch khèi c¦u b¡n k½nh 6cm l  V= 4 363= 288 (cm 3).Chån ¡p ¡n B dC¥u 5. T¼m nguy¶n h m cõa h m sè f(x ) = 7 x. A Z7xdx = 7 xln 7 + C. B Z7xdx = 7x ln 7+C. C Z7xdx = 7 x+1+C. D Z7xdx = 7x+1 x+ 1 +C. ÊLíi gi£i.Ta câ Z7xdx = 7x ln 7+C.277/305 277/305 pGV: L¶ Quang Xe  Ô0967.003.131— SÈ 18 NÌI N€O CÂ Þ CH, NÌI  C CON ×ÍNG278Chån ¡p ¡n B dC¥u 6. Cho h m sè f(x ) câ b£ng bi¸n thi¶n nh÷ sau xy0 y �1�1 0 1 +1 �0 +0 � ++1 +1 �4 �4 �3 �3 �4 �4 +1 +1 H m sè ¤t cüc ¤i t¤ix0 b¬ng A �3. B �4. C 0. D 1. ÊLíi gi£i.Tø b£ng bi¸n thi¶n suy ra h m sè ¤t cüc ¤i t¤i x0 = 0.Chån ¡p ¡n C dC¥u 7. Bi¸t r¬ng S l  tªp nghi»m cõa b§t ph÷ìng tr¼nh log(�x2+ 100 x� 2400) <2câ d¤ngS = ( a;b) n f x0g. Gi¡ trà a+ b� x0 b¬ng A 50. B 150. C 30. D 100. ÊLíi gi£i.i·u ki»n �x2+ 100 x� 2400 >0, 40< x < 60:Ta câlog(�x2+ 100 x� 2400) <2, � x2+ 100 x� 2400 <102, (x � 50) 2> 0, x6= 50 :Vªy S= (40; 60) n f50g. Do â a+ b� x0 = 50.Chån ¡p ¡n A dC¥u 8. Khèi châp câ di»n t½ch ¡y l  Sv  chi·u cao l  hth¼ thº t½ch cõa khèi châp â l  A 12Sh . B 13Sh . C Sh . D 16Sh . ÊLíi gi£i.Ta câ cæng thùc t½nh thº t½ch khèi châp l  V= 1 3S h .Chån ¡p ¡n B dC¥u 9. Tªp x¡c ành Dcõa h m sè y= (2 �x)�3l  A D=R. B D=Rn f 2g . C D=Rn f 0g . D D= ( �1 ; 2). ÊLíi gi£i.i·u ki»n 2� x6= 0 ,x6= 2 . Do â D=Rn f 2g .Chån ¡p ¡n B dC¥u 10. T¼m tªp nghi»m Scõa ph÷ìng tr¼nh log4x= 3 . A S= f12 g. B S= ?. C S= f64 g. D S= f81 g. ÊLíi gi£i.278/305 278/305 pGV: L¶ Quang Xe  Ô0967.003.131PHT TRIšN — THAM KHƒO 2022 NÌI N€O CÂ Þ CH, NÌI  C CON ×ÍNG279Ta câlog4x= 3 ,x= 4 3= 64 .Chån ¡p ¡n C dC¥u 11. N¸u2Z1 f(x ) d x= 3 v 3Z2 f(x ) d x= 4 th¼ 3Z1 f(x ) d xcâ gi¡ trà b¬ng A �1. B 7. C 1. D 12. ÊLíi gi£i.�p döng t½nh ch§t t½ch ph¥n ta câ3Z1 f(x ) d x= 2Z1 f(x ) d x+ 3Z2 f(x ) d x= 3 + 4 = 7 :Chån ¡p ¡n B dC¥u 12. Cho hai sè phùc: z1 = 1�2i, z2 = 2 + 3i. T¼m sè phùc w= z1 �2z2. A w= �3 + 8 i. B w= �5 + i. C w= �3� 8i. D w= �3 + i. ÊLíi gi£i.Ta câ w= z1 �2z2 = (1�2i) � 2(2 + 3 i) = �3� 8i.Chån ¡p ¡n C dC¥u 13. Trong khæng gian vîi h» tåa ë Oxyz, cho m°t ph¯ng ( ) : 2 x+ y� 3z � 1 = 0 . V²c-tìn o sau ¥y l  v²c-tì ph¡p tuy¸n cõa ( )? A #n = (2; �1; 3) . B #n = ( �2; 1; 3) . C #n = ( �4; �2; 6) . D #n = (2; 1; 3) . ÊLíi gi£i.( ) câ mët VTPT l  #n 0= (2; �1; 3) n¶n#n = �2#n 0= ( �4; �2; 6) công l  mët VTPT cõa ( ).Chån ¡p ¡n C dC¥u 14. Trong khæng gian Oxyz, cho v²c-tì # OA =#j � 2#k . Tåa ë iºm Al  A (0; 1;�2) . B (1;�2; 0) . C (1; 0;�2) . D (0;�1; 2) . ÊLíi gi£i.Ta câ # OA = 0#i + 1 #j � 2#k . Vªy tåa ë iºm Al  (0; 1; �2) .Chån ¡p ¡n A dC¥u 15. Trong m°t ph¯ng tåa ë Oxy, cho iºm M(� 2; 1) . Häi iºm Ml  iºm biºu di¹n cõasè phùc n o sau ¥y? A z= 2 �i. B z= �2 + i. C z= �1 + 2 i. D z= 1 �2i. ÊLíi gi£i.M (� 2; 1) )z= �2 + i:Chån ¡p ¡n B dC¥u 16. T¼m ti»m cªn ngang cõa ç thà h m sè y= 1 + x� 1 x+ 1 .279/305 279/305 pGV: L¶ Quang Xe  Ô0967.003.131— SÈ 18 NÌI N€O CÂ Þ CH, NÌI  C CON ×ÍNG280A y= 2 . B y= 1 . C x= �1. D x= 1 . ÊLíi gi£i.Ta câ: limx ! +1 y= 2 . Vªy y= 2 l  ti»m cªn ngang cõa ç thà h m sè.Chån ¡p ¡n A dC¥u 17. Choa; bl  c¡c sè thüc d÷ìng. M»nh · n o sau ¥y óng? A logab= log a+ log b. B logab= log alog b. C loga b=loga logb. D loga b= logb� log a. ÊLíi gi£i.M»nh · óng l  logab= log a+ log b.Chån ¡p ¡n A dC¥u 18.÷íng cong trong h¼nh b¶n l  ç thà cõa mët h m sè trong bèn h m ÷ñcli»t k¶ ð bèn ph÷ìng ¡n A; B; C; Dd÷îi ¥y. Häi h m sè â l  h m sèn o? A y= �x4� 2x 2. B y= x4� 3x 2+ 1 . C y= x4+ 2 x2. D y= x4� 2x 2. xyO�1 1ÊLíi gi£i.Nh¼n v o ç thà ta nhªn th§y ¥y l  ç thà cõa h m sè tròng ph÷ìng y= ax 4+ bx2+ c.Do nh¡nh cuèi còng cõa ç thà i l¶n n¶n a >0.Do f(0) = 0 )c= 0 .H m sè câ 3 cüc trà n¶n ab < 0. Suy ra b <0.Chån ¡p ¡n D dC¥u 19. Trong khæng gian Oxyz, cho ÷íng th¯ng câ ph÷ìng tr¼nh x� 1 2=y+ 1 �1 =z� 2 3.iºm n o d÷îi ¥y thuëc ÷íng th¯ng ¢ cho? A Q(� 2; 1; �3) . B P(2; �1; 3) . CM(� 1; 1; �2) . D N(1; �1; 2) . ÊLíi gi£i.L¦n l÷ñt thay tåa ë cõa c¡c iºm ¢ cho v o ph÷ìng tr¼nh ÷íng th¯ng, ta th§y N(1; �1; 2) thuëc÷íng th¯ng ¢ cho.Chån ¡p ¡n D dC¥u 20. Câ bao nhi¶u c¡ch th nh lªp mët ban c¡n sü lîp gçm 3ng÷íi ÷ñc chån tø 16håcsinh trong lîp? A A316 . B 163. C 316. D C316 . ÊLíi gi£i.Sè c¡ch chån ra mët ban c¡n sü 3ng÷íi tø 16håc sinh l  mët tê chñp chªp 3cõa 16ph¦n tû v  b¬ngC 316 .Chån ¡p ¡n D 280/305 280/305 pGV: L¶ Quang Xe  Ô0967.003.131PHT TRIšN — THAM KHƒO 2022 NÌI N€O CÂ Þ CH, NÌI  C CON ×ÍNG281dC¥u 21. Mët khèi châp câ thº t½ch b¬ng 15v  di»n t½ch ¡y b¬ng 9. Chi·u cao cõa khèi châpâ b¬ng A 53. B 5. C 95. D 6. ÊLíi gi£i.Thº t½ch khèi châp l  V= 1 3B h vîi Bl  di»n t½ch ¡y, hl  chi·u cao.Do â h= 3V B=315 9= 5.Chån ¡p ¡n B dC¥u 22. T½nh ¤o h m cõa h m sè y= log3(2x+ 1) . A y0= 1 (2x+ 1) ln 3 . B y0= 1 2x + 1 . C y0= 2 (2x+ 1) ln 3 . D y0= (2 x+ 1) ln 3 . ÊLíi gi£i.Ta câ y0= (2x+ 1) 0 (2x+ 1) ln 3 =2 (2x+ 1) ln 3 .Chån ¡p ¡n C dC¥u 23. Cho h m sè y= f(x ) câ b£ng x²t d§u ¤o h m nh÷ sau xf0( x ) �1�1 1 4 +1 �0 +0 �0 +H m sèy= f(x ) çng bi¸n trong kho£ng n o? A (4; +1). B (�1 ;� 1) . C (1; +1). D (� 1; 4) . ÊLíi gi£i.H m sè çng bi¸n tr¶n c¡c kho£ng (� 1; 1) v (4; + 1).Chån ¡p ¡n A dC¥u 24. T½ch ph¥n I= 1Z0 e2xdx b¬ng A I= 2(e 2� 1). B I= e2 2. C I= e2� 1 2. D I= e 2� 1. ÊLíi gi£i.I = 1Z0 e2xdx = e2x 2 10 =e2� 1 2:Chån ¡p ¡n C dC¥u 25. Mët c§p sè cëng câ 7sè h¤ng. Bi¸t r¬ng têng cõa sè h¤ng ¦u v  sè h¤ng cuèi b¬ng30 , cán têng cõa sè h¤ng thù ba v  sè h¤ng thù s¡u b¬ng 35. Khi â, sè h¤ng thù b£y cõa c§p sècëng â câ gi¡ trà b¬ng A 25. B 30. C 35. D 40. ÊLíi gi£i.281/305 281/305 pGV: L¶ Quang Xe  Ô0967.003.131— SÈ 18 NÌI N€O CÂ Þ CH, NÌI  C CON ×ÍNG282Gåiu1; u1; : : : ; u7l¦n l÷ñt l 6sè h¤ng cõa c§p sè cëng · b i cho. Gåi dl  cæng sai cõa c§p sè cëngn y.Ta câ ¨u1 +u7 = 30u 3 +u6 = 35 ,¨u1 +u1 + 6d= 30u 1 + 2d+ u1 + 5d= 35 ,¨2u1 + 6d= 302 u1 + 7d= 35 ,¨u1 = 0d = 5 :Vªy sè h¤ng thù b£y cõa c§p sè cëng ¢ cho l  u7 =u1 + 6d= 30 .Chån ¡p ¡n B dC¥u 26. T¼m nguy¶n h m F(x ) = Z�1 x2 + 2 ‹dx . A F(x ) = �1 x+ 2x+ C. B F(x ) = 1 x+ 2x+ C. C F(x ) = �1 x+ 2 +C. D F(x ) = �1 x3 + 2x+ C. ÊLíi gi£i.Ta câ F(x ) = Z�1 x2 + 2 ‹dx = �1 x+ 2x+ C.Chån ¡p ¡n A dC¥u 27. ç thà cõa h m sè n o sao ¥y khæng i qua iºm M(1; �2) ? A y= 3x � 1 x� 2. B y= x3� 3x . C y= �x3+ 3 x2� 1. D y= x4� x2� 2. ÊLíi gi£i.Thay gia trà èi sè x= 1 v o h m sè y= �x3+ 3 x2� 1, ta câ y= �1 + 3 �1 = 1 )M(1; �2) khængthuëc ç thà h m sè y= �x3+ 3 x2� 1.Chån ¡p ¡n C dC¥u 28. Cho h m sè f(x ) = x� m2 x+ 8 vîiml  tham sè thüc. Gi£ sû m0l  gi¡ trà d÷ìng cõatham sè º h m sè câ gi¡ trà nhä nh§t tr¶n o¤n [0; 3]b¬ng �3. Gi¡ trà m0thuëc kho£ng n otrong c¡c kho£ng cho d÷îi ¥y? A (5; 6). B (6; 9). C (20; 25). D (2; 5). ÊLíi gi£i.H m sè f(x ) = x� m2 x+ 8 x¡c ành v  li¶n töc tr¶n o¤n[0; 3].Ta câ: f0( x ) = 8 +m2 (x + 8) 2)f0( x ) > 0; 8x 2 [0; 3] n¶n h m sè ¢ cho çng bi¸n tr¶n o¤n [0; 3].Do â min[0;3] f(x ) = f(0) = �m 2 8. Theo gi£ thi¸tmin[0;3] f(x ) = �3, �m 2 8=�3, –m =�2p 6m = 2 p 6:Vªy m0= 2 p 62 (2; 5) (dom > 0).Chån ¡p ¡n D dC¥u 29. T¼m c¡c kho£ng çng bi¸n cõa h m sè y= x3� 3x 2+ 1 : A (�1 ;� 1) v  (1; + 1). B (� 1; 1) . C (�1 ; 0)v (2; + 1). D (0; 2). ÊLíi gi£i.282/305 282/305 pGV: L¶ Quang Xe  Ô0967.003.131PHT TRIšN — THAM KHƒO 2022 NÌI N€O CÂ Þ CH, NÌI  C CON ×ÍNG283Ta câ:y0= 3 x2� 6x ) y0= 0 ,–x = 0x = 2 :Tø b£ng bi¸n thi¶n suy ra h m sè çng bi¸n tr¶n kho£ng (�1 ; 0)v (2; + 1): xy0 y �10 2 +1 +0 �0 +�1�1 11�3 �3 +1 +1 Chån ¡p ¡n C dC¥u 30. Cholog27 =a, log37 =b. Gi¡ trà log67t½nh theo av  bl  A a+ b. B a+ b ab. C 1a+ b. D aba+ b. ÊLíi gi£i.Ta câ log67 = 1 log76 =1 log72 + log73 =1 1a+1 b=ab a+ b.Chån ¡p ¡n D dC¥u 31. Cho tù di»n OAB Cbi¸tOA; OB; OC æi mët vuæng gâc vîi nhau, bi¸t OA=3 ; OB = 4v  thº t½ch khèi tù di»n OAB Cb¬ng6. Khi â kho£ng c¡ch tø O¸n m°t ph¯ng( ABC )b¬ng A 3. B p4112. C 144p41. D 12p41. ÊLíi gi£i.Ta câ VOABC =1 6OAOB OC )OC =66 34 = 3.Gåi dl  kho£ng c¡ch tø O¸n m°t ph¯ng (ABC ), ta câ:1 d2 = 1 OA2+ 1 OB2+ 1 OC2= 1 9+1 9+1 16=41 144)d= 12 p41:Chån ¡p ¡n D dC¥u 32. T½nh1Z0 e�xdx . A �1 e+ 1. B 1. C 1e. D �1 + 1 e. ÊLíi gi£i.Ta câ 1Z0 e�xdx = �e�x 10 =�e�1+ e 0= �1 e+ 1.Chån ¡p ¡n A 283/305 283/305 pGV: L¶ Quang Xe  Ô0967.003.131— SÈ 18 NÌI N€O CÂ Þ CH, NÌI  C CON ×ÍNG284dC¥u 33. Trong khæng gian Oxyz, cho bèn iºm A(� 1; 3; 1) ,B(1; �1; 2) ,C(2; 1; 3) v D (0; 1; �1) . M°t ph¯ng (P )chùa ABv  song song vîi C Dcâ ph÷ìng tr¼nh l  A (P ) : 8 x+ 3 y� 4z + 3 = 0 . B (P ) : x+ 2 y+ 6 z� 11 = 0 . C (P ) : x+ 2 z� 4 = 0 . D (P ) : 2 x+ y� 1 = 0 . ÊLíi gi£i.Ta câ # AB = (2; �4; 1) ,# C D = (�2; 0; �4) ) [# AB; # C D ] = (8; 3; �4) .M°t ph¯ng (P )i qua A(� 1; 3; 1) , nhªn #n = [ # AB; # C D ] = (8; 3; �4) l  v²c-tì ph¡p tuy¸n, câ ph÷ìngtr¼nh l  8(x+ 1) + 3( y� 3) �4(z� 1) = 0 ,8x + 3 y� 4z + 3 = 0(thäa m¢n song song C Dn¶n thäa m¢n · b i).Chån ¡p ¡n A dC¥u 34. Cho sè phùc zthäa m¢n z(2 �i) + 13 i= 1 . T½nh mæ-un cõa sè phùc zA jz j = p 34. B jz j = p 343. C jz j = 5p 343. D jz j = 34 . ÊLíi gi£i.C¡ch 1 Ta câz(2 �i) + 13 i= 1 ) jz(2 �i) j = j1 � 13ij , j zj  p 5 =p 170, jzj = p 34:C¡ch 2 Ta câz(2 �i) + 13 i= 1 ,z(2 �i) = 1 �13i, z(2 �i)(2 + i) = (1 �13i)(2 + i) , 5z = 15 �25i, z= 3 �5i) j zj = p 32+ 5 2= p 34.Chån ¡p ¡n A dC¥u 35. Cho h¼nh châp S:ABC Dcâ ¡y l  h¼nh vuæng c¤nh a. C¤nh b¶n S Avuæng gâc vîim°t ph¯ng ¡y, c¤nh b¶n S Ct¤o vîi m°t ph¯ng (S AB )mët gâc 30. Thº t½ch cõa khèi châp âb¬ng A a3p 33. B a3p 24. C a3p 22. D a3p 23. ÊLíi gi£i.Ta câ: ¨BC ?S ABC ?AB )BC ?(S AB )) S B l  h¼nh chi¸u cõa S Cl¶n m°t ph¯ng (S AB )) Û�( S C; (S AB )) =Ù�( S C; S B ) =ÕBS C = 30.Trong 4S C B ,ta câ tanÕBS C =BC S B=a S B, S B =ap 3; S A =p S B2� AB 2= ap 2.Vªy thº t½ch khèi châp l VS ABC D =1 3S ASABC D =1 3a p 2a 2= a3p 23: B AC DSChån ¡p ¡n D dC¥u 36. Mët cuëc håp câ sü tham gia cõa 5nh  To¡n håc trong â câ 3nam v  2nú, 6nh Vªt lþ trong â câ 3nam v  3nú, 7nh  Hâa håc trong â câ 4nam v  3nú. Ng÷íi ta muèn lªpmët ban th÷ k½ gçm 4nh  khoa håc vîi y¶u c¦u ph£i câ c£ ba l¾nh vüc ( To¡n, Lþ, Hâa) v  câc£ nam l¨n nú. N¸u måi ng÷íi ·u b¼nh ¯ng nh÷ nhau th¼ sè c¡ch lªp mët ban th÷ k½ nh÷ th¸l 284/305 284/305 pGV: L¶ Quang Xe  Ô0967.003.131PHT TRIšN — THAM KHƒO 2022 NÌI N€O CÂ Þ CH, NÌI  C CON ×ÍNG285A 1575. B 1440. C 1404. D 171. ÊLíi gi£i.Ban th÷ k½ gçm 4nh  khoa håc thäa m¢n y¶u c¦u ph£i câ c£ ba l¾nh vüc To¡n- Lþ - Hâa câ c¡c kh£n«ng sau:Ì Kh£ n«ng 1: 2To¡n, 1Lþ, 1Hâa câ C25 C 16 C 17 = 420c¡ch lªp ban th÷ k½.Trong kh£ n«ng n y, câ C23 C 13 C 14 = 36c¡ch lªp ban th÷ k½ gçm to n nh  khoa håc nam v  câC 22 C 13 C 13 = 9c¡ch lªp ban th÷ k½ gçm to n nh  khoa håc nú.Ì Kh£ n«ng 2: 1To¡n, 2Lþ, 1Hâa câ C15 C 26 C 17 = 525c¡ch lªp ban th÷ k½.Trong kh£ n«ng n y, câ C13 C 23 C 14 = 36c¡ch lªp ban th÷ k½ gçm to n nh  khoa håc nam v  câC 12 C 23 C 13 = 18c¡ch lªp ban th÷ k½ gçm to n nh  khoa håc nú.Ì Kh£ n«ng 3: 1To¡n, 1Lþ, 2Hâa câ C15 C 16 C 27 = 630c¡ch lªp ban th÷ k½.Trong kh£ n«ng n y, câ C13 C 13 C 24 = 54c¡ch lªp ban th÷ k½ gçm to n nh  khoa håc nam v  câC 12 C 13 C 23 = 18c¡ch lªp ban th÷ k½ gçm to n nh  khoa håc nú.Vªy sè c¡ch lªp ban th÷ k½ thäa m¢n ¦u b i l :420 + 525 + 630�36 �9� 36 �18 �54 �18 = 1404 :Chån ¡p ¡n C dC¥u 37. Trong khæng gian Oxyzcho hai ÷íng th¯ng 1: 8><>: x= ty = tz = 2 v 2: x� 3 �1 =y� 1 2=z 1.÷íng vuæng gâc chung cõa 1v 2i qua iºm n o d÷îi ¥y? A P�2; 32 11;7 11‹. B N�� 2; 32 11;7 11‹. C M�2; �32 11;7 11‹. D Q�� 2; 32 11;� 7 11‹. ÊLíi gi£i.Ta câ 1: 8><>: x= ty = tz = 2 ,2: 8><>: x= 3 �sy = 1 + 2 sz = s v #u 1 = (1; 1; 0);#u 2 = (�1; 2; 1) l¦n l÷ñt l  v²c-tì ch¿ ph÷ìngcõa 1;2.Gi£ sû A21; B22khi âA(t; t; 2) ; B(3�s; 1 + 2 s;s ) ) # AB = (3 �s� t; 1 + 2 s� t; s � 2).AB l  ÷íng vuæng gâc chung cõa 1;2khi v  ch¿ khi # AB #u 1= # AB #u 2= 0.Khi â ¨1(3 �s� t) + 1(1 + 2 s� t) + 0( s� 2) = 0� 1(3 �s� t) + 2(1 + 2 s� t) + 1( s� 2) = 0 ,¨2t � s= 4t � 6s = �3 , 8><>: t= 27 11s = 10 11:Suy raA�27 11;27 11; 2‹; B �23 11;31 11;10 11‹.# AB =�� 4 11;4 11;� 12 11‹còng ph÷ìng vîi #u = (1; �1; 3) :÷íng th¯ng ABcâ ph÷ìng tr¼nh 8>>>><>>>>: x= 27 11+ty = 27 11�tz = 2 + 3 t:Tø â suy ÷íng th¯ng ABi qua iºm P�2; 32 11;7 11‹.Chån ¡p ¡n A 285/305 285/305 pGV: L¶ Quang Xe  Ô0967.003.131— SÈ 18 NÌI N€O CÂ Þ CH, NÌI  C CON ×ÍNG286dC¥u 38. Câ bao nhi¶u sè nguy¶n xnghi»m óng b§t ph÷ìng tr¼nh 1 logx2 +1 logx42 <10? A 1. B 2. C 4. D 3. ÊLíi gi£i.i·u ki»n x >0; x 6= 1 .Khi â1 logx2 +1 logx42 <10 , 1 logx2 +4 logx2 <10, log2x+ 4 log2x <10, log2x <2, x < 4:Tø i·u ki»n v  xnguy¶n, ta câ x2 f 2;3 g . Vªy câ 2 gi¡ trà nguy¶n cõa xthäa m¢n y¶u c¦u b i to¡n.Chån ¡p ¡n B dC¥u 39. Cho h m sè y= x4+ 2 mx2+ m (vîi ml  tham sè thüc). Tªp t§t c£ c¡c gi¡ trà cõatham sè mº ç thà h m sè ¢ cho c­t ÷íng th¯ng y= �3 t¤i bèn iºm ph¥n bi»t, trong âcâ mët iºm câ ho nh ë lîn hìn 2cán ba iºm kia câ ho nh ë nhä hìn 1, l  kho£ng (a ;b) (vîia; b 2Q,a bl  ph¥n sè tèi gi£n). Khi â,15ab nhªn gi¡ trà n o sau ¥y? A �63 . B 63. C 95. D �95 . ÊLíi gi£i.X²t ph÷ìng tr¼nh ho nh ë giao iºm �3 = x4+ 2 mx2+ m. °t x2= t, t  0. Khi â ph÷ìng tr¼nhtrð th nh t2+ 2 mt+m + 3 = 0 (1) v  °tf(t) = t2+ 2 mt+m + 3 .º ç thà h m sè c­t ÷íng th¯ng y= �3 t¤i 4iºm ph¥n bi»t th¼ ph÷ìng tr¼nh (1)câ hai nghi»mthäa m¢n 0< t1< t2v  khi â ho nh ë bèn giao iºm l �p t2 <�p t1 < p t1 < p t2 .Do â, tø i·u ki»n cõa b i to¡n suy ra ¨p t2 >2p t1 <1hay0< t1<1< 4< t2.i·u n y x£y ra khi v  ch¿ khi 8><>: f(0) >0f (1) <0f (4) <0, 8><>: m+ 3 >03 m + 4 <09 m + 19 <0, �3< m < �19 9:Vªy a= �3, b = �19 9n¶n15ab = 95 .Chån ¡p ¡n C dC¥u 40. Cho c¡c h m sè f(x ) = 20x2� 30x+ 7 p2x � 3 ,F (x ) = ( ax2+ bx +c)p 2x � 3vîi x > 3 2.Gåi (a ;b; c) l  bë sè thäa m¢n F(x ) l  mët nguy¶n h m cõa f(x ). Khi â a+ b+ cb¬ng A 1. B 5. C 3. D 7. ÊLíi gi£i.Ta câf(x ) = 10 xp 2x � 3 + 7 p2x � 3= 5(2x� 3)p 2x � 3 + 15 p 2x � 3 + 7 p2x � 3= 5(2 x� 3)3 2+ 15(2 x� 3)1 2+ 7 p2x � 3:286/305 286/305 pGV: L¶ Quang Xe  Ô0967.003.131PHT TRIšN — THAM KHƒO 2022 NÌI N€O CÂ Þ CH, NÌI  C CON ×ÍNG287Suy raZf(x ) d x= (2 x� 3)5 2+ 5(2 x� 3)3 2+ 7 p 2x � 3 + C= (4 x2� 2x + 1) p 2x � 3 + C.Suy ra F(x ) = (4 x2� 2x + 1) p 2x � 3hay a= 4 ; b=�2; c = 1 )a+ b+ c= 3 .Chån ¡p ¡n C dC¥u 41. Mët h¼nh l«ng trö ùng tam gi¡c câ ë d i c¡c c¤nh l  9;3 ;4 ;3 ;4 ;5 ;9 ;5 ;9 . Thº t½chcõa khèi l«ng trö n y b¬ng bao nhi¶u? A 46. B 50. C Khæng t½nh ÷ñc. D 54. ÊLíi gi£i.L«ng trö ùng tam gi¡c câ 3c¤nh b¶n b¬ng nhau v  câ hai a gi¡c ¡y b¬ngnhau.Do â h¼nh l«ng trö ùng tam gi¡c câ chi·u cao b¬ng 9, ¡y l  tam gi¡c câc¤nh b¬ng 3; 4; 5 (tam gi¡c vuæng).Vªy thº t½ch Vcõa khèi l«ng trö ùng ¢ cho b¬ngV= 1 234 9 = 54 : 53 4 9Chån ¡p ¡n D dC¥u 42. Cho sè phùc z= m+ 1 + mivîi m2R. Häi câ bao nhi¶u gi¡ trà nguy¶n cõam 2(� 5; 5) sao cho jz � 2ij > 1? A 5. B 4. C 0. D 9. ÊLíi gi£i.Ta câjz � 2ij > 1 , j m+ 1 + mi�2ij > 1, È (m + 1) 2+ ( m�2)2> 1, 2m 2� 2m + 5 >1, 2m 2� 2m + 4 >0, m2R:V¼ m2(� 5; 5) v ml  sè nguy¶n n¶n m2 f� 4;�3; �2; �1; 0; 1; 2; 3; 4 g.Chån ¡p ¡n D dC¥u 43. T¼m mæun cõa sè phùc z, bi¸t z+ 2 z= 3 �2i A p10. B p2. C p5. D p13. ÊLíi gi£i.Ta câ z+ 2 z= 3 �2i ) z+ 2 z= 3 + 2 i) 4z + 2 z= 6 + 4 i) z+ 3 z= 3 + 6 i) z= 1 + 2 i) j zj = p 5.Chån ¡p ¡n C dC¥u 44.287/305 287/305 pGV: L¶ Quang Xe  Ô0967.003.131— SÈ 18 NÌI N€O CÂ Þ CH, NÌI  C CON ×ÍNG288Cho h m sèy= f(x ) câ ¤o h m tr¶n Rv  câ ç thà nh÷ h¼nhv³. Häi ç thà h m sè y= f2(x ) câ bao nhi¶u iºm cüc ¤i, cüctiºu? A 1iºm cüc ¤i, 2iºm cüc tiºu. B 2iºm cüc ¤i, 3iºm cüc tiºu. C 3iºm cüc ¤i, 2iºm cüc tiºu. D 2iºm cüc ¤i, 2iºm cüc tiºu. xyO 13ÊLíi gi£i.Ta câ y0= 2 f(x )f 0( x ), y0= 0 ,–f (x ) = 0f 0( x ) = 0 :Tø ç thà suy raf(x ) = 0 ,264 x= 0x = 1x = 3 ;f0( x ) = 0 ,264 x= m; m 2(0; 1)x = 1x = n; n 2(1; 3) :Tø â ta câ b£ng bi¸n thi¶n xf0( x ) f(x ) y0 y �10 m1 n3 +1 � �0 +0 �0 + ++0 � �0 � �0 +�0 +0 �0 +0 �0 ++1 +1 CTCT CCCTCT CCCTCT +1 +1 Vªy h m sè câ2iºm cüc ¤i, 3iºm cüc tiºu.Chån ¡p ¡n B dC¥u 45. Trong khæng gian Oxyz, cho tam gi¡c ABCvîiA(3; 0; 0) ,B(0; 6; 0) ,C(0; 0; 6) .Ph÷ìng tr¼nh n o d÷îi ¥y l  ph÷ìng tr¼nh ÷íng th¯ng i qua trüc t¥m cõa tam gi¡c ABCv  vuæng gâc vîi m°t ph¯ng (ABC )? A x� 2 2=y� 1 1=z� 1 1. B x� 1 2=y� 1 1=z� 1 1. C x� 3 2=y� 6 1=z� 6 1. D x� 1 2=y� 3 1=z� 3 1. ÊLíi gi£i.Ta th§y tù di»n OAB Cl  tù di»n vuæng v¼ câ OA,OB ,OC æi mët vuæng gâc vîi O(0; 0; 0) l  gècto¤ ë. Do â, h¼nh chi¸u vuæng gâc cõa Ol¶n (ABC )ch½nh l  trüc t¥m cõa tam gi¡c ABC.Vªy ÷íng th¯ng c¦n t¼m i qua Ov  vuæng gâc vîi m°t ph¯ng (ABC ).Ta câ # AB = (�3; 6; 0) ,# AC = (�3; 0; 6) n¶n ÷íng th¯ng i qua Ov  nhªn #u = 1 2”# AB; # AC —= (2; 1; 1) l m v²c-tì ch¿ph÷ìng câ ph÷ìng tr¼nh x 2=y 1=z 1l  ÷íng th¯ng c¦n t¼m.Tø â suy ra ÷íng th¯ng câ ph÷ìng tr¼nh x� 2 2=y� 1 1=z� 1 1tho£ m¢n y¶u c¦u b i to¡n.Chån ¡p ¡n A 288/305 288/305 pGV: L¶ Quang Xe  Ô0967.003.131PHT TRIšN — THAM KHƒO 2022 NÌI N€O CÂ Þ CH, NÌI  C CON ×ÍNG289dC¥u 46. Cho h¼nh nân câ chi·u cao b¬ng 2p 5. Mët m°t ph¯ng i qua ¿nh h¼nh nân v  c­th¼nh nân theo mët thi¸t di»n l  tam gi¡c ·u, m°t ph¯ng n y c¡ch t¥m cõa ÷íng trán ¡y mëtkho£ng h= 2p 353p 3. Thº t½ch cõa khèi nân ÷ñc giîi h¤n bði h¼nh nân ¢ cho b¬ng A 32p 5 3. B 32. C 32p 5 . D 96. ÊLíi gi£i.Gåi Ol  ¿nh h¼nh nân, Il  t¥m ÷íng trán ¡y h¼nh nân, thi¸t di»n l  tamgi¡c ·u OAB. GåiKl  trung iºm cõa ABkhi â I K?AB . K´ I H?OKkhi â kho£ng c¡ch tø I¸n (OAB )ch½nh l  I Hhay I H=2p 353p 3.Tam gi¡c OI Kvuæng t¤i Iv  I H l  ÷íng cao n¶n:1 I K2= 1 I H2� 1 OI2= 1 7,I K 2= 7 . ABIOK HX²t tam gi¡cOI K:OK =p OI2+ I K 2= 3 p 3.M  OK l  ÷íng cao cõa tam gi¡c ·u OABn¶nOK =AB p 32,OA =23p 3p3= 6.Do â I A=p OA2� OI 2= q 36�€2p 5Š2= 4 .Khèi nân c¦n t¼m câ b¡n k½nh ¡y I A= 4 , chi·u cao OI= 2p 5n¶n câ thº t½ch l :V = 1 3Sd h = 1 3I A 2OI =1 316 2p 5 =32p 53.Chån ¡p ¡n A dC¥u 47. T½nh sè nghi»m cõa ph÷ìng tr¼nh cotx= 2 xtrong kho£ng �11  12; 2019‹. A 2020. B 2019. C 2018. D 1. ÊLíi gi£i.i·u ki»n x6= k ,k 2 Z. Ta câ cotx= 2 x, cot x� 2x= 0 . (1)X²t h m sè f(x ) = cot x� 2xtr¶n �11  12; ‹,( ; 2 ),. . . , (2018 ; 2019 ).Ta câ f0( x ) = � 1 sin2x �2xln 2 <0vîi 8x 2 �11  12; ‹,( ; 2 ),. . . , (2018 ; 2019 ).Suy ra h m sè f(x ) nghàch bi¸n tr¶n tøng kho£ng x¡c ành.Tr¶n kho£ng �11  12; ‹ta câ f(x ) < f �11  12‹) f(x ) < cot �11  12‹� 211 12<0.) f(x ) = 0 væ nghi»m.Ta câ h m sè f(x ) nghàch bi¸n tr¶n tøng kho£ng ( ; 2 ),. . . , (2018 ; 2019 ) v  tr¶n méi kho£ng âh m sè câ tªp gi¡ trà l  R.Suy ra tr¶n méi kho£ng ( ; 2 ),. . . , (2018 ; 2019 ) th¼ ph÷ìng tr¼nh f(x ) = 0 câ nghi»m duy nh§t.Vªy ph÷ìng tr¼nh (1)câ2018 nghi»m.Chån ¡p ¡n C 289/305 289/305 pGV: L¶ Quang Xe  Ô0967.003.131MÖC LÖCNÌI N€O CÂ Þ CH, NÌI  C CON ×ÍNG290dC¥u 48. Trong khæng gian Oxyz, cho m°t c¦u (S ) : x2+ y2+ z2+ 2 x� 2y + 6 z� 5 = 0 v  m°tph¯ng (P ) : x� 2y + 2 z+ 3 = 0 . GåiMl  ti¸p iºm cõa (S ) v  m°t ph¯ng (Q )di ëng vuænggâc vîi m°t ph¯ng (P ). Tªp hñp c¡c iºm Ml  A ÷íng trán:x2+ y2+ z2+ 2 x� 2y + 6 z� 5 = 0 ;x � 2y + 2 z+ 9 = 0 . B M°t ph¯ng:x� 2y + 2 z� 9 = 0 . C ÷íng trán:x2+ y2+ z2+ 2 x� 2y + 6 z� 5 = 0 ;x � 2y + 2 z� 9 = 0 . D M°t ph¯ng:x� 2y + 2 z+ 9 = 0 . ÊLíi gi£i.Ta câ ph÷ìng tr¼nh m°t c¦u (S ) : ( x+ 1) 2+ ( y� 1)2+ ( z+ 3) 2= 16 . Gåi I, R l  t¥m v  b¡n kinhm°t c¦u suy ra I(� 1; 1; �3) v  R= 4 . Khi â d (I; ( P)) = j�1� 2� 6 + 3 j 3= 2suy ra m°t ph¯ng( P )luæn c­t m°t c¦u (S ) theo mët ÷íng trán. Do gi£ thi¸t suy ra I M?(Q )suy ra Mthuëc m°tph¯ng ( ) chùa I M.Do â ( ) (P )n¶n ph÷ìng tr¼nh ( ) : x� 2y + 2 z+ m = 0 (m 6= 3 ). V¼ I2 ( ) n¶n �1� 2� 6 + m=0 , m= 9 suy ra ph÷ìng tr¼nh ( ) : x� 2y + 2 z+ 9 = 0 . Do â Mthuëc ÷íng trán l  giao cõa m°tph¯ng ( ) v  m°t c¦u (S ).Chån ¡p ¡n A dC¥u 49. T¼m t§t c£ c¡c gi¡ trà cõa tham sè mº h m sè y= jx j3� (2 m �1) x2+ ( m�1) jx j � 2câ óng ba iºm cüc trà. A m1. B m � 2. C �2 m 1. D m >1. ÊLíi gi£i.°t f(x ) = x3� (2m �1)x2+ ( m�1)x� 2) f(jx j) = jx j3� (2m �1)x2+ ( m�1)jx j � 2.H m sè y= f(jx j) câ óng 3iºm cüc trà tr¶n R, H m sè y= f(x ) câ óng 1iºm cüc trà tr¶n (0; +1), Tr¶n R, h m sè y= f(x ) câ 2iºm cüc trà x1; x2thäa m¢nx1 <0< x2ho°c0 =x1 < x2, Tr¶n R, ph÷ìng tr¼nh f0( x ) = 0 câ2nghi»m x1; x2thäa m¢nx1 <0< x2ho°c0 =x1 < x2.Ta câ f0( x ) = 0 ,3x 2� 2(2 m�1)x+ m �1 = 0 .Ì Ph÷ìng tr¼nh câ 2nghi»m x1,x2 thäa m¢nx1 <0< x2,3(m �1) <0, m < 1.Ì Ph÷ìng tr¼nh câ 2nghi»m x1,x2 thäa m¢nx1 = 0< x2)3(m �1) = 0 ,m= 1 .Vîi m= 1 , ph÷ìng tr¼nh câ 2nghi»m x1 = 0,x2 = 2 3>0(thäa m¢n).Vªy h m sè y= f(jx j) câ óng 3iºm cüc trà khi v  ch¿ khi m1.Chån ¡p ¡n A H˜T290/305 290/305 pGV: L¶ Quang Xe  Ô0967.003.131PHT TRIšN — THAM KHƒO 2022 NÌI N€O CÂ Þ CH, NÌI  C CON ×ÍNG291BË GIO DÖC & €O T„OTR×ÍNG THPT NGUY™N T‡T TH€NH GV: L– QUANG XE - 0967.003.131 — SÈ 19 PHT TRIšN — THAM KHƒO 2022N‹M HÅC 2021 - 2022Mæn:To¡nThíi gian l m b i: 90 phót — THAM KHƒO HKIIdC¥u 1. Sè phùc n o d÷îi ¥y l  sè thu¦n £o? A z= p 2. B z= �1 + i. C z= p 2i. D z= p 2 +i. ÊLíi gi£i.Sè thu¦n £o l  sè câ d¤ng z= bi,b 2 R:Chån ¡p ¡n C dC¥u 2. Trong khæng gian vîi h» tåa ë Oxyz, cho m°t c¦u (S ) : ( x� 2)2+ y2+ ( z+ 1) 2= 4 .Tåa ë t¥m Icõa m°t c¦u (S ) l  A I(2; 1 �1). B I(2; 0; �1) . C I(� 2; 0; 1) . D I(� 2; 1; 1) . ÊLíi gi£i.T¥m cõa m°t c¦u (S ) l  I(2; 0; �1) .Chån ¡p ¡n B dC¥u 3. iºm n o sau ¥y thuëc ç thà (C )cõa h m sè y= x2+ 3 x+ 3 x+ 1 ? A (3; 0). B (2; 1). C (0; 3). D (� 2; 1) . ÊLíi gi£i.Ta câ y(0) = 3 n¶n iºm câ to¤ ë (0; 3)thuëc ç thà (C )cõa h m sè â cho.Chån ¡p ¡n C dC¥u 4. Cho m°t c¦u (S ) câ t¥m I, b¡n k½nh Rv  m°t ph¯ng ( ). Bi¸t kho£ng c¡ch tø t¥mcõa m°t c¦u (S ) tîi m°t ph¯ng ( ) b¬ng R. Trong c¡c kh¯ng ành sau kh¯ng ành n o óng? A M°t ph¯ng( ) ti¸p xóc vîi m°t c¦u (S ). B M°t ph¯ng( ) c­t m°t c¦u (S ). C M°t ph¯ng( ) v  m°t c¦u (S ) khæng câ iºm chung. D Thi¸t di»n cõa m°t ph¯ng( ) vîi m°t c¦u (S ) l  mët ÷íng trán. ÊLíi gi£i.V¼ d (I; ( )) = Rn¶n ( ) ti¸p xóc vîi m°t c¦u (S ).Chån ¡p ¡n A dC¥u 5. Zxdx b¬ng A x+ C. B x� 1+ C. C x ln+C. D x+1 + 1 +C. ÊLíi gi£i.291/305 291/305 pGV: L¶ Quang Xe  Ô0967.003.131— SÈ 19 NÌI N€O CÂ Þ CH, NÌI  C CON ×ÍNG292Zxdx = x+1 + 1 +C:Chån ¡p ¡n D dC¥u 6. Cho h m sè y= f(x ) câ b£ng bi¸n thi¶n sau: xf0( x ) f(x ) �1�2 2 +1 +0 �0 +�1�1 3300 +1 +1 T½nh gi¡ trà cüc ¤iyC v  gi¡ trà cüc tiºuyCT cõa h m sè ¢ cho A yC =�2 v  yCT = 2. B yC = 3v yCT = 0. C yC = 22v yCT = 0. D yC = 3v yCT =�2. ÊLíi gi£i.Tø b£ng bi¸n thi¶n suy ra h m sè ¢ cho câ yC = 3v yCT = 0.Chån ¡p ¡n B dC¥u 7. T¼m nghi»m cõa b§t ph÷ìng tr¼nh �1 2‹x� 1 1 4. A x 3. B x >3. C x 3. D 1< x 3. ÊLíi gi£i.Ta câ �1 2‹x� 1 1 4, �1 2‹x� 1 �1 2‹2, x� 1 2, x 3.Chån ¡p ¡n A dC¥u 8. Trong khæng gian, cho khèi hëp chú nhªt ABC D:A0B 0C 0D 0câ ë d i AB= 1 m,AA 0= 3 m v  BC= 2 m. T½nh thº t½ch Vcõa khèi hëp chú nhªt ABC D:A0B 0C 0D 0? A V= 6 m3. B V=p 5m3. C V= 3 m3. D V= 3 p 5m3. ÊLíi gi£i.Thº t½ch khèi hëp chú nhªt l  VABC D:A 0B 0C 0D 0= AA 0 AB AD =AA 0 AB BC = 31 2 = 6 m3.Chån ¡p ¡n A dC¥u 9. Tªp x¡c ành Dcõa h m sè y= (1 �x)  2019l  A D=Rnf 1g . B D= (1; + 1). C D= (0; + 1). D D= ( �1 ; 1). ÊLíi gi£i.V¼  2019l  sè khæng nguy¶n n¶n1� x > 0, x < 1.Vªy tªp x¡c ành cõa h m sè ¢ cho l  D= ( �1 ; 1).Chån ¡p ¡n D dC¥u 10. X¡c ành sè thüc xº d¢y sè log 2,log 7 ;log xtheo thù tü â lªp th nh mët c§p sècëng.292/305 292/305 pGV: L¶ Quang Xe  Ô0967.003.131PHT TRIšN — THAM KHƒO 2022 NÌI N€O CÂ Þ CH, NÌI  C CON ×ÍNG293A x= 49 2. B x= 2 49. C x= 2 7. D x= 7 2. ÊLíi gi£i.i·u ki»n x >0.º log 2 ,log 7 ;log xtheo thù tü â lªp th nh mët c§p sè cëng th¼log 2 + log x= 2 log 7 ,log 2 x= log 7 2, log 2 x= log 49 ,2x = 49 ,x= 49 2:Chån ¡p ¡n A dC¥u 11. ChoF(x ) v  G(x ) t÷ìng ùng l  nguy¶n h m cõa h m sè f(x ) = x; g(x ) = ex. M»nh· n o sau ¥y óng A Z(x + ex) d x= F(x ) G (x ). B Z(x � ex) d x= F(x ) � G(x ). C Z(x + ex) d x= F(x ) + G(x ) + C,C l  h¬ng sè. D Zxexdx = F(x ) G (x ). ÊLíi gi£i.Theo t½nh ch§t cõa nguy¶n h m ta câ Z(x + ex) d x= F(x ) + G(x ) + C,C l  h¬ng sè.Chån ¡p ¡n C dC¥u 12. T½nhi4+ i2. A �2. B �1. C 0. D 2. ÊLíi gi£i.Ta câ i2= �1 n¶n i4= ( i2)2= ( �1) 2= 1 .Suy ra i4+ i2= 1 �1 = 0 .Chån ¡p ¡n C dC¥u 13. Trong khæng gian Oxyz, cho m°t ph¯ng (P ) : x� 2y + 3 = 0 . T¼m mët v²c-tì ph¡ptuy¸n #n (P ) cõa m°t ph¯ng(P ). A #n (P ) = (1;�2; 0) . B #n (P ) = (1;�2; 3) . C #n (P ) = (1; 0;�2) . D #n (P ) = (0; 1;�2) . ÊLíi gi£i.L÷u þ: N¸u m°t ph¯ng (P )câ ph÷ìng tr¼nh l  ax+by +cz +d= 0 th¼ nâ câ mët v²c-tì ph¡p tuy¸nl  #n = ( a;b; c). V  khi â, måi v²c-tì ph¡p tuy¸n cõa (P )·u câ d¤ng k#n .Vi¸t l¤i ph÷ìng tr¼nh cõa (P )nh÷ sau x� 2y + 0 z+ 3 = 0 , suy ra #n (P ) = (1;�2; 0) l  mët v²c-tìph¡p tuy¸n cõa (P ).Chån ¡p ¡n A dC¥u 14. Trong khæng gian Oxyz, cho iºm A(� 2; 3; �1) . Gåi A0l  iºm èi xùng vîi Aquatröc ho nh. T¼m tåa ë iºm A0. A A0(2; �3; 1) . B A0(0; �3; 1) . C A0( � 2; �3; 1) . D A0( � 2; 0; 0) . ÊLíi gi£i.293/305 293/305 pGV: L¶ Quang Xe  Ô0967.003.131— SÈ 19 NÌI N€O CÂ Þ CH, NÌI  C CON ×ÍNG294iºm èi xùng cõa iºmA(x ;y ;z ) qua tröc ho nh l  iºm câ d¤ng A0( x ;� y;� z).Suy ra iºm èi xùng cõa iºm A(� 2; 3; �1) qua tröc ho nh l  iºm A0( � 2; �3; 1) .Chån ¡p ¡n C dC¥u 15.iºm n o trong h¼nh v³ b¶n l  iºm biºu di¹n sè phùc z= 2 +i ? A D. B B. C C. D A. Ox�2 �1 1 2y�1 12A DCBÊLíi gi£i.iºm biºu di¹n sè phùc z= a+ bi l  I(a ;b). Vªy ¡p ¡n óng l  D(2; 1) .Chån ¡p ¡n A dC¥u 16. Sè ÷íng ti»m cªn cõa ç thà h m sè y= x� 3 x+ 4 l  A 4. B 3. C 2. D 1. ÊLíi gi£i.ç thà h m sè câ ÷íng ti»m cªn ùng l  x= �4 v  ti»m cªn ngang l  y= 1 .Chån ¡p ¡n C dC¥u 17. Vîial  sè thüc d÷ìng b§t k¼, m»nh · n o d÷îi ¥y óng? A lna2019= 1 2019lna. B lna2019= 2019 ln a. C ln(2019a) = 2019 ln a. D ln(2019a) = 1 2019lna. ÊLíi gi£i.Vîi a > 0, ta câÌ ln(2019 a) = ln 2019 + ln a.Ì lna2019= 2019 ln a.Chån ¡p ¡n B dC¥u 18.÷íng cong trong h¼nh sau l  ç thà cõa h m sè n o? A y= �x3+ 3 x2+ 1 . B y= 1 3x3� x2+ 1 . C y= x3+ 3 x2+ 1 . D y= x3� 3x 2+ 1 . O xy�2 �1 321�3 �2 �1 1294/305 294/305 pGV: L¶ Quang Xe  Ô0967.003.131PHT TRIšN — THAM KHƒO 2022 NÌI N€O CÂ Þ CH, NÌI  C CON ×ÍNG295ÊLíi gi£i.Tø h¼nh v³ ta th§y ¥y l  ç thà cõa mët h m sè bªc ba.Suy ra h m sè câ d¤ng y= ax 3+ bx2+ cx2+ dvîi a6= 0 .Tø â ta câ y0= 3 ax2+ 2 bx+c.Cho x= 0 ta ÷ñc d= 1 .Tø h¼nh v³ ta th§y ç thà h m sè câ hai iºm cüc trà (0; 1)v (2; �3) .Do â x= 0 v x= 2 l  hai nghi»m cõa ph÷ìng tr¼nh 3ax 2+ 2 bx+c= 0 (1) .Ta câ x= 0 l  nghi»m cõa (1)n¶n c= 0 .Suy ra y= ax 3+ bx2+ 1 .Ta câ x= 2 l  nghi»m cõa (1)n¶n 12a+ 4 b= 0 (2) .iºm (2;�3) thuëc ç thà h m sè suy ra 8a + 4 b+ 1 = �3 (3) .Tø (2);(3) ta câ a= 1 v b= �3.Vªy y= x3� 3x 2+ 1 l  h m sè c¦n t¼m.Chån ¡p ¡n D dC¥u 19. Trong h» tåa ë Oxyz, cho ÷íng th¯ng  :x� x0 a=y� y0 b=z� z0 c. iºmMn¬mtr¶n th¼ tåa ë cõa Mcâ d¤ng n o sau ¥y? A M(x0 +at;y0 +bt;z0 +ct). B M(a + x0t; b + y0t; c + z0t) . C M(at ;bt ;ct ). D M(x0t; y0t; z0t) . ÊLíi gi£i.Ph÷ìng tr¼nh tham sè cõa l 8><>: x= x0 +aty = y0 +btz = z0 +at . Do â tåa ë iºm câ d¤ngM(x0 +at;y0 +bt;z0 +ct).Chån ¡p ¡n A dC¥u 20. Câ bao nhi¶u c¡ch x¸p 10ng÷íi ngçi v o 10gh¸ h ng ngang? A 3028800. B 3628800. C 3628008. D 3628880. ÊLíi gi£i.Sè c¡ch x¸p 10ng÷íi ngçi v o 10gh¸ h ng ngang l  10! = 3628800.Chån ¡p ¡n B dC¥u 21. Cho l«ng trö ùng câ c¤nh b¶n b¬ng 3, ¡y l  h¼nh vuæng c¤nh b¬ng 6. Thº t½ch khèil«ng trö l  A 108. B 72. C 96. D 84. ÊLíi gi£i.Ta câ V= 3 62= 108 .Chån ¡p ¡n A dC¥u 22. T½nh ¤o h m cõa h m sè y= 2 x+1. A y0= 2 x+1log 2 . B y0= 2 x+1ln 2 . C y0= ( x+ 1)2 xln 2 . D y0= 2x+1 ln 2. ÊLíi gi£i.Ta câ y0= (2 x+1)0= ( x+ 1) 0 2x+1ln 2 = 2 x+1ln 2 :Chån ¡p ¡n B 295/305 295/305 pGV: L¶ Quang Xe  Ô0967.003.131— SÈ 19 NÌI N€O CÂ Þ CH, NÌI  C CON ×ÍNG296dC¥u 23.Cho b£ng bi¸n thi¶n nh÷ h¼nh v³ b¶n. M»nh · n osau ¥y sai? A H m sè ¢ cho çng bi¸n tr¶n kho£ng(�1 ; 1). B H m sè ¢ cho nghàch bi¸n tr¶n kho£ng(0; 3). C H m sè ¢ cho çng bi¸n tr¶n kho£ng(2; +1). D H m sè ¢ cho çng bi¸n tr¶n kho£ng(3; +1). xy0 y �11 2 +1 +0 �0 +�1�1 3300 +1 +1 ÊLíi gi£i.Düa v o b£ng bi¸n thi¶n ta câ c¡c k¸t luªn sau:Ì H m sè çng bi¸n tr¶n c¡c kho£ng (�1 ; 1),(2; + 1).Ì H m sè nghàch bi¸n tr¶n kho£ng (1; 2).Ì H m sè ¤t cüc ¤i t¤i x= 1 v  gi¡ trà cüc ¤i cõa h m sè b¬ng 3.Ì H m sè ¤t cüc tiºu t¤i x= 2 v  gi¡ trà cüc tiºu cõa h m sè b¬ng 0.Do â, x²t h m sè tr¶n kho£ng (0; 3)th¼ h m sè çng bi¸n tr¶n c¡c kho£ng (0; 1),(2; 3) v  nghàchbi¸n tr¶n kho£ng (1; 2).Chån ¡p ¡n B dC¥u 24. N¸u2Z� 1 f(x ) d x= 2 v p 5Z2 f(x ) d x= 1 th¼p 5Z� 1 f(x ) d xb¬ng A �3. B �1. C 1. D 3. ÊLíi gi£i.Ta câ p 5Z� 1 f(x ) d x= 2Z� 1 f(x ) d x+ p 5Z2 f(x ) d x= 2 + 1 = 3 .Chån ¡p ¡n D dC¥u 25. Cho c§p sè cëng câ u4 =�12 ,u14 = 18. Khi â sè h¤ng ¦u ti¶n v  cæng sai l  A u1 =�22 ,d = 3 . B u1 =�21 ,d = 3 . C u1 =�21 ,d = �3. D u1 =�20 ,d = �3. ÊLíi gi£i.Theo b i ra ta câ ¨u1 + 3d= �12u 1 + 13d= 18 ,¨u1 =�21d = 3 :Chån ¡p ¡n B dC¥u 26. Hå nguy¶n h m cõa h m sè f(x ) = sin x� 1l  A cosx� x+ C. B �cos x+ C. C �cos x� x+ C. D cosx� x+ C. ÊLíi gi£i.Ta câ Zf(x )dx =Z(sin x� 1) d x= �cos x� x+ C.Chån ¡p ¡n C 296/305 296/305 pGV: L¶ Quang Xe  Ô0967.003.131PHT TRIšN — THAM KHƒO 2022 NÌI N€O CÂ Þ CH, NÌI  C CON ×ÍNG297dC¥u 27. ç thà h m sè y= 2x + 1 3� x câ t¥m èi xùng l  A I(� 2; 3) . B I(3; �2) . C I(3; �1) . D I(3; 2) . ÊLíi gi£i.ç thà h m sè y= ax+b cx+dnhªn giao iºm cõa hai ti»m cªn l  t¥m èi xùng.Ta th§y x= 3 ; y=�2 l¦n l÷ñt l  ti»m cªn ùng v  ti»m cªn ngang cõa ç thà y= 2x + 1 3� x.Chån ¡p ¡n B dC¥u 28. Gi¡ trà lîn nh§t cõa h m sè f(x ) = x4� 4x 2+ 5 tr¶n o¤n [� 2; 3] b¬ng A 50. B 5. C 1. D 122. ÊLíi gi£i.H m sè f(x ) = x4� 4x 2+ 5 x¡c ành v  li¶n töc tr¶n o¤n [� 2; 3] .Ta câ f0( x ) = 0 ,4x 3� 8x = 0 ,–x = 0x = p 2(c¡c nghi»m n y ·u thuëc o¤n[� 2; 3] )M  f(0) = 5 ,f €p 2Š= f€� p 2Š= 1 ,f(� 2) = 5 ,f(3) = 50 .Suy ra max[ � 2;3] f(x ) = f(3) = 50 .Chån ¡p ¡n A dC¥u 29. H m sèy= x3� 3x çng bi¸n tr¶n kho£ng n o trong c¡c kho£ng sau? A (� 2018; �2) . B (� 2; 0) . C (� 1; 0) . D (0; 1). ÊLíi gi£i.Tªp x¡c ành cõa h m sè D=R:Ta câ y0= 3 x2� 3) y0= 0 ,x= 1.B£ng bi¸n thi¶n xy0 y �1�1 1 +1 +0 �0 +�1�1 22�2 �2 +1 +1 Tø b£ng bi¸n thi¶n ta th§y h m sè çng bi¸n tr¶n kho£ng(� 2018; �2) .Chån ¡p ¡n A dC¥u 30. T½nhlog120 600theo a= log23v  b= log35. A log120 600 = 1 + ab ab+a+ 3 . B log120 600 = 1 + ab ab+b+ 3 . C log120 600 = 1 + ab 3ab +a+ 1 . D log120 600 = 1 + ab 3ab +b+ 1 . ÊLíi gi£i.Tø gi£ thi¸t ta câ ab= log23log35 = log25.Suy ra log120 600 = log2(2 33 52) log2(2 33 5) =3 + log23 + 2 log25 3 + log23 + log25 =3 +a+ 2 ab 3 +a+ ab .Chån ¡p ¡n A 297/305 297/305 pGV: L¶ Quang Xe  Ô0967.003.131— SÈ 19 NÌI N€O CÂ Þ CH, NÌI  C CON ×ÍNG298dC¥u 31. Mët bº c¡ h¼nh hëp chú nhªt câ thº t½ch 0;36 m3. Bi¸t k½ch th÷îc cõa ¡y bº l¦n l÷ñtb¬ng 0;5 m v  1;2 m. Ta câ chi·u cao cõa bº c¡ b¬ng A 0;65 m. B 0;6 m. C 0;7 m. D 0;5 m. ÊLíi gi£i.Ta câ V=h0;5 1;2 ) h= 0;36 0;5 1;2 = 0;6 m.Chån ¡p ¡n B dC¥u 32. Chof(x ) l  h m sè li¶n töc tr¶n o¤n [0; 10]thäa m¢n 10Z0 f(x ) d x= 7 v 6Z2 f(x ) d x=3 . T½nh 2Z0 f(x ) d x+ 10Z6 f(x ) d x. A 7. B �4. C 4. D 10. ÊLíi gi£i.2Z0 f(x ) d x+ 10Z6 f(x ) d x= 10Z0 f(x ) d x� 6Z2 f(x ) d x= 7 �3 = 4 :Chån ¡p ¡n C dC¥u 33. Trong khæng gian vîi h» tåa ë Oxyz, cho hai iºm A(1; 6; �7) v  B(3; 2; 1) . Ph÷ìngtr¼nh m°t ph¯ng trung trüc cõa o¤n th¯ng ABl  A x� 2y + 4 z+ 2 = 0 . B x� 2y � 3z � 1 = 0 . C x� 2y + 3 z+ 17 = 0 . D x� 2y + 4 z+ 18 = 0 . ÊLíi gi£i.Gåi (P )l  m°t ph¯ng trung trüc cõa o¤n th¯ng AB.Ta câ # AB = (2; �4; 8) . Suy ra mët v²c-tì ph¡p tuy¸n cõa (P )l  #n (P ) = (1;�2; 4)Hìn núa, trung iºm cõa ABl I(2; 4; �3) thuëc m°t ph¯ng (P )n¶n( P ) : ( x� 2) �2(y� 4) + 4( z+ 3) = 0 ,x� 2y + 4 z+ 18 = 0 :Chån ¡p ¡n D dC¥u 34. Cho tªpX=f1; 3; 5; 7; 9 g. Câ bao nhi¶u sè phùc z= x+ yi câ ph¦n thüc, ph¦n £o·u thuëc Xv  câ têng x+ y 10? A 20. B 10. C 15. D 24. ÊLíi gi£i.X²t sè phùc z= x+ yi (x; y 2X).V¼ sè phùc z= x+ yi thäa m¢n x+ y 10 n¶n ta x²t c¡c tr÷íng hñp saua) (x ;y ) 2 f (1; 3) ;(1; 5) ;(1; 7) ;(1; 9) ;(3; 5) ;(3; 7) g, câ 2 6 = 12 sè phùc thäa m¢n.b) (x ;y ) 2 f (1; 1) ;(3; 3) ;(5; 5) g, câ 3sè phùc thäa m¢n.Vªy câ 12 + 3 = 15 sè phùc thäa m¢n · b i.Chån ¡p ¡n C 298/305 298/305 pGV: L¶ Quang Xe  Ô0967.003.131PHT TRIšN — THAM KHƒO 2022 NÌI N€O CÂ Þ CH, NÌI  C CON ×ÍNG299dC¥u 35. Cho h¼nh l«ng trö tam gi¡c ·u ABC:A0B 0C 0câ AB =a; AA 0= 2 a. T½nh kho£ngc¡ch giúa hai ÷íng th¯ng AB0v  A0C . A ap 5. B 2a p 1717. C ap 32. D 2a p 55. ÊLíi gi£i.Gåi I ; M l¦n l÷ñt l  trung iºm cõa A0B; B C) I M A0C ) A0C (AB 0M ): d (AB 0; A 0C ) = d(C; (AB 0M )) =3 VB0AM C SAB 0M . Ta câVB0:AM C =1 3BB0 SAM C =1 6BB0 SAM C =a3p 312M B 0M =p BM2+ BB 02= ap 172. L¤i câAM?(BC C 0B 0) ) AM ?B 0M . Di»n t½ch tam gi¡c ABMl SABM =1 2AMB 0M =a2p 518:Vªy d(AB 0; A 0C ) = 2a p 1717: A IB CMA0 C0 B0 Chån ¡p ¡n B dC¥u 36. Mët a gi¡c lçi câ 10c¤nh, x²t c¡c tam gi¡c m  3¿nh l  ¿nh cõa a gi¡c. Häi trongsè c¡c tam gi¡c n y câ bao nhi¶u tam gi¡c m  c£ 3c¤nh ·u khæng ph£i l  c¤nh cõa a gi¡c? A 60. B 70. C 120. D 50. ÊLíi gi£i.* Sè tam gi¡c t¤o th nh tø 3 ¿nh cõa a gi¡c l  C310 .* Sè tam gi¡c t¤o th nh tø 3 ¿nh cõa a gi¡c câ 1 c¤nh l  c¤nh cõa a gi¡c:Chån 2 ¿nh k· nhau: câ 10 c¡ch chån.Chån ¿nh cán l¤i khæng k· vîi 1 trong 2 ¿nh ¢ chån: câ 6 c¡ch.Vªy câ 10:6 = 60 tam gi¡c.* Sè tam gi¡c t¤o th nh tø 3 ¿nh cõa a gi¡c câ 2 c¤nh l  c¤nh cõa a gi¡cChån 2 c¤nh k· nhau: câ 10 c¡ch.Vªy sè tam gi¡c c¦n t¼m l  C310 �60 �10 = 50 tam gi¡c.Chån ¡p ¡n D dC¥u 37. Trong khæng gian Oxyz, cho iºm A(1; 2; 3) v  ÷íng th¯ng d: x� 3 2=y� 1 1=z + 7 �2 . ÷íng th¯ng i quaA, vuæng gâc vîi dv  c­t tröc Oxcâ ph÷ìng tr¼nh l  A 8><>: x= �1 + 2 ty = �2tz = t . B 8><>: x= 1 + ty = 2 + 2 tz = 3 + 3 t. C 8><>: x= 1 + ty = 2 + 2 tz = 3 + 2 t. D 8><>: x= �1 + 2 ty = 2 tz = 3 t . ÊLíi gi£i.Gåi l  ÷íng th¯ng c¦n t¼m v  B(b ; 0; 0) l  giao iºm cõa v  Ox .Ta câ # AB = (b� 1;�2; �3) ,#u = (2; 1; �2) l  v²c-tì ch¿ ph÷ìng cõa d.Suy ra ?d, # AB #u = 0 ,2b + 2 = 0 ,b= �1.299/305 299/305 pGV: L¶ Quang Xe  Ô0967.003.131— SÈ 19 NÌI N€O CÂ Þ CH, NÌI  C CON ×ÍNG300i qua B(� 1; 0; 0) v  nhªn # AB = (�2; �2; �3) l m v²c-tì ch¿ ph÷ìng n¶n câ ph÷ìng tr¼nh8><>: x= �1 + 2 ty = 2 tz = 3 t:Chån ¡p ¡n D dC¥u 38. Cho b§t ph÷ìng tr¼nh log 5 + log (x2+ 1) log ( mx2+ 4 x+ m), m l  tham sè thüc.Câ bao nhi¶u gi¡ trà nguy¶n cõa tham sè mº b§t ph÷ìng tr¼nh nghi»m óng vîi måi x2 R. A 3. B 2. C 0. D 1. ÊLíi gi£i.Ta câ log 5 + log ( x2+ 1) log ( mx2+ 4 x+ m), ¨5x 2+ 5 mx 2+ 4 x+ mmx 2+ 4 x+ m > 0 .Vªy b§t ph÷ìng tr¼nh nghi»m óng vîi måi xkhi v  ch¿ khi¨ 5x 2+ 5 mx 2+ 4 x+ m; 8x 2 Rmx 2+ 4 x+ m > 0;8 x 2 R, ¨(5 �m)x 2� 4x + (5 �m) 0;8 x 2 Rmx 2+ 4 x+ m > 0;8 x 2 R, 8>>><>>>: 5� m > 04 � (5 �m)2< 0m > 02 � m2< 0, 0< m < p 2Tø â suy ra ch¿ câ mët gi¡ trà nguy¶n m= 1 tho£ m¢n y¶u c¦u b i to¡n.Chån ¡p ¡n D dC¥u 39. Bi¸t r¬ng ÷íng th¯ng y= 3 x+ 4 c­t ç thà h m sè y= 4x + 2 x� 1 t¤i hai iºm ph¥nbi»t câ tung ë y1 v y2. T½nhy1 +y2. A y1 +y2 = 11. B y1 +y2 = 9. C y1 +y2 = 1. D y1 +y2 = 10. ÊLíi gi£i.i·u ki»n x¡c ành x6= 1 .X²t ph÷ìng tr¼nh ho nh ë giao iºm3x + 4 = 4x + 2 x� 1 ,(3x+ 4)( x� 1) = 4 x+ 2 ,x2� x� 2 = 0 : (1)Gåi x1; x2l  nghi»m cõa ph÷ìng tr¼nh(1), ta câ ¨x1 +x2 = 1x 1x2 =�2:M  y1 = 3x1 + 4v y2 = 3x2 + 4, do vªy y1 +y2 = 3(x1 +x2) + 8 = 11.Chån ¡p ¡n A dC¥u 40. Cho h m sè f(x ) li¶n töc tr¶n R. Bi¸t cosx+ xl  mët nguy¶n h m cõa h m sèf (x )e 2x, hå t§t c£ c¡c nguy¶n h m cõa h m sè f0( x )e 2xl  A �2x + 2 cos x+ sin x+ C (C 2R). B 2 cosx+ sin x+ C (C 2R). C 2 cosx� sin x+ C (C 2R). D �2x � 2 cos x� sin x+ C (C 2R).300/305 300/305 pGV: L¶ Quang Xe  Ô0967.003.131PHT TRIšN — THAM KHƒO 2022 NÌI N€O CÂ Þ CH, NÌI  C CON ×ÍNG301ÊLíi gi£i.Do cosx+ xl  mët nguy¶n h m cõa h m sè f(x )e 2xtr¶n Rn¶nf (x )e 2x= (cos x+ x)0; 8x 2 R, f(x )e 2x= �sin x+ 1 ;8x 2 R, f(x ) = (1 �sin x)e �2x; 8x 2 R) f0( x ) = ( �cos x)e �2x� 2(1 �sin x)e �2x; 8x 2 R) f0( x ) = �(2 �2 sin x+ cos x)e �2x; 8x 2 R:Do â, hå t§t c£ c¡c nguy¶n h m cõa h m sè f0( x )e 2xl Zf0( x )e 2xdx = Z� (2 �2 sin x+ cos x)e �2x e2xdx= Z(� 2 + 2 sin x� cos x) d x= �2x � 2 cos x� sin x+ C(C 2R):Chån ¡p ¡n D dC¥u 41. Cho h¼nh châp S:ABCcâ ¡y l  4ABC vuæng t¤i C,AB = 2a, AC =a, v  S Avuæng gâc vîi m°t ph¯ng (ABC ). Bi¸t gâc giúa hai m°t ph¯ng (S AB )v  (S B C )b¬ng 60. T½nhthº t½ch cõa khèi châp S:ABC. A a3p 26. B a3p 612. C a3p 64. D a3p 22. ÊLíi gi£i.Trong 4ABC k´C H ?AB) C H ?(S AB )) C H ?S B (1).BC =p AB2� AC 2= ap 3,BH BA =BC 2,) BH =3a 2,C H =p BC2� BH 2= ap 32.Trong 4S AB k´K H ?S B )C K ?S B .Tø (1) v (2) suy ra H K?S B .Gâc giúa hai m°t ph¯ng (S AB )v  (S B C )l  ÖC K H =60 . Trong 4C K H vuæng t¤i Hcâ H K =C H cot 60 = a 2,BK =p BH2� H K 2= ap 2.4 S AB v4H K B n¶nS A H K=AB BK=2a ap 2.Thº t½ch h¼nh châp S:ABCl V = 1 3S AS4 ABC =1 3a p21 2a p 3a = a3p 612: 60 CA BKSHa 2a ap 3Chån ¡p ¡n B dC¥u 42. Câ bao nhi¶u sè thùc thäa m¢n z+ jz j2i � 1� 3 4i= 0 ? A 1. B 3. C 2. D 0.ÊLíi gi£i.301/305 301/305 pGV: L¶ Quang Xe  Ô0967.003.131— SÈ 19 NÌI N€O CÂ Þ CH, NÌI  C CON ×ÍNG302°tz= a+ bi (a; b 2R): Thay v o biºu thùc cõa b i to¡n ta câ:( a � 1) + �a2+ b2+ b� 3 4‹i= 0 )8<: a= 1b 2+ b+ 1 4= 0 )8<: a= 1b = �1 2:Vªy ch¿ câ óng mët sè phùc thäa m¢n b i to¡n.Chån ¡p ¡n A dC¥u 43. Cho sè phùc z= 1 + ip 3. Sè phùc li¶n hñp cõa zl  A z= 1 �ip 3. B z= �p 3� i. C z= �1 + ip 3. D z= p 3 +i. ÊLíi gi£i.z = a+ ib ) z= a� bi.Chån ¡p ¡n A dC¥u 44.Cho h m sè y= f(x ) vîi ¤o h m f0( x ) câ ç thà nh÷ h¼nh v³. H m sèg (x ) = f(x ) � x3 3+x2� x+ 2 ¤t cüc ¤i t¤i iºm n o? A x= �1. B x= 2 . C x= 0 . D x= 1 . xyO�1 �2 211ÊLíi gi£i.X²t h m sè g(x ) = f(x ) � x3 3+x2� x+ 2 , câ g0( x ) = f0( x ) � x2+ 2 x� 1;8x 2 RTa câ g0( x ) = 0 ,f0( x ) = ( x� 1)2, 264 x= 0x = 1x = 2 :(*)B£ng x²t d§u cõa g0( x ) xg0( x ) �10 1 2 +1 �0 +0 �0 +Vªy h m sèg(x ) ¤t cüc ¤i t¤i x= 1 .Chån ¡p ¡n D dC¥u 45. Trong khæng gian vîi h» tröc tåa ë Oxyz, cho ba iºmA (1; 1; 0) ; B(1;�2; 4) ; C(13; 1; 0) v dl  ÷íng th¯ng m  måi iºm cõa dluæn c¡ch ·uA; B; C . Ph÷ìng tr¼nh ÷íng th¯ng dl  A 8>><>>: x= 7y = �1 2+ 4tz = 2 + 3 t. B 8><>: x= 7y = 1 �8tz = �6t . C 8>><>>: x= 1y = �1 2�24tz = 2 �18t . D 8>><>>: x= 5y = 48 tz = 4 3+ 36t. ÊLíi gi£i.302/305 302/305 pGV: L¶ Quang Xe  Ô0967.003.131PHT TRIšN — THAM KHƒO 2022 NÌI N€O CÂ Þ CH, NÌI  C CON ×ÍNG303dluæn c¡ch ·u A; B; Cn¶nd? (ABC )t¤i t¥m I(I l  t¥m ÷íng trán ngo¤i ti¸p ABC )Nhªn x²t :# AB = (0; �3; 4) ;# AC = (12; 0; 0) )# AB # AC = 0 )ABC vuæng t¤i An¶n Il  trungiºm c¤nh BC)I�7; �1 2; 2 ‹.d qua Iv  câ VTCP #u = ”# AB; # AC —= (0; 48; 36) hay#u 0= (0; 4; 3) . Vªyd: 8>><>>: x= 7y = �1 2+ 4tz = 2 + 3 t.Chån ¡p ¡n A dC¥u 46. Cho hai m°t c¦u (S1)câ t¥m I1, b¡n k½nhR1= 1,(S2)câ t¥m I2 b¡n k½nhR2= 5.L¦n l÷ñt l§y hai iºm M1,M2thuëc hai m°t c¦u(S1), (S2). Gåi Kl  trung iºm cõa M1M2. KhiM 1,M2di chuyºn tr¶n(S1), (S2)th¼ Kqu²t mi·n khæng gian l  mët khèi trán xoay câ thº t½chb¬ng? A 55 3. B 68 3. C 76 3. D 82 3. ÊLíi gi£i. I1 I2 E M2 M1 KGåiEl  trung iºm cõa o¤n I1I2. Khi â, ta câ# I 1M+# I 2N=€# I 1E+ # E M Š+ €# I 2E+ # E N Š= €# I 1E+ # I 2E Š+ €# E M +# E N Š= #0 + 2 # E K) j I1M�I2Nj  2E K I1M+I2N) 2 E K 3:Thº t½ch khèi trán xoay c¦n t¼m l  V= 4 3(3 3� 23) = 76 3.Chån ¡p ¡n C dC¥u 47. Sè gi¡ trà nguy¶n d÷ìng cõa tham sè mº b§t ph÷ìng tr¼nh €2x+2�p 2Š(2 x� m)< 0câ khæng qu¡ 6nghi»m nguy¶n l  A 29. B 33. C 32. D 31. ÊLíi gi£i.Ta câ €2x+2�p 2Š(2 x� m)< 0vîi x2 Z v  y2 Z+.303/305 303/305 pGV: L¶ Quang Xe  Ô0967.003.131— SÈ 19 NÌI N€O CÂ Þ CH, NÌI  C CON ×ÍNG304ÌTr÷íng hñp 1. N¸u¨2x+2�p 2< 02 x� m > 0 ,8<: x+ 2 <1 2x > log2m , 8<: x <�3 2x > log2m:Theo y¶u c¦u b i to¡n, mët mcâ khæng qu¡ 6sè nguy¶n x, m  x <�3 2n¶n� 8 log2m � 2, 2�8< m 2�2:M  m2Z+) Khæng tçn t¤i mthäa m¢n y¶u c¦u b i to¡n.Ì Tr÷íng hñp 2. N¸u¨2x+2�p 2> 02 x� m < 0 ,8<: x+ 2 >1 2x < log2m , 8<: x >�3 2x < log2m:Theo y¶u c¦u b i to¡n, mët mcâ khæng qu¡ 6sè nguy¶n x, m  x >�3 2n¶n� 1 log2m5, 1 2m 25= 32 :M  m2Z+) m2 f 1; 2; 3 : : :; 31; 32 gcâ 32 gi¡ trà.Vªy câ 32gi¡ trà nguy¶n d÷ìng cõa tham sè mthäa m¢n b§t ph÷ìng tr¼nh ¢ cho.Chån ¡p ¡n C dC¥u 48. Trong khæng gian Oxyzcho m°t c¦u (S ) : x2+ y2+ z2� 2x + 2 z+ 1 = 0 v  ÷íngth¯ng d:x 1=y� 2 1=z �1. Hai m°t ph¯ng(P ); (P 0) chùa dv  ti¸p xóc vîi (S ) t¤i Tv  T0. T¼mtåa ë trung iºm Hcõa T T0. A H�5 6;1 3;� 5 6‹. B H�5 6;2 3;� 7 6‹. C H�� 5 6;1 3;5 6‹. D H�� 7 6;1 3;� 7 6‹. ÊLíi gi£i.M°t c¦u (S ) câ t¥m I(1; 0; �1) v  b¡n k½nh R= 1 .¨I T ?(P )) I T ?dI T 0? (P 0) ) I T0? d)d? (I T T 0) .Gåi N=d\ (I T T 0) ) Nl  h¼nh chi¸u cõa I tr¶n d.÷íng th¯ng dcâ ph÷ìng tr¼nh tham sè 8><>: x= ty = 2 + tz = �t t2 R) N(t; 2 + t; � t) v  # I N = (t� 1; 2 + t; � t+ 1) . IT0 TH N# I N #u = 0 ,t� 1 + 2 + t+ t� 1 = 0 ,t= 0 )N(0; 2; 0) )¨I N =p 6# I N = (�1; 2; 1) :Ta câ I HI N =I T 2) I H =1 p6.Ph÷ìng tr¼nh ÷íng th¯ng I N:8><>: x= �uy = 2 + 2 uz = u )H(� u; 2 + 2 u;u ) u 2 Rv  # I H = (�u � 1; 2 + 2 u;u + 1) .304/305 304/305 pGV: L¶ Quang Xe  Ô0967.003.131MÖC LÖCNÌI N€O CÂ Þ CH, NÌI  C CON ×ÍNG305I H=1 p6,I H 2= 1 6,(� u � 1)2+ (2 u+ 2) 2+ ( u+ 1) 2= 1 6, 2664 u= �5 6)H�5 6;1 3;� 5 6‹) # I H =�� 1 6;1 3;1 6‹u = �7 6)H�7 6;� 1 3;� 7 6‹) # I H =�1 6;� 1 3;� 1 6‹:V¼ # I H còng h÷îng vîi # I N )H�5 6;1 3;� 5 6‹.Chån ¡p ¡n A dC¥u 49.Cho h m sè y= f(x ). H m sè y= f0( x ) câ ç thà nh÷ h¼nh v³ H m sèy = f(x 2) câ bao nhi¶u iºm cüc trà? A 3. B 5. C 4. D 3. xyO 4�1 1ÊLíi gi£i.Düa v o ç thà ta th§y y= f0( x ) l  h m bªc 3n¶n ta câf 0( x ) = (x + 1)( x� 1)( x� 4) vîi 2 R.) g0( x ) = f0( x 2) 2x = (x 2+ 1)( x2� 1)( x2� 4) 2x = 0 ,264 x= 2x = 0x = 1:Ta câ b£ng bi¸n thi¶n xg0( x ) g(x ) �1�2 �1 0 1 2 +1 �0 +0 �0 +0 �0 ++1 +1 �2 �2 �1 �1 00 1122 +1 +1 Düa v o b£ng bi¸n thi¶n ta th§y h m sèg(x ) = f(x 2) câ 5cüc trà.Chån ¡p ¡n B H˜T305/305 305/305 pGV: L¶ Quang Xe  Ô0967.003.131

- Xem thêm -

Tài liệu liên quan

Bình luận